Текст
                    Г.Д ГяльлЕРим, ДК.Тотып?
№скодкие
МЛГЕМАТИУЕСКИЕ
СМИМПИДДЫ


ББК 22.1 Г17 Рецензенты: доктор физико-математических наук, профессор МГУ В. М. Тихомиров, кандидат педагогических наук, зав. кабинетом математики МГИУУ С. М. Саакян. Гальперин Г. А., Толпыго А. К. Г17 Московские математические олимпиады: Кн. для уча- щихся/Под ред. А. Н. Колмогорова.— М.: Просвещение, 1986.—303 с.» ил. Книга содержит задачи всех Московских математических олимпиад за 50 лет их про- ведения. К большинству задач даны ответы, указания, решения. D книге много интерес- ных задач, связанных с современными научными проблемами. Книга предназначена для учащихся VII—X классов средней школы, интересующихся математикой, а также может быть использована учителями во внеклассной работе. 4306020000—774 103(03)—86 227—86 ББК 22.1 51 © Издательство «Просвещение», 1986
ПРЕДИСЛОВИЕ РЕДАКТОРА Нашей стране необходимо иметь много математиков-иссле- дователей, способных делать открытия в самой математике и применять ее нестандартным образом, требующим большой изобретательности. Обычно серьезных успехов достигают те на- учные работники, которые начали тренироваться в такого рода деятельности еще в школьные годы. В возрасте 17—19 лет мно- гие из них уже начинают делать настоящие открытия. Откла- дывая вовлечение молодых людей в напряженную научную работу, мы безвозвратно теряем многих из тех, кто мог бы сделаться творчески активным ученым. Обращаясь к самим школьникам, всерьез собравшимся стать настоящими математиками, скажу следующее. Как и в спорте, тренировка юного математика требует затраты большого вре- мени. Будет очень хорошо, если вы возьметесь самостоятельно просматривать предлагаемый сборник задач, выберете из их числа какую-нибудь задачу, которая покажется вам наиболее ин- тересной по формулировке, и приметесь, нс заглядывая в реше- ния, размышлять над ней, нс боясь потратить на нес многие, многие часы. Напомню по этому поводу высказывание одного из самых замечательных советских математиков — Бориса Николаевича Делоне, по мнению которого большое на- учное открытие отличается от хорошей олимпиадной задачи только тем, что для решения олимпиадной задачи требуется 5 ча- сов, а получение крупного научного результата требует затраты 5000 часов. Борис Николаевич любил несколько преувеличенные формулировки, не понимайте его «5000 часов» слишком букваль- но. Но типичным для математика, который атакует трудную проблему, является способность напряженного размышления над ней целыми днями. Если задача упорно не выходит, то разум- но взяться за другую. Но хорошо также после некоторого пере- рыва вернуться к первоначальной. Зрелым математикам тоже иногда бывает полезно на некоторое время отложить занятие какой-либо неподдающейся проблемой. Нередко после некото- рого перерыва решение неожиданно выплывает из подсознания. Своим успехам на олимпиаде естественно радоваться и даже гордиться ими. Неудачи же на олимпиаде нс должны чрезмер- но огорчать и приводить к разочарованию в своих способностях к математике. Для успеха на олимпиаде необходимы некото- рые специальные типы одаренности, которые вовсе не обяза- тельны для успешной исследовательской работы. Уже само на- личие назначенного очень ограниченного срока для решения задач многих делает совершенно беспомощными. Но существуют з
и такие математические проблемы, которые могут быть решены лишь в результате очень длительного и спокойного размышле- ния и формирования новых понятий. Много такого рода проб- лем было решено замечательным советским топологом П. С. Алек- сандровым. Нс случайно Павел Сергеевич Александ- ров говорил, что если бы во времена его юности были ма- тематические олимпиады, то, возможно, он вообще не сделался бы математиком: его главные достижения в математике явились нс плодом быстро работающей изобретательности, а итогом дли- тельного и углубленного созерцания. Я надеюсь, что наш сборник окажется неоценимым пособием для всех руководителей школьных кружков и местных олимпиад. Для них я хочу высказать два замечания. Вначале Московские математические олимпиады были рас- считаны на учащихся IX—X классов. Начиная же с 1940 г. к участию в олимпиадах приглашались также семиклассники и восьмиклассники. Такой выбор начального возраста представ- ляется мне обоснованным. Это тот возраст, когда склонности и способности к математике уже начинают проявляться достаточно явственно. Можно, конечно, устраивать олимпиады и для млад- шеклассников, но при этом следует иметь в виду, что из числе мальчиков и девочек, выделившихся в V—VI классах в состяза- нии по решению задач, большинство в старших классах эти свои особые способности, а часто и сам интерес к математике поте- ряют При организации олимпиад для того или иного контингента участников чрезвычайно существенно, чтобы уровень трудности задач был надлежащим образом заранее правильно оценен. Сле- дует планировать его так, чтобы наиболее сильные участники могли решить большую часть задач, а с другой стороны, чтобы нс было чрезмерного преобладания участников, нс решивших ни одной задачи. Некоторые сведения о фактически обнару- жившейся трудности задач можно найти в отчетах об олим- пиадах, печатающихся в журналах «Математика в школе» и «Квант». К сожалению, в Московских математических олимпиа- дах уровень трудности нс всегда выбирался правильно. Но содер- жание задач было обычно на очень высоком уровне. В предисловии составителей подробно рассказывается об огромном опыте Московских математических олимпиад, о том, как процесс создания олимпиадных задач шел в неразрывной связи с работой математических кружков при МГУ. Коллектив руководителей университетских математических кружков про- делал огромную, уникальную работу, итоги которой сейчас перед вами. Замечательна и заслуживает большой благодарности работа составителей — Г. А. Гальперина и А. К. Толпыго. Академик А. Н. Колмогоров
ПРЕДИСЛОВИЕ АВТОРОВ Из истории Московских олимпиад В этой книге собраны задачи всех Московских математи- ческих олимпиад, начиная с первой, состоявшейся в 1935 г., к большинству из них даны ответы, указания и решения. В сво- ей совокупности эти задачи представляют собой плод многолет- ней коллективной работы студентов, аспирантов и преподава- телей механико-математического факультета МГУ ряда поколе- ний, а в последнее время также математиков МГПИ им. В. И. Ле- нина, МИИТа и факультета вычислительной математики и ки- бернетики МГУ. Наша книга создавалась не на пустом месте. Двадцать лет назад задачи Московских олимпиад 30—50-х годов почти в пол- ном объеме были собраны и опубликованы в «Сборнике задач Московских математических олимпиад» (см. [21J в списке ли- тературы). При создании этого сборника была проделана огром- ная работа как его составителем А. А. Леманом, так и всеми, кто способствовал его изданию. Однако сборник [21] давно уже стал библиографической редкостью и для большинства сегод- няшних школьников практически недоступен. Кроме того, в нем содержатся задачи только первых 27 олимпиад (до 1964 г включительно). Критический пересмотр задач этих олимпиад позволил нам обнаружить ряд недочетов, которые мы по мере своих сил и воз- можностей ликвидировали. Таким образом, ряд задач первых олимпиад, а также задачи последних 20 олимпиад в нашей кни- ге впервые предлагаются широкому кругу читателей. В ре- зультате новое поколение школьников получает возможность по- знакомиться с тем огромным набором интересных и красивых идей, который заключен в более чем тысяче задач Московских олимпиад, а также и с историей этих олимпиад. Об истории пер- вых 27 олимпиад хорошо рассказано в статье, написанной В. Г. Болтянским и И. М. Ягломом для книги [21] ; в ней они сумели ярко отразить дух Московских олимпиад, царивший в те годы. Мы приносим благодарность авторам за любезное разре- шение воспользоваться материалами их статьи. 5
о Московские математические олимпиады уже давно стали тра- диционными. Какова их цель и как они проводятся? В 30-е годы была осознана необходимость участия ученых- математиков в работе со школьниками. Инициаторами такой работы выступили в Ленинграде член-корреспондент АН СССР Б. Н. Делоне и профессор В. А. Тартаковский, а в Москве — член-корреспондент АН СССР Л. Г. Ш н и ре л ь- м а н и профессор (впоследствии член-корреспондент АН СССР) Л. А. Л ю с т е р н и к. Весной 1934 г. в Ленинграде была про- ведена первая в СССР школьная математическая олимпиада, а с осени 1934 г. в Москве, в Институте математики АН СССР, начали регулярно читаться лекции по математике для учащихся старших классов. Одновременно по инициативе Л. А. Люстер- ника начала выходить серия «Популярная библиотека по ма- тематике», предназначенная специально для школьников. С целью привлечения к активным занятиям способных школь- ников, интересующихся математикой, весной 1935 года правле- ние Московского математического общества, подхватив ини- циативу ленинградцев, приняло решение о проведении I Москов- ской математической олимпиады. К этому мероприятию москов- ские математики отнеслись с большим воодушевлением. В орг- комитет олимпиады вошли профессора-математики МГУ, среди них А. Н. Колмогоров, Л. А. Л ю с т е р н и к, Л. Г. Ш н и- рельман, В. Ф. Каган, С. А. Яновская и др. Предсе- дателем оргкомитета стал президент Московского математи- ческого общества П. С. Александров. Олимпиада ставила своей целью выявить наиболее способных учащихся, привлечь внимание широких масс школьной молодежи к важнейшим проб- лемам и методам современной математики и хотя бы частично показать, над чем работает отечественная математическая наука, каковы ее достижения и какие задачи стоят перед ней. В I олимпиаде приняло участие 314 школьников — большой успех в то время. Во втором (заключительном) туре приняло участие 120 человек, из которых трое получили первые премии, а пятеро школьников — вторые; кроме того, 44 школьника полу- чили почетные призы. Для многих школьников победа на олим- пиаде определила характер их будущей научной деятельности. Любопытен был набор задач второго тура первой олимпиады. Были предложены три серии задач: А, В и С. Как рассказал нам А. Н. Колмогоров, это было сделано по его инициативе, чтобы дать возможность проявить себя ученикам с разным скла- дом математического мышления: вычислительным (или «алго- ритмическим»), геометрическим, комбинаторно-логическим (под- робно об этом можно прочесть в брошюре «О профессии мате- матика» [13]). В соответствии с этими типами мышления и под- бирались серии задач на первую олимпиаду. 6
Успех I Московской олимпиады способствовал полной пере- стройке всей работы со школьниками, в частности возник Школь- ный математический кружок при МГУ. Организаторами его яви- лись Л. А. Л юс те р н и к, Л. Г. Шнирельман и И. М. Гельфанд. Работа кружка проводилась в двух направ- лениях: в чтении разнообразных по тематике лекций и в засе- даниях кружка. На лекции приходили сначала десятки, а затем и сотни школьников Москвы. Первоначально проводились лек- ции для учащихся VIII—X классов, а затем, с 1940 г., были об- разованы две группы: для VII—VIII и IX—X классов. В своих выступлениях лекторы излагали в популярной форме серьезные математические результаты, включая научные достижения самых последних лет. В 1950 г. Гостехиздат (переименованный затем в Физматгиз и вошедший теперь в издательство «Наука») начал издавать спе- циальную серию книг «Популярные лекции по математике», большинство которых возникло при обработке лекций, прочи- танных в математическом кружке при МГУ. Часть лекций была также опубликована в сборниках «Математическое просве- щение». Наряду с лекциями регулярно проводились и секционные за- седания кружка. Ими руководили, как правило, студенты стар- ших курсов и аспиранты мехмата МГУ (за исключением двух секций 1936—1937 гг.: секции геометрических методов теории чисел, руководимой Л. Г. Шнирельманом, и секции ка- чественной геометрии, руководимой А. Н. Колмогоро- вым). Сначала на секциях в основном делали доклады школь- ники, но вскоре выяснилось, что такие доклады являются мало- эффективной формой работы кружка. Дело в том, что большин- ство сообщений школьников на секциях оказывались скучными для всех участников кружка (кроме, быть может, самого до- кладчика). Ведь для того, чтобы сделать хороший доклад, не- достаточно полностью понять все то, что сказано в отрывке ма- тематического текста, указанного руководителем секции. До- клад должен заинтересовать слушателей и заставить их заду- маться над услышанным; в нем должны быть выпукло препод- несены подлежащие рассмотрению задачи, должны быть отте- нены основные идеи решения, ярко обрисованы красивые, ори- гинальные места доказательств и т. д. Кроме того, известно, что хорошую лекцию редко может прочесть человек, познания ко- торого в данном вопросе ограничиваются лишь конкретным ма- териалом, связанным лишь с данной темой. Поэтому доклад школьника обычно уступает по качеству рассказу опытного лектора. Решительная перестройка работы секций связана с именем талантливого математика и блестящего педагога Д. О. Шкля- р с к о г о, в то время студента МГУ, руководившего рабо- той кружков в 1938—1941 гг. (В 1942 г. Д. О. Шклярский 7
в возрасте 23 лет погиб в партизанском отряде, сражавшемся с немецко-фашистскими захватчиками в тылу врага.) Доклады школьников на заседаниях его секции, за редким исключением, не заслушивались, а вместо этого руководитель каждый раз сам читал краткую лекцию по той или иной теме. В конце лекции школьники рассказывали о решенных ими задачах, предлагав- шихся на предыдущих заседаниях. Наличие задач разной труд- ности позволяло Д. О. Шклярскому вовлекать в активную работу практически всех участников секции, а повторение рас- сказанного школьником решения задачи достигало сразу двух целей: слушатели лучше усваивали это решение, а автор решения получал наглядный урок ясного и четкого изложения матема- тического доказательства. Такая система принесла замечательные плоды: в 1938 г. на IV олимпиаде участники секции Д. О. Шклярского за- воевали половину всех премий (12 из 24), в том числе все 4 пер- вые премии! Итоги IV олимпиады произвели на руководителей других секций настолько сильное впечатление, что уже со сле- дующего года практически все они пересмотрели форму заня- тий с учащимися. С тех пор форма работы кружка, найденная Д. О. Ш к л я р с к и м, стала господствующей. С самых первых лет работы кружка возникла традиция из- дания ежегодного небольшого сборника подготовительных задач к олимпиаде, который вручался участникам кружка и всем желающим принять участие в олимпиаде. Задачи первых четырнадцати Московских математических олимпиад и другие материалы секций кружка частично были опубликованы в трех книгах серии «Избранные задачи и тео- ремы элементарной математики»; первая книга [25] посвящена арифметике и алгебре, вторая [26] и третья [27] — геометрии (планиметрии и стереометрии соответственно). Среди авторов этих трех книг первым указан Д. О. Ш к л я р с к и й, формально нс участвовавший в подготовке издания, но всей своей деятель- ностью содействовавший появлению этих книг. Если кружок привлекал к систематической работе несколько сот московских школьников, то число участников Московской олимпиады всегда было значительно больше и достигало не- скольких тысяч. Все аудитории во время проведения олимпиад в указанные годы были переполнены, и приходилось размещать часть школьников в лабораториях физического, химического и биологического факультетов МГУ. Форма проведения олимпиады практически нс изменилась со времени I олимпиады 1935 г. Первые 36 олимпиад (1935— 1973 гг.) проводились в два тура, по воскресеньям в конце мар- та— начале апреля. 1-й тур являлся отборочным; на нем каж- дому из участников предлагалось решить 4—6 сравнительно не- сложных (но с изюминкой!) задач, причем участники уведомля- лись о том, что для прохождения на 2-й тур достаточно решить 8
две задачи. Через неделю после 1-го тура проводился разбор предложенных задач с указанием различных решений и типичных ошибок и объявлялись результаты тура. Еще через неделю про- ходил 2-й тур, на который приглашались все успешно прошед- шие 1-й тур (30—50% его участников) ; в некоторых случаях ко 2-му туру допускались школьники, не прошедшие 1-й тур (это происходило по рекомендации одного из членов оргкомитета, усмотревшего нетривиальность мышления школьника, а иногда по личной просьбе школьника), а также школьники, по каким-либо причинам не сумевшие прийти на 1-й тур (как правило, из-за бо- лезни). Задачи 2-го тура были уже существенно сложнее за- дач 1-го тура. На решение задач на каждом туре отводилось 5—6 часов. Наконец, через неделю после 2-го тура проводился оконча- тельный разбор задач. Обычно для разбора задач, придуман- ных в основном студентами и аспирантами, привлекали крупных математиков. Делалось это для того, чтобы соединить рассказы о решениях задач, предложенных на олимпиаде, с указанием более широких перспектив в математике. Так, один из нас был свидетелем того, как А. Н. Колмогоров перед аудиторией, состоявшей из нескольких сот восьмиклассников, разбирая ре- шение двух задач XXXVIII олимпиады, перешел к обсуждению современных проблем дискретной математики (теории графов и теории информации). В заключение проходило награждение победителей олим- пиады. Им вручались призы — математические книги с дарствен- ными надписями. Победителям присуждалось в среднем около десяти первых премий (по разным классам), около двадцати вторых и около тридцати третьих; кроме того, выдавалось не- сколько десятков похвальных отзывов l-й и 2-й степени. При под- ведении итогов олимпиады и награждении победителей резуль- таты 1-го тура, как правило, не учитывались. Задачи первых пяти олимпиад предлагались всем школьни- кам без разделения их на классы. Начиная с VI олимпиады (1940 г.) учащиеся разделялись на два потока: отдельно сорев- новались школьники VII—VIII классов и отдельно — старше- классники. Во время Великой Отечественной войны произошел трехлетний перерыв в проведении Московских олимпиад — с 1942 по 1944 г.; в эти годы московские математики провели олимпиады в Ашхабаде и Казани. К сожалению, никаких материалов тех олимпиад не сохранилось. Начиная с XV олимпиады (1952 г.) соревнования проводи- лись уже по каждому классу в отдельности, хотя некоторые наибо- лее интересные задачи предлагались параллельно в нескольких классах. С самого начала проведения олимпиад большую организа- ционную работу взяли на себя Московский городской отдел на- родного образования и Московский городской институт усовер- 9
шенствования учителей. Сотрудники института совместно с наи- более опытными учителями и преподавателями МГУ с 1949 г. стали проводить районные математические олимпиады; задачи этих олимпиад приведены в книге [11]. Это позволило привлечь к занятиям математикой еще более широкий круг школьников, нс только старшеклассников, но и учеников V—VII классов. Если школьный математический кружок на протяжении при- мерно четверти века был господствующей формой внеклассной математической работы со школьниками, а Московская олим- пиада была, так сказать, фокусом, в котором сходились все линии этой работы, то за последние 20 лет формы такой работы стали заметно разнообразнее. Возникли специализированные матема- тические школы; в 1963 г. была создана вечерняя математи- ческая школа (ВМШ), а через год возникла заочная математи- ческая школа (ЗМШ). Вслед за МГУ свои математические олим- пиады стали проводить другие вузы Москвы; наряду с город- скими олимпиадами возникла система республиканских, Всесо- юзных и, наконец, Международных олимпиад. Тем не менее Московская олимпиада долгое время нс переставала играть осо- бую роль: получить на ней награду считалось весьма почетным, а по уровню сложности задач она превосходила все остальные математические олимпиады. Иногда время проведения несколь- ких олимпиад совпадало, и некоторые победители Московской олимпиады часто нс имели возможности прийти на ее закры- тие, поскольку в это же самое время они участвовали в другой олимпиаде. Тогда им высылалось специальное приглашение для получения положенной награды. Текст приглашения носил шутливый характер: «Награждение наградами награжденных, не награжденных наградами на награждении 17 марта, состо- ится 1 апреля». В 1961 г. на XXIV Московскую олимпиаду были приглашены команды областей, краев и союзных республик — так было по- ложено начало проведению единой многоэтапной математи- ческой олимпиады по всей стране. Первая Всесоюзная математическая олимпиада состоялась 16 апреля 1967 г. в г. Тбилиси. Поскольку 2-й тур XXX Московской олим- пиады проходил в тот же день, команду от Москвы пришлось фор- мировать по результатам 1-го тура. В последующие годы сроки Московских олимпиад были перенесены с апреля на март, с тем чтобы успеть отобрать команду на проходящую в середине апре- ля Всесоюзную олимпиаду. Сначала команда Москвы отбиралась непосредственно по результатам Московской олимпиады. Позже стали устраивать- ся дополнительные «отборочные туры», на которые приглаша- лись 15—20 человек, получившие первые, вторые, иногда даже третьи премии. Задачи отборочного тура выбирались, как пра- вило, из первоначально предлагавшихся для олимпиады и нс включенных в нес по разным причинам. Отборочный тур, одна- 10
ко, никогда не считался дополнительным (завершающим) туром олимпиады. Дополнительный тур был проведен только один раз — на XXXIII Москов- ской олимпиаде для учащихся VII класса (1970 г.); он получил название «День Пифагора». Этот тур был проведен с отклонениями от сложившихся тради- ций. Сначала школьникам предложили решить три задачи. Через два часа все работы участников были собраны и объявлен получасовой перерыв, после окон- чания которого были даны еще три задачи. Интересным было и другое новшество: одну из задач «Дня Пифагора» (33.42) оргкомитет дал, не предполагая определенного на нее ответа (точнее, этот ответ был неизвестен): школьникам объявили, что задача эта исследо- вательская и надо попытаться продвинуться в ее решении как можно дальше. Другая задача «Дня Пифагора» (33.45), как сейчас выяснилось, полу- чила неожиданное распространение за рубежом, и в недавно изданной книге «Математический цветник» [18] в заметке «Занимательное столоверчение» приведено ее обобщение на случай «многорукого Али-Бабы». В заметке ска- зано, что сначала она побывала на страницах февральского номера журнала „Scientific American" за 1979 г., где ее опубликовал Мартин Гарднер — известный популяризатор математики, хорошо знакомый советскому читателю по целому ряду книг и статей ([5] — [8]); однако сам Гарднер призна- ется, что задачу ему сообщил «Роберт Таппей из Торонто, по мнению ко- торого задача пришла к нам из Советского Союза» [18]. Таким образом, за- дача совершила большой путь, прежде чем вернуться домой, впрочем, в обоб- щенном виде. Если до 60-х годов организацией и проведением Москов- ских олимпиад занимались преподаватели и аспиранты мех- мата МГУ и сотрудники Института математики АН СССР им. В. А. Стеклова, то позже и в других вузах и научных учрежде- ниях Москвы появилось много молодых математиков, которые с увлечением стали заниматься подготовкой задач и проведением олимпиад. Некоторые вузы столицы стали проводить свои соб- ственные олимпиады; кроме того, намного поднялся уровень районных олимпиад. Поэтому было решено привлечь к прове- дению Московских олимпиад, кроме МГУ, еще два вуза: М Г П И им. В. И. Ленина и Московский институт инженеров транспорта (М И И Т), что и было сделано в 1975 г., а также проводить Мо- сковскую олимпиаду для младших школьников только в один тур. Начиная с 1981 г. олимпиаду по VII классу проводил факуль- тет вычислительной математики и кибернетики МГУ; в настоя- щее время он проводит олимпиаду и по IX классу. Заседания оргкомитета по обсуждению задач, подведению итогов олим- пиады и другим вопросам проводились совместные, как правило, в МГУ; там же проводились разбор задач олимпиады и награж- дение победителей. Первый тур XXXVII олимпиады (1974 г.) проводился толь- ко для учащихся IX—X классов, а на XXXVIII—XL олим- пиадах — только для десятиклассников. Результаты 1-го тура на этих олимпиадах учитывались при подведении итогов, а школьники, решившие все задачи 1-го тура, получали премию или отзыв на разряд выше, чем могли бы получить просто по результатам 2-го тура. Начиная с XLI олимпиады 1978 г. было и
решено проводить по всем классам только один, заклю- чительный тур, к участию в котором допускались побе- дители районных математических олимпиад. Мы не будем здесь подробно останавливаться на вопросах, связанных с работой оргкомитета олимпиады. Скажем только, что проведение олимпиады, выпуск сборника подготовительных задач, составление большого списка новых задач и отбор вари- антов задач олимпиады, а также проверка работ школьников и обсуждение различных организационных вопросов — колоссаль- ная работа, о которой большинство школьников и не подозре- вает и которая не могла бы осуществляться без энтузиазма сту- дентов и аспирантов, без помощи Московского математического общества и других организаций. В период проведения олимпи- ады рабочий день членов оргкомитета не нормирован: нередко он длится с раннего утра до поздней ночи. Работы проверяются и перепроверяются по нескольку раз, и немало школьников обя- заны своими премиями внимательному отношению членов орг- комитета, не поленившихся еще и еще раз вчитаться в их работы и найти в них рациональное зерно. Что касается самих задач олимпиады, то, помимо требова- ний красоты, яркости, идейной содержательности и новизны, предъявляемых к каждой отдельной задаче, весьма серьезные требования предъявляются ко всему ансамблю задач в целом. Задачи должны быть разнообразны как по форме, так и по ле- жащим в основе их решения идеям, однако эти решения не долж- ны выходить за рамки существующей школьной программы. Как правило, среди пяти-шести задач две задачи более простые; алгебраические и текстовые задачи перемежаются с геометри- ческими, и сложность задач обычно возрастает с увеличением номера задачи в варианте. Естественно, что из наиболее удач- ных задач, отобранных для олимпиады, более легкие предлага- лись на 1-й тур или в младшие классы, а более сложные остав- лялись для заключительного тура. Отметим, что все предлагающиеся на олимпиаде задачи не- стандартны. Их новизна и привлекательность в значительной степени объясняются тем, что они одухотворены свежими идея- ми современной математики и каждая из них — это маленькая научная работа, открывающая решающему задачу новые гори- зонты. Многие задачи связаны с различными разделами мате- матики, физики, механики. Приведем несколько примеров таких задач. Задача 9.14 появилась из кристаллографии и связана с ростом кристаллов. Задача 9.15 относится к т е о- рии конечных проективных плоскостей. За- дача 13.06 взята из знаменитого труда создателя неевклидо- вой геометрии Н. И. Лобачевского <Воображаемая геометрия». Задача 15.32 связана с задачей Лагранжа в небес- ной механике. С понятием «притягивающей» и «отталкиваю- 12
щей» точек в методе итераций связана задача 20.37 (все уравнения этой задачи имеют один и тот же вид: они, как говорят, «итерируют» первое уравнение). В решениях задач 21.19 и 31.24 используется понятие мировой линии в пространстве-времени. 26.31 — задача о диа- граммах Юнга, применяющихся в теории представлений симметрических групп. 29.13 и 29.18 — типичные задачи теории информации. Задача 30.31 — «преобразова- ние пекаря» из эргодической теории. 31.13 — пер- вая задача теории кодирования («проверка на четность» Хэмминга). Вопрос, поставленный в задаче 47.19, относится к одному из способов плотной упаковки ин- формации в памяти ЭВМ, а задачи 45.01, 45.17 и 48.18 косвенно связаны с теорией алгоритмов и вы- числений. Задача 48.15 взята из записных книжек вы- дающегося математика Леонарда Эйлера. Подобные примеры можно было бы продолжить (например, указать ряд задач по теории чисел); к сожалению, в большинстве случаев, оставаясь на «школьном уровне», невоз- можно объяснить суть дела более подробно. Подготавливая решения, мы, где это возможно, стремились в примечаниях к задачам осветить связь с современной мате- матикой. Мы старались также указать идейные взаимосвязи между задачами разных олимпиад или одной и той же олимпи- ады (так, например, вы легко заметите, что задача 40.23 явля- ется развитием задачи 40.01). Кроме того, за последние 50 лет школьные программы не раз менялись и новые веяния в програм- мах тут же отражались на содержании задач Московских олим- пиад: в соответствующие годы предлагались задачи на комп- лексные числа, задачи с производной и т. п. Со- ставленные нами решения и указания ориентированы на нынеш- нюю школьную программу, хотя стоит сказать, что в ряде слу- чаев от участников олимпиады предполагалось иное, зачастую более громоздкое решение. Так, в задаче 15.32 мы воспользо- вались известными нынешним школьникам свойствами интегра- ла и привели решение, занимающее несколько строк (в отли- чие от более длинного решения этой задачи в книге [25]). При всей огромной помощи энтузиастов составление и отбор задач олимпиады представляет одну из самых трудных частей работы оргкомитета. Этому посвящается целый ряд многоча- совых заседаний, на которых члены оргкомитета спорят до хри- поты, отстаивая одни задачи и отвергая другие. Бывает, что за- дачи на глазах меняются до неузнаваемости: иногда несколь- ко, казалось бы, совершенно разных идей формируются в одну задачу, иногда же, напротив, одна задача распадается на две- три другие, быть может, даже из разных разделов математики. При отборе задач на олимпиаду их приходилось, с одной стороны, «засекречивать», а с другой стороны, пытаться разны- 13
ми способами выяснять, не давали ли руководители кружков (никогда нс являвшиеся членами оргкомитета олимпиады) та- кой же или похожей задачи своим питомцам. Выяснение этого обстоятельства всегда было тонким делом: неизвестная ранее задача могла случайно «рассекретиться», широко распростра- нившись среди студентов. Поэтому отбор задач представлял всегда большие трудности. Список олимпиадных задач нередко оказывался утвержден- ным за день-два до тура, а бывало, что этот список печатался в ночь перед открытием олимпиады. Составление окончательных вариантов буквально накануне соответствующих туров позволя- ло сохранить задачи в секрете, а также воспользоваться всеми счастливыми находками, сделанными зачастую в последний мо- мент. Трудность задач на олимпиадах в разные годы заметно ко- лебалась. Самые трудные задачи первых олимпиад, которые решались отдельными их участниками, ныне стоят на уровне рядовых задач. Внимательный читатель заметит, что сам стиль задач поздних олимпиад существенно изменился по сравнению со стилем задач первых олимпиад. Однако сложность олимпиады в каждый конкретный год всегда была очень высокой. Иногда даже при всем желании облегчить варианты задач это не уда- валось сделать. Бывали и совсем трудные олимпиады. Таковы XXVII (1964 г.), XXIX (1966 г.), XXXI (1968 г.), XXXV (1972 г.); на этих олимпиадах некоторые задачи не решил никто (в ка- честве примеров можно привести задачи 29.15, 31.26, 31.27, 35.32). а некоторые — только один участник (например, зада- чу 35.30). Юбилейную XLVIII олимпиаду (1985 г.) нельзя от- нести к разряду очень трудных, но и на ней были задачи, кото- рые не решил ни один школьник в своем классе (задачи 48.04, 48.10, 48.15). Но это скорее исключение, чем правило; на большинстве олимпиад каждая задача были решена хотя бы одним участником и были довольно сложные задачи, которые решались многими участниками олимпиады. Самое сильное влияние на дух и характер олимпиады, содер- жание и трудность ее задач оказывали профессора мехмата МГУ, возглавлявшие в разные годы оргкомитет Московской олим- пиады и направлявшиеся на эту работу правлением Московского математического общества. Председателями оргкомитетов в раз- ные годы являлись: академики АН СССР П. С. Александ- р о в (1935 г.), А. Н.'К олмогоров (1937 г., 1963 г., 1975 г.), И. Г. Петровский (1947 г.), Л. С. П о н т р я г и н (1940 г.), И. М. Г е л ь ф а н д (1945 г.); члены-корреспонденты АН СССР Б. Н.*Делоне (1951 г.), Л. А. Люстерник (1939 г.), Д. Е. Меньшов (1953 г.), А. О. Гельфонд (1941 г.), И. Р. Шафаревич (1960 г., 1964 г., 1971 г.), Н. В. Ефи- мов (1962 г., 1965 г.), Н. С. Бахвалов (1968 г.), В. И. А р- нольд (1974 г.), О. Б. Лупанов (1981 —1986 гг.); дей- 14
ствительный член АПН СССР А. И. Маркушевич (1949 г.); член-корреспондент АПН СССР В. Г. Болтянский (1958 г.); профессора, доктора физико-математических наук Н. А. Гла- голев (1936 г.), А. Г. К у р о ш (1938 г.), С. А. Г а л ь п е р н (1946 г.), В. В. Немыцкий (1948 г., 1967 г.), М. А. К р е fl- не с (1950 г.), П. К. Рашевский (1952 г.), С. В. Б а х в а- лов (1954 г.), Г. Е. Шилов (1955 г.), О. А. Олейник (1957 г.), Е. М. Ландис (1959 г.), В. А. Ефремович (1961 г., 1969 г.), А. А. Кронрод (1966 г.), В. М. Алексе- ев (1970 г.), Б. П. Демидович (1972 г.), А. А. Кирил- лов (1973 г.), А. В. А р х а н ге л ь с к и й (1976 г.), В. А. У с- пенский (1977 г.), Ю. И. Манин (1978 г.), В. М. Ти- хомиров (1979 г.), А. С. Мищенко (1980 г.). Предсе- датели оргкомитетов не только проводили большую организа- ционную работу, но и руководили многочисленными заседа- ниями оргкомитетов. В обязанности каждого председателя вме- нялось также проведение разбора задач в одном из классов. Председатель имел основных помощников: заместите- ля (з а м п р е д а; до 60-х годов он назывался ответствен- ным секретарем) и старших по классам. У зам- преда была масса обязанностей: договориться о проведении лек- ций для школьников; дать указания о расклейке афиш об олим- пиаде; организовать проведение самой олимпиады; напечатать подготовительный сборник задач и бланки грамот для награжде- ния победителей; распределить аудитории, в которых будет про- ходить олимпиада, а позже участвовать в разборе задач и на- граждении победителей. Кроме того, зампред обязан принимать участие в обсуждении и составлении задач. Один из авторов этой книги, будучи зампредом, испытал на себе всю сложность его функций. Старшие по классам также очень сильно загружены: кроме помощи зампреду в организационных вопросах, они должны от- бирать наиболее понравившиеся им задачи для своего класса, из которых оргкомитет составляет потом окончательный вариант задач по этому классу; во время олимпиады следить за ее про- ведением (по своему классу), обходить аудитории и отвечать на вопросы участников (в заранее предусмотренном оргкоми- тетом объеме); организовывать проверку работ участников сво- его класса; выявлять лучшие работы, а все свои мнения выно- сить на регулярно проводившиеся заседания оргкомитета. Во время проведения самой олимпиады ее обслуживают мно- гочисленные дежурные-студенты, которые помогают школьникам разойтись по аудиториям, сидят в аудиториях, следят за тем, чтобы коридоры и другие помещения не превращались в дискус- сионные клубы участников олимпиады, а к концу тура собирают работы участников. Затем старшие по классам распределяют собранные работы между студентами для проверки. Проверка составляет самую ответственную часть работы; ка- 15
ждую сколько-нибудь интересную работу читают несколько чле- нов оргкомитета, а работы, выдвигаемые на премию, рассматри- вают все члены оргкомитета. Разнообразие требований, предъ- являемых к работам участников, делает оценку каждого отдель- ного члена оргкомитета довольно субъективной, и лишь коллек- тивный характер процедуры присуждения премий обеспечивает правильность и объективность окончательного решения. Для оценки работы применяется не балльная система, а бо- лее гибкая система плюсов и минусов. Основные оценки, кото- рые могут быть выставлены за задачу, таковы: (+) —задача полностью решена; ( + ) — задача решена, но решение содержит мелкие пропуски или огрехи; (±) — задача решена не полностью; (ч=)—задача не решена, но подход к решению правилен; ( —) — задача не решена, или решена неправильно; (-г-) или (е) — задача не решена, но в черновиках или чистовой работе обнаружены некоторые разумные соображения; (—?) —ре- шение неверно и содержит очень грубые ошибки. Употребля- ется еще знак (!) (например, (-Н) или (Ян!)), который озна- чает, что в решении, полном или неполном, имеются яркие и нео- жиданные (иногда непредвиденные оргкомитетом) идеи. Напри- мер, на IX олимпиаде первую премию получил ученик X класса Эрик Балаш. Все отведенное на олимпиаде время он затратил на решение задачи 9.17, которая предполагалась оргкомитетом сравнительно несложной. Однако вместо ответа на конкретный вопрос, поставленный в задаче, Эрик провел подробное иссле- дование, указав закон повторяемости тех членов последователь- ности Фибоначчи, которые оканчиваются четырьмя нулями, и правильно определил номер первого такого члена (равный 7501). Решение было удостоено оценки (±!) (Эрик не успел на олим- пиаде довести свое исследование до конца), и, несмотря на то что к решению остальных задач он и не приступал, ему была при- суждена первая премия. —о--------- Над основным материалом нашей книги — задачами — тру- дилось несколько поколений замечательных математиков, и боль- шинство задач несет на себе отпечаток незаурядности и кра- соты. Установление авторства большинства задач Московских олимпиад (а их около 1300) потребовало бы в данный момент со- вершенно непосильной исследовательской работы. Составить да- же приблизительный список авторов задач невозможно хотя бы потому, что часть задач возникала уже «на месте», в оргкоми- тете, и автор у них коллективный. Однако наиболее красивые (по признанию оргкомитета) и оригинальные задачи олимпиады придуманы вполне конкретны- ми авторами. Такие задачи получают сначала в оргкомитете (а после окончания олимпиады и в других кругах) широкое рас- 16
пространение, превращаясь в математический фольклор; «зна- токи» сразу вспоминают их по кратким специфическим назва- ниям. Приведем фамилии нескольких авторов подобных задач: Н. Н. Константинов — 23.27 («Улитка» ); Н. Б. Васильев — 25.39 («Коробочка»), 26.35 («Джентльмены»); А. С. Шварц — 24.32 («Матрица Шварца»); Б. Д. Гинзбург— 23.31 («Обход конем»); А. Г. Кушниренко — 33.36 («Апельсин»); А. П. Савин — 33.11 («Крайние короли»); А. Л. Тоом — 35.12 («Лес»); Д. Б. Фукс — 24.07 («Игра в скалярное произведение»), 26.50 и 31.31 («Фук- совы дуги»); Г. А. Гальперин — 34.20, 38.23, 39.04 («Прожекто- ры»), 33.25 («Придворные короля Люловика»), 38.10 («Двойки, тройки и четверки»), 39.20 («Пятаки»), 48.12 («Аэродромы»); А. К- Толпыго— 31.13 («Телефоны»), 35.37 («Игра в карточки»); О. В. Ляшко — 35.23 («Странствующий рыцарь»); А. В. Зеле- винский — 34.33 («Банкир и Игрок»); М. Р. Ковтун — 34.15 («Спички»); А. В. Климов — 37.24 («Полоски»), 37.27 («Асте- роиды»); А. И. Грюнталь — 36.14 («Многогранник»); С. А. Ели- сеев— 38.19 («Невыпуклое разрезание»); С. В. Конягин — 42.09, 42.14 («Химики и алхимики»); С. Б. Гашков — 45.17 («Машин- ка Тьюринга»), 48.04 («Волки и заяц»); И. Н. Сергеев — 48.09 («Дядька Черномор»). Этот список было бы легко продолжить, но почти невозмож- но завершить, и мы приносим свои извинения всем неупомяну- тым авторам задач Московских олимпиад. Считаем также своим долгом особо отметить многолетнюю работу Н. Н. Константино- ва по проведению Московских олимпиад и других математиче- ских соревнований школьников, душой которых он неизменно являлся. Академик АН СССР А. Н. Колмогоров, активно участво- вавший в проведении Московских математических олимпиад на протяжении всего их существования, много сделал и для нашей книги в качестве ее редактора и научною консультанта; мы поль- зуемся случаем выразить ему нашу самую горячую признатель- ность. Мы признательны также всем тем, кто помогал нам в ра- боте над сборником и в подготовке его публикации, в первую очередь В. М. Тихомирову, Н. Б. Васильеву, А. М. Абрамову, а также А. П. Савину, С. М. Саакяну, А. Л. Тоому, Е. А. Моро- зовой, Р. С. Черкасову. Мы надеемся, что еще много лет в аудиториях МГУ будут шуметь звонкие голоса школьников и будут раздаваться во- просы: — Когда следующая олимпиада? Г. А. Гальперин А. К. Толпыго
Указания к работе с книгой Наша книга состоит из двух частей. В первой части «Зада- чи Московских математических олимпиад» приведены тексты задач всех Московских олимпиад с 1935 по 1985 г. включитель- но в том виде, как они предлагались на олимпиадах. Условия почти всех задач первых 27 олимпиад взяты из книги [21]; задачи олимпиад начиная с 1965 г. были собраны авторами настоящей книги. Следует, однако, отметить ряд пропусков в списке задач, приводимом в [21]. Так, в результате работы с литературой и архивами нам удалось обнаружить отсутствовавшие в [21] за- дачи 2.10; 10.04; 10.05; 10.10; 10.11; 11.11; 11.12; 11.17; 12.17; 15.13; 15.18; 17.25. Кроме того, был найден полный вариант XIV олимпиады (см. [37]), мало совпадающий, как выяснилось, с опубликованным в [21]. XIV олимпиада полностью реконструи- рована. Отыскались также задачи 1-го тура II олимпиады и комплект задач для VII класса, предлагавшийся на 2-м туре XIX олимпиады. Однако по-прежнему не обнаружен 1-й тур IV олимпиады. Вторая часть нашей книги содержит ответы, решения и ука- зания к большинству задач. Большинство задач снабжено указаниями или ответами, и лишь некоторые наиболее интересные задачи — подробными ре- шениями. Задачи во второй части книги имеют двойную нумера- цию: первое число означает номер олимпиады, второе — поряд- ковый номер задачи в этой олимпиаде. Например, 12.04 озна- чает: XII олимпиада, задача 4. Тех же обозначений мы при- держиваемся и при ссылках на ту или иную задачу. В заключение приведем некоторый обозначения, часто повто- ряющиеся в условиях и решениях различных задач, и дадим к ним пояснения. 1) [х] —«целая часть числа х» — наибольшее целое число, не превосходящее х (примеры: [5]=5, [1,5]= 1, Г4-1 = 0, [л]=3, [ — 1,5]= —2 и т. д.). L4J 2) n! = 1 • 2-3*4«...-(п— 1)-п (читается: «п — факториал»); 0! = 1!= 1; (2п)!!=2’4-6-...-(2п — 2)-2п (произведение всех четных чисел от 2 до 2п); (2п— 1)!! = 1 -3-5-...-(2п — 3) (2n— 1) (произве- дение всех нечетных чисел от 1 до 2м —1). 3) = — число сочетаний из п по /г (равному числу способов выбрать k элементов из м). 4) ab...c — натуральное число, записанное цифрами n, Ь, ..., с. 5) Последовательность Фибоначчи — это последовательность (хл), в которой хп = Хг;~\ 4-хл-2. Чаще всего это последователь- ность 0,1,1,2,3,5,8,13,21,34, ... . 18
6) Во всех задачах о турнирах предполагается, если не ого- ворено другое условие, что каждый играет с каждым по одному разу (турнир в один круг). В шахматном турнире победа дает 1 очко, ничья--поражение — 0; в футболе все очки вдвое больше. В баскетболе и теннисе ничьих не бывает. 7) Многоугольник называется вписанным в другой многоуголь- ник, если его вершины лежат на сторонах (в том числе и в вер- шинах) этого многоугольника. 8) Во всех задачах о луче света, бильярдном шаре и т. д. предполагается, что выполнен закон: угол падения равен углу отражения. 9) Все многоугольники, если нет оговорки, предполагаются несамопересекающимися и выпуклыми. 10) Длиной конечной последовательности называется число ее членов. —о— Предлагаемая книга может быть полезна педагогам — руко- водителям факультативов и математических кружков; вероятно, она заинтересует студентов и научных работников. Но в первую очередь книга предназначается для школьников старших классов, любящих математику. Здесь есть задачи довольно простые (это, как правило, задачи первых туров), есть и трудные. Особенно трудные задачи отмечены звездочкой. Не спешите преждевременно заглядывать во вторую часть книги. Не жалейте потратить несколько часов или даже дней на одну задачу, и в случае, если упорные раздумья не при- ведут вас к цели, заглянит^ в ответ или решение. Если вы сами справились с решением задачи, просмотрите на всякий случай приводимое в книге решение или указание: вдруг в нем вы обна- ружите что-то ценное для себя. Итак, попытайтесь помериться силами с участниками про- шедших Московских олимпиад! Желаем вам успехов! Авторы.
ЧАСТЬ I ЗАДАЧИ МОСКОВСКИХ МАТЕМАТИЧЕСКИХ ОЛИМПИАД I ОЛИМПИАДА (1935 г.) 1-м т у р 1. Поезд проходит мимо наблюдателя в течение t\ с, а мимо моста длиной / м в течение 6 с. Считается, что поезд проходит мимо моста начиная с того момента, когда локомотив въезжает на мост, и кончая моментом, когда последний вагон покидает мост. Определить длину и скорость поезда. 2. Построить квадрат, три вершины которого лежат на трех данных параллельных прямых. 3. Найти объем правильной четырехугольной пирамиды со сторонами основания а и плоскими углами при вершине, рав- ными углам наклона боковых ребер к плоскости основания. 2-й тур. Серия А 4. Построить треугольник по точкам пересечения с описан- ной окружностью продолжений медианы, биссектрисы и высоты, проведенных из одной вершины. 5. На поверхности куба найти точки, из которых диагональ видна под наименьшим углом. 2-й тур. Серия В 6. Сколько действительных решений имеет система X + у = 2, ху — Z2— 1? 7. Решить систему f х3 —1/3 = 26, I х2у — ху2 = Ь. 8. Вычислить сумму 13 + 33 + 53 + ...+(2п — I)3. 2-й тур. Серия С 9. Сколькими способами можно раскрасить куб в шесть дан- ных цветов, если любые две различные грани должны быть раскрашены разными красками? (Различными считаются те рас- краски, которые не совмещаются при поворотах куба.) 20
10. Сколькими способами можно представить число п в ви- де суммы трех целых положительных слагаемых? 11. Обозначим через М (а, Ь, с, k) наименьшее общее кратное, а через D (а, Ь, с, .... k) — наибольший общий дели- тель чисел а, Ь, с, .... k. Доказать: а) М (a, b)-D (а, Ь) = аЬ\ б) ^^-D(a,byD{b,c).D{a,c)=abc. II ОЛИМПИАДА (1936 г.) 1-й т у р 1. Найти четырехзначное число, являющееся полным квад- ратом, первая цифра которого равна второй, а третья — чет- вертой. 2. Построить такой равнобедренный треугольник, чтобы у любого вписанного в него прямоугольника, две вершины кото- рого лежат на основании треугольника, периметр был величи- ной постоянной. 3. Представить произвольное натуральное число в виде вы- ражения, в запись которого входят только 3 двойки и произ- вольные математические знаки. (Задача П. Дирака.) 4. Дан круг и точка Р вне его. С помощью циркуля и ли- нейки построить диаметр круга, который виден из точки Р под данным углом. 5. Найти 4 последовательных натуральных числа, произве- дение которых равно 1680. 2-й тур 6. Решить систему J х + «/=а, Ь5+у5=*5. 7. Дан угол, меньший 180°, и точка М вне его. Провести через точку М прямую так, чтобы треугольник, одна из вер- шин которого совпадает с вершиной данного угла, а две другие есть точки пересечения его сторон с проведенной прямой, имел данный периметр. 8. Доказать, что если стороны прямоугольника и его диа- гональ — целые числа, то его площадь — целое число, крат- ное 12. 9. Сколькими способами можно разложить миллион в про- изведение трех множителей? Разложения, отличающиеся поряд- ком множителей, считаются различными. 10. В пространстве расположены три плоскости и шар. Сколь- кими различными способами можно поместить в пространстве 21
второй шар так, чтобы он касался трех данных плоскостей и первого шара? Ill ОЛИМПИАДА (1937 г.) 1-й тур 1. Решить систему x2 + y2 + z2 = a2, х3 + у3 + ^ = ^. 2*. Даны прямая и две точки Л и В по одну сторону от нее. Найти на прямой такую точку М, чтобы сумма МА-\-МВ равнялась заданному отрезку. 3. По двум скрещивающимся прямым скользят два отрезка. Доказать, что объем тетраэдра с вершинами в концах этих отрезков не зависит от положения последних. 2-й тур 4. Даны три точки, не лежащие на одной прямой. Через каж- дые две из них провести окружности так, чтобы три проведен- ные окружности имели в точках пересечения взаимно перпен- дикулярные касательные. 5*. В пространстве расположен правильный додекаэдр. Сколь- кими способами можно провести плоскость так, чтобы она вы- секла на додекаэдре правильный шестиугольник? 6. На сколько частей разделяют n-угольник его диагонали, если никакие три диагонали не пересекаются в одной точке? IV ОЛИМПИАДА (1938 г.) 2-й т у р I. В пространстве даны точки Oi, Ог, О3 и точка А. Точка А симметрично отражается относительно точки О|, полученная точка Ai —относительно Ог, полученная точка А? — относитель- но Оз. Получаем некоторую точку Л3, которую также после- довательно отражаем относительно Oi, О2, О3. Доказать, что последняя полученная точка совпадает с А. 2. На сколько частей могут разделить пространство п пло- скостей? 3. Построить треугольник по основанию, высоте и разности углов при основании. 4. Сколько существует натуральных чисел, меньших тысячи, которые не делятся ни на 5, ни на 7? 22
V ОЛИМПИАДА |1939 г.) 1-й т у р 1. Решить систему уравнений {Зхуз — х3—у3 — z3 — Ь3, x+y + z = 2b, х2 -|-у2 — z2 = ft2. 2. Доказать, что cos 5р,~Ь cos • 3. Даны три точки А, В, С. Через точку А' провести прямую так, чтобы сумма расстояний от точек В и С до этой прямой была равна заданному отрезку. 4. Решить уравнение ~^а — ^а+х=х. 5. Доказать, что во всяком неравнобедренном треугольни- ке биссектриса лежит между медианой и высотой, проведенными из той же вершины. 2-й тур 6. Разложить на целые рациональные множители выражение а,0 + о5-|-1. 7. Даны два многочлена от переменной х с целыми коэффи- циентами. Произведение их есть многочлен с четными коэффици- ентами, не все из которых делятся на 4. Доказать, что в одном из многочленов все коэффициенты четные, а в другом — хотя бы один нечетный. 8. Даны две точки Л и В и окружность. Найти на окруж- ности точку X так, чтобы прямые АХ и ВХ отсекли на окружно- сти хорду CD, параллельную данной прямой MN. 9. Найти остаток от деления на 7 числа 10'°+10'°’+ 1О'°3-|-... 4- ю10'0. 10. Дана правильная пирамида. Из произвольной точки Р ее основания восставлен перпендикуляр к плоскости основа* ния. Доказать, что сумма отрезков от точки Р до точек пере- сечения перпендикуляра с плоскостями граней пирамиды не за- висит от выбора точки Р на основании. 11. На какое самое большое число частей можно разбить пространство пятью сферами? VI ОЛИМПИАДА (1940 г.) 1-й тур VII—VIII классы 1. Разложить на множители: (fe — с)3 + (с — а)3 + (а — 6)3. 2. Пароход от Горького до Астрахани идет 5 суток, а от 23
Астрахани до Горького — 7 суток. Сколько дней будут плыть по течению плоты от Горького до Астрахани? 3. Сколькими нулями оканчивается произведение всех целых чисел от 1 до 100 включительно? 4. Провести окружность данного радиуса, касающуюся дан- ной прямой и данной окружности. Сколько решений имеет за- дача? IX—X классы 5. Решить систему уравнений | (х34-у3)(х24-«/2)=2&5, I х+у = Ь. 6. Все целые числа, начиная с единицы, выписаны подряд. Таким образом, получается следующий ряд цифр: 123456789101112131415... . Определить, какая цифра стоит на 206788-м месте. 7. Построить окружность, равноудаленную от четырех точек плоскости. Сколько решений имеет задача? 8. В плоскости даны две прямые. Найти множество всех точек, разность расстояний которых от этих прямых равна за- данному отрезку. 9. Найти все трехзначные числа, равные сумме факториалов своих цифр. 2-й т у р VII — VIII классы 10. Найти четырехзначное число, являющееся точным квад- ратом, если первые две цифры, а также последние две цифры его одинаковы. 11. Точки А, В, С—вершины вписанного в окружность правильного треугольника. Точка D лежит на меньшей дуге АВ. Доказать, что AD-\-BD = DC. 12. Данным четырехугольником неправильной формы наст- лать паркет, т. е. покрыть всю плоскость четырехугольниками, равными данному, без пропусков и перекрытий. 13. Сколько существует таких пар целых чисел х, */, за- ключенных между 1 и 1000, что х2 + </2 делится на 49? IX — X классы 14. На бесконечном конусе, угол развертки которого равен а, взята точка. Из этой точки в обе стороны проводится ли- ния так, что после развертки она превращается в отрезки пря- мых. Найти число ее самопересечений. 24
15. Что больше: 300! или 1ОО300? 16. Центр О описанной около треугольника АВС окружно- сти отражается симметрично относительно каждой из прямых, содержащих сторону ДЛВС. По трем полученным точкам Оь Ог> Оз восстановить треугольник АВС, если все остальное стерто. 17. Доказать неравенство Д| а О2 । О.| । । Ом - I । Ол ------- г-Н - «2 аз Щ-----------------ап Oi (oi, az, ...» ап-1, ап — положительные числа). 18. Сколько существует положительных целых чисел х, меньших 10 000, для которых 2х — х2 делится на 7? VII ОЛИМПИАДА |1941 г.) 1-й тур VII — VIII классы 1. Построить треугольник, если известны его высота и ме- диана, выходящие из одной вершины, и радиус описанной около треугольника окружности. 2. Дописать к 523 ... три цифры так, чтобы полученное шести- значное число делилось на 7, 8 и 9. 3. Дан четырехугольник; А, В, С, D — последовательные се- редины его сторон, Р, Q — середины диагоналей. Доказать, что треугольник ВСР равен треугольнику ADQ. 4. Через точку Р, лежащую вне окружности, проводятся все- возможные прямые, пересекающие эту окружность. Найти мно- жество середин хорд, отсекаемых данной окружностью на этих прямых. 5. Доказать, что произведение четырех последовательных целых чисел в сумме с единицей дает полный квадрат. IX — X классы 6. См. задачу 2. 7. На сторонах параллелограмма вне его построены квадра- ты. Доказать, что их центры являются вершинами некоторого квадрата. 8. Доказать, что многочлен с целыми коэффициентами аохп -|- atxn~1 -|-... + an-ix-f-an, принимающий при х=0 и х= 1 нечетные значения, не имеет целых корней. 9. Построить треугольник АВС по точкам М и N — осно- ваниям высот AM и BN, и прямой, на которой лежит сторо- на АВ. 25
10. Решить уравнение |х-|-1| —|х| 4-3|х—1|—2|х—2|=х-|-2. II. Сколько корней имеет уравнение sinx=-j^? 2-й т у р VII—VIII классы 12. Доказать, что из пяти различных по величине квадратов нельзя сложить прямоугольник. 13. Дан треугольник АВС. Требуется разрезать его на наи- меньшее число частей так, чтобы, перевернув эти части на дру- гую сторону, из них можно было сложить тот же треуголь- ник АВС. 14. Дан треугольник АВС. Точка М, лежащая внутри него, движется параллельно стороне ВС до пересечения со стороной СЛ, затем параллельно стороне АВ до пересечения со стороной ВС, затем параллельно стороне СА и т. д. Доказать, что через некоторое число таких шагов точка вернется в исходное поло- жение, и найти это число. 15. Найти целое число а, при котором (х — а) (х— 10)+ I раз- лагается в произведение (х + &)(х + с) с целыми b н с. 16. Доказать, что квадрат любого простого числа р>3 при делении на 12 дает в остатке 1. 17. Построить треугольник АВС по трем точкам Я|, Яг, Я3, которые являются симметричными отражениями точки пересе- чения высот искомого треугольника относительно его сторон. IX—X классы 18. Доказать, что из шести попарно различных по величине квадратов нельзя сложить прямоугольник. 19. Некоторое количество точек расположено на плоскости так, что каждые 3 из них можно заключить в круг радиуса r= 1. Доказать, что тогда и все точки можно заключить в круг радиу- са 1. 20. Найти такие отличные от нуля неравные между собой целые числа а, &, с, чтобы выражение х (х — а) (х—Ь) (х — с)+ 1 представлялось в виде произведения двух многочленов с целы- ми коэффициентами. 21. Решить в целых числах уравнение х + «/ = х2 — ху + у2. 22. В пространстве даны две скрещивающиеся перпендику- лярные прямые. Найти множество середин всех отрезков данной длины, концы которых лежат на этих прямых. 23. Построить прямоугольный треугольник по двум медиа- нам, проведенным к катетам. 26
VIII ОЛИМПИАДА (1945 г.) 1-й тур VII—VIII классы 1. См. задачу 6 для k=7. 2. Доказать, что при любом целом положительном п спра- ведливо неравенство -^-+7^2 +•• Двузначное число в сумме с числом, записанным теми же цифрами, но в обратном порядке, дает полный квадрат. Найти все такие числа. 4. Доказать, что разносторонний треугольник нельзя разре- зать на два равных треугольника. 5. К двум окружностям, касающимся извне, проведены об- щие внешние касательные, и точки касания соединены между собой. Доказать, что в полученном четырехугольнике суммы противоположных сторон равны. IX—X классы 6. Разделить а2к — Ь2* на (a4-fe)(a2 + &2)(a4 + fe4)-...-(a2*_,+ + &2* '). 7. Найти трехзначное число, всякая натуральная степень которого оканчивается на три цифры, составляющие исходное число (в том же порядке). 8. Система уравнений второго порядка { х2 —</2 = 0, I (х — а)2+у2=\ имеет, вообще говоря, четыре решения. При каких значениях а число решений системы уменьшается до трех или до двух? 9. Прямоугольный треугольник АВС с катетами а и b дви- жется по плоскости так, что вершины А и В его острых углов скользят по сторонам данного прямого угла. Доказать, что мно- жеством точек С является отрезок, и найти его длину. 2-й т у р VII—VIII классы 10. Даны шесть цифр: 0, 1, 2, 3, 4, 5. Найти сумму всех четырехзначных четных чисел, которые можно написать этими цифрами (одна и та же цифра может повторяться). 11. Из картона вырезали два равных многоугольника, сов- местили их и проткнули в некоторой точке булавкой. При пово- роте одного из многоугольников около этой «оси» на 25°30' он снова совместился со вторым многоугольником. Каково наимень- шее возможное число сторон у таких многоугольников? 12. Сторона AD параллелограмма ABCD разделена на п рав- ных частей. Первая точка деления Р соединена с вершиной В. 27
Доказать, что прямая ВР пересекает диагональ АС в точке Q такой, что = АС. 13. Вершины А, В, С треугольника АВС соединены отрез- ками соответственно с точками Ai, Bt, С|, лежащими на про- тивоположных сторонах треугольника. Доказать, что середины отрезков A At, BBi, CCt не лежат на одной прямой. IX—X классы 14. Решить в целых числах уравнение ху-|-Зх—5у= —3. • 15. Некоторые из чисел ai, аг, .... ап равны 4-1, остальные равны — 1. Доказать, что 0102 . О|О2О1 . , О|О2 ... Од “2 1 4 -I 2„_| = а Гу 2 -|- a?-\j 2 4- аз^2 4- • •• + . В частности, при а} =а2 =... = ап = I имеем: 2 sin(1 +1+4-+ - 4~)f=2 cos1i7=V2+^ + - +& 16. Окружность радиуса, равного высоте некоторого правиль- ного треугольника, катится по стороне этого треугольника. До- казать, что дуга, высекаемая сторонами треугольника на ок- ружности, все время равна 60°. 2 sinf ai 4 IX ОЛИМПИАДА (1946 г.) 1-й тур VII—VIII классы 1. Какое наибольшее число острых углов может встретиться в выпуклом многоугольнике? 2. На прямой даны три точки A, Bt С. На отрезке АВ построен равносторонний треугольник ABCit на отрезке ВС по- строен равносторонний треугольник BCAi. Точка М — середи- на отрезка АА\. точка N — середина отрезка СС|. Доказать, что треугольник BMN равносторонний. (Точка В лежит между точками А и С; точки At и С\ расположены по одну сторону от прямой АВ.) 3. Найти четырехзначное число, которое при делении на 131 дает в остатке 112, а при делении на 132 дает в остатке 98. 4. Решить систему уравнений {*1 4" *24"*3 = 6, *2 4“ *3 + *4 — 9, *3 4" *4 4" *5 = 3, *4 4-*5 4-*6= —3, *5 4" *6 4" *7 = —9, *6 4" *7 4" *8 = —6, *74"*в4"*1 = —2, *в + *1 4" *2 = 2. 28
5. Доказать» что в произведении (1 —х-|-х2 —х3 +...—х"+х,0°) (1 4-%-|-х24-...4-х99+х100) после раскрытия скобок и приведения подобных членов не оста- нется членов, содержащих х в нечетной степени. IX—X классы 6. В пространстве даны две пересекающиеся плоскости а и ₽. На линии их пересечения дана точка Л. Доказать, что из всех прямых, лежащих в плоскости а и проходящих через точку Л, наибольший угол с плоскостью р образует та, которая перпен- дикулярна к линии пересечения плоскостей аир. 7. Через точку Л, лежащую внутри угла, проведена пря- мая, отсекающая от этого угла наименьший по площади тре- угольник. Доказать, что отрезок этой прямой, заключенный между сторонами угла, делится в точке А пополам. 8- Доказать, что д2+Зл4-5 ни при каком натуральном п не делится на 121. 9. Доказать, что для любого натурального п справедливо соотношение 10. Доказать, ^=2"-(2л-1)!! что если аир — острые углы и а < р, то а 0 2-й т у р VII—VIII классы 11. В шахматном турнире участвовали два ученика VII клас- са и некоторое число учеников VIII класса. Два семиклассника набрали вместе восемь очков, а все восьмиклассники набрали по одинаковому числу очков. Сколько восьмиклассников участвова- ло в турнире? 12. Доказать, что выражение Xs + Зх4у — 5х3у2 — 1 5х2у3 4- Аху ’ 4- 12у5 не равно 33 ни при каких целых значениях х и у. 13. На сторонах угла АОВ от вершины О отложены отрезки ОА и ОВ,. причем ОА > ОВ. На отрезке О А взята точка М, на отрезке О В — точка Л/ так, что АМ = BN=x. Найти значение х, при котором отрезок MN имеет наименьшую длину. 14* . Из тридцати пунктов Д|, Дг, .... Дзо. расположенных на прямой MN на равных расстояниях друг от друга, выходят тридцать прямых дорог. Эти дороги располагаются по одну сторону от прямой MN и образуют с нею следующие углы: 29
№ 1 2 3 4 5 6 7 8 9 10 60° 30° 15° 20° 155° 45° 10° 35° 140° 50° № II 12 13 14 15 16 17 18 19 20 125° 65° 85° 86° 80° 75° 78° 115° 95° 25° № 21 22 23 24 25 26 27 28 29 30 28° 158° 30° 25° 5° 15° 160° 170° 20° 158° Из всех тридцати пунктов выезжают одновременно трид- цать автомобилей, едущих, никуда не сворачивая, по этим доро- гам с одинаковой скоростью. На каждом из перекрестков уста- новлено по шлагбауму. Как только первая (по времени) машина проезжает перекресток, шлагбаум закрывается и преграждает путь всем следующим машинам, попадающим на этот перекре- сток. Какие из машин проедут все перекрестки на своем пути, а какие застрянут? 15. Автобусная сеть города устроена следующим образом: 1) с любой остановки на любую другую остановку можно попасть без пересадки; 2) для любой пары маршрутов найдется, и притом единственная, остановка, на которой можно пересесть с одного из этих маршрутов на другой; 3) на каждом маршруте ровно три остановки. Сколько автобусных маршрутов в городе? IX—X классы 16. В шахматном турнире участвовали ученики IX и X клас- сов. Десятиклассников было в десять раз больше, чем девятиклассников, и они набрали вместе в 4,5 раза больше очков, чем все девятиклассники. Сколько очков набрали девяти- классники? 17. Дана последовательность Фибоначчи 0, 1, 1, 2, 3, 5, 8, ... . Найдется ли среди 100 000 001 ее первых членов число, оканчи- вающееся четырьмя нулями? 18. На сторонах PQ, Q/?, RP треугольника PQR отложены отрезки АВ, CD, EF. Внутри треугольника задана точка So. Найти множество точек S, для которых сумма площадей треуголь- ников SAB, SCD и SEF равна сумме площадей треугольников SqAB, SqCD, SqEF. Рассмотреть особо случай, когда —— QR~ RP' •9. В городе 57 автобусных маршрутов. Известно, что: 1) с любой остановки на любую другую остановку можно попасть без пересадки; 2)для любой пары маршрутов найдется, и притом зо
только одна, остановка, на которой можно пересесть с одного из маршрутов на другой; 3) на каждом маршруте не менее трех остановок. Сколько остановок имеет каждый из 57 марш- рутов? 20. См. задачу 14. X ОЛИМПИАДА (1947 г.| 1-й т у р VII-VIII классы 1. Найти остаток от деления многочлена х243-|-х8,-|-х274-х9+ +х11 * 3+х на х— 1. 2. Доказать, что из 9 последовательных натуральных чисел всегда можно выбрать одно число, взаимно простое с остальными. 3. Найти коэффициенты при х17 и х'8 после раскрытия скобок и приведения подобных членов в выражении (х74-х8+1)20. 4. Дан выпуклый пятиугольник ABCDE. Сторонами, противо- положными вершинам А, В, С, D, Е, назовем соответственно отрезки CD, DE, ЕА, АВ, ВС. Доказать, что если произволь- ную точку М, лежащую внутри пятиугольника, соединить прямыми со всеми его вершинами, то из этих прямых либо ровно одна, либо ровно три, либо ровно пять пересекут стороны пятиуголь- ника, противоположные вершинам, через которые они проходят. 5. Точка О является точкой пересечения высот остроуголь- ного треугольника АВС. Доказать, что три окружности, проходя- щие: первая через точки О, А, В, вторая — через точки О, В, С и третья — через точки О, С, А, равны между собой. IX—X классы 6. Найти коэффициент при х2 после раскрытия скобок и приведения подобных членов в выражении ((...(((х—2)2 —2)2— —2)2—... —2)2—2)2 (k скобок). 7. См. задачу 2 для 16 последовательных чисел. 8. Сколько различных по величине или по расположению квадратов, состоящих из целого числа клеток, можно начертить на шахматной доске в 64 клетки? 9. В каком из выражений (1 + х2—х3)1000 или (1 —х24-х3)1000 после раскрытия скобок и приведения подобных членов будет стоять больший коэффициент при х20? 10. Вычислить с точностью до 0,00001 произведение 11. Найти множество всех прямых в пространстве, проходящих через данную точку М на данном расстоянии d от данной прямой АВ. 31
1-й тур VII—VIII классы 12. Некоторые из 20 металлических кубиков, одинаковых по размерам и внешнему виду, алюминиевые, остальные — дюралевые (более тяжелые). Как при помощи не более чем 11 взвешиваний на чашечных весах без гирь определить число дюралевых кубиков? (Предполагается, что все кубики могут быть алюминиевыми, но не могут быть все дюралевыми.) 13. Сколько цифр в десятичной записи числа 2Г0°? 14. На плоскости даны пять точек, из которых никакие три не лежат на одной прямой. Доказать, что из этих точек можно выбрать четыре, являющиеся вершинами выпуклого четы- рехугольника. 15. Доказать, что никакой выпуклый 13-угольник нельзя разре- зать на параллелограммы. 16. Из чисел 1, 2, ... , 200 выбрано 101 число. Доказать, что среди выбранных чисел найдется пара таких, что одно из них делится на другое. IX—X классы 17. п проволочных треугольников расположены в простран- стве так, что каждые два из них имеют общую вершину и в каждой вершине сходится одно и то же число k треугольников. Найдите все значения п и k, при которых такое расположение возможно. 18. В числовом треугольнике 1 111 12321 1367631 каждое число равно сумме чисел, расположенных в предыдущей строке над этим числом и над его соседями справа и слева (отсутствующие числа считаются равными нулю). Доказать, что в каждой строке, начиная с третьей, найдется четное число. 19. Внутри квадрата расположен выпуклый четырехугольник, внутри которого взята точка А. Известно, что девять точек— вершины квадрата, вершины четырехугольника и точка А — расположены так, что никакие три из них не лежат на одной прямой. Доказать, что из них можно выбрать пять точек, лежащих в вершинах выпуклого пятиугольника. 20. Из чисел 1, 2, ..., 200 выбрали одно число, меньшее 16, и еще 99 чисел. Доказать, что среди выбранных чисел найдутся два таких, одно из которых делится на другое. 21. Доказать, что если четыре грани тетраэдра равновелики, то они равны между собой. 32
XI ОЛИМПИАДА (1948 г.) 1-й тур VII—VIII классы 1. Сумма обратных величин трех целых положительных чисел равна 1. Каковы эти числа? Найти все решения. 2. Найти все возможные расположения четырех точек на плоскости, при которых попарное расстояние между этими точка- ми принимают только два значения а и Ь. При каких значе- ниях отношения -j- такие расположения возможны? 3. На плоскости проведено п прямых. Доказать, что облас- ти, на которые эти прямые разбивают плоскость, можно так закрасить двумя красками, что никакие две соседние области (т. е. области, соприкасающиеся по отрезку прямой) не будут закрашены одной и той же краской. IX—X классы 2п 2 22"-1 2 4. Если число- целое, то число ——— целое. Доказать. П £ — 1 5. Доказать без помощи таблиц, что т-!—к-г-—>2. logjn ' logs я 6. Даны две треугольные пирамиды ABCD и A'BCD. Точка А’ лежит внутри пирамиды ABCD. Доказать, что сумма плоских углов при вершине А’ пирамиды A'BCD больше суммы плоских углов при вершине А пирамиды ABCD. 7. Даны окружность и точка А вне ее. Из этой точки мы совершаем путь по некоторой замкнутой ломаной, состоящей из отрезков прямых, касательных к окружности, и заканчиваем путь в точке А (рис. 1). Участки пути, по которым мы приближаемся к центру окружности, берем со знаком «плюс», остальные — со знаком «минус». Доказать, что алгебраическая сумма длин участков пути с указанными знаками равна нулю. 2-й т у р VII—VIII классы 8. Решить в натуральных числах урав- нение Xs = у* (х ф у). Найти все решения. 9. Доказать, что в любом треуголь- нике имеет место неравенство R^2r (Rar — радиусы описанного и вписан- ного кругов соответственно), причем ра- венство R = 2r имеет место только для правильного треугольника. 2 Заказ 247 33
10. Может ли фигура иметь более одного, но конечное число центров симметрии? 11. Доказать, что если отрезок, соединяющий середины про- тивоположных сторон выпуклого четырехугольника, равен полу- сумме двух оставшихся сторон, то эти две стороны параллельны. 12. На плоскости расположены два прямолинейных зеркала, образующих угол а. Доказать, что, если а=^-, и только в этом случае, любой луч, несколько раз отразившись от зеркал, в конце концов уйдет в направлении, противоположном тому, по которому пришел. Найти число его отражений. IX—X классы 13. Найти все рациональные положительные решения уравне- ния ху = ух (х^у) (указать формулу, дающую все решения). 14* . Поместить в куб окружность наибольшего возможного радиуса. 15. Сколько различных целочисленных решений имеет нера- венство | х | 4- | у | < 100? 16. Каково наибольшее возможное число лучей в простран- стве, выходящих из одной точки и образующих попарно тупые углы? 17. В пространстве расположены три плоских зеркала, обра- зующих октант (трехгранный угол с прямыми плоскими угла- ми). Доказать, что любой луч света, попавший в этот зеркаль- ный октант, после нескольких отражений от его граней уйдет в направлении, противоположном тому, по которому пришел. Найти число его отражений. (Ср. с задачей 12.) XII ОЛИМПИАДА (1949 г.| 1-й т у р VII— VIII классы 1. Показать, что 27 I958— 10 8878-|-10 152е делится на 26460 без остатка. 2. Доказать, что если плоский многоугольник имеет несколько осей симметрии, то все они пересекаются в одной точке. 3. Доказать, что равенство зг+у2+з?=2xyz для целых чисел х, у, z возможно только при x=y = z=0. 4. Дана плоская замкнутая ломаная периметра 1. Доказать, что можно начертить круг радиуса покрывающий всю ломаную. 5. Доказать, что для любого треугольника отрезок, соединя- ющий центры вписанной и вневписанной окружностей, делится описанной окружностью пополам. 34
IX—X классы 6. Найти такие целые числа х, у, г, и, что х2+у2+г2 + и2 = 2xyzu. 7. Как расположены плоскости симметрии ограниченного тела, если оно имеет две различные оси вращения? 8. Найти действительные корни уравнения ^ + 2ах+-^=-а+уа2+х—~ (о<а<-Ь) . 9. Имеется набор из 4л положительных чисел. Из любых четырех попарно различных чисел этого набора можно составить геометрическую прогрессию. Доказать, что в таком случае в наборе имеются по крайней мере п одинаковых чисел. 10. Доказать, что если у шестиугольника противоположные стороны параллельны и диагонали, соединяющие противополож- ные вершины, равны, то вокруг него можно описать окружность. 1-й тур VII— VIII классы II. Двенадцать полей расположены по окружности. На четы- рех соседних полях стоят четыре разноцветные фишки: красная, желтая, зеленая и синяя. Одним ходом можно передвинуть любую фишку с поля, на котором она стоит, через четыре поля на пятое, если оно свободно, в любом из двух возможных направле- ний. После нескольких ходов фишки стали опять на те же четыре поля. Как они могут при этом переставиться? 12. Даны два треугольника АВС и DEF и точка О. Берется любая точка X в треугольнике АВС и любая точка Y в тре- угольнике DEF, после чего треугольник OXY достраивается до параллелограмма OXYZ. Доказать, что все полученные таким образом точки Z образуют многоугольник. Сколько сторон он мо- жет иметь? Доказать, что его периметр равен сумме периметров исходных треугольников. 13. Имеется тринадцать гирь, масса каждой из которых равна целому числу граммов. Известно, что любые двенадцать из них можно так разложить на две чаши весов, по шесть на каждую, что наступит равновесие. Доказать, что все гири имеют одну и ту же массу. 14. В произвольном шестиугольнике середины сторон соедине- ны через одну. Доказать, что точки пересечения медиан двух образовавшихся треугольников совпадают. 2» 35
15. Если имеется сто любых целых чи- сел, то среди них всегда можно взять не- сколько (или может быть одно) так, что в сумме они дадут число, делящееся на 100. Доказать. IX—X классы 16. См. задачу II 17. Сложить из одинаковых кирпичи- ков, один из которых изображен на рисун- ке 2, выпуклый многогранник. 18. В данный треугольник вписать цент- рально-симметричный многоугольник мак- симальной площади. 19*. Доказать, что числа вида 2П при различных натуральных п могут начинаться на любую заранее заданную комбинацию цифр. 20. Доказать, что к квадрату нельзя приложить более восьми не налегающих друг на друга квадратов такого же размера. XIII ОЛИМПИАДА (1950 г.) 1-й тур VII — VIII классы 1. Имеется шахматная доска с обычной раскраской (границы квадратов считаются окрашенными в черный цвет). Начертить на ней окружность наибольшего радиуса, целиком лежащую на черных полях, и доказать, что большей окружности того же рода начертить нельзя. 2. Имеется 555 гирь массой 1 г, 2 г, 3 г, 4 г, ..., 555 г. Разложить их на три равные по массе кучи. 3. См. задачу 10 для п — 3 окружностей. 4. Пусть а, b и с — длины сторон треугольника; Л, В и С — величины противоположных углов. Доказать, что Aa + Bb + Cc^-^-(Ab-l-Ac+Ba + Bc + Ca + Cb). 5. Из пункта А в другие можно попасть двумя способами: выйти сразу и идти пешком или, вызвав машину и подождав ее определенное время, ехать на ней. В каждом случае используется способ передвижения, требующий меньшего времени. При этом оказывается, что если конечный пункт отстоит на 1 км, 2 км, 3 км, то на дорогу понадобится соответственно 10 мин, 15 мин, 17,5 мин. Скорости пешехода и машины, а также время ожида- ния машины принимаются неизменными. Сколько времени пона- добится для достижения пункта, отстоящего от Л на б км? 36
IX — X классы 6. Пусть А — произвольный угол, В и С — острые углы. Всегда ли существует такой угол X, что sinX = ______ sin В sin С 1 — cos В cos С cos А 7. Из двух треугольных пирамид с общим основанием одна лежит внутри другой. Может ли сумма ребер внутренней пирамиды быть больше суммы ребер внешней? 8. Имеется 81 гиря массами I2 г, 22 г, З2 г, .... 812 г. Разло- жить их на три равные по массе кучи. 9. Решить уравнение V-v + 3-4^-1 + V-v+8-6vv-1 = 1 10. Даны п окружностей: О1э О2, О3, ...» О/ь проходящих через одну точку О. Вторые точки пересечения О\ с О2, Ог с Оз, ..., Оп с Oj обозначим соответственно через Л|, Л2, Ап. На О| берем произвольную точку В\. Проводим через At и Bi прямую до второго пересечения с О2 в точке В2, затем про- водим через Л2 и В2 прямую до пересечения с О3 в точке Вз и т. д., пока не получим точку Вп на окружности Оп. Прове- дем через точки Вп и Ап прямую до второго пересечения с Oi в точке Вл + 1. Доказать, что Вп + | совпадает с Вь Замечание. В случае совпадения каких-либо точек Bk и Ak нужно через точку А1{ проводить касательную к окружности Ok до пересечения с в точке В* + |. 2-й тур VII—VIII классы 11. В выпуклом 13-угольнике проведены все диагонали. Они разбивают его на многоугольники. Возьмем среди них многоугольник с наибольшим числом сторон. Какое самое боль- шое число сторон он может иметь? гг _ 1 з 5 7 99 I 12. Доказать, что <-. 13. В треугольник вписана окружность, около которой описан квадрат так, что ни одна сторона квадрата не параллельна ни одной стороне треугольника. Доказать, что длина части границы квадрата, лежащей вне треугольника, меньше половины периметра квадрата. 14* . На окружности расположены 20 точек. Эти точки соеди- няются десятью хордами, не имеющими общих концов и не пересекающимися между собой. Сколькими способами это можно сделать? 37
IX—X классы 15. См. задачу И для 1950-угольника. 16. Числа 1, 2, 3, ..., 101 выписаны в ряд в каком-то порядке. Доказать, что из этого ряда всегда можно вычеркнуть 90 чисел так, что оставшиеся 11 будут расположены в порядке возраста- ния или убывания. 17. Около сферы описан пространственный четырехугольник. Доказать, что все точки касания лежат в одной плоскости. 18. Можно ли провести в городе 10 автобусных маршру- тов и установить на них остановки так, что для любых 8 марш- рутов найдется остановка, не лежащая ни на одном из них, а любые 9 маршрутов проходят через все остановки? XIV ОЛИМПИАДА (1951 г.) 1-й т у р VII—VIII классы I. Доказать, что многочлен х12—х9 + х4 — х+l при всех зна- чениях х положителен. 2. Пусть ABCD и A'B'C'D' — два выпуклых четырехугольника с соответственно равными сторонами (ЛВ = Л'В\ ВС = В'С' и т. д.). Доказать, что если Z_A> Z-А', то ZBCZB', Z.OZ.C', Z.D<Z.D'. 3. Какое число больше. (1 О4)<.|_2,оо 04 ИЛИ(!,оо...о2)2 4-2.00.. 02’ (Во всех указанных числах после запятой сначала идут 10 нулей.) 4. На плоскости даны равнобочная трапеция ABCD и точка Р. Доказать, что из отрезков РА, РВ, PC, PD можно построить четырехугольник. 5. Имеется кусок цепи из 60 звеньев, каждое массой 1 г. Какое наименьшее число звеньев надо расковать, чтобы из об- разовавшихся частей можно было составить все массы в 1 г, 2 г, ..., 59 г, 60 г? (Раскованное звено также имеет массу 1 г.) IX—X классы 6. Найти первые три цифры после запятой в десятичной записи частного от деления числа 0,123456789101112...495051 на число 0,515049...121110987654321. 7. Из всех выпуклых многоугольников, у которых одна сторона равна а и сумма внешних углов при вершинах, не прилегающих к этой стороне, равна 120°, найти многоугольник наибольшей площади. 8. Имеются две концентрические окружности. Вокруг меньшей из них описан многоугольник, целиком находящийся внутри 38
большей окружности. Из общего центра на стороны многоуголь- ника опущены перпендикуляры, которые продолжены до пересе- чения с большей окружностью; каждая из полученных точек пересечения соединена с концами соответствующей стороны много- угольника. При каком условии построенный так звездчатый много- угольник будет разверткой пирамиды? 9. См. задачу 5 для цепи из 150 звеньев. 10. Даны три параллельные прямые на равных расстояниях друг от друга. Как надо изображать точками соответствующих прямых величины сопротивления, напряжения и силы тока в проводнике, чтобы, прикладывая линейку к точкам, изображаю- щим значения сопротивления R и значения силы тока I, получить на шкале напряжения точку, изображающую величину напряже- ния U = hR? (Точка каждой шкалы изображает одно и только одно число.) 2-й тур VII—VIII классы 11. Доказать, что число I00...00500...001 не является кубом 49 нулей 99 нулей никакого целого числа. 12*. На плоскости даны три точки Л, В, С и три угла Z.D, Z.E, Z.F, каждый из которых меньше 180°, и в сумме составляющие 360°. Построить с помощью линейки и транспорти- ра точку О плоскости такую, что Z_AOB= Z.D, Z_BOC= Z.E и Z_COA = Z_F. (С помощью транспортира можно перенести любой данный угол так, чтобы одна из его сторон заняла произ- вольное наперед заданное положение.) 13. Доказать, что сумма I3 + 23 +... + п3 при всех п есть полный квадрат. 14. Какой фигурой может быть образ треугольника при цент- ральной симметрии, центр которой не лежит в плоскости треугольника? 15. На консультации было 20 школьников и разбиралось 20 задач. Оказалось, что каждый решил две задачи и каждую задачу решили двое. Доказать, что можно организовать разбор задач так, чтобы каждый школьник рассказал одну из решенных им задач и все задачи были разобраны. 16. При делении многочлена х1961—1 на многочлен Р(х)= = х4 4-х3 + 2х2 + х+ 1 получается частное и остаток. Найти в част- ном коэффициент при х14. IX—X классы 17. В и-угольную пирамиду вписана сфера. Доказать, что если совместить все боковые грани пирамиды с плоскостью 39
основания, повернув их вокруг соответствующих ребер основания, то все точки касания этих граней со сферой сольются в одну точку //, а вершины граней расположатся на окружности с центром Н. 18*. Имеется несколько чисел, каждое из которых меньше чем 1951. Наименьшее общее кратное любых двух из них больше чем 1951. Доказать, что сумма обратных величин этих чисел меньше 2. 19. Из всех ортогональных проекций правильного тетраэдра на различные плоскости найти проекцию наибольшей площади. 20. Окружность обладает тем свойством, что внутри ее можно двигать вписанный правильный треугольник так, чтобы каждая вершина треугольника двигалась по этой окружности и описывала ее целиком. Найти плоскую замкнутую несамопере- секающуюся кривую с тем же свойством, отличную от окруж- ности. 21*. Автобусный маршрут содержит 14 остановок (считая две конечные). В автобусе одновременно могут ехать не более 25 пассажиров. Доказать, что во время поездки автобуса из одного конца в другой: а) найдутся 8 различных остановок Aif В|, А 2, B2f Аз, Вз, А4, В4, таких, что для всех Л=1, 2, 3, 4 ни один пассажир не едет от А* до Bk\ б) может оказаться, что не существует 10 различных остановок Ah В|, ..., А6, В5, которые обладали бы аналогичными свойствами. (Считается, что пассажир едет от остановки А до остановки В, если он вошел в автобус на остановке А, а вышел на остановке В.) XV ОЛИМПИАДА |1952 г.) 1-й тур VII класс 1. В треугольник АВС вписана окружность, касающаяся его сторон в точках L, М, N. Доказать, что треугольник LMN всегда остроугольный. 2. Доказать тождество (ax + 6y + cz)2+(fcx+cy+az)24-(cx+ + ay + bz)2 = (ex 4- by 4- az)2 4- (bx 4- ay 4- cz)2 4- (ax 4- cy 4- bz)2. 3. Если все грани параллелепипеда — равные между собой параллелограммы, то они являются ромбами. Доказать. 4. См. задачу б и ответьте на вопрос: когда должен выйти из N пешеход С, чтобы А и В прибыли в N одновременно? VIII класс 5. Доказать, что если ортоцентр делит все высоты треугольника в одном и том же отношении, то треугольник равносторон- ний. (Ортоцентром называется точка пересечения высот треуголь- ника.) 40
6. Два человека А и В должны попасть из пункта М в пункт 7V, расположенный в 15 км от М. Пешком они могут пе- редвигаться со скоростью 6 км/ч. Кроме того, в их распоря- жении имеется велосипед, на котором можно ехать со скоростью 15 км/ч. А и В отправляются в путь одновременно, причем А идет пешком, а В едет на велосипеде до встречи с пешехо- дом С, идущим из N в М. Дальше В передает С велосипед и продолжает путь пешком, а С едет на велосипеде до встречи с А, передает ему велосипед, на котором тот и приезжает в М С идет пешком с той же скоростью, что А и В. Время, затраченное Л и Б на дорогу, считается от момента их отправления из М до момента прибытия последнего из них в N. Когда должен выйти из N пешеход С, чтобы это время было наимень- шим? 7. Доказать тождество (ах + by+cz+duf +(bx + cy + dz + аи)2 + +(сх+dy+az+ bu)2 +(dx+ay4-bz4-си)2 = =(dx+су + bz аи)2 4~(cx4-by -\-az-\-duy 4- 4- (bx 4- ay 4- dz4-cu)24-(ax+dy4-cz 4- b u)2. 8. См. задачу 3. IX класс 9. Дана геометрическая прогрессия, знаменатель которой — целое число, не равное 0 и —1. Доказать, что сумма двух или большего числа произвольно выбранных членов не может равняться никакому члену этой прогрессии. 10. Доказать, что если |х| < 1, |у| < 1, то | <1. 11. Треугольник АВС разбит прямой BD на два треуголь- ника. Доказать, что сумма радиусов окружностей, вписанных в треугольники ABD и DBC, больше радиуса окруж- ности, вписанной в треугольник АВС. 12. Дана последовательность целых чисел, построенная сле- дующим образом: ai — произвольное трехзначное число; аг— сумма квадратов его цифр; аз — сумма квадратов цифр числа аг и т. д. Доказать, что в последовательности О|, аг, аз, ... обязательно встретится либо 1, либо 4. 13. См. задачу 18. X класс 14. Найти соотношение между arcsincosarcsih х и arccossinarccos х. 15. Доказать, что при целом п^2 и |х|<1 справедливо неравенство (1-хУ4-(14-хГ<2". 41
16. В трехгранный угол с вершиной S вписана сфера с центром в точке О. Доказать, что плоскость, проходящая через три точки касания, перпендикулярна прямой OS. 17. Доказать, что если при любом положительном р оба корня уравнения ах1 2-|-Ьх-|-с-|-р=0 действительны и положитель- ны, то о=0. 18. Даны три попарно скрещивающиеся прямые. Доказать, что они являются общими перпендикулярами к своим общим перпендикуляра м. 2-й т у р VI! класс 19. См. задачу 27 при п=15. 20. В выпуклом четырехугольнике ABCD выполнено соотноше- ние AB + CD=BC+AD. Доказать, что окружность, вписанная в ДЛВС, касается окружности, вписанной в ДЛСД. 21. Доказать, что если квадрат числа начинается с 0,9...99 (100 девяток), то и само число начинается с 0.9...9 (не менее чем 100 девяток). (Ср. с задачей 23.) 22. Дан отрезок АВ. Найти множество вершин С остроуголь- ных треугольников АВС. VIII класс 23. Найти первые шестьдесят десятичных знаков числа V0,999...99 (60 девяток). 24. Из точки С проведены касательные СА и СВ к окружности О. Из произвольной точки N окружности опущены перпен- дикуляры ND, NE, NF соответственно на прямые АВ, СА и СВ. Доказать, что ND есть среднее пропорциональное между NE и NF. 25. Имеется семь жетонов с цифрами 1, 2, 3, 4, 5, 6, 7. Доказать, что никакие 2 семизначных числа, составленных с помощью этих жетонов, не делятся одно на другое. 26. 99 прямых разбивают плоскость на п частей. Найти все возможные значения п, меньшие 199. IX класс 27. Решить систему уравнений 1 —Х|Х2=0, 1 — *2X3=0, 1 —хп_|Х„=0, 1 — х„Х| =0 и исследовать решение при различных п. 42
28. Поместить в полый куб с ребром а три цилиндра диаметра у и высоты а так, чтобы они не могли менять своего положения внутри куба. 29. См. задачу 25. 30*. В равнобедренном треугольнике АВС угол АВС равен 20° и ВС=АВ. На сторонах АВ и ВС взяты соответ- ственно точки Q и Р такие, что Z_/MC=50°, Z.QCA =60°. Доказать, что угол PQC равен 30°. 31. Двести учеников выстроены прямоугольником по 10 чело- век в каждом поперечном ряду и по 20 человек в каждом продольном ряду. В каждом продольном ряду выбран самый высокий ученик (если таких несколько, выбирается любой), а затем из отобранных 10 человек выбран самый низкий. С другой стороны, в каждом поперечном ряду выбран самый низкий ученик, а затем среди отобранных 20 человек выбран самый высокий. Какой из этих двух отобранных учеников окажется выше? X класс 32. Доказать, что при фиксированных а(, ..., Дз1 и переменном х сумма cos 32x4-a3i cos 31x4-... + a? cos 2x4-0i cos х принимает как положительные, так и отрицательные значения. 33. См. задачу 28. 34. Доказать, что ни при каком целом а ни один много- член вида Зх2л + ахл + 2 не делится на многочлен 2х2от + ахт + 3. 35. См. задачу 30. 36. См. задачу 31. XVI ОЛИМПИАДА (1953 г.| 1-й тур VII класс 1. Доказать, что в трапеции сумма углов при большем ос- новании меньше, чем при меньшем. 2. Найти наименьшее число, записываемое одними единица- ми, которое делилось бы на 33...33 (100 троек). 3. Разделить отрезок пополам с помощью угольника (с по- мощью угольника можно проводить прямые и восставлять перпендикуляры, опускать перпендикуляры нельзя). 4. При любом натуральном п сумма п2 + 8п+15 не делится на п+4. Доказать. VIII класс 5. Три окружности попарно касаются друг друга. Через три точки касания проводим окружность. Доказать, что эта окруж- 43
ность перпендикулярна каждой из исходных. (Углом между двумя окружностями называется угол между касательными к ним в точке пересечения.) 6. Доказать неравенство 2--\/2+-У2+-+^ >_1_ 2--\/2+л/2+-+л/2 4 (в числителе п радикалов, в знаменателе на один меньше). 7. См. задачу 2. 8. См. задачу 3. IX класс 9. Найти множество точек плоскости, координаты которых удовлетворяют соотношению sin (*+{/)=0. 10. АВ и Д|В| — два отрезка, лежащие на скрещивающихся прямых, О и О\ — их середины. Доказать, что отрезок ОО\ меньше полусуммы отрезков AAt и ВВ|. И. Доказать, что многочлен х20() • г/200 +1 нельзя представить в виде произведения многочленов от одного только х и от одного только уу т. е. в виде f(x)-g(y). 12. Л — вершина правильного звездчатого пятиугольника, угол при которой меньше развернутого. Ломаная АА\BBiCC\DD\EEi является его внешним контуром. Прямые АВ и DE пересекаются в точке В. Доказать, что многоугольник ABB\CC\DED\A равновелик четырехугольнику AD\EF. 13. См. задачу 6. X класс 14. См. задачу 9. 15. Даны Прямой круговой конус и точка А. Найти множество вершин конусов, равных данному, с осями, параллельными оси данного конуса, и содержащих внутри данную точку А. 16. См. задачу 11. 17. См. задачу 12. 18. См. задачу 6. 2-й т у р VII класс 19. Доказать, что наибольший общий делитель суммы двух чисел и их наименьшего общего кратного равен наибольшему общему делителю самих чисел. 20. Около окружности описан четырехугольник. Его диагонали пересекаются в центре этой окружности. Доказать, что этот четырехугольник — ромб. 44
21. См. задачу 27 при я = 11. 22. 1000 точек являются вершинами выпуклого тысячеуголь- ника, внутри которого расположено еще 500 точек так. что никакие три из этих 1500 точек не лежат на одной пря- мой. Данный тысячеугольник разрезан на треугольники таким образом, что все указанные 1500 точек являются вершинами треугольников и эти треугольники не имеют никаких других вер- шин. Сколько треугольников получится при таком разреза- нии? 23. См. задачу 28 при л =5. VIII класс 24. а, Ь, с и d — длины последовательных сторон четырех- угольника, S — его площадь. Доказать, что (а4-с)(Ь+</). 25. 1953 цифры выписаны по окружности. Известно, что если читать эти цифры по часовой стрелке, начиная с некоторого определенного места, то полученное 1953-значное число делится на 27. Доказать, что если начать читать по часовой стрелке с любого другого места, то полученное число также будет делиться на 27. ' 26. На окружности дано множество точек Ai, Аг. Аз..А„. Построим все возможные выпуклые многоугольники, вершинами которых являются точки из данного множества. Разобьем эти многоугольники на 2 группы. В первую группу входят все многоугольники, у которых Ai является вершиной. Во вторую груп- пу входят все многоугольники, у которых Ai вершиной не является. В какой группе больше многоугольников? 27. В плоскости расположено и зубчатых колес таким образом, что первое колесо сцеплено своими зубцами со вторым, второе — с третьим и т. д. Наконец, последнее колесо сцеплено с первым. Могут ли вращаться колеса такой системы? 28. Решить систему уравнений при я = 100: Xi-р2х-г2хз2x4-|-2x54- ••• 4“2хя=1» X| -j-3x2-4" 4хз 4~ 4x4 4- 4xs 4* ••• 4“4хп = 2, Х|4-Зхг4-5хз4-6x4+6xs+ ••• -4-6хп=3, Х14-Зх2 4-5хз4-7х4 4-9х5-4- — 4-(2я — 1)хп = я. IX класс 29. См. задачу 25. 30. В плоскости даны дДИгЛэ и прямая I вне его, образующая с продолжениями сторон треугольника А1А2, АзАз, Д3Д1 соответственно углы аз, ai. аг. Через точки Ai, Аг, А3 проводятся прямые, образующие с / соответственно углы 45
л—at, л —аг, л —аз. Доказать, что эти прямые пересекаются в одной точке. (Все углы отсчитываются от прямой I в одном направлении.) 31. Даны уравнения ах2 4- bx 4-с=0, (1) —ах2 + Ьх+с=О. (2) Доказать, что если Х| и хз (соответственно) — какие- либо корни уравнений (1) и (2), то найдется такой корень хз уравнения -|-х24-Ьх4-с=0, что либо Х|^хз^хг, либо х( >х3>х2. 32. Дан лист клетчатой бумаги размером 101X200 клеток. Начиная от какой-либо угловой клетки идем по диагонали и каждый раз, дойдя до границы, меняем направление движения по правилу отражения света. Попадем ли мы когда-нибудь в какую-либо угловую клетку? 33. Разрезать куб на три равные пирамиды. X класс 34. Найти корни уравнения ]f| х(х 0 _|_(__________ 1 у» х(х—1)‘...*(х—/1-1-1) Q 35. См. задачу 30. 36. Пусть хо=Ю9, хп = 2 • Д°казать, что 0 < х3б—-\/2 < <10"9. 37. См. задачу 33. 38. На бесконечной шахматной доске стоит конь. Найти все клетки, куда он может попасть ровно за 2п ходов. XVII ОЛИМПИАДА (1954 г.) 1-й т у р VZZ класс 1. Правильный звездчатый шестиугольник разрезать на 4 ча- сти так, чтобы из них можно было составить выпуклый много- угольник. 2. Даны два выпуклых многоугольника Д|Дг...Дп и В|Вг...Вп. Известно, что Д|Дг=В|В2, ^2^3 = 6263,.... ЛПЛ|=ВПВ| и п—3 уг- ла одного многоугольника равны соответственным углам другого. Будут ли эти многоугольники равны? 3. Определить четырехзначное число, если деление этого числа на два разных однозначных производится по таким схемам: 46
**** I *r * r.- 4r*4t4t I 4r If 4. Существуют ли целые числа тип, удовлетворяющие уравнению т2+ 1954 = п2? 5. Определить наибольшее значение отношения трехзначно- го числа к сумме его цифр. VIII класс 6*. Из квадрата размером 3X3 вырезать одну фигуру, ко- торая представляет собой развертку полной поверхности куба, длина ребра которого равна 1. 7. Из произвольной внутренней точки О выпуклого п-уголь- иика опущены перпендикуляры на стороны (или на их продол- жения). На каждом перпендикуляре от точки О в направле- нии к соответствующей стороне отложен вектор, длина кото- рого равна половине длины этой стороны. Определить сумму по- лученных векторов. 8. См. задачу 3. 9. Найти все решения системы, состоящей из уравнений вида J'(1-^+«(1-STT)+2(|-5^)=0’ ™е * 3' 10. См. задачу 4. IX класс 11. Доказать, что если хо4-а|ХоЦ-а2Хо-|-азХо-|-а4=О и 4хо-|- •+-За|Хо4-2а2Хо4-аз=О, то многочлен х4+О|Х34-агХ2-|-азХ-|-а4 делится на (х—х©)2. 12. Дано число 123456789101112131415...9899100. Вычерк- нуть 100 цифр так, чтобы оставшееся число было наибольшим. 13. Дано 100 чисел at, аг, .... а кто, удовлетворяющих условиям at — Заг Ц- 2аз 0, аг — Заз 2а^ 0, аг®—3aioo + 2ai >0, аюо—3ai 2аг 0. Доказать, что все эти числа равны между собой. 14. Дан треугольник АВС. Пусть S — любая точка внутри дЛВС, Л|, В|, С|— точки пересечения прямых BS, CS соот ветственно со сторонами ВС, АС, АВ треугольника АВС. До-
казать, что по крайней мере в одном из полученных четырех- угольников ABtSCt, CiSAiB, AtSBtC оба угла при С| и Bi, или С| и Ai, или Л| и В| не острые. 15. Существуют ли в пространстве такие четыре точки А, В, С, D, что AB = CD = 8 см, ЛС=ВД=10 см и AD = BC=\3 см? X класс 16. Найти все действительные решения уравнения х24-2х sin (ху)4-1 =0. 17. См. задачу 12. 18. Дано 100 чисел ai, аг, ..., аюо, удовлетворяющих усло- виям а,—4а,+ i -|-3ai+2^:0 для всех i = l, 2, 3, ..., 100. Известно, что а( = 1. Найти аг, аз, .... аюо- 19. См. задачу 14. 20. См. задачу 15. 2-й т у р VII класс 21. Дан лист клетчатой бумаги, каждый узел сетки которого обозначен некоторой буквой. Известно, что на любом отрезке, соединяющем два узла, обозначенные одной буквой и лежащие на одной линии, найдется по крайней мере один узел, обозна- ченный другой буквой. Какое наименьшее число различных букв требуется для этого? 22*. Решить систему х 1 Ох । 4“ 3x2 4“ 4хз -f- Х4 -|- Х5 == О, 11 Хг 4- 2хз -4 2x4 4” ЗХ5 “I- Х6 == О, 15хз 4- 4х4+5xs 4- 4х6 4- х7=О, \ 2X|-J-X2 —Зхз+12X4 —ЗХ5-|-Хб4-Х7 = 0, 6Х| — 5x2 + Зхз — Х4 + 17X5 4 Хб = О, ЗХ| 4-2x2 —Зхз +4х4-|-xs— 16хб4*2х7=0, 4х । — 8x2 4- Хз -J- Х4 4“ Зхб 19x7 0. 23. Сколько осей симметрии может иметь семиугольник? 24. Дано число /V = 2-3-5-7-11 • 13-17-19-23-29-31 (произве- дение последовательных простых чисел). Пусть 1, 2, 3, 5, .... N — все его делители, выписанные в порядке возрастания. Под рядом делителей выпишем ряд из 4" 1 и — 1 по следующему правилу: под 1 пишем 4-1; под числом, разлагающимся на четное число простых сомножителей, пишем 4-1. под остальными чис- лами пишем —1. Доказать, что сумма чисел построенного ряда равна нулю. 25. План города представляет собой плоскость, разбитую на одинаковые правильные треугольники. Стороны треугольни- 48
ков — шоссейные дороги, а вершины треугольников — пере- крестки; таким образом, на каждом перекрестке сходятся шесть дорог. Из точек А и В, расположенных на одной дороге (на одной стороне треугольника), одновременно в одном направле- нии с одинаковыми скоростями выезжают две машины. Доехав до любого перекрестка, каждая машина может или продолжить свое движение в первоначальном направлении, или же повернуть на 120° вправо или влево. Смогут ли эти машины встретиться? (Требуется либо доказать, что при любом движении машин они не встретятся, либо указать такое их движение, при котором они встречаются.) VIII класс 26. Из клетчатой бумаги вырезан квадрат 17X17. В каждой клетке квадрата написано одно из чисел 1, 2, 3, ..., 70. Дока- зать, что существуют четыре различные клетки, центры которых А, В, С, D являются вершинами параллелограмма (AB\\CD) и сумма чисел, стоящих в клетках с центрами А и С, равна сумме чисел, стоящих в клетках с центрами В и D. 27. Даны четыре прямые mi, m2, m3, m4, пересекающиеся в одной точке О, занумерованные относительно О по часовой стрелке. Через произвольную точку At прямой mi проводим пря- мую, параллельную т4, до пересечения с прямой тг в точке Аг. Через Аг проводим прямую, параллельную mi, до пересечения с тз в точке Аз. Через А3 проводим прямую, параллельную т2, до пересечения с т4 в точке Л4. Через А4 проводим прямую, па- раллельную тз, до пересечения с mi в точке В. Доказать, что 28. См. задачу 22. 29. См. задачу 23. 30. См. задачу 24 при W=2-3-5-7-11 • 13-17- 19-23-29-3I-37. IX класс 31. На двух лучах Л и 1г, проходящих через точку О, отло- жены отрезки ОЛ| и ОВ\ на луче /1 и ОАг и ОВг на луче /г; известно, что Найти множество всех точек пересече- ОАг ОВг г ния прямых AAt и ВВ\ при вращении луча 1г вокруг точки О (луч Л неподвижен). 32. См. задачу 27; доказать, что . 33*. Сто положительных чисел Х|, хг, .... хюо удовлетворяют условиям f х? -J- хг ... “I- xfoo > 10 000, 1 Х| -(-Ха-)-... 4-*ioo< 300. 49
Доказать, что среди них найдутся три числа, сумма которых больше 100. 34. Если дана последовательность 15 чисел ait az, аз, .... a is, то можно построить новую последовательность чисел bi, bz, Ьз, .„, 6 is, где bi равно количеству чисел первой последователь- ности, меньших а,, <=1, 2, ..., 15. Существует ли последователь- ность (а*), для которой последовательность (Ьь) выглядит так: 1, 0, 3, 6, 9, 4, 7, 2, 5, 8, 8, 5, 10, 13, 13? 35. Из точки А выходит пять отрезков: ABt, ABz, АВ3, АВ4, АВъ. Из каждой точки Д могут выходить еще пять отрезков или ни одного отрезка и т. д., причем концы никаких двух отрезков построенной системы не совпадают. Может ли число свободных концов построенных отрезков равняться 1001? (Под свободным концом мы понимаем точку, из которой не выходит ни одного отрезка.) X класс 36. Сколько плоскостей симметрии может иметь треуголь- ная пирамида? 37. См. задачу 32. 38. См. задачу 33. 39. Известно, что модули всех корней уравнений х1 2-|-Лх-|-/?=0 и x2-|-Cx-|-D=0 меньше 1. Доказать, что модули корней уравнения х2+^±£х+-^=0 также меньше 1. 40. Рассматриваются всевозможные десятизначные числа, записываемые при помощи цифр 1 и 2. Разбить их на два класса так, чтобы при сложении любых двух чисел одного класса по- лучалось число, в написании которого содержится не менее двух троек. XVIII ОЛИМПИАДА (1955 г.) 1-й т у р VII класс 1. См. задачу 11 при fe=7. 2. Дан прямоугольный треугольник АВС. Из вершины В пря- мого угла проведена медиана BD. Пусть К — точка касания стороны AD треугольника ABD с окружностью, вписанной в этот треугольник. Найти углы треугольника АВС, если К делит AD пополам. 50
3. Дан равносторонний треугольник АВС. На сторонах АВ и ВС взяты точки D и Е так, что AE=CD. Найти множество то- чек пересечения отрезков АЕ и CD. 4. Существует ли такое целое п, что n2-|-n + 1 делится на 1955? 5. Найти все прямоугольники, которые можно разрезать на 13 равных квадратов. VIII класс в. Дано: 2"=10а-|-б. Доказать, что если л>3, то ab де- лится на 6. Здесь п, а, b — целые положительные числа, b < 10. 7. Дан четырехугольник ABCD. На стороне АВ взята точка К, на стороне ВС — точка L, на стороне CD — точка М и на стороне AD — точка N так, что KB=BL=a, MD=DN=b. Пусть KL X MN. Найти множество всех точек пересечения прямых KL и MN при изменении а и Ь. 8. См. задачу 16 при п = 7. 9. Какие выпуклые плоские области могут содержать прямую? 10. На окружности даны четыре точки А, В, С, D. Через каждую пару соседних точек проведена окружность. Вторые точки пересечения соседних окружностей обозначим через Дь Bi, Ci, Di (некоторые из них могут совпадать с прежними). Доказать, что точки Д>, В\, Ci, Dt лежат на одной окружности. IX класс 11. Числа 1, 2, .... k2 расположены в квадратной таблице: 1, 2....... k, Л4-1, Л4-2, ...» 2k, (Л-1)Л+Т (k-1)^+2* k2’. Произвольное число выписывается, после чего из таблицы вычер- киваются строка и столбец, содержащие это число. То же самое проделывается с оставшейся таблицей из (k — I)2 чисел и т. д. k раз. Найти сумму выписанных чисел. 12. Найти множество середин всех отрезков с концами на двух различных непересекающихся окружностях, лежащих одна вне другой. 13. Найти все действительные решения системы (х3+у3=1, \х4+д4=1. 14. р простых чисел at, аг, .... ар образуют возрастающую арифметическую прогрессию и ai>p. Доказать, что если р — простое число, то разность прогрессии делится на р. 15. Дан дЛВС и точка D внутри него, причем AC—DA> 1 и ВС—BD>\. Берется произвольная точка Е внутри отрезка АВ. Доказать, что ЕС—ED>1. 51
X класс 16. Квадратная таблица в п2 клеток заполнена числами от 1 до п так, что в каждой строке и в каждом столбце встре- чаются все эти числа. Если п нечетно и таблица симметрична относительно диагонали, идущей из левого верхнего угла в пра- вый нижний, то на этой диагонали встретятся все числа 1, 2, 3, ..., п. Доказать. 17. См. задачу 13. 18. См. задачу 15. 19. Дан трехгранный угол с вершиной О. Можно ли найти такое плоское сечение ЛВС, чтобы углы ОЛВ, ОВЛ, ОСВ, ОАС, ОСА, ОВС были острыми? 2-й т у р VII класс 20. Решить в целых числах уравнение х3 — 2у3 —• 4г3 = 0. 21. Доказать, что если трехчлен ах2 + Ьх+с при всех целых х является точной четвертой степенью целого числа, то a=fr = 0. 22. Дан дЛВС. Центры вневписанных окружностей Оь Ог и Оз соединены прямыми. Доказать, что ДО1О2О3 остроуголь- ный. 23. В турнире собираются принять участие 25 шахматистов. Все они играют в разную силу, и при встрече всегда побеждает сильнейший. Какое наименьшее число партий требуется, чтобы определить двух сильнейших игроков? 24. Разрезать прямоугольник на 18 прямоугольников так, чтобы никакие два соседних не образовывали прямоугольник. VIII класс 25*. Трехчлен ах2 + Ьх + с при всех целых х является точ- ным квадратом. Доказать, что тогда ax2 + bx + c — (dx+ef. 26*. Две окружности касаются друг друга внешним образом и третьей изнутри. Проводятся внешняя и внутренняя общие ка- сательные к первым двум окружностям. Доказать, что внутрен- няя касательная делит пополам дугу, отсекаемую внешней ка- сательной на третьей окружности. 27. Точка О лежит внутри выпуклого n-угольника Л|Л2... Ап и соединена отрезками с его вершинами. Стороны п-угольника нумеруются числами от 1 до п, причем разные стороны нуме- руются разными числами. То же самое делается с отрезками ОЛь ОЛ2, ..., ОАп. а) Для и = 9 найти нумерацию, при которой сумма номеров сторон для всех треугольников Л1ОЛ2, Л2ОЛ3, ...» AnOAi одинакова, б) Доказать, что при п=10 такой нумерации осуществить нельзя. 52
28. Неравенство Аа (Bb-j-Cc)-j-Bb (Aa-j-Cc)-j-Cc (Аа + ВЬ)> >~(АВс2+ВСа2 + САЬг) (где а>0, 6>0, с>0 —данные числа) выполняется при всех Л>0, В>0, С>0. Можно ли из отрезков длины а, Ь, с составить треугольник? 29. Найти все такие числа а, что все числа [а], [2а], .... [ДОа] различны между собой и все числа [-i-J, , .... также различны между собой (натуральное N фиксировано). IX класс 30. Дан- треугольник АВС. На сторонах АВ, ВС, СА взяты соответственно точки Ci, At, В, так, что ДС, _ ВА, СВ, = _ 1 С\В А\С В\А п На сторонах Л|ВЬ BiCi, С1Л1 треугольника Л1В1С1 взяты соот- ветственно точки С2, Л2, В2 так, что А1С2 В\А2 С1В2 С2Вх “ Л2С| В2Л1 ' Доказать, что Л2С2ЦЛС, С2В2||СВ, В2Л2||ВЛ. 31. Расположить на прямой систему отрезков длины 1, не имеющих общих точек (в том числе и общих концов), так, чтобы любая бесконечная арифметическая прогрессия (с любой раз- ностью и любым первым членом) имела общую точку с некото- рым отрезком системы. 32. Дано уравнение хл—Я|ХЛ“| —я2хл~2——ял-1%—яп = 0. где ai^O, я2^0, Яз^О, ..., ял^0. Доказать, что это уравнение не может иметь двух положительных корней. 33. См. задачу 26. 34. Пять человек играют несколько партий в домино (двое на двое) так, что каждый играющий имеет каждого один раз партнером и два раза противником. Найти количество сыгран- ных партий и все способы распределения играющих. X класс 35. Доказать, что если -£-несократимая рациональная дробь, являющаяся корнем полинома f (х)==яоХл4-Я|Хл~1 + ... 4-ял с целыми коэффициентами, то р — kq есть делитель числа f (k) при любом целом k. 36. См. задачу 31. 37. На плоскости Р стоит прямой круговой конус. Радиус основания его равен г, высота — Л. На расстоянии Н от пло- скости и 1 от высоты конуса находится источник света. Ка- 53
кую часть окружности радиуса R, лежащей в плоскости Р и кон- центрической с окружностью, лежащей в основании конуса, осветит этот источник? 38. Имеется 1955 точек. Какое максимальное число троек точек можно из них выбрать так, чтобы каждые две тройки имели одну общую точку? 39. Дан треугольник АоВоСо- На его сторонах Лобо, ВоСо, С0Ап взяты соответственно точки С,, Ль Bt, на сторонах Л|ВЬ B|Ci, С|Л| треугольника Л|В(С1 взяты соответственно точки Сг, Л2, В2 и т. д. Известно, что ЛоВ|__BqCi __С&Л । * А^Вг В1С2___С1Л2__ I BtCti С|Ло Л|во * BiCi С2Л1 Л2в( k2 ’ и вообще л„в„-|.| _ в„сл+| ___ сплл+1 _ J k2" при четном п, вл+,сл сл+,л„ Лл+1вл I _L при нечетном п. v k2‘ Доказать, что треугольник ЛВС, образованный прямыми Л0Л1, BOB|, CoCi, содержится в треугольнике AnBnCn при любом п. XIX ОЛИМПИАДА (19S6 г.) 1-й т у р VII класс 1. Докажите, что на плоскости не существует таких четырех точек Л, В, С, D, что все треугольники ЛВС, BCD, CDA, DAB остроугольные. 2. Найти все двузначные числа, сумма цифр которых не ме- няется при умножении числа на 2, 3, 4, 5, 6, 7, 8, 9. 3. Имеется замкнутая самопересекающаяся ломаная. Извест- но, что она пересекает каждое свое звено ровно один раз. Дока- зать, что число звеньев четно. 4. Найти все числа, на которые может быть сократима дробь 5/4-6 , gjq-? при целом значении /. 5. Какое наименьшее число точек можно выбрать на окруж- ности, имеющей длину 1956, так, чтобы для каждой из этих то- чек нашлась ровно одна выбранная точка на расстоянии 1 и ровно одна на расстоянии 2 (расстояния измеряются по окружности)? VIII класс 6. На сторонах АВ и СВ треугольника ЛВС откладываются равные отрезки AD и СЕ произвольной длины. Найти множество середин отрезков DE. 54
7. В десятичной записи положительного числа а отброше- ны все десятичные знаки, начиная с третьего знака после запя- той (т. е. взято приближение числа а по недостатку с точностью до 0,01). Полученное число делится на а, и частное снова округ- ляется по недостатку с той же точностью. Какие числа при этом могут получиться? (Указать все значения.) 8. На окружности длины 15 выбрано п точек так, что для каждой имеется ровно одна выбранная точка на расстоянии 1 и ровно одна на расстоянии 2 (расстояния измеряются по окружно- сти). Доказать, что п делится на 10. (Ср. с задачей 5.) 9. Пусть а, fr, с, d, I — целые числа. Докажите, что если дробь сократима на число fe, то ad — bc делится на k. (Ср. с задачей 4.) 10. В каждой клетке бесконечного клетчатого листа бумаги стоит число, равное среднему арифметическому четырех чисел, стоящих в соседних клетках. Из этого листа вырезана прямо- угольная таблица. Доказать, что наибольшее из всех чисел таб- лицы стоит с края. IX класс 11. Дан выпуклый четырехугольник ABCD. Четырехугольник KLMN образован центрами тяжести треугольников ЛВС, BCD, DBA, CDA. Доказать, что прямые, соединяющие середины про- тивоположных сторон четырехугольника ABCD, пересекаются в той же точке, что и прямые, соединяющие середины противопо- ложных сторон четырехугольника KLMN. 12. См. задачу 7, причем округление производится с точ- ностью до 0,001. 13. См. задачу 10. 14. Даны положительные числа ft, s2 и расположенный в пространстве треугольник ЛВС. Сколькими способами можно выбрать точку D так, чтобы в тетраэдре ABCD высота, опу- щенная из вершины D, была равна h, а площади граней ACD и BCD равнялись соответственно si и $2? (Исследовать все воз- можные случаи.) 15. См. задачу 9. X класс 16. В треугольник вписан квадрат так, что две его верши- ны лежат на основании, а две другие вершины — на боковых сторонах треугольника. Доказать, что сторона квадрата меньше 2г, но больше г-\/2, где г — радиус окружности, вписанной в треугольник. 17. См. задачу 7, причем округление производится с точно- стью до 0,0001. 55
18. См. задачу 9. 19. Дана замкнутая пространственная ломаная A tA2A3...An. Некоторая плоскость пересекает все ее звенья: Л1Л2 в точке В(, Л2Л3 в точке В2.....AnAi в точке В„. „ Л|в1 А2В2 А3В3 А„В„ Доказать, что =1. 20. Доказать, что система уравнений xt — х2=а, Хз — Х4 = Ь, х» 4-*2+4" •*« = 1 имеет хотя бы одно решение Х|>0, х2>0, х3>0, тогда и только тогда, когда |а| 4-|Ь1 <1- 2-й т у р VII класс 21. Точка О — центр круга, описанного около треугольни- ка АВС. Точки Л|, Bi, Ct симметричны точке О относительно прямых, содержащих стороны треугольника АВС. Доказать, что все высоты треугольника Л1в|С> проходят через точку О, а все высоты треугольника АВС проходят через центр круга, опи- санного около треугольника Л1В1С1. 22. Точки At, А2, А3, Л«, А3, Ае делят окружность радиуса 1 на шесть равных частей. Из точки Л| проведен луч h в направле- нии А2, из А2 — луч 12 в направлении А3, ..., из Лв — луч /6 в направлении At. Из точки В(, взятой на луче /,, опущен перпен- дикуляр на луч /в. из основания этого перпендикуляра опускается перпендикуляр на Z5 и т. д. Основание шестого перпендикуляра совпало с В|. Найти длину отрезка Л<В|. (Ср. с задачей 40.) 23. См. задачу 29. 24. См. условие задачи 27, в которой нужно доказать, что сумма чисел в любом столбце меньше 1035. 25*. Доказать, что у любого дерева можно оборвать его 1 о 8 листьев, оставив при этом не менее — тени, которую давало дерево. (Число листьев считать кратным 15; тенью от ствола и веток пренебречь.) VI// класс 26*. Груз массой 13,5 т упакован в некоторое число неве- сомых ящиков. Масса каждого ящика с грузом не превосходит 350 кг. Доказать, что этот груз можно перевезти на 11 полуто- ратонках. 56
27. 64 неотрицательных числа, сумма которых равна 1956, расположены в форме квадратной таблицы по 8 чисел в каждой строке и в каждом столбце. Сумма чисел, стоящих на двух диаго- налях, равна 112. Числа, расположенные симметрично относи- тельно любой диагонали, равны. Доказать, что сумма чисел в любой строке меньше 518. 28. Все точки данного отрезка АВ ортогонально проектиру- ются на всевозможные прямые, проходящие через данную точку О. Найти множество всех этих проекций. 29. 100 чисел, среди которых есть положительные и отрица- тельные, выписаны в ряд. Подчеркнуто, во-первых, каждое поло- жительное число, во-вторых, каждое число, сумма которого со следующим положительна, и, в-третьих, каждое число, сумма ко- торого с двумя следующими положительна. Может ли сумма всех подчеркнутых чисел оказаться отрицательной? равной нулю? 30*. В прямоугольнике площадью 5 кв. ед. расположены девять прямоугольников, площадь каждого из которых равна единице. Доказать, что площадь общей части каких-нибудь двух прямо- угольников больше или равна IX класс 31. См. задачу 26. 32. В кубе, ребро которого равно 13, выбрано 1956 точек. Можно ли в этот куб поместить кубик с ребром 1 так, чтобы внутри него не было ни одной выбранной точки? 33. Взяли три числа х, у, г. Абсолютные величины их попар- ных разностей обозначили через Xi, yt, zt. Тем же способом по числам Х|, </|, Zi построили числа Хг, Уг, гг и т. д. Оказалось, что при некотором п справедливы равенства х„=х, ул = у, z„ — z. Зная, что х=1, найти у и z. 34. Четырехугольник описан около окружности. Доказать, что прямые, соединяющие соседние точки касания, пересекаются на продолжении диагонали или параллельны ей. 35*. На клетчатой бумаге выбраны три точки А, В, С, нахо- дящиеся в вершинах клеток. Доказать, что если треугольник АВС остроугольный, то внутри или на сторонах его есть по крайней мере еще одна вершина клетки. X класс 36. Подряд выписаны п чисел, среди которых есть положи- тельные и отрицательные. Подчеркивается каждое положитель- ное число, а также каждое число, сумма которого с несколькими непосредственно следующими за ним числами положительна. Доказать, что сумма всех подчеркнутых чисел положительна. (Ср. с задачей 29.) 57
37. См. задачу 30 для девяти произвольных многоугольников. 38. См. задачу 33. 39*. Доказать, что если в треугольной пирамиде трехгран- ные углы при каждой вершине составлены из одних и тех же троек плоских углов, то все грани этой пирамиды попарно равны. 40. На продолжениях сторон Д1Д2, Д2Л3, .... AnAt правиль- ного n-угольника Л1Л2 ... А„ построить точки Bt, В2, .... В„ так, чтобы В1В2 было перпендикулярно Л(Л2, В2В3 — перпендику- лярно Л2Л3, .... BnBt — перпендикулярно A„At. XX ОЛИМПИАДА (1957 г.| 1-й т у р VII класс 1. Равнобочная трапеция разбивается диагональю на два равнобедренных треугольника. Найти углы трапеции. 2. Известно, что ax3 + bx2 + cx+d (a, b, с, d — данные на- туральные числа) при любом целом х делится на 5. Доказать, что каждое из чисел a, b, с, d делится на 5/ 3. Улитка ползет по столу с постоянной скоростью. Через каждые 15 мин она поворачивает на 90°, а в промежутках меж- ду поворотами ползет по прямой. Доказать, что она может вер- нуться в исходный пункт только через целое число часов. 4. См. задачу 9. 5. От Л до В 999 км. Вдоль дороги стоят километровые столбы, на которых написаны расстояния до А и до В: 0 999 1 998 998 1 999 0 Сколько среди них таких, на которых оба расстояния записаны только двумя различными цифрами? VIII класс 6. Найти множество четвертых вершин всех таких прямоуголь- ников, три вершины которых лежат на двух данных концентриче- ских окружностях (две вершины лежат на одной из окружностей, а третья — на другой), а стороны параллельны двум данным прямым. 7. См. задачу 3. 8. Из всех параллелограммов данной площади найти тот, у которого большая диагональ минимальна. 9. В прямоугольной таблице произведение суммы чисел любого столбца на сумму чисел любой строки равно числу, стоя- щему на их пересечении. Доказать, что сумма всех чисел в табли- це равна единице или все числа равны нулю. 58
10. Известно, что ax* + bx^+cx2+dx+e (a, b, с, d, е — дан- ные натуральные числа) при всех целых х делится на 7. Дока- зать, что все числа а, b, с, d, е делятся на 7. (Ср. с задачей 2.) IX класс 11. См. задачу 9. 12. Решить уравнение х3— [х]=3. 13. В четырехугольнике ABCD точка М — середина диагонали AC, N — середина диагонали BD. Прямая, проходящая через точки М и N, пересекает стороны АВ и CD в точках М' и N'. До- казать, что если ММ' = NN', то AD\\BC. 14. Школьник едет на олимпиаду в метро, платит рубль и получает сдачу. Доказать, что если он обратно поедет трамваем, то он сможет уплатить за проезд без сдачи. Примечание. До денежной реформы 1961 г. проезд в метро стоил 50 к., в трамвае — 30 к. и в обращении находились монеты достоинством в 1, 2, 3, 5, 10, 15, 20 к. 15. См. задачу 20. X класс 16. При каких целых п число 20"+ 16"—3"— 1 делится на 323? 17. В пространстве построена замкнутая ломаная так, что все звенья имеют одинаковую длину и каждые три последователь- ные звена попарно перпендикулярны. Доказать, что число звень- ев делится на 6. (Ср. с задачей 3.) 18. См. задачу 13. 19. Школьник едет на занятия кружка трамваем, платит рубль и получает сдачу. Доказать, что на обратном пути он смо- жет уплатить за проезд в трамвае без сдачи (см. примечание к задаче 14). 20. Плоский многоугольник Л(ЛгЛз — ЛЯ_|Л„ составлен из п>4 твердых стержней, соединенных шарнирно. Можно ли его деформировать в треугольник? 2-й тур VII класс 21. Прямые О А и ОВ перпендикулярны. Найти множество концов М всех ломаных ОМ длины I, пересекающихся с каждой прямой, параллельной ОА или ОВ, не более чем в одной точке. 22. Радиолампа имеет 7 контактов, расположенных по кругу и включенных в штепсель, имеющий 7 отверстий. Можно ли так занумеровать контакты лампы и отверстия штепселя, чтобы при включении лампы хотя бы один контакт попал на свое место (т. е. в отверстие с тем же номером)? 59
23. В треугольнике известны две стороны а и Ь. Какой должна быть третья сторона, чтобы наибольший угол треуголь- ника имел наименьшую величину? 24. В треугольник вписана окружность. Точки касания яв- ляются вершинами второго треугольника. В него вписана окруж- ность, причем точки касания являются вершинами третьего тре- угольника, который имеет те же углы, что и первоначальный тре- угольник. Найти эти углы. 25. В последовательности Фибоначчи 1, 2, 3, 5, 8, 13, 21, ... выбрано 8 идущих подряд чисел. Доказать, что их сумма не входит в последовательность. VIII класс 26. В треугольнике известны две стороны а и Ь. Какой должна быть третья сторона, чтобы наименьший угол треугольника имел наибольшую величину? (Ср. с задачей 23.) 27. См. задачу 39 при й = 8. 28. Внутри равностороннего треугольника АВС находится точка О. Прямая OG, соединяющая О с центром тяжести G треугольника, пересекает стороны треугольника (или их продол- жения) в точках Д', В', С'. Доказать, что А'О I В'° I с'° — Q A'G + B'G C'G 29. Решить систему уравнений 2х? 1 4-х? 2x1 I + Ха *2, *3, 2хз 1 4~*з Х|. 30. В неравносторонний треугольник вписана окружность. Точки касания являются вершинами нового треугольника. В него вписана окружность, точки касания которой — вершины третьего треугольника, в третий вписана третья окружность и т. д. Дока- зать, что в образовавшейся последовательности треугольников нет подобных. (Ср. с задачей 24.) IX класс 31. Два прямоугольника расположены на плоскости так, что их стороны имеют восемь точек пересечения. Эти точки соедине- ны через одну. Доказать, что площадь полученного четырехуголь- ника не изменится при параллельном переносе одного из прямо- угольников. 32. См. задачу 37 при и = 99. 60
33. См. задачу 22 для 20 контактов, изменив вопрос. Всегда ли можно так соединить разъем, чтобы ни один контакт не был замкнут, т. е. не попал бы на свое место? 34. Разбить 1957 на 12 целых положительных слагаемых «и 02, аз» ...» О|2 так, чтобы произведение а|!-а2!-азЬ...«а|2! было минимально. 35*. Три равные окружности касаются друг друга. Из произ- вольной точки четвертой окружности, касающейся внешним обра- зом всех данных окружностей, проведены касательные к ним. Доказать, что сумма длин двух касательных равна длине третьей. X класс 36. Дан четырехугольник ABCD. Вписать в него прямоуголь- ник с заданными направлениями сторон. 37*. Найти все действительные решения системы 1 — л? = х2, 1 — Х2 = *3, 1-Л2п = Л. 38. Точка G — центр шара, вписанного в правильный тетраэдр ABCD. Прямая OG, соединяющая G с точкой О, лежащей внутри тетраэдра, пересекает плоскости граней в точках Л', В'. С'. D'. Доказать, что А'О . В'О . СО . D'O __д A'G'B'G* C G ' D'G~^' (Ср. с задачей 28.) 39. Доказать, что число всех цифр в последовательности I, 2, 3, ... ,10* равно числу всех нулей в последовательности 1, 2, 3. ... , 10*+1. 40. Дано п целых чисел: О| = 1, аг, ... , ап, причем (/=!, 2, 3...............и—I) и сумма всех чисел четна. Можно ли эти числа разбить на две группы так, чтобы суммы чисел в этих группах были равны? XXI ОЛИМПИАДА (1958 г.) 1-й тур VII класс I. Имеется система уравнений +*</+*£ = 0, *x + *i/ + *z = 0, *x + *(/ + *z = 0. Si
Два человека поочередно вписывают вместо звездочек числа. Доказать, что начинающий всегда может добиться того, чтобы система имела ненулевое решение. 2. В круге проведены два диаметра АВ и CD. Доказать, что если М — произвольная точка окружности, а Р и Q — ее проекции на диаметры АВ и CD, то длина отрезка PQ не зависит от выбора точки М. 3. Сколько существует четырехзначных номеров (от 0000 до 9999), у которых сумма двух первых цифр равна сумме двух последних цифр? 4. На плоскости даны точки А и В. Построить такой квадрат, чтобы точки А и В лежали на его границе и сумма расстояний от точки А до вершин квадрата была наименьшей. 5. Дана следующая треугольная таблица чисел: 0 1 2 ..................... 1957 1958 1 3 ...................... 3915 Каждое число (кроме чисел верхней строки) равно сумме двух ближайших чисел предыдущей строки. Доказать, что число, стоящее в самой нижней строке, делится на 1958. VIII класс 6. Внутри треугольника АВС взята точка О. На лучах ОД, ОВ, ОС построены векторы единичной длины. Доказать, что сумма этих векторов имеет длину, меньшую единицы. 7. Доказать, что если уравнения с целыми коэффициентами *?+pix + qt=0, x2 + p2x+q2 = 0 имеют общий не целый корень, то рх=р2, q\—q?. 8. На круглой поляне радиуса R растут три круглые со- сны одинакового диаметра. Центры их стволов находятся р на расстоянии — от центра поляны в вершинах равносторон- него треугольника. Два человека, выйдя одновременно из диамет- рально противоположных точек поляны, обходят поляну по краю с одинаковой скоростью в одном направлении и все время не видят друг друга. Увидят ли друг друга три человека, если они будут так же обходить поляну, выйдя из точек, находящих- ся в вершинах вписанного в поляну правильного треугольника? 9. См. задачу 13 при я = 1958. 10*. Проекции многоугольника на ось ОХ, биссектрису первого и третьего координатных углов, ось OY и биссектрису второго и четвертого координатных углов равны соответственно 4, 3^/2, 5, 4-у/2. Площадь многоугольника равна S. Доказать, что S<17,5. 62
IX класс 11. Бесконечная плоская ломаная АоА। ... Ап ... , все углы кото- рой прямые, начинается в точке Ло с координатами л = 0, у=1 и обходит начало координат О по часовой стрелке. Первое звено ломаной имеет длину 2 и параллельно биссектрисе четвер- того координатного угла. Каждое из следующих звеньев пересе- кает одну из координатных осей и имеет наименьшую возможную при этом целочисленную длину. Расстояние ОАп обозначается через гЛ, сумма длин первых п звеньев ломаной — через Sn. Sn Доказать, что найдется п, для которого —> 1958. ГЛ 12. Какое наибольшее число осей симметрии может иметь пространственная фигура, состоящая из трех прямых, из которых никакие две не параллельны и не совпадают? 13. Решить в целых положительных числах уравнение 14. Отрезок длины Зп разбивается на три равные части. Первая и третья из них называются отмеченными. Каждый из отмеченных отрезков разбивается на три равные части, из которых первая и третья снова называются отмеченными, и т. д. до тех пор, пока не получатся отрезки длины 1. Концы всех отме- ченных отрезков называются отмеченными точками. Доказать, что для любого целого k можно найти две отмеченные точки, расстояние между которыми равно fe(l^fe^3n). X класс 15*. См. условие задачи 10. Доказать, что 10. 16. Доказать, что 1155,9584-34,958=Лп2 ни для какого целого п. 17. См. задачу 12. 18. На листе бумаги нарисован правильный 100-угольник, в вершинах которого по порядку написаны 1, 2, ..., 100. Затем эти числа переписывают в порядке удаления от переднего края листа. Если вершины находятся на равном расстоянии от края, сначала выписывается левое число, затем правое. Выписаны всевозможные различные наборы чисел, соответству- ющие разным положениям правильного 100-угольника на листе. Вычислить сумму чисел, стоящих во всех этих наборах на три- надцатом месте слева. 63
19*. Из четырех прямых на плоскости никакие две не парал- лельны, никакие три не пересекаются в одной точке. По каждой прямой с постоянной скоростью идет пешеход. Известно, что первый пешеход встречается со вторым, третьим и четвертым, а второй — с третьим и четвертым. Доказать, что третий пешеход встречается с четвертым. 2-й тур VII класс 20. Доказать, что на плоскости нельзя расположить больше четырех выпуклых многоугольников так, чтобы каждые два из них имели общую сторону. 21. Имеются два набора из +1 и —1, в каждом по 1958 чисел. Доказать, что за некоторое число шагов можно превратить первый набор во второй, если на каждом шаге разрешается одновременно изменить знак у любых 11 чисел пер- вого набора. (Два набора считаются одинаковыми, если у них на одинаковых местах стоят одинаковые числа.) 22. Каждая грань куба заклеивается двумя равными прямо- угольными треугольниками с общей гипотенузой, один из которых белый, другой черный. Можно ли эти треугольники располо- жить так, чтобы при каждой вершине куба сумма белых углов была равна сумме черных углов? 23. Доказать, что если целое п>2, то (п!)2>п". 24. Сторона клетки клетчатой бумаги равна I. По линиям сетки построен прямоугольник размером тХи. Можно ли в прямоуголь- нике провести по линиям сетки ломаную, которая ровно один раз проходила бы через каждый узел сетки, расположен- ный внутри или на границе прямоугольника? Если можно, то како- ва ее длина? VIII класс 25. Из бумаги вырезан многоугольник. Через две точки его границы проводится прямая, по которой многоугольник перегиба- ется. Доказать, что периметр многоугольника, получающегося по- сле перегибания, не больше периметра исходного многоугольника. 26. Доказать, что для любых чисел а\ и а2, таких, что 0,^0. 02^0, а,+о2=1, можно найти такие числа Ь\ и 62, что 6,>0, 62>0, 6,4-62=1 и (1,25-0,)6,+3(1,25-о2)&2>1. 27. Внутри /LAOB взята точка С. Из нее опущены перпен- дикуляры: CD — на сторону ОД, СЕ — на сторону ОВ. Из точек D и Е опущены перпендикуляры: DN — на сторону ОВ, ЕМ — на сторону ОД. Доказать, что OC.LMN. 28. Доказать, что если целое п>\. то л(/1+ I) 1'-22-33- 2 . 64
29. Обозначим через а наибольшее число непересекающихся кругов диаметра I, центры которых лежат внутри многоуголь- ника М, через Ь — наименьшее число кругов радиуса I, которыми можно покрыть весь многоугольник М. Какое число больше- fl или 6? IX класс 30. См. задачу 35. 31. Из точки О провести п лучей на плоскости так, чтобы сумма всех попарных углов между ними была наибольшей. 32. Игральная доска имеет форму ромба с углом 60°. Каждая сторона ромба разделена на 9 частей. Через точки деления про- ведены прямые, параллельные сторонам и малой диагонали ромба, разбивающие доску на треугольные клетки. Если на некоторой клетке поставлена фишка, проведем через центр этой клетки три прямые, параллельные сторонам и малой диагонали ромба. Клетки, которые они пересекут, будут считаться побитыми фишкой. Каким наименьшим числом фишек можно побить все клетки доски? 33. Обозначим через а наименьшее число кругов радиуса 1, которыми можно полностью покрыть заданный многоугольник М, через b — наибольшее число непересекающихся кругов радиуса 1 с центрами внутри многоугольника М. Что больше: а или 6? 34. Между зажимами А и В включено несколько сопротивле- ний. Каждое сопротивление имеет входной и выходной зажимы. Какое наименьшее число сопротивлений необходимо иметь и како- ва должна быть схема их соединения, чтобы при порче любых 9 сопротивлений между зажимами А и В цепь осталась замкнутой, но не было короткого замыкания? (Порча сопротивле- ния: короткое замыкание или обрыв.) X класс 35. Решить в целых положительных числах уравнение Х^ + (Х_|_|)2У = (Х + 2)Ч 36. В многоугольнике существуют такие точки А и В, что любая соединяющая их ломаная, проходящая внутри или по границе многоугольника, имеет длину больше I. Доказать, что периметр многоугольника больше 2. 37*. В школе изучают 2л предметов. Все ученики учатся на «4» и «5». Никакие два ученика не учатся одинаково, ни про каких двух нельзя сказать, что один из них учится лучше другого. Доказать, что число учеников в школе не больше чем С$„. Примечание. Мы считаем, что один ученик учится лучше другого, если у него по всем предметам не хуже, чем у второго ученика, а по некоторым предметам — лучше. 3 Заказ 247 65
38. Стороны параллелограмма равны а и Ь. Найти отношение объемов тел, полученных при вращении параллелограмма во- круг стороны а и вокруг стороны Ь. 39. На п карточках написаны с разных сторон числа: на первой 0 и 1, на второй 1 и 2, на пи числа п — 1 и п. Одни человек берет из стопки несколько карточек и пока- зывает одну сторону. Указать все случаи, в которых второй может определить число, написанное на обороте последней показанной ему карточки. XXII ОЛИМПИАДА (19S9 г.) 1-й тур VII класс 1. Пусть а и b — натуральные числа. Напишем число b справа от числа а. Если число а четное, то разделим его на 2, если оно нечетное, то сначала вычтем из него единицу, а потом разделим его на 2. Получившееся число 0| напишем под числом а. Справа от числа 0j напишем число 2b. С числом ai проделаем ту же операцию, что и с числом а, и, получив число а2» напишем его под числом Оь Справа от числа 02 за- пишем число 46. Получив аналогичным образом число 0з, напи- шем справа от него число 86, и т. д. Этот процесс про- должаем до тех пор, пока не получим в левом столбце число I. Доказать, что сумма тех чисел правого столбца, слева от которых стоят нечетные числа, равна произведению ab. 2. Доказать, что число 2 — I делится на 3. 3*. Можно ли расположить все трехзиачные числа, не окан- чивающиеся нулями, в последовательность так, чтобы последняя цифра каждого числа была равна первой цифре следующего за ним? 4. Как должна двигаться ладья по шахматной доске, чтобы побывать на каждом поле по одному разу и сделать наимень- шее число поворотов? 5. Дан квадрат со стороной 1. Найти множество всех точек, сумма расстояний от которых до сторон этого квадрата или их продолжений равна 4. VIII класс 6. Даны две бочки бесконечно большой емкости. Можно ли, пользуясь двумя ковшами емкостью (2 — \/2) л и \/2 л, перелить из одной в другую ровно 1 л? 7. Заметим, что если перевернуть лист, на котором написаны цифры, то цифры 0, 1, 8 не изменятся, 6 и 9 поменяются местами, остальные потеряют смысл. Сколько существует девятн- 66
значных чисел, которые при переворачивании листа нс изме- няются? 8. Дан выпуклый четырехугольник A BCD. Середины сторон АВ и CD обозначим соответственно через К и Л1, точку пересечения отрезков AM и DK — через О, точку пересече- ния отрезков ВМ и СК — через Р. Доказать, что площадь четырехугольника МОКР равна сумме площадей треугольников В PC и AOD. 9. См. задачу 4. 10. Даны две непересекающиеся окружности с центрами в точках О| и О2. Пусть at и аг — внутренние касательные к этим окружностям, а3 и а4 — внешние касательные к ним. Пусть, далее, а5 и — касательные к окружности с центром в Oi, проведенные из точки О2, я7, а* — касательные к окружности с центром в точке О2, проведенные из точки О|. Обозначим через О точку пересечения прямых С| и а2. Доказать, что можно провести две окружности с центром в точке О так, чтобы первая касалась а3 и а4, вторая касалась а5, а6, а7, 08, причем радиус второй в два раза меньше радиуса первой. IX класс 11. Имеется 1959 положительных чисел: а2, ...» а то, сум- ма которых равна 1. Рассматриваются всевозможные комби- нации из 1000 чисел, причем комбинации считаются совпадающи- ми, если они отличаются только порядком чисел. Для каждой комбинации рассматривается произведение входящих в нее чисел. Доказать, что сумма всех этих произведений меньше 1. 12. См. задачу 7. 13. Построить окружность, проходящую через две данные точки и отсекающую от данной окружности хорду данной длины. 14. Рассмотрим лист клетчатой бумаги со стороной клетки, равной 1. Пусть pk — число всех ломаных линий длины Л. начи- нающихся в точке О — некотором фиксированном узле сетки, причем все ломаные составлены из звеньев сетки. Доказать, что для любого k справедливо неравенство р*<2-3*. 15* . Доказать, что не существует тетраэдра, в котором каждое ребро являлось бы стороной плоского тупого угла. X класс 16. Доказать, что не существует таких целых чисел х, у, г. что +</’ = £* при условии z>0, 0<Zx<Zk9 0<Zy<Zk, k — нату- ральное число. 17. См. задачу 8. 18. Существует ли тетраэдр, каждое ребро которого являлось бы стороной плоского тупого угла? (Ср. с задачей 15.) 3* 67
19. В квадратную таблицу WXW записаны все целые числа от 1 до № по следующему закону: I стоит на любом месте; 2 стоит в строке с номером, равным номеру столбца, содержащего I; 3 стоит в строке с номером, равным номеру столбца, содержащего 2, и т. д. На сколько сумма чисел в строке, содержащей 1, отличается от суммы чисел в столбце, содержащем №? 20. Дана невозрастающая последовательность положительных чисел: 0|>а2>а3> ... >ort> ...; at=^, at + a2 + ... 4-ал-|-.•.= = 1. Доказать, что найдутся k чисел, из которых самое малень- кое больше половины самого большого. 2-й т у р VII класс 21. Имеется два набора чисел: at>a2> ...>ап и b\>b2> >...>Ьп. Доказать, что а\Ь। + а2Ь24- ... -V anbriz> aibn-V a2bn — 14- ... 4"О«6ь 22. Дан треугольник АВС. Найти такую точку, что ее образ при осевой симметрии относительно прямой, содержащей любую сторону треугольника, лежит на описанной окружности. 23. На какое целое число надо умножить 999 999 999, чтобы получить число, состоящее из одних единиц? 24. Доказать, что в любом шестизначном числе можно пере- ставить цифры так, чтобы сумма первых трех цифр нового числа отличалась от суммы вторых трех цифр меньше чем на 10. 25. Дано п чисел: Л|, х2,...» хп, при этом Xk = ± 1, Л = 1,2,... , п. Доказать, что если Х1Х2 + х2Хз + ... + xn_tXn + xnxi=0, то п де- лится на 4. VIII класс 26. См. задачу 25. 27. Даны 12 чисел а2» о3, ... , О|2, причем имеют место следующие неравенства: ( a2(ai — о24-о3)<0, I о3 (о2 —а34-о4)<0, О|| (О|0 — О| I 4" 0|2)<СО. Доказать, что среди этих чисел найдется по крайней мере 3 положительных и 3 отрицательных. 28. Дан треугольник АВС; Оь О2, Оз — его вневписанные окружности. Для каждой пары из этих окружностей построим вторую общую внешнюю касательную (одна такая касательная уже проведена — это продолжение стороны ДЛВС). Три по- 68
строенные прямые образуют треугольник. Найти его углы, если углы треугольника АВС известны. 29. Даны два пересекающихся отрезка АВ и CD длины 1. Доказать, что по крайней мере одна из сторон четырехуголь- ника ABCD не меньше 30*. Доказать, что шахматную доску размером 4X4 нель- зя обойти ходом шахматного коня, побывав на каждом поле ровно один раз. IX класс 31. Даны такие сто чисел Xi, х2, ...» Хюо, сумма которых равна 1, что каждая из абсолютных величин разностей хь+1— Xk меньше Доказать, что из этих 100 чисел можно выбрать 50 чисел так, чтобы их сумма отличалась от 4- не больше чем на . Z 1 ии 32. п отрезков длины 1 пересекаются в одной точке. Доказать, что хотя бы одна сторона 2и-угольника с вершинами в концах этих отрезков не меньше стороны правильного 2и-угольника, вписанного в окружность диаметра 1. 33*. Доказать, что не более одной вершины тетраэдра об- ладает тем свойством, что сумма любых двух плоских углов при этой вершине больше 180°. 34. Доказать, что существует бесконечно много чисел, не представимых в виде суммы трех кубов. 35. В углах шахматной доски размером 3x3 стоят кони: в верхних углах белые, а в нижних — черные. Одним ходом разрешается переставить любого коня на любое свободное поле в соответствии с правилами шахмат. Мы хотим поставить белых коней в нижние углы доски, а черных коней в верхние углы. Доказать, что для этого потребуется не меньше 16 ходов. X класс 36. См. задачу 34. 37. Пусть ABCD — пространственный четырехугольник, точки Ki и К? делят соответственно стороны А В и CD в отношении а, точки Кз и делят соответственно стороны ВС и AD в отношении р. Доказать, что отрезки К1К2 и КзК* пересекаются. 38*. На плоскости дано несколько пересекающихся кругов, объединение которых имеет площадь, равную 1. Доказать, что из них можно выбрать некоторое количество попарно неперекры- вающихся, чтобы сумма их площадей была не менее 69
39*. Дано п комплексных чисел ci, С2» ..., сп таких, что если их представить себе как точки плоскости, то они являются вершинами выпуклого n-угольника. Доказать, что если комплексное число z обладает тем свойством, что то точка плоскости, соответствующая z, лежит внутри этого п-угольника. 40. Два диска разного диаметра разделены на 2л равных сектора каждый, и полученные при этом секторы выкрашены в бе- лый и черный цвет таким образом, что на каждом диске оказалось п белых и л черных секторов. Если укрепить оба диска на одной осн, проходящей через их центры, то окажется, что окружность — край меньшего диска — окрашена дважды: изнутри и снаружи. При этом одни части окружности окрашены в разные цвета, а остальные — в один цвет с обеих сторон. Доказать, что можно так повернуть меныпий диск, что части первого рода составят не меньше половины длины окружности. XXIII ОЛИМПИАДА (1960 г.) 1-й т у р VII класс 1. Указать все денежные суммы, выраженные целым числом рублей, которые могут быть представлены как четным, так и нечетным числом денежных билетов. Примечание. Считать, что в обращении имеются билеты достоинством в I, 3, 5, 10, 25, 50 и 100 р. 2. Три равные окружности с центрами Oi, О2. Оз пере- секаются в данной точке. Д|, Дг, Дз— остальные точки пересе- чения. Доказать, что треугольники O1O2O3 и Д1Д2Д3 равны. 3. В составлении 40 задач приняло участие 30 студентов со всех 5 курсов. Любые два однокурсника придумали одинаковое число задач. Любые два студента с разных курсов придумали разное число задач. Сколько человек придумало по одной задаче? 4. М и N — точки пересечения двух окружностей с центра- ми О| и Ог. Прямая О|Л4 пересекает первую окружность в точке Д|, а вторую—в точке Д2. Прямая О2М пересекает первую окружность в точке В|, а вторую окружность — в точ- ке Д2. Доказать, что прямые A1B1, A2B2t MN пересекаются в одной точке. 5. Доказать, что число делителей числа п не превосходит 2у[п. 70
VIII класс 6. Доказать, что число, в десятичной записи которого имеется триста единиц, а остальные цифры — нули, не является точным квадратом. 7. В турнире каждый шахматист половину всех очков набрал во встречах с участниками, занявшими три последних места. Сколько человек принимало участие в турнире? 8. Через данную вершину А выпуклого четырехугольника ABCD провести прямую, делящую его площадь пополам. 9. Даны отрезки ЛВ, CD и точка О, причем никакие три из точек А, В, С, D, О не лежат на одной прямой. Конец отрезка называется «отмеченным», если прямая, проходящая через него и точку О, не пересекает другого отрезка. Сколько может быть «отмеченных» концов? 10*. Доказать, что существует бесконечно много натуральных чисел, не представимых в виде ни при каких простых р и натуральных п и k. IX класс 11. Доказать, что любая правильная дробь может быть пред- ставлена в виде конечной суммы обратных величин попарно различных целых чисел. 12. См. задачу 10. 13. Даны выпуклый многоугольник и точка О внутри его. Любая прямая, проходящая через точку О, делит площадь много- угольника пополам. Доказать, что многоугольник — центрально- симметричный и О — центр симметрии. 14. Найти множество четвертых вершин прямоугольников, у которых две вершины лежат на данной окружности, а третья — в данной точке внутри окружности. X класс 15. Два равных правильных треугольника расположены в про- странстве в параллельных плоскостях Pi и Рг, причем от- резок, соединяющий их центры, перпендикулярен этим плоскостям. Найти множество точек, являющихся серединами всех отрезков, соединяющих точки одного треугольника с точками другого треугольника. 16. Числа а, Ь, п натуральные. Доказать, что если ап+Ьп и । t ап4-6" а + b одновременно делятся на п, то в случае, если оно целое, также делится на п. 17. См. задачу 14. 18. В десятичной записи целого числа А все цифры, кроме первой и последней,— нули, первая и последняя — не нули, число 71
цифр — не меньше трех. Доказать, что А не является точным квадратом. 19. Даны числа сц, а2, аз, ... , а*, причем для всех нечетных п имеет место равенство ai-f-aS-f- ... 4-aj?=O. Доказать, что те из чисел он, £2, , а*, которые не равны нулю, можно разбить на пары таким образом, чтобы два числа, входящих в одну и ту же пару, были бы равны по абсолютной величине, но противоположны по знаку. 2-й т у р VII класс 20. Даны четыре точки А, В, С, D. Найти такую точку О, что сумма расстояний от нее до данных точек минимальна. 21. Доказать, что стороны произвольного четырехугольника являются сторонами некоторой трапеции. 22. Доказать, что любой несамопересекающийся пятиугольник лежит по одну сторону хотя бы от одной из своих сторон. 23. В каком-то году некоторое число ни в одном месяце не было воскресеньем. Найти это число. VIII класс 24. Каково наибольшее п, при котором можно так располо- жить п точек на плоскости, чтобы каждые три из них служили вершинами прямоугольного треугольника? 25. Имеется бесконечная шахматная доска. Обозначим через (а, Ь) поле, расположенное на пересечении горизонтали с номером а и вертикали с номером Ь. Фишка с поля (а, Ь) может сделать ход на любое из восьми полей: (а±/п, Ь±п), (а±п, Ь±:т\ где т, п — фиксированные числа, а знаки ( + ) и ( —) комбини- руются произвольно. Сделав х ходов, фишка вернулась на исход- ное поле. Доказать, что х четно. 26. См. задачу 21. 27*. Улитка ползет вперед (не поворачивая назад) с не- постоянной скоростью. Несколько человек наблюдают за ней по очереди в течение 6 мин. Каждый начинает наблюдать раньше, чем кончает предыдущий, и наблюдает ровно 1 мин, причем замечает, что за эту минуту улитка проползла ровно 1 м. Дока- зать, что за 6 мин улитка сможет проползти самое большее 10 м. 28. Дан пятиугольник ABCDE, в котором AB = BC = CD = DE, Z_B = Z_D = 9()°. Доказать, что пятиугольниками, равными данно- му, можно покрыть плоскость (без щелей и перекрытий). 72
IX класс 29. Имеется т точек, некоторые из которых соединены от- резками так, что каждая соединена с / точками. Какие значе- ния может принимать /? 30. Дан произвольный центрально-симметричный шестиуголь- ник. На его сторонах как на основаниях построены во внешнюю сторону правильные треугольники. Доказать, что середины отрез- ков, соединяющих вершины соседних треугольников, являются вершинами правильного шестиугольника. 31. Доказать, что никакую прямоугольную шахматную доску шириной 4 клетки нельзя обойти ходом шахматного коня, побы- вав на каждом поле по одному разу и последним ходом вернувшись на исходную клетку. 32. Найти множество центров симметрии всех прямоугольни- ков, описанных около данного остроугольного треугольника. 33*. В квадрате со стороной 100 расположено N кругов радиуса 1, причем всякий отрезок длины 10, целиком располо- женный внутри квадрата, пересекает хотя бы один круг. Доказать, что Л^400. X класс 34. Число А делится на 1, 2, 3, ... , 9. Доказать, что если 2А представлено в виде суммы натуральных чисел, меньших 10, т. е. 2Л = а| + а2+ ... + то из чисел а2, ... , ak можно выбрать несколько таких, что их сумма равна Л. 35. бп-значное число делится на 7. Последнюю цифру пере- несли в начало. Доказать, что полученное число также делится на 7. 36. Собралось п человек. Некоторые из них знакомы между собой, причем каждые два незнакомых имеют ровно двух общих знакомых, а каждые два знакомых не имеют общих знакомых. Доказать, что каждый из присутствующих знаком с одинаковым числом человек. 37. См. задачу 32. 38. Улитка должна проползти вдоль линий клетчатой бумаги путь длины 2п, начав и кончив свой путь в данном узле. Доказать, что число различных ее маршрутов равно (С2л)2. XXIV ОЛИМПИАДА (1961 г.) 1-й тур VII класс 1. См. задачу 13 при четном п. __ ____________________ 2. Имеется трехзначное число abc, возьмем число cba и вычтем из большего меньшее. Получим число aibiCi, сделаем 73
с ним то же самое и т. д. (случай flj=0 допускается). До- казать, что на каком-то шаге мы получим или число 495, или 0. 3. Дан остроугольный треугольник AqBoCq. Пусть точки Bi, Ci — центры квадратов, построенных на сторонах BqCq, CqAq, AgBq. С полученным треугольником Л|В|С| делаем то же самое и получаем треугольник А2В2С2 и т. д. Доказать, что гра- ницы треугольников АпВпСп и Лп+|Вп+|Сп+| пересекаются ровно в шести точках. 4. Имеется 100 точек на плоскости, причем расстояние между любыми двумя из них не превосходит 1, и если Л, В, С — любые три точки из данных, то треугольник АВС тупоугольный. Доказать, что можно провести такую окружность радиуса 0,5, что все данные точки окажутся внутри ее или на ней самой. 5. На шахматной доске выбраны две клетки одинакового цвета. Доказать, что ладья, начав с первой из этих клеток, может обойти все клетки по разу, а на второй выбранной клетке побывать два раза. VIII класс 6. Даны треугольник АВС и точка О. Обозначим через Afj, М2, Мз центры тяжести треугольников ОАВ, ОВС, ОСА соответ- ственно. Доказать, что площадь треугольника М|М2М3 равна -i- площади треугольника АВС. 7. Играют двое, один из них загадывает набор из целых однозначных чисел х2» ... , хП9 как положительных, так и отрицательных. Второму разрешается спрашивать, чему равна сумма GiXi4-а2х2+ ... + апхп, где ai, а2, ..., ап — любой набор чисел. Каково наименьшее число вопросов, за которое отгады- вающий может узнать задуманный набор? 8. См. задачу 3. 9. Доказать, что ладья может обойти все клетки прямоуголь- ной шахматной доски, побывав на каждой клетке ровно один раз, и вернуться в начальную клетку только в случае, если число клеток на доске четно. (Ср. с задачей 5.) 10. Набор чисел а, а-|-1, а + 2, ..., a + k называется отрезком натурального ряда. Два отрезка натурального ряда, каждый длины 1961, подписаны один под другим. Доказать, что можно так переставить числа в каждом из отрезков, что после сложения чисел, стоящих друг под другом, снова получится отре- зок натурального ряда. IX класс 11. См. задачу 6. 12. См. задачу 7. 13. Доказать, что можно так расположить числа от 1 до п2 74
в таблице пХл, чтобы суммы чисел каждого столбца были равны. 14. 'См. задачу 18 в предположении, что k делится на 4. 15. На плоскости расположены п точек таких, что если Л, В, С — любые три из них, то внутри треугольника АВС нет ни одной точки из данных. Доказать, что эти точки можно зануме- ровать так, что многоугольник Л1Л2Л3... Ап будет выпуклым. X класс 16. Дана последовательность Uk чисел Фибоначчи 1, 1, 2, 3, 5, ... . Доказать, что иы делится на 5 при всех Л= 1, 2, 3, ... . 17. На плоскости проведено несколько полос разной ширины. Никакие две из них не параллельны. Как нужно параллельно перенести каждую из этих полос, чтобы площадь их общей части была наибольшей? 18. k человек ехали в автобусе без кондуктора. Известно, что ни у кого из пассажиров не было медных денег и монет крупнее 20 коп. Известно далее, что каждый пассажир уплатил за проезд и получил сдачу. Доказать, что наименьшее число монет, которое могло для этого потребоваться, равно . 19. Окружность S и точка О лежат в одной плоскости, причем О находится вне окружности. Построим произвольную сферу, проходящую через окружность S, и опишем конус с вершиной в точке О, касающийся этой сферы. Найти множество центров всех окружностей, по которым конусы касаются сфер. 20. Известно, что Z|-f-z2+ — -|-zrt = 0, где zt — комплексные числа. Доказать, что среди этих чисел найдутся два таких числа, что разность их аргументов больше или равна 120°. 2-й т у р VII класс 21. Стороны произвольного выпуклого многоугольника покра- шены снаружи. Проводим несколько диагоналей многоугольника. Каждая из этих диагоналей тоже покрашена с одной стороны, т. е. с одной стороны отрезка проведена узкая цветная полоска. Доказать, что хотя бы один из многоугольников, на которые разбит диагоналями исходный многоугольник, весь покрашен сна- ружи. (Допускается, чтобы в вершине многоугольника окраска заходила внутрь.) 22. В квадрате ABCD на сторонах ЛВ, ВС, CD и AD взяты точки Р, Q, Р, S соответственно; оказалось, что фигура PQRS — прямоугольник. Доказать, что тогда PQRS — либо квад- рат, либо обладает тем свойством, что его стороны параллельны диагоналям квадрата. 75
)И------- -----— 23. Доказать, что среди любых 39 по- r j следовательных натуральных чисел обя- зательно найдется такое, у которого • ' * * сумма цифр делится на 11. 24. Дана таблица размером 4X4. Рис 3 Показать, что можно так расставить семь звездочек в клетках этой таблицы, чтобы при вычеркивании любых двух строк и любых двух столбцов этой таблицы в остав- шихся клетках всегда была хотя бы одна звездочка. Доказать, что если звездочек меньше семи, то всегда можно так вычерк- нуть две строки и два столбца, что все оставшиеся клетки будут пустыми. 25. Доказать, что не существует целых чисел a, b, с, d, удовлетворяющих равенствам abed — а= 1961, abed — b = 961, abed —с = 61, abcd — d = 1. VIII класс 26. Дана фигура, состоящая из 16 отрезков (рис. 3). Дока- зать, что нельзя провести ломаную, пересекающую каждый из отрезков ровно один раз. «Ломаная может быть незамкнутой и самопересекающейся, но ее вершины не должны лежать на отрезках, а стороны не могут проходить через общие концы отрезков. 27*. Диагональ прямоугольника равна d. Из его вершин как из центров описаны четыре окружности радиусами и. Го, г3, г4, причем Г|4-Г2 = ^з + п<</. Проведены две пары внешних каса- тельных к окружностям с номерами 1, 3 и 2, 4. Доказать, что в четырехугольник, образованный этими четырьмя прямыми, мож- но вписать окружность. 28. У каждого из целых чисел k и £-|-/ сумма цифр делится на 11, и между ними нет числа, обладающего тем же свойством. Какое наибольшее значение может принимать /? 29. См. задачу 24. 30. Дана четверка чисел a, b, с, d. Из нее получается новая ab, be. cd, da умножением каждого числа на следующее, а четвертого — на первое. Из второй четверки по этому же принци- пу получается третья и т. д. Доказать, что среди полученных четверок никогда не встретится вновь четверка а, 6, с, d, кроме случая, когда a = b = c = d=l. IX класс 31. Точки А \\ В движутся равномерно и с равными угловыми скоростями по окружностям О| и Ог соответственно 76
(по часовой стрелке). Доказать, что вершина С правильного ДЛВС также движется равномерно по некоторой окружности. 32. В клетки таблицы /пХл вписаны некоторые числа. Разрешается одновременно менять знак у всех чисел некоторого столбца или некоторой строки. Доказать, что, применяя несколь- ко раз эту операцию, можно превратить данную таблицу в такую, у которой суммы чисел, стоящих в любом столбце и в любой строке, неотрицательны. 33. п точек соединены отрезками так, что каждая точка соеди- нена с любой другой некоторым путем и нет таких двух точек, которые соединялись бы двумя разными путями. Доказать, что общее число отрезков равно п — 1. 34* . а, Ь. р — любые натуральные числа. Доказать, что най- дутся такие взаимно простые числа Л, Z, что ak-\-bl делится на р. 35. Коля и Петя делят 2п+1 орехов, п^2, причем каждый хочет получить возможно больше. Предлагается три способа деления. Каждый проходит в три этапа, причем 1-й и 2-й этапы общие для всех трех способов. 1-й этап: Петя делит все орехи на две части, в каждой не меньше двух орехов. 2-й этап: Коля делит каждую часть снова на две, в каждой не меньше одного ореха. 3-й этап: при первом способе Коля берет наибольшую и наименьшую части. При втором способе Коля берет себе обе средние части. При третьем способе Коля берет либо большую и меньшую части, либо средние части, но за право выбора отдает Пете один орех. Определить, какой способ самый выгодный для Коли и какой наименее выгоден для него. X класс 36. Доказать, что для любых трех бесконечных последова- тельностей натуральных чисел ah ...» ап, ...; Ь\. Ь?. .... Ьп, ...; Ci, С2, ...» сп, ... найдутся такие номера р и q. что ap^aq. Ьр bq. Ср Cq. 37. В прямоугольник со сторонами 20 и 25 бросают 120 квадратов со стороной 1. Доказать, что в прямоугольник можно поместить круг диаметра 1, не пересекающийся ни с одним из квадратов. 38. См. задачу 32. 39. Расстояния от фиксированной точки Р плоскости до двух вершин Л, В равностороннего треугольника АВС равны ЛР = 2, ВР=3. Определить, какое максимальное значение может иметь длина отрезка PC. 40. Дан произвольный набор из 4-1 и —1 длиной 2*. Из него получается новый набор по правилу: каждое число умно- жается на следующее за ним; последнее, 2*-е число, умножается на первое. С новым набором из 4" 1 и — 1 проделывается то же самое и т. д. Доказать, что в конце концов получится набор, состоящий из одних единиц. 77
XXV ОЛИМПИАДА (1962 г.) 1-й т у р VII класс 1. Дана прямая /, перпендикулярная отрезку АВ и пересе- кающая его. Для любой точки М прямой / строится такая точка /V, что Z_NAB = 2Z_MAB; Z_NBA=2Z_MBA. Доказать, что модуль разности AN — BN не зависит от выбора точки М на прямой /. 2. Правильный треугольник, одна сторона которого отмечена, отражается симметрично относительно одной из своих сторон. Полученный треугольник в свою очередь отражается аналогично и т. д., пока на некотором шаге треугольник пе придет в первона- чальное положение. Доказать, что при этом отмеченная сторона также займет исходное положение. 3. Пусть a, b,c,d — стороны четырехугольника, не являющего- ся ромбом. Доказать, что из отрезков а, 6, с, d можно сло- жить самопересекающийся четырехугольник. 4. Сумму цифр числа а обозначим через s(a). Доказать, что если s(a)=s(2a), то число а делится на 9. 5. Даны п карточек; на обеих сторонах каждой карточки написано по одному из чисел 1, 2, ..., п, причем так, что каждое число встречается на всех п карточках ровно два раза. Доказать, что карточки можно разложить на столе так, что сверху окажутся все числа: 1, 2, ..., /г. VIII класс 6. На сторонах АВ, ВС, СА правильного треугольника АВС найти такие точки X, Y, Z (соответственно), чтобы площадь треугольника, образованного прямыми СХ, BZ, AY, была вчетверо меньше площади треугольника АВС и чтобы было выполнено условие = 7. См. задачу 2. 8. Доказать, что для любого целого d найдутся такие • п — 2т + 1 целые т, п, что d=---?—5—. т~ — п 9. См. задачу 4. 10. См. задачу 5. IX класс 11. Даны два пересекающихся отрезка АВ и CD. На них выбираются точки М и N (соответственно) так, что AM = CN. Найти положение точек М и N, при котором длина отрезка MN минимальна. 78
12. Конем называется фигура, ход которой состоит в перемеще- нии на п клеток по горизонтали и на 1 по вертикали (или наоборот). Конь стоит на некотором поле бесконечной шахмат* пой доски. При каких п он может попасть на любое задан- ное поле? При каких п это невозможно? 13. См. задачу 4. 14. Дана система уравнений г Х\Х2Хз ... *1962 = I, *1—Х2Х3 ... *1962= 1 » ◄ *1*2— Х3Х4 ... *1962= 1» *1*2*3 ... *1961 —*1962= 1 . Какие значения может принимать x2s? 15. Доказать, что в прямоугольнике площади 1 можно рас- положить непересекающиеся круги так, чтобы сумма их радиу- сов была равна 1962. X класс 16. В задаче 11 заменить отрезки пересекающимися лучами. 17. На сторонах квадрата как на основаниях построены во внешнюю сторону равные равнобедренные треугольники с острым углом при вершине. Доказать, что получившуюся фигуру нельзя разбить на параллелограммы. 18. Доказать, что любое натуральное число можно предста- вить в виде суммы нескольких различных членов последователь- ности Фибоначчи 1, 2, 3, 5, 8, 13, ... . 19. См. задачу 14. 20. См. задачу 15. 2-й т у р VII класс 21. У края бильярда, имеющего форму правильного 2л-уголь- ника, стоит шар. Как надо пустить шар от борта, чтобы он, отразившись от всех бортов, вернулся в ту же точку? Доказать, что при этом длина пути шара не зависит от выбора началь- ной точки. 22*. Дан равнобедренный треугольник АВС, в котором ЛВ=ВС, BII — высота, М — середина стороны АВ, К — точка пересечения ВН с окружностью, проходящей через В, М и С. Доказать, что ВК=-~-/?, где R — радиус описанной около треу- гольника АВС окружности. 23. Уголком называется фигура, составленная из трех квадра- тов со стороной 1 в виде буквы «Г». Доказать, что прямоуголь- 79
ник размером 1961 X 1963 нельзя разбить на уголки, а прямоуголь- ник размером 1963X1965 — можно. 24. Дано число 100 ... 01; число нулей в нем равно 1961. Доказать, что это число составное. 25. На плоскости даны 25 точек; известно, что из любых трех точек можно выбрать две, расстояние между которыми меньше I. Доказать, что среди данных точек найдутся 13, лежащих в круге радиуса 1. VIII класс 26. Проведем в выпуклом многоугольнике некоторые диаго- нали так, что никакие две из них не пересекаются (из одной вершины могут выходить несколько диагоналей). Доказать, что найдутся по крайней мере две вершины многоугольника, из кото- рых не проведено ни одной диагонали. 27. Как надо расположить числа 1, 2, ..., 1962 в после- довательности ai, а2, ... • Я|962, чтобы сумма |а|—О2| + |02— 031 4“ ... +|О|961—01962 I + 1^1962 — 011 была наибольшей? 28. В окружность вписан неправильный л-угольник, который при повороте окружности около центра на некоторый угол а=/=2л совмещается сам с собой. Доказать, что п — число составное. 29*. Из чисел хь х2, х3, х4, хе можно образовать десять попарных сумм; обозначим их через а2, ... , аю. Доказать, что, зная числа аь а2, ..., ою (но не зная, разумеется, суммой каких именно двух чисел является каждое из них), можно восстановить числа хь х2, ..., Х5. 30. Две окружности О| и О2 пересекаются в точках М и Р. Обозначим через МА хорду окружности Оь касающуюся окруж- ности 01 в точке М, а через МВ — хорду окружности О2, касающуюся окружности О\ в точке Af. На прямой МР отложен отрезок РН—МР. Доказать, что около четырехугольника МАНВ можно описать окружность. IX класс 31. Школьник в течение учебного года должен решать ровно по 25 задач за каждые идущие подряд 7 дней. Время, необхо- димое на решение одной задачи (любой), не меняется в течение дня, но меняется в течение учебного года по известному школьнику закону и всегда меньше 45 мин. Школьник хочет затратить на решение задач в общей сложности наименьшее время. Доказать, что для этого он может выбрать некоторый день недели и в этот день (каждую неделю) решать по 25 задач. 32. См. задачу 27, где вместо чисел 1, 2, ..., 1962 взяты 25 произвольных различных чисел. 80
33. Стороны выпуклого многоугольника, периметр которого равен 12, параллельно переносятся на расстояние d=l во внеш- нюю сторону и продолжаются до пересечения. Доказать, что пло- щадь многоугольника увеличится по крайней мере на 15. 34. См. задачу 29. 35. Даны 2п конечных последовательностей из нулей и единиц, причем ни одна из них не является началом никакой другой. Доказать, что сумма длин этих последовательностей не меньше п*2п. X класс 36. На данной прямой /, проходящей через центр О дан- ной окружности, фиксирована точка С. Точки А и Д' распо- ложены на окружности по одну сторону от / так, что углы, образованные прямыми АС и А'С с прямой /, равны. Обозначим через В точку пересечения ДД' и /. Доказать, что положение точки В не зависит от точки Д. 37. См. задачу 32. 38. См. задачу 33. 39. Как надо расположить в пространстве прямоугольный параллелепипед, чтобы площадь его проекции на горизонтальную плоскость была наибольшей? 40. В шахматном турнире каждый участник сыграл с каждым другим одну партию. Доказать, что участников можно так зануме- ровать, что окажется, что ни один участник не проиграл непо- средственно следующему за ним. XXVI ОЛИМПИАДА (1963 г.) 1-й т у р VII класс 1. Из вершины В произвольного треугольника АВС проведе- ны вне треугольника прямые ВМ и BN так, что Z-ABM = Z.CBN. Точки Д' и С' симметричны точкам Д и С относительно прямых ВМ и BN (соответственно). Доказать, что ДС' = Д'С. 2. а, Ь, с — такие три числа, что а + Ь-{-с=0. Доказать, что ab-Vbc-Vca^O. 3. Имеется 200 карточек размером 1X2, на каждой из кото- рых написаны числа 4-1 и —1. Можно ли так заполнить этими карточками лист клетчатой бумаги размером 4Х 100, чтобы произведения чисел в каждом столбце и в каждой строке образовавшейся таблицы были положительны? 4. См. задачу 9. 5. Можно ли так провести прямую на листе клетчатой бумаги размером 20X30, чтобы она пересекала 50 клеток? (Ср. с задачей 18.) 81
VIII класс 6. fli, аз, ... , an — такие числа, что ai4-tz24- ...4-a,t=0. Доказать, что в этом случае справедливо соотношение S =П|П2 -J-flifla + ... в сумму S входят все возможные произведения ад,, i^=j. 7. Даны выпуклый четырехугольник ABCD площади S и точка М внутри его. Точки Р, Q, Р, S симметричны точке М отно- сительно середин сторон четырехугольника ABCD. Найти площадь четырехугольника PQRS. 8. Решить в целых числах уравнение “'+““4_”"=3. 9. На плоскости даны семь прямых, никакие две из которых не параллельны. Доказать, что найдутся две из них такие, что угол между ними меньше 26°. 10. Лист клетчатой бумаги размером заполнен кар- точками размером 1X2. На каждой карточке написаны числа 4-1 и —1. Известно, что произведения чисел по строкам и столбцам образовавшейся таблицы положительны. При каких п это возможно? (Ср. с задачей 3.) IX класс 11. Первый член и разность арифметической прогрессии — целые числа. Доказать, что найдется такой член прогрессии, в записи которого встречается цифра 9. 12. См. задачу 10. 13. а, Ь, с — любые положительные числа. Доказать, что а I I е з 6 4 с *а4(’*«-|6хХ2 14. Из любых четырех точек на плоскости, никакие три из которых не лежат на одной прямой, можно так выбрать три, что треугольник с вершинами в этих точках имеет хотя бы один угол, не больший 45°. Доказать. 15. Можно ли в прямоугольник с отношением сторон 9:16 вписать прямоугольник с отношением сторон 4:7 так, чтобы на каждой стороне первого прямоугольника лежала вершина второго? X класс 16. См. задачу 11. 17. Из любых шести точек на плоскости (из которых ника- кие три нс лежат на одной прямой) можно так выбрать три, что треугольник с вершинами в этих точках имеет хотя бы один угол, не больший 30°. Доказать. (Ср. с задачей 14.) 82
18. Какое наибольшее число клеток может пересечь прямая, проведенная на листке клетчатой бумаги размером /пХн клеток? 19. а, Ь, с — такие три числа, что abc>0 и л + 6 + оО. Доказать, что ал-f-+ с" >0 при любом натуральном п. 20. Дан произвольный треугольник АВС. Найти множество всех таких точек Л4, что перпендикуляры к прямым AM, ВМ, СМ, проведенные из точек А, В, С соответственно, пересекаются в одной точке. XI класс 21. Положительные числа х, z обладают тем свойством, что arctg х-{-arctg //-{-arctg z< л. Доказать, что сумма этих чисел больше их произведения. 22. Дана система из 25 различных отрезков с общим началом в данной точке Лис концами на прямой /, не проходящей через эту точку. Доказать, что не существует замкнутой 25-звенной ломаной, для каждого звена которой нашелся бы отрезок системы, равный и параллельный этому звену. 23. См. задачу 20. 24. Из цифр 1, 2, 3, 4, 5, 6, 7 составляются всевозмож- ные семизначные числа, в записи которых каждая из этих цифр встречается ровно один раз. Доказать, что сумма всех таких чисел делится на 9. 25. Каждое ребро правильного тетраэдра разделено на три равные части. Через каждую полученную точку деления проведены две плоскости, параллельные соответственно двум граням тетра- эдра, не проходящим через эту точку. На сколько частей постро- енные плоскости разбивают тетраэдр? 2-й т у р VII класс 26. Завод выпускает погремушки в виде кольца с надетыми на него 3 красными и 7 синими шариками. Сколько различных погремушек может быть выпущено? (Две погремушки считаются одинаковыми, если одна из них может быть получена из другой только передвижением шариков по кольцу и переворачиванием.) 27. См. задачу 37. 28. Дан произвольный треугольник АВС и проведена та- кая прямая, пересекающая треугольник, что расстояние до нее от точки А равно сумме расстояний до этой прямой от точек В и С. Доказать, что все такие прямые проходят через одну точку. 29. Какое наибольшее количество чисел можно выбрать из набора 1, 2, ..., 1963, чтобы сумма двух любых выбранных чисел делилась на 26? 30. Система точек, соединенных отрезками, называется связ- ной, если из любой точки можно попасть в любую другую по этим 83
отрезкам. Можно ли соединить пять точек отрезками в связную систему так, чтобы при стирании любого отрезка образовались ровно две связные системы точек, не связанные друг с другом? (Мы считаем, что в местах пересечения отрезков переход с од- ного из них на другой невозможен.) VIII класс 31. tij, 6Z2, ап — произвольные натуральные числа. Обозна- чим через bk количество чисел из набора а2, ...» ап, удовлетво- ряющих условию ai^k. Доказать, что ai 4-Q24-...=bi 4-^2 + .... 32. В таблицу 8X8 вписаны все целые числа от 1 до 64. Доказать, что в ней найдутся два соседних числа, разность между которыми не меньше 5. (Соседними называются клетки, имеющие общую сторону.) 33. Найти множество центров тяжести всех остроугольных треугольников, вписанных в данную окружность. 34. Какое наибольшее количество чисел можно выбрать из набора 1, 2, ..., 1963, чтобы сумма никаких двух чисел не делилась на их разность? 35*. По аллее длиной 100 м идут три джентльмена со скоро- стями 1, 2, 3 км/ч. Дойдя до конца аллеи, каждый из них пово- рачивает и идет назад с той же скоростью. Доказать, что найдется отрезок времени в 1 мин, когда все трое будут идти в одном направлении. IX класс 36*. Даны произвольный треугольник АВС и точка X вне его. AM, BN, CQ — медианы треугольника АВС. Доказать, что площадь одного из треугольников ХАМ, XBN, XCQ равна сумме площадей двух других. 37. Какое наибольшее число точек самопересечения может иметь замкнутая 14-звенная ломаная, проходящая по линиям клетчатой бумаги так, что ни на какой линии не лежит более одного звена ломаной? 38. В правильном 10-угольнике проведены все диагонали. Сколько попарно неподобных треугольников при этом образова- лось? 39. В таблицу 9X9 вписаны все целые числа от 1 до 81. Доказать, что найдутся два соседних числа, разность между ко- торыми не меньше 6. 40. См. задачу 35. 84
X класс 41. Доказать, что при нечетном п уравнение xn-\-yn = zn не может иметь решений в целых числах, если (х-|-у)— простое число. 42. На листе бумаги нанесена сетка из п горизонтальных и п вертикальных прямых. Сколько различных 2/г-звенных замкну- тых ломаных линий можно провести по линиям сетки так, чтобы каждая ломаная проходила по всем горизонтальным и всем вер- тикальным прямым? 43. Из центра правильного 25-угольника отложены векторы во все его вершины. Как надо выбрать несколько векторов из этих 25-ти, чтобы их сумма имела наибольшую длину? 44. Д', В', С', £)', Е' — середины сторон выпуклого пяти- угольника ABCDE. Доказать, что площади пятиугольников ABCDE и A'B'C'D'E' связаны соотношением 25л'е с*О'£' SABCDE- 45* . Последовательность чисел а>, «2, ... образуется по закону а|=а2=1; ап — (л 3). Доказать, что все Оп - 2 числа в последовательности целые. XI класс 46. Доказать, что не существует попарно различных нату- ральных чисел х, у, z, /, для которых было бы справедливо соотно- шение xx + yy = zz + tl. 47. Доказать, что из одиннадцати произвольных бесконечных десятичных дробей можно выбрать две дроби, разность которых имеет в десятичной записи либо бесконечное число нулей, либо бесконечное число девяток. 48. Найти все многочлены Р(х), для которых справедливо тождество хР (х — 1)=(х — 26) Р (х). 49. См. задачу 44. 50*. Доказать, что на сфере нельзя так расположить три дуги в 300° каждая, чтобы никакие две из них не имели общих точек (в том числе общих концов). XXVII ОЛИМПИАДА (1964 г.) 1-й т у р VII класс 1. В треугольнике АВС высоты, опущенные на стороны АВ и ВС, не меньше этих сторон (соответственно). Найти углы треугольника. 85
2. На данной окружности выбраны диаметрально противопо- ложные точки А и В и третья точка С. Касательная, проведен- ная к окружности в точке В, и прямая АС пересекаются в точ- ке М. Доказать, что касательная, проведенная к окружности в точке С, делит пополам отрезок ВМ. 3. Доказать, что сумма цифр числа, являющегося точным квадратом, не может равняться пяти. 4. На листе бумаги проведено 11 горизонтальных и 11 верти- кальных прямых, точки пересечения которых называются узлами. Звеном мы будем называть отрезок, соединяющий два соседних узла одной прямой. Какое наименьшее число звеньев надо сте- реть, чтобы после этого в каждом узле сходилось не более трех звеньев? 5. Последовательность а0, Оь fl2, ... образована по закону ao = fl| = l; azl+i =ап-+ 1, л = 1, 2, .... Доказать, что число а 1964 не делится на 4. V1H класс 6. См. задачу 1. 7. Найти все такие натуральные числа л, что число (л —1)! не делится на лI 2. 8. Решить в целых числах уравнение с переменными х, у, z д/х-рд/х4-...4-^/x = z, (у радикалов). 9. См. задачу 14 а). 10. Рассмотрим суммы цифр всех чисел от 1 до 1000 000 включительно. У полученных чисел снова рассмотрим суммы цифр и т. д., пока не получим миллион однозначных чисел. Каких чисел больше среди них: единиц или двоек? IX класс II. Решить в положительных числах систему f xy = z, I ^=1/ 12. Доказать, что произведение двух последовательных нату- ральных чисел не является степенью никакого целого числа. 13. Известно, что при любом целом k^27 число a — kA де- лится без остатка на 27 — k. Найти а. 14. Доказать, что: а) если все углы шестиугольника равны, то его стороны удовлетворяют соотношениям ai—a4=as— <22 = -<2з ас,; б) если длины отрезков ..., а^ удовлетворяют при- веденным соотношениям, то из этих отрезков можно построить шестиугольник с равными углами. 15. В четырехугольнике ABCD из вершин А и С опущены перпендикуляры на диагональ BD, а из вершин В и D опущены 86
перпендикуляры на диагональ АС. Доказать, что четырехуголь- ники ABCD и MNPQ подобны (М, N, Р, Q — основания пер- пендикуляров) . X — XI классы 16. Натуральное число N является квадратом и не окан- чивается нулем. После зачеркивания у этого числа двух пос- ледних цифр снова получился точный квадрат. Найти наибольшее число N с таким свойством. 17. См. задачу 8. 18. Известно, что при любом целом Л^=27 число а —А'964 делится без остатка на 27— k. Найти а. (Ср. с задачей 13.) 19. См. задачу 14. 20. На какое наименьшее число непересекающихся тетраэд- ров можно разбить куб? 1-й тур VII класс 21. На отрезке АС выбрана произвольно точка В и на отрезках АВ, ВС и АС как на диаметрах построены окружности Оь 0-2 и О. Через точку В проводится произвольная прямая, пересекающая окружность О в точках Р и Q, а окружности О\ и Ог — в точках R и S (соответственно). Доказать, что PR = QS. 22. Собрались 2л человек, каждый из которых знаком не менее чем с л присутствующими. Доказать, что можно вы- брать из них четырех человек и рассадить их за круглым столом так, что при этом каждый будет сидеть рядом со своими знако- мыми. 23. В квадрате со стороной 1 выбраны 102 точки, из ко- торых никакие три не лежат на одной прямой. Доказать, что найдется треугольник с вершинами в этих точках, площадь ко- торого меньше чем 0,01. 24. Через противоположные вершины Л и С четырехуголь- ника ABCD проведена окружность, пересекающая стороны АВ, ВС, CD и AD соответственно в точках М, N, Р и Q. Известно, что DP~DQ = BM =BN=R, где R — радиус данной окружности. Доказать, что в этом случае сумма углов АВС и ADC данного четырехугольника равна 120°. 25. При каких натуральных числах а существуют такие нату- ральные числа х и у, что х2-\-у2 = аху? VIII класс 26. В л стаканов достаточно большой вместимости налито поровну воды. Разрешается переливать из любого стакана в лю- 87
бой другой столько воды, сколько имеется в этом последнем. При каких п можно за конечное число операций слить всю воду в один стакан? 27. Даны три точки Л, В, С, лежащие на одной прямой, и точка О вне этой прямой. Обозначим через Oi, О2, О3 центры окружностей, описанных около треугольников ОЛВ, ОВС, ОАС. Доказать, что точки О2, Оз и О лежат на одной окружности. 28. На квадратном клетчатом поле 99 X 99 клеток играют двое. Первый игрок ставит крестик на центр поля, вслед за этим второй игрок может поставить нолик на любую из восьми кле- ток, окружающих крестик первого игрока. После этого первый ставит крестик на любое из полей рядом с уже занятыми и т. д. Первый игрок выигрывает, если ему удается поставить крестик на любую угловую клетку. Доказать, что при любой игре второго игрока первый всегда может выиграть. 29. Внутри равностороннего (не обязательно правильного) семиугольника Л|Л2Л3...Л7 взята пройзвольно точка О. Обозна- чим через //i, Н7 основания перпендикуляров, опущенных из точки О на стороны Л1Л2, Л2Л3, ..., А7А\ соответственно. Известно, что точки Hi, Нг, ...» Н7 лежат на самих сторонах, а не на их продолжениях. Доказать, что А\Н{ +Л2//2 + ...-\-А7Н7 = = //1Л2 + Я2Л3 + ... + //бЛ7 + Я7Ль 30*. В квадрате со стороной 1 взята произвольно 101 точ- ка (не обязательно внутри квадрата), причем никакие три из них не лежат на одной прямой. Доказать, что существует треуголь- ник с вершинами в этих точках, площадь которого не больше 0,01. IX класс 31. См. задачу 29. 32. См. задачу 26. 33. Доказать, что любое четное число 2п может быть единст- венным образом представлено в виде 2n==(x-}-yf + Зх-\-у, где х и у — целые неотрицательные числа. 34. В треугольнике АВС сторона ВС равна полусумме двух других сторон. Доказать, что биссектриса угла ВАС перпенди- кулярна отрезку, соединяющему центры вписанной и описан- ной окружностей треугольника. 35* . На клетчатой бумаге проведена замкнутая ломаная с вершинами в узлах сетки, все звенья которой равны. Дока- зать, что число звеньев такой ломаной четно. X класс 36. В п мензурок налиты п разных жидкостей; кроме того, имеется одна пустая мензурка. Можно ли за конечное число операций составить равномерные смеси в каждой мензурке (т. е. сделать так, чтобы в каждой мензурке было ровно от пер- 88
воначального количества каждой жидкости и при этом одна мензурка была бы пустой)? Мензурка имеет деления, позво- ляющие измерять объем налитой жидкости. 37. Дана система из п точек на плоскости, причем известно, что для любых двух точек данной системы можно указать дви- жение плоскости, при котором первая точка перейдет во вторую, а система перейдет сама в себя. Доказать, что все точки такой системы лежат на одной окружности. 38. В треугольнике АВС сторона ВС равна полусумме двух других сторон. Доказать, что вершина А, середины сторон АВ и АС и центры вписанной и описанной окружностей лежат на одной окружности. 39. См. задачу 35. 40*. Имеется бесконечное число карточек, на каждой из ко- торых написаны какие-то натуральные числа. Известно, что для любого натурального числа п можно указать ровно п карто- чек, на которых написаны делители этого числа. Доказать, что любое натуральное число встречается хотя бы на одной карточке. XI класс 41. Из точки О на плоскости отложено несколько векторов, сумма длин которых равна 4. Доказать, что можно выбрать не- сколько векторов (или, быть может, один вектор), длина суммы которых больше 1. 42. См. задачу 33. 43. В треугольнике АВС сторона ВС равна полусумме двух других сторон. Через точку А и середины сторон АВ и АС про- ведена окружность и к ней из центра тяжести треугольника про- ведены касательные. Доказать, что одна из точек касания яв- ляется центром окружности, вписанной в треугольник АВС. (Ср. с задачами 34 и 38.) 44*. Пирог имеет форму правильного л-угольника, вписан- ного в окружность радиуса 1. Из середины каждой стороны про- ведены прямолинейные надрезы длины 1. Доказать, что при этом от пирога всегда будет отрезан какой-нибудь кусок. 45*. При дворе короля Артура собрались 2л рыцарей, при- чем каждый из них имеет среди присутствующих не более л—1 врага. Доказать, что Мерлин, советник Артура, может так рас- садить рыцарей за Круглым Столом, что ни один из них не будет сидеть рядом со своим врагом. XXVIII ОЛИМПИАДА (1965 г.) 1-й т у р VIИ класс 1. Даны окружность О, прямая а, пересекающая ее, и точка М. Через точку М провести секущую b так, чтобы ее часть, 89
заключенная внутри окружности О, делилась пополам в точке ее пересечения с прямой а. 2. Докажите следующий признак делимости на 37. Для того чтобы узнать, делится ли число на 37, надо разбить его на грани справа налево, по три цифры в каждой грани. Если сумма полученных трехзначных чисел делится на 37, то и данное число делится на 37. (Слово «трехзначное» употреблено условно: неко- торые из граней могут начинаться с нулей и быть на самом деле двузначными или однозначными числами; не трехзначным числом будет и самая левая грань, если количество цифр нашего числа не делится на 3.) 3. Даны прямая а и два непараллельных отрезка АВ и CD по одну сторону от нее. Найти на прямой а такую точку М, чтобы треугольники АВМ и CDM были равновелики. 4. 30 команд участвуют в розыгрыше первенства по футболу. Доказать, что в любой момент состязаний имеются две команды, сыгравшие к этому моменту одинаковое число матчей. IX класс 5. Шестизначное число делится на 37. Все его цифры раз личны. Доказать, что из тех же цифр можно составить друго* шестизначное число, также делящееся на 37. 6. Внутри данного треугольника АВС найти такую точку О. чтобы площади треугольников АОВ, ВОС, СОА относились как 1:2:3. 7. Дан треугольник АВС, в котором сторона АВ больше ВС. Проведены биссектрисы АК и СМ (К лежит на ВС, М лежит на АВ). Доказать, что АМ>МК>КС. 8. В стране Иллирии некоторые пары городов связаны пря- мым воздушным сообщением. Доказать, что там есть два города, связанные с равным количеством других городов. (Ср. с зада- чей 4.) 9. Вдоль коридора положено несколько кусков ковровой до- рожки. Куски покрывают весь коридор из конца в конец без пропусков и даже накладываются друг на друга, так что над некоторыми местами пола они лежат в несколько слоев. Дока- зать, что можно убрать несколько кусков, возможно достав их из-под других и оставив остальные в точности на тех же местах, где они лежали прежде, так, что коридор по-прежнему будет полностью покрыт, а общая длина оставленных кусков будет меньше удвоенной длины коридора. X класс 10. Окружности О| и 01 лежат внутри треугольника АВС и касаются друг друга извне, причем окружность С\ касается сторон АВ и ВС, а окружность О2 — сторон АВ и АС. Доказать, 90
что сумма радиусов этих окружностей больше радиуса окруж- ности, вписанной в треугольник. 11. См. задачу 5. 12. Концы отрезка постоянной длины скользят по сторонам данного угла. Из середины этого отрезка к нему восставлен перпендикуляр. Доказать, что отрезок перпендикуляра от его основания до точки пересечения с биссектрисой угла имеет пос- тоянную длину. 13. х — число, большее двух. Некто пишет на карточках числа: 1, х, х2, х3, ..., хк (каждое число только на одной кар- точке). Потом часть карточек он кладет себе в правый карман, часть в левый, остальные выбрасывает. Доказать, что сумма чисел в правом кармане не может быть равна сумме чисел в ле- вом. (Ср. с задачей 15.) 14. Бумажный квадрат был проколот в 1965 точках. Из то- чек-проколов и вершин квадрата никакие три не лежат на одной прямой. Потом сделали несколько прямолинейных, не пересе- кающихся между собой разрезов, каждый из которых начинался и кончался только в проколотых точках или вершинах квадрата. Оказалось, что квадрат разрезан на треугольники, внутри кото- рых проколов нет. Сколько было сделано разрезов и сколько получилось треугольников? XI класс 15. Все коэффициенты многочлена равны единице, нулю или минус единице. Доказать, что все действительные корни данного многочлена, если они существуют, заключены в отрезке [ — 2; 2]. 16. На плоскости даны три точки. Построить три окружности, касающиеся друг друга в этих точках. Разобрать все возмож- ные случаи. 17. В квадратном уравнении х2+рх + </=0 коэффициенты р, q независимо пробегают все значения от —1 до +1 вклю- чительно. Найти множество значений, которые при этом могут принимать действительные корни данного уравнения. 18. Даны окружность О, точка Л, лежащая на ней, пер- пендикуляр к плоскости окружности О, восставленный из точ- ки Л, и точка В, лежащая на этом перпендикуляре. Найти геометрическое место (множество) оснований перпендикуляров, опущенных из точки Л на прямые, проходящие через точку В и произвольную точку окружности О. 19* . Даны двадцать карточек. Каждая из цифр от нуля до девяти включительно написана на двух из этих карточек. Можно ли расположить эти карточки в ряд так, чтобы нули стояли рядом, между единицами лежала ровно одна карточка, между двойками — две и т. д. до девяток, между которыми должно лежать девять карточек? 91
2-й т у р V1H класс 20*. Дана последовательность ...» a_rt, ...» а_|, а0, аь ап. .... бесконечная в обе стороны, причем каждый ее член 1 гт равен — суммы двух соседних. Доказать, что если какие-то два ее члена (не обязательно соседние) равны, то в ней есть беско- нечное число пар равных между собой чисел. 21. Дан прямоугольный бильярд размером 26 X 1965. Из нижней левой лузы под углом 45° к бортам выпускается шар. Доказать, что после нескольких отражений от бортов он упадет в верхнюю левую лузу. 22. Два неравных картонных диска разделены на 1965 равных секторов. На каждом диске произвольно выбраны 200 секторов и раскрашены в красный цвет. Меньший диск наложен на боль- ший так, что их центры совпадают, а секторы целиком лежат один против другого. Меньший диск поворачивают на всевоз- можные углы, кратные -и части окружности, оставляя боль- ший диск неподвижным. Доказать, что по крайней мере при 60 положениях на дисках совпадут не более 20 красных секторов. 23*. Посередине между двумя параллельными улицами стоят в один ряд одинаковые дома-квадраты со стороной, равной а (см. рис. 4). Расстояние между улицами — За, а расстояние меж- ду двумя соседними домами — 2а. Одна улица патрулируется полицейскими, которые движутся на расстоянии 9а друг от друга со скоростью v. К тому времени, как первый полицейский про- ходит мимо середины некоторого дома, точно напротив него на другой улице появляется гангстер. С какой постоянной скоростью и в какую сторону должен двигаться по этой улице гангстер, чтобы ни один полицейский его не заметил? IX класс 24. См. задачу 29. 25. Дан бильярд прямоугольной формы. В его углах имеют- ся лузы, попадая в которые шарик останавливается. Шарик вы- пускают из одного угла бильярда под углом 45° к стороне. В ка- ____________А_______________ и г р 0 о В Рис. 4. кой-то момент он попал в сере- дину некоторой стороны. Дока- зать, что в середине противопо- ложной стороны он побывать не мог. 26. См. задачу 20. 27. См. задачу 30. 28. Найти геометрическое мес- 92
то (множество) центров равносторонних треугольников, опи- санных около данного произвольного треугольника. X класс 29*. Имеются 11 мешков с монетами и весы с двумя чашка- ми и стрелкой, которые показывают, на какой чашке груз тяже- лее и на сколько именно. Известно, что в одном мешке все монеты фальшивые, а в остальных — все монеты настоящие. Все настоя- щие монеты имеют одинаковую массу, а все фальшивые — тоже одинаковую, но другую. За какое наименьшее число взвешива- ний можно определить, в каком мешке фальшивые монеты? 30. На лист клетчатой бумаги размером п X и клеток кла- дутся черные и белые кубики, причем каждый кубик занимает ровно одну клетку. Первый слой кубиков положили произвольно, а затем вспомнили, что каждый черный кубик должен граничить с четным числом белых, а каждый белый — с нечетным числом черных. Кубики во второй слой положили так, чтобы для всех кубиков первого слоя выполнялось это условие. Если для всех кубиков второго слоя это условие уже выполняется, то больше кубиков не кладут, если же нет, то кладут третий слой так, чтобы для всех кубиков второго слоя выполнялось это условие, и т. д. Существует ли такое расположение ку- биков первого слоя, что этот процесс никогда не кончится? 31. В прямоугольном бильярде размером р X 2р, где р и q — целые нечетные числа, сделаны лузы в каждом углу и в се- редине каждой стороны длиной 2q. Из угла выпущен шарик под углом 45° к стороне. Доказать, что шарик обязательно по- падет в одну из средних луз. (Ср. с задачей 25.) 32. Все целые числа от 1 до 2п выписаны в строчку. Затем к каждому числу прибавили номер того места, на котором оно стоит. Доказать, что среди полученных сумм найдутся хотя бы две, дающие при делении на 2п одинаковый остаток. 33*. В ящике лежат два ящика поменьше, в каждом из них еще по два ящика и т. д. п раз. В каждом из 2п маленьких ящиков лежит по монете, причем одни — вверх гербом, а осталь- ные — вверх решкой. За один ход разрешается переворачивать один любой ящик вместе со всем, что в нем лежит. Доказать, что не больше чем за п ходов можно расположить ящики так, что число монет, лежащих вверх гербом, будет равно числу монет, лежащих вверх решкой. XI класс 34. Найти все простые числа вида рр+1 (р — натуральное), содержащие не более девятнадцати цифр. 35. Доказать, что последние цифры чисел пп (п — нату- ральное) образуют периодическую последовательность. 93
36*. Даны плоскость Р и две точки А и В по разные стороны от нее. Построить сферу, проходящую через эти точки и высе- кающую из Р наименьший круг. 37. Дан многоугольник на плоскости, невыпуклый и нссамо- пересекающийся. D — множество точек, принадлежащих тем диагоналям многоугольника, которые не выходят за его пре- делы (т. е. ни одна точка диагонали не лежит вне многоуголь- ника). Доказать, что любые две точки из D можно соединить ломаной, целиком принадлежащей D. 38. В каждой клетке квадратной таблицы т X т клеток стоит либо натуральное число, либо нуль. При этом если на пересечении строки и столбца стоит нуль, то сумма чисел в этой строке и в этом столбце не меньше т. Доказать, что сумма всех чисел в таблице не меньше чем . XXIX ОЛИМПИАДА (1966 г.) 1-й т у р VIII класс I. Найти геометрическое место (множество) центров вписан- ных в треугольник АВС прямоугольников (одна сторона пря- моугольника лежит на АВ). 2. Найти все двузначные числа такие, что при умножении на некоторое целое число получается число, предпоследняя цифра которого равна пяти. 3. См. задачу 6. 4. См. задачу Ю. 5. Из 28 костей домино убрали все кости с шестерками. Мож- но ли остальные кости выложить в цепь? IX—XI классы 6. Решить в целых положительных числах систему ( *+y = zt. I ?-Н=лт/. - п t л 19*4 66* 7. При каком значении к величина А/.=——— максимальна? 8. Внутри окружности расположен выпуклый пятиугольник (его вершины могут лежать как внутри, так и на окружности). Доказать, что хотя бы одна из его сторон не больше стороны правильного пятиугольника, вписанного в эту окружность. 9. Доказать, что те натуральные k. для которых число kk+l делится на 30, образуют арифметическую прогрессию. Найти ее. 10. Какое максимальное число дамок можно расставить на черных полях шахматной доски размером 8X8 так, чтобы каж- дую дамку била хотя бы одна из остальных? 94
2-й т у р VI11 класс 11. Разделить циркулем и линейкой отрезок на 6 равных частей, проведя не более 8 линий (прямых, окружностей). 12*. Дано: ai = l, 0л=НагНН02 +...1] при ft>l. Най- ти 01000. 13*. Из 19 шаров 2 радиоактивны. В условиях задачи 18 обнаружить оба радиоактивных шара за 8 проверок. 14. Сеть метро имеет на каждой линии не менее четырех станции, из них не более трех пересадочных. Ни на какой пере- садочной станции не пересекается более двух линий. Какое наи- большее число линий может иметь такая сеть, если с любой стан- ции на любую можно попасть, сделав нс больше двух пересадок? 15*. Доказать, что существует такое ft, что число ft! начи- нается с цифр 1966. IX—XI классы 16. См. задачу 11. 17. См. задачу 12, где 01 = 1966 и требуется найти Пюбб. 18*. Из 11 шаров два радиоактивны. Про любой набор шаров за одну проверку можно узнать, имеется ли в нем хотя бы один радиоактивный шар (но нельзя узнать, сколько их). Доказать, что менее чем за 7 проверок нельзя гарантировать нахождение обоих радиоактивных шаров, а за 7 проверок их всегда можно обнаружить. 19*. Из набора гирь массой I, 2, ...» 26 выбрать 6 гирь так, чтобы из них нельзя было составить два набора равных масс. Доказать, что нельзя выбрать 7 гирь, обладающих тем же свойством. 20. На клетчатой доске 11X11 отмечено 22 клетки так, что на каждой вертикали и на каждой горизонтали отмечено ровно 2 клетки. Два расположения отмеченных клеток эквива- лентны, если, меняя любое число раз вертикали между собой и горизонтали между собой, мы из одного расположения можем получить другое. Сколько существует неэквивалентных распо- ложений отмеченных клеток? XXX ОЛИМПИАДА (1967 г.) 1-й т у р VIII класс 1. Существуют ли два последовательных натуральных числа таких, что сумма цифр каждого из них делится на 125? Найти наименьшую пару таких чисел или доказать, что их не сущест- вует 95
2. Дан треугольник ЛВС. Найти на прямой АВ точку М та- кую, чтобы сумма радиусов окружностей, описанных около тре- угольников АСМ и ВСМ, была минимальна. 3. Для зашифровки телеграфных сообщений требуется раз- бить всевозможные десятичные «слова» — наборы из десяти то- чек и тире — на две группы так, чтобы любые два слова одной группы отличались не менее чем в трех разрядах. Указать способ такого разбиения или доказать, что его не существует. 4. Дан треугольник ЛВС. Найти геометрическое место (множество) точек М таких, что треугольники АВМ и ВСМ — равнобедренные. 5. Остап Бендер организовал в городе Фуксе раздачу слонов населению. На раздачу явилось 28 членов шахматного клуба и 37 не членов, причем Остап раздавал слонов поровну всем членам клуба и поровну не членам. Оказалось, что существует лишь один способ такой раздачи (так, чтобы раздать всех слонов). Какое наибольшее число слонов могло быть у О. Бендера? IX класс 6. Имеется лабиринт, состоящий из п окружностей, касаю- щихся прямой / в точке М. Все окружности расположены по одну сторону от прямой, а их длины составляют геометри- ческую прогрессию со знаменателем 2. Два пешехода в разное время начали ходить по этому лабиринту. Их скорости одинаковы, а направления движения различны. Каждый из них проходит все окружности по порядку и, пройдя наибольшую, снова идет в меньшую. Доказать, что пешеходы встретятся. 7. Можно ли разрезать квадратный пирог на 9 равновеликих частей таким образом: 'выбрать внутри квадрата две точки и соединить каждую из них прямолинейными разрезами со всеми четырьмя вершинами квадрата? Если можно, то какие две точки следует выбрать? 8. См. задачу 2. 9. Чему равна максимальная разность между соседними чи- слами среди тех, сумма цифр которых делится на 7? 10. Имеется 120-значное число. Его первые 12 цифр перестав- ляются всеми возможными способами. Из полученных таким образом 120-значных чисел наугад выбирают 120 чисел. Дока- зать, что их сумма делится на 120. X класс 11. В квадрате расположено k точек (ft>2). На какое наименьшее число треугольников нужно разбить квадрат, чтобы в каждом треугольнике находилось не более одной точки? 12. Доказать, что в круге радиуса 1 нельзя найти более 5 точек, все попарные расстояния между которыми больше 1. 9G
13. Доказать, что уравнение 19х3 — 17ул = 50 не имеет реше- ний в целых числах. 14. В бесконечно большой каравай, занимающий все прост- ранство, в точках с целыми координатами впечены изюминки диаметра 0,1. Каравай разрезали на части несколькими плоско- стями. Доказать, что найдется неразрезанная изюминка. 15. Из первых k простых чисел 2, 3, 5, ...» pk(k>4) состав- лены всевозможные произведения, в которые каждое из чисел входит не более одного раза (например, 3*5, З-5-...-р*, 11-13, 7 и т. д.). Обозначим сумму таких чисел через S. Доказать, что S4-1 разлагается в произведение более 2 k простых сомножи- телей. 2-й т у р VII класс 16. В треугольнике АВС проведены высоты AEt ВМ и СР. Известно, что ЕМ\\АВ9 ЕР\\АС. Доказать, что МР\\ВС. 17. Над квадратным катком нужно повесить четыре лампы так, чтобы они его полностью освещали. На какой наименьшей высоте нужно повесить лампы, если каждая лампа освещает круг радиуса, равного высоте, на которой она висит? 18* . Доказать, что существует целое число q такое, что в десятичной записи числа 47-2,00° нет ни одного нуля. 19. Число у получается из натурального х некоторой пере- становкой его цифр. Доказать, что, каково бы ни было %, * + # =7^999...99 (1967 девяток). 20. В четырех заданных точках на плоскости расположены точечные прожекторы, каждый из которых может освещать пря- мой угол. Стороны этих углов могут быть направлены на север, юг, запад или восток. Доказать, что эти прожекторы можно направить так, что они осветят всю плоскость. VIII класс 21. См. задачу 18 для числа </-21967. 22. Обозначим через d (N) число делителей /V. Найти такие, что — целое простое число (числа 1 и N все N также считаются делителями). 23. На каждой стороне прямоугольного треугольника пост- роено по квадрату («Пифагоровы штаны»), и около полученной фигуры описана окружность. Для каких прямоугольных треуголь- ников это можно сделать? 24. На шахматной доске размером 1000 X 1000 находятся черный король и 499 белых ладей. Черные и белые фигуры ходят по очереди. Доказать, что, как бы ни ходили ладьи, король всегда может за несколько ходов встать под бой одной из них. 4 Заказ 247 97
25. Семь школьников решили за воскресенье обойти семь ки- нотеатров, в каждом из которых сеансы начинаются в 9.00, 10.40, 12.20, 14.00, 15.40, 17.20, 19.00 и 20.40 (8 сеансов). На каждый сеанс шестеро шли вместе, а кто-нибудь один (ые обязательно один и тот же) шел в другой кинотеатр. К вечеру каждый побы- вал в каждом кинотеатре. Доказать, что в каждом кинотеатре был сеанс, на котором не был ни один из этих школьников. IX класс 26. Число у получается из натурального числа х некоторой перестановкой его цифр. Известно, что % + «/= 100...00 ( 200 ну- лей). Доказать, что х делится на 50. (Ср. с задачей 19.) 27. Дана последовательность целых положительных чисел Xi, %2, ...» все элементы которой не превосходят некоторого числа М, Известно, что । —х*_2| при всех k>2. Какой может быть максимальная длина этой последователь- ности? 28. На каждой стороне прямоугольного А ЛВС построено по квадрату во внешнюю сторону («Пифагоровы штаны»). Оказа- лось, что внешние вершины квадратов лежат на одной окружно- сти. Доказать, что треугольник АВС равнобедренный. (Ср. с задачей 23.) 29*. Задано натуральное число К такое, что для любого •4— натурального N, делящегося на К, число N тоже делится на ч - К (N — число, состоящее из тех же цифр, что и N, но записан- ч— ных в обратном порядке, например 1967=7691» 450 = 54). Доказать, что К является делителем числа 99. 30. Испанский король решил перевесить по-своему портреты своих предшественников в круглой башне замка. Однако он хо- чет, чтобы за один раз меняли местами только два портрета, висящие рядом, причем это не должны быть портреты двух коро- лей, один из которых царствовал сразу после другого. Кроме того, ему важно лишь взаимное расположение портре/ов (два расположения, отличающиеся поворотом круга, он считает оди- наковыми). Доказать, что, как бы сначала ни висели портреты, король может по этим правилам добиться любого нового их расположения. X класс 31*. Дана таблица пХп клеток. Таблица заполняется сле- дующим образом: в первую строчку записываются числа 1, 2, ..., и, и если в некоторой строчке записаны числа ai, .... ал, а*+ь flm + i, ап (т и n — k взаимно просты), то в сле- 98
дующей строчке записываются те же числа, но в таком порядке: Дгп + 1. ...» fl/ь л* + 1, ...» ат, ai, ... Qk. Доказать, что после заполне- ния таблицы в каждом столбце будут написаны все числа от 1 до п. 32. См. задачу 28. 33. Можно ли расставить на окружности числа I, 2, ..., 12 так, чтобы разность между двумя рядом стоящими числами была 3, 4 или 5? 34. В восьми данных точках пространства установлено по точечному прожектору, каждый из которых может осветить в про- странстве октант (трехгранный угол со взаимно перпендикуляр- ными ребрами). Доказать, что можно повернуть прожекторы так, чтобы они осветили все пространство. (Ср. с задачей 20.) 35*. Рассматриваются всевозможные n-значные числа, со- ставленные из цифр 1, 2 и 3. В конце каждого из этих чисел при- писывается цифра 1, 2 «ли 3 так, что к двум числам, у которых во всех разрядах стоят разные цифры, приписываются разные цифры. Доказать, что найдется л-значное число, в записи ко- торого участвует лишь одна единица и к которому приписывается единица. XXXI ОЛИМПИАДА (1968 г.| 1-й т у р VII класс 1. Число 4 обладает тем свойством, что при делении его па q2 получается остаток меньше , каково бы ни было q. Пере- числить все числа, обладающие этим свойством. 2. Расставить в таблице 4X4 шестнадцать чисел так, чтобы сумма чисел по любой вертикали, горизонтали и диагонали рав- нялась нулю. Примечание. Таблица имеет 14 диагоналей, включая все малые, состоящие из трех, двух и одной клеток. 3. Доказать, что для любых трех чисел, меньших 1 000 000, найдется число, меньшее 100, взаимно простое с каждым из них. 4. Как соединить 50 городов наименьшим числом авиалиний так, чтобы из любого города можно было попасть в любой дру- гой, сделав не более двух пересадок? VIII класс 5. В шахматном турнире участвовало 12 человек. После окончания турнира каждый участник составил 12 списков. В п е р- в ы й список входит только он сам, во второй — он и те, у кого он выиграл, в т р е т и й — все люди из второго списка и те, у кого они выиграли, ит. д. В двенадцатый список вхо- дят все люди из одиннадцатого списка и те, у кого они выиграли. 4‘ 99
Известно, что для любого участника турнира в его двенадцатый список попал человек, которого не было в его одиннадцатом списке. Сколько ничейных партий было сыграно в турнире? 6. Даны числа 4, 14, 24, ..., 94, 104. Доказать, что из них нельзя вычеркнуть сначала одно число, затем из оставшихся еще два, затем еще три и, наконец, еще четыре числа так, чтобы после каждого вычеркивания сумма оставшихся чисел делилась на 11. 7. Можно ли вписать в окружность выпуклый семиугольник Л|Л2ЛзЛ4Л5ЛеЛ7 с углами Z4| = 14O°, ^Л2=120°, /1Лз=130о, /_Л4 = 120°, ^Л5=130°, /1Ле= 110°, zM7=150°? 8. Выбрать 100 чисел, удовлетворяющих условиям %1 = 1; 0^x2^2xg 0^хз^2х2; 0^Х99^2х98; О^хюо^ 2x99, так, чтобы выражение Xi — Х2+Х3 — Х4 + ...4-Х99—Хюо было максимально. 9. Можно ли расположить на плоскости 1000 отрезков так, чтобы каждый отрезок своими концами упирался строго внутрь других отрезков? IX класс 10. Существует ли четырехугольник ABCD площади 1 такой, что для любой точки О внутри его площадь хотя бы одного из треугольников ОЛВ, ОВС. OCD, OAD есть иррациональное число? 11. См. задачу 9 для 1968 отрезков. 12. В коридоре длиной 100 м постелено 20 ковровых дорожек общей длиной 1000 м. Каково может быть наибольшее число не- застеленных кусков? (Ширина дорожки равна ширине коридора.) 13. Можно ли выбрать 100 000 номеров телефонов из 6 цифр каждый так, чтобы при одновременном вычеркивании из всех этих номеров ft-й цифры (й=1, 2, 3, 4, 5, 6) получились все пятизначные номера от 00 000 до 99 999? 14. Доказать, что если р и q — два простых числа, причем <7 = р + 2, то pq + qp делится на X класс 15*. Из пункта Л одновременно вылетают 100 самолетов (флагманский и 99 дополнительных). С полным баком горючего самолет может пролететь 1000 км. В полете самолеты могут пере- давать друг другу горючее. Самолет, полностью отдавший го- рючее другим, совершает планирующую посадку. Каким обра- зом надо совершать перелет, чтобы флагман пролетел возможно дальше? 16. Двое играют в следующую игру: имеется две кучки кон- фет Играющие делают ход по очереди. Ход состоит в том, что играющий съедает одну из кучек, а другую делит на две (равные 10Ф
или неравные) части. Если он не может разделить кучку, так как в ней всего одна конфета, то он ее съедает и выигрывает. В на- чале в кучках было 33 и 35 конфет. Кто выигрывает, начи- нающий или его партнер, и как для этого надо играть? 17. Можно ли разбить множество всех целых неотрицатель- ных чисел на 1968 классов так, чтобы в каждом классе было хотя бы одно число и выполнялось бы следующее условие: если число т получается из числа п вычеркиванием двух рядом стоящих одинаковых цифр или одинаковых групп цифр, то т и п принадлежат одному классу (например, числа 7, 9339337, 93223393447, 932239447 обязательно принадлежат одному классу)? 18. По заданной последовательности положительных чисел qi9 q*2, ...» qn, ... строится последовательность многочленов следующим образом: /0 (*)=1, fi (x)=x, fn + i (*) = (! +^п)лг/л (*) — — qnfn_i(x) для п^1. Доказать, что все действительные корни n-го многочлена заключены между —1 и 4-1. 19. В пространстве расположены 4 попарно скрещивающиеся прямые li, Z2, /з» /4, причем никакие три из них не параллельны одной плоскости. Провести плоскость Р так, чтобы точки Ль Л2, Лз, Л4 пересечения этих прямых с Р являлись верши- нами параллелограмма. Сколько решений имеет задача? 2-й т у р VII класс 20. На плоскости отмечено 1968 точек, являющихся вершина- ми правильного 1968-угольника. Двое играющих по очереди соединяют две вершины многоугольника отрезком, соблюдая следующие правила: нельзя соединять две точки, хотя бы одна из которых уже соединена с другой, и нельзя пересекать уже проведенные отрезки. Проигрывает тот, кто не может сделать очередного хода согласно этим правилам. Как нужно играть, чтобы выиграть? Кто выиграет при правильной игре? 21. На плоскости даны три точки. Из них выбираются любые две, строится серединный перпендикуляр к отрезку, их соединяю- щему, и все точки отражаются симметрично относительно этой прямой; затем из всех точек (старых и новых) снова выбирают какие-то две точки и весь процесс повторяют. Так делается беско- нечно много раз. Доказать, что в плоскости найдется такая пря- мая, что все полученные точки будут лежать по одну сторону от нее. 22. Два маляра красят забор, огораживающий 100 дачных участков. Они приходят через день и красят по одному участку в красный или зеленый цвет. Первый маляр дальтоник и путает цвета: он помнит, что и в какой цвет он сам покрасил, и видит что покрасил второй маляр, но не знает, в какой цвет. Первый маляр добивается того, чтобы в наибольшем числе мест зеленый 101
участок граничил с красным. Какого наибольшего числа перехо- дов может добиться первый маляр (как бы ни действовал вто- рой маляр)? 23. Двухсотзначное число 89252525 ... 2525 умножено на число 444х18(/27 (х и у — неизвестные цифры). Оказалось, что 53-я цифра полученного числа (считая справа) есть 1, а 54-я — 0. Найти х и у. 24*. Ковбой Джимми поспорил с друзьями, что сумеет од- ним выстрелом пробить все четыре лопасти вертилятора. (Верти- лятор устроен следующим образом: на оси, вращающейся со скоростью 50 об/с, расположены на равных расстояниях друг от друга четыре полудиска, повернутые друг относительно друга под какими-то углами и перпендикулярные оси.) Джимми может стрелять в любой момент и добиваться произвольной скорости пуль. Доказать, что Джимми выиграет пари. VIII класс 25. Разобьем все натуральные числа на группы так, чтобы в первой группе было одно число, во второй — два, в третьей — три и т. д. Можно ли это сделать таким образом, чтобы из суммы чисел в каждой группе нацело извлекался корень седьмой сте- пени? 26*. Две прямые на плоскости пересекаются под углом а. На одной из них сидит кузнечик. Каждую секунду он прыгает с одной прямой на другую (точка пересечения считается при- надлежащей обеим прямым). Известно, что длина каждого его прыжка равна 1 и что он никогда не возвращается на то место, где был секунду назад. Через некоторое время кузнечик вернулся в первоначальную точку. Доказать, что угол а измеряется ра- циональным числом градусов. 27*. Круглый пирог режут таким образом. Вырезают сектор с углом а, переворачивают его на другую сторону, а затем весь пирог поворачивают на угол р. Дано, что р<а<180°. Дока- зать, что после некоторого конечного числа таких операций каждая точка пирога будет находиться на своем первоначаль- ном месте. 28. На бумажной ленте напечатаны автобусные билеты с но- мерами от 000 000 до 999 999. Затем синей краской отмечены те билеты, у которых сумма цифр, стоящих на четных местах, равна сумме цифр, стоящих на нечетных местах. Какая наи- большая разность может оказаться между номерами двух сосед- них синих билетов? 29. Страна Фарра расположена на 1000 000 000 островов. Между некоторыми островами каждый день курсируют пароходы. Маршруты пароходов устроены так, что с каждого острова можно попасть на любой другой (возможно, за несколько дней). Вор и полицейский могут совершать не более одного рейса в день 102
на пароходе и не имеют никакой другой возможности попасть с острова на остров. Вор не ездит на пароходе 13-го числа каж- дого месяца, а полицейский не суеверен и всегда знает, где на- ходится вор. Доказать, что полицейский сможет поймать вора (т. е. оказаться с ним на одном острове). IX класс 30. На плоскости нарисован правильный пятиугольник А1А2А3А4А5. Можно ли выбрать на плоскости множество точек, обладающее следующим свойством: через любую точку, не ле- жащую внутри пятиугольника, можно провести отрезок, концы которого являются точками нашего множества, а через точки, лежащие внутри пятиугольника, такого отрезка провести нельзя? 31. На окружности радиуса 1 отмечена точка О, и из нее циркулем делается на окружности засечка по часовой стрелке радиусом /. Из полученной точки О\ в том же направлении тем же радиусом делается вторая засечка, и так повторяется 1968 раз. После этого окружность разрезается во всех 1968 за- сечках и получается 1968 дуг. Сколько различных длин дуг мо- жет при этом получиться? 32. Белые и черные играют в следующую игру. В углах шах- матной доски стоят два короля: белый на al, черный на Л8. Играющие делают ходы по очереди (начинают белые). Играющий может ставить своего короля на любое соседнее поле (если толь- ко оно свободно), соблюдая следующие правила: нельзя уве- личить расстояние между королями (расстоянием между двумя королями называется наименьшее число ходов короля, за которое он может пройти с одного поля на другое: так, в начале игры расстояние между королями — 7 ходов). Выигрывает тот, кто поставит своего короля на противопо- ложную кромку доски (белого короля на вертикаль h или вось- мую горизонталь, черного — на вертикаль а или первую гори- зонталь). Как нужно играть, чтобы выиграть? Кто выигрывает при правильной игре? 33*. Известно, что ап — Ьп делится на п (а, 6, п — натураль- ные числа, а=/=6). Доказать, что --делится на п. а — о 34*. Дано натуральное число N. С ним производится сле- дующая операция: каждая цифра этого числа заносится на отдельную карточку (при этом разрешается добавлять или выбра- сывать любое число карточек, на которых написана цифра 0). Затем эти карточки разбивают на две группы. В каждой из них карточки располагаются в произвольном порядке и получен- ные два числа складываются. С полученным числом N\ проде- лывается такая же операция и т. д. Докажите, что за 15 шагов из N можно получить однозначное число. юз
X класс 35. Внутри выпуклого многоугольника М помещена окруж- ность максимально возможного радиуса R (это значит, что внут- ри М нельзя поместить окружность большего радиуса). Известно, что, двигая единичный отрезок (как твердый стержень) внутри многоугольника Л4, его можно повернуть на любой угол. До- казать, что 36. В таблице А размером 10X10 написаны какие-то числа. Обозначим сумму всех чисел в первой строке через Si, во второй — s? и т. д. Аналогично сумму чисел в первом столбце обозначим через 6, во втором — G и т. д. Составлена новая таблица В размером 10X10, в нее вписаны числа следующим образом: в первой клетке первой строки пишется наименьшее из чисел Si и Л, в третьей клетке пятой строки пишется наименьшее из чисел $5 и /з, аналогично заполнена вся таблица. Оказалось, что можно так занумеровать клетки таблицы В числами от 1 до 100, что в клетке с Л-м номером будет стоять число, меньшее или равное k. Какое максимальное значение может принимать при этих усло- виях сумма всех чисел таблицы А? 37. Дана система уравнений < Х| 4~хг 4" ••• 4~х* = 0, х? 4~ х! 4~ ••• 4~ х* = 0, х? 4~ *2 4~ ••• 4~ =0, х174-Х274- -4-xf=0, х!94-Х294--+^9=0, <х?14-х|,4-...4-хГ = 1 (все Xi — действительные числа). Доказать, что при некоторых k такая система имеет решение. 38*. Правильный треугольник АВС разбит на N выпуклых многоугольников так, что каждая прямая пересекает не более 40 из них (мы говорим, что прямая пересекает многоугольник, если они имеют общую точку, в том числе если прямая про- ходит через вершину многоугольника). Может ли N быть больше миллиона? 39. На поверхности кубика мелом отмечено 100 различных точек. Доказать, что можно двумя различными способами поста- вить кубик на черный стол (причем в точности на одно и то же место) так, чтобы отпечатки от мела на столе при этих спо- собах были разными. (Если точка отмечена на ребре или вер- шине, она тоже дает отпечаток.) 104
XXXII ОЛИМПИАДА (1969 г.) 1-й т у р VII класс 1. Белая ладья преследует черного слона на доске 3X1969 клеток (они ходят по очереди по обычным правилам). Как должна играть ладья, чтобы взять слона? Первый ход делают белые. 2. Старинный замок был обнесен треугольной стеной. Каждая сторона стены была поделена на три равные части, и в этих точ- ках, а также в вершинах были построены башни. Всего вдоль стены было 9 башен: А, Е, F, В, К, -L, С, М, N. Со временем все стены и башни, кроме башен Е, К, М, разрушились. Как по оставшимся башням определить, где находились башни А, В, С, если известно, что башни А, В, С стояли в вершинах? 3. В Чили в феврале проходил международный турнир по футболу. Первое место с 8-ью очками занял местный клуб «Ко- ло-Коло». На очко отстало московское «Динамо» и заняло второе место. 3-е место с 4-мя очками занял бразильский клуб «Корин- тианс». 4-е место занял югославский клуб «Црвёна Звезда», также набравший 4 очка. Доказать, что по этим данным можно точно определить, сколько еще команд участвовало в турнире и по сколько очков они набрали. 4. Доказать, что никакая степень числа 2 не оканчивается четырьмя одинаковыми цифрами. 5. Имеется 1000 деревянных правильных 100-угольников, при- битых к полу. Всю эту систему мы обтягиваем веревкой. Натя- нутая веревка будет ограничивать некоторый многоугольник. До- казать, что у него более 99 вершин. VIII класс 6. См. задачу 4. 7. См. задачу 5 с заменой чисел 1000, 100, 99 на числа 57, 57, 56 соответственно. 8. См. задачу 1, заменив слона конем. 9. Дан отрезок АВ. Найти геометрическое место (множест- во) точек С таких, что в треугольнике АВС медиана, проведенная из вершины В, равна высоте, проведенной из вершины А. 10. Можно ли записать в строку 20 чисел так, чтобы сумма любых трех последовательных чисел была положительна, а сумма всех 20 чисел была отрицательна? IX класс 11. Найти все натуральные числа х, обладающие следую- щим свойством: из каждой цифры числа х можно вычесть одну и ту же цифру а^0 (т. е. все цифры его не меньше а) и при этом получится (х—а)2. 105
12. Остров Толпыго имеет форму многоугольника. На нем расположено несколько стран, каждая из которых имеет форму треугольника, причем каждые две граничащие страны имеют целую общую сторону (т. е. вершина одного треугольника не лежит на стороне другого). Доказать, что карту этого острова можно так раскрасить тремя красками, чтобы каждая страна была закрашена одним цветом и любые две граничащие страны были закрашены в разные цвета. 13. Можно ли записать в строку 50 чисел так, чтобы сумма любых 17 последовательных чисел была положительна, а сумма любых 10 последовательных чисел была отрицательна? (Ср. с за- дачей 10.) 14. См. задачу 4. 15. В государстве царя Додона расположено 500 городов, каждый из которых имеет форму правильной 37-угольной звезды, в вершинах которой находятся башни. Додон решил обнести их выпуклой стеной так, чтобы каждый отрезок стены соединял две башни. Доказать, что стена будет состоять не менее чем из 37 отрезков. (Если несколько отрезков лежат на одной пря- мой, то они считаются за один. Ср. с задачей 5.) X класс 16. На выходе из бетатрона установлены два одинаковых тон- ких обруча, расположенные во взаимно перпендикулярных пло- скостях, причем каждый обруч проходит через центр другого. Частицы, вылетающие из бетатрона, пролетают по прямой сквозь оба обруча. По какой прямой они должны двигаться, чтобы находиться на наибольшем расстоянии от обручей, т. е. чтобы наименьшее расстояние, на которое частицы приближаются к обручу, было как можно больше? 17. Дана бесконечная последовательность чисел at, .... а„. Она периодична с периодом 100, т. е. a, =aioi, 02 = 0102, ... . Известно, что ai>0, а। + 02^0, О|-|-О2-|-аз>0 и, вообще, сум- ма Oi4-o2+. .-Ьоп неотрицательна при нечетном п и неполо- жительна при четном п. Доказать, что |o99l>|o(ool. 18. Колода перфокарт четырех цветов разложена в один ряд. Если две перфокарты одного цвета лежат рядом или через одну, то можно выбрасывать ту из них, которая левее. Кроме того, можно подкладывать справа любое количество перфокарт из дру- гих колод. Доказать, что можно подкладывать и выбрасывать перфокарты таким образом, чтобы в конце концов их осталось только четыре. 19. Существует ли такое число Л, что ни для какого на- турального п число [Л.1969”] не делится на [Л* 1969"_|]? 20. Дан квадрат ABCD. Найти геометрическое место (мно- жество) таких точек М, что выполняется равенство £.АМВ = = Z.CMD. 106
i-Й т у р VII класс 21. Даны два целых положительных числа тип. Выписы- ваются все различные делители числа т — числа а, Ь,k — и все различные делители числа п — числа s, t, ..., г. (Само число и 1 тоже включаются в число делителей.) Оказалось, что и -+-+•+v=t+t+-+t- Доказать, что m=n. 22. С числом 123456789101112...9989991000 производится сле- дующая операция: зачеркиваются две соседние цифры а и b (а стоит перед Ь) и на их место вставляется число а+26 (можно в качестве а взять нуль, «стоящий» перед числом, а в качестве Ь — первую цифру числа). С полученным числом про- изводится такая же операция и т. д. (Например, из числа 118 307 можно на первом шаге получить числа 218 307, 38 307, 117 307, 111 407, 11 837, 118 314.) Доказать, что таким способом можно получить число 1. 23. Некий фермер приобрел квадратный участок земли, обнес его забором и получил у доверчивого арендатора документ, в котором сказано, что он имеет право несколько раз произвести следующую операцию: провести прямую через любые две точки забора, огораживающего его участок, снести участок забора меж- ду этими двумя точками по одну сторону от прямой и достроить такой же кусок забора с другой стороны симметрично снесенной части относительно выбранной прямой. Сможет ли он такими операциями увеличить площадь своего участка? 24. Двое играют в следующую игру. Каждый игрок по оче- реди вычеркивает 9 чисел (по своему выбору) из последователь- ности 1, 2, 3, ..., 100, 101. После одиннадцати таких вычерки- ваний останутся два числа. Затем второй игрок присуждает первому столько очков, какова разница между этими оставшимися числами. Доказать, что первый игрок всегда сможет набрать по крайней мере 55 очков, как бы ни играл второй. 25*. В круглый пудинг радиуса 10 см запечена жемчужина радиуса 3 мм. Мы хотим ее найти. Для этого разрешается разрезать пудинг острым ножом по прямой на две (одинаковые или разные) части. Если жемчужина не попадет под нож, можно одну из этих частей снова разрезать; если она снова не будет обнару- жена, можно разрезать одну из трех получившихся частей и т. д. Доказать, что, как бы мы ни резали, может случиться, что после 32 разрезов жемчужина все еще не будет обнаружена. Доказать, что можно так сделать 33 разреза, что жемчужина обязательно будет обнаружена, где бы она ни находилась. 107
VIII класс 26. См. задачу 2. 27. На шахматной доске на поле al стоит белый конь. Двое по очереди замазывают по одной клетке шахматной доски бокси- товым клеем. При этом они должны замазывать так, чтобы конь мог пройти в любую незамазанную клетку, нигде не при- клеившись (конь ходит по обычным шахматным правилам). Про- игрывает тот, кто не может сделать очередного хода. Кто выигры- вает при правильной игре: сделавший первый ход или его партнер? 28*. Имеется два правильных пятиугольника с одной общей вершиной. Вершины каждого пятиугольника нумеруются по ча- совой стрелке цифрами от I до 5, причем в общей вершине ставится цифра I. Вершины с одинаковыми номерами соединены прямыми. Доказать, что полученные четыре прямые пересекаются в одной точке. 29. Из натуральных чисел составляются последовательности, в которых каждое последующее число больше квадрата предыду- щего, а последнее число в последовательности равно 1969 (по- следовательности могут иметь разную длину). Доказать, что различных последовательностей такого вида меньше чем 1969. 30*. В ряд поставлено 100 кубиков: 77 черных и 23 белых. Они расставлены приблизительно равномерно, т. е. если в лю- бом месте отметить некоторое количество кубиков подряд и потом в другом месте отметить такое же число кубиков подряд, то число черных кубиков в первом наборе отличается от числа черных кубиков во втором наборе не более чем на 1; при этом, если первый набор стоит на л е в о м краю, то число черных кубиков в нем не больше, чем во втором наборе, а если первый набор стоит на правом краю, то число черных кубиков в нем не меньше, чем во втором наборе. Доказать, что если расставить другой набор из 77 черных и 23 белых кубиков так, чтобы выпол- нялись те же условия, то белые кубики будут стоять на тех же местах. IX класс 31. Двое играют в следующую игру. Каждый игрок по очере- ди вычеркивает одно число из ряда 1, 2, 3, ..., 27 до тех пор, пока не останется два числа. Если сумма этих чисел делится на 5, то выигрывает первый игрок, если не делится — то второй. Кто выиграет при правильной игре: первый или второй? 32. Имеется пятак, с помощью которого можно чертить окруж- ности на плоскости (обводить пятак). Разрешается с его помо- щью проводить окружность через одну или две данные точки (расположенные достаточно близко друг от друга). На плоскости заданы три точки, которые можно закрыть одним пятаком и ко- 108
торые не лежат на одной прямой или на одной окружности, равной окружности пятака. Построить четвертую точку так, чтобы полученные четыре точки лежали в вершинах параллелограмма. Разрешается использовать только один пятак. 33*. Одна под другой выписаны 2Л“| различных последова- тельностей из нулей и единиц длины п. Известно, что для любых трех из выписанных последовательностей найдется такой номер р, что в р-м разряде у них стоит 1 (у всех трех). Доказать, что в некотором разряде у всех выписанных последовательностей стоит 1 и такой разряд только один. 34*. В стране Анчурии, где правит президент Мирафлорес, приблизилось время новых президентских выборов. В стране ров- но 20 миллионов избирателей, из которых только один процент поддерживает Мирафлореса (регулярная армия Анчурии). Ми- рафлорес, естественно, хочет быть избранным, но, с другой сторо- ны, он хочет, чтобы выборы были «демократическими». «Демокра- тическим голосованием» Мирафлорес называет вот что: все изби- ратели разбиваются на равные группы; каждая из этих групп вновь разбивается на некоторое количество равных групп, при- чем большие группы могут разбиваться на разное количество меньших групп, затем эти группы снова разбиваются и т. д. В са- мых мелких группах выбирают представителя группы — выбор- щика — для голосования в большей группе: выборщики в этой большей группе выбирают выборщика для голосования в еще большей группе и т. д. Наконец, представители самых больших групп выбирают президента. Мирафлорес делит избирателей на группы по своей воле и инструктирует своих сторонников, как им голосовать. Сможет ли он так организовать «демократические» выборы, чтобы его выбрали? (В каждой группе выборщики выбирают своего представителя простым большинством. При равенстве голосов побеждает оппозиция.) 35*. Правильный 1000-угольник разбит непересекающимися диагоналями на треугольники. Доказать, что среди этих диагона- лей найдется не менее 8 диагоналей, длины которых попарно различны. X класс 36. Два мудреца играют в следующую игру. Выписаны числа 0, 1, 2, ..., 1024. Первый мудрец зачеркивает 512 чисел (по своему выбору), второй зачеркивает 256 из оставшихся, затем снова первый зачеркивает 128 чисел и т. д. На десятом шаге второй мудрец зачеркивает одно число; остаются два числа. После этого второй мудрец платит первому разницу между этими числами. Как выгоднее играть первому мудрецу? Как второму? Сколько уплатит второй мудрец первому, если оба будут играть наилучшим образом? (Ср. с задачами 24 и 31.) 109
37. Жесткая проволока выгнута в форме правильного тре- угольника, и края ее запаяны. Разрешается перегибать кусок проволоки, заключенный между любыми двумя ее точками так, чтобы отогнутый кусок был симметричен прежнему относитель- но прямой, проходящей через эти точки (если эти точки совпа- дают, то прямую через них можно проводить произвольным образом). Так разрешается делать несколько раз. Можно ли не- сколькими такими операциями получить правильный шестиуголь- ник того же периметра? (Ср. с задачей 23.) 38. См. задачу 25, заменив круг на шар радиуса 20 см и количество разрезов 32 и 33 на 65 и 66 соответственно. 39. На клетках шахматной доски 8x8 написаны числа, сум- ма которых равна нулю. Затем каждую клетку разбили на 4 ячейки (вертикальными и горизонтальными линиями). Можно ли так вписать числа в ячейки, чтобы одновременно выполнялись условия: а) в ячейках у края доски стояли нули; б) для каждой клетки суммы чисел, стоящих в четырех ячейках этой клетки, равнялись числу, которое раньше было в ней написано; в) для каждого узла шахматной доски (там, где встречаются 4 клетки) сумма чисел, стоящих в примыкающих к нему четырех ячейках, равнялась нулю? 40*. Требуется расставить 1969 кубиков в ряд так, чтобы часть из них — число между 0 и 1969 — были белыми, а осталь- ные — черными, причем цвета должны распределяться прибли- зительно равномерно (см. условие задачи 30). Доказать, что существует по крайней мере 1970 различных способов расстанов- ки кубиков в ряд так, чтобы выполнялись эти требования. XXXIII ОЛИМПИАДА (1970 г.) 1-й т у р VII класс 1. На бесконечной шахматной доске на двух соседних по диагонали черных полях стоят две черные шашки. Можно ли дополнительно поставить на эту доску некоторое число черных шашек и одну белую таким образом, чтобы белая о д н и м х о- д о м взяла все черные шашки, включая две первоначально стоявшие? 2. На 99 карточках пишутся числа 1, 2, 3, ..., 99. Затем карточки перемешиваются, раскладываются чистыми сторонами вверх и на чистых сторонах снова пишутся числа 1, 2, 3, 4, ..., 99. Для каждой карточки числа, стоящие на ней, складываются и 99 полученных сумм перемножаются. Доказать, что в результате получится четное число. 3. Внутри правильного треугольника АВС лежит точка О. Известно, что Z. АОВ= 113°, Z_BOC= 123°. Найти углы треуголь- ника, стороны которого равны отрезкам ОА, ОВ9 ОС. по
4. В наборе имеется 100 гирь, каждые две из которых отли- чаются по массе не более чем на 20 г. Доказать, что эти ги- ри можно положить на две чашки весов, по 50 штук на каж- дую, так, чтобы одна чашка весов была легче другой не более чем на 20 г. 5. В городе X имеется 1000 коттеджей, в каждом из которых живет по одному человеку. В один прекрасный день каждый че- ловек переезжает из своего дома в какой-либо другой (переезд осуществляется так, что после него в каждом доме живет один жилец). Доказать, что после переезда можно так покрасить все 1000 коттеджей синей, зеленой и красной красками, чтобы у каждого хозяина цвет его нового дома отличался от цвета ста- рого дома. V111 класс 6. См. задачу 2. 7. Около окружности описан пятиугольник ABCDE, стороны которого —-целые числа и АВ = CD = 1. Окружность каса- ется стороны ВС в точке К. Найти длину отрезка ВК. 8. На прямоугольном листе бумаги расставлены 16 черных точек. Для каждой пары точек проделывается следующая опе- рация: они соединяются отрезком, и затем прямоугольник, диаго- налью которого является этот отрезок, а стороны параллельны сторонам листа, закрашивается красным цветом (имеется в виду, что черные точки просвечивают на красном фоне). Сколько сторон может иметь закрашенная фигура? (Указать все ответы, которые могут получиться при различных расположениях точек.) 9. На каждую чашку весов положили k гирь, занумерован- ных числами от 1 до k, причем левая чашка перевесила. Ока- залось, что если поменять чашками любые две гири с одинаковыми номерами, то всегда либо правая чашка начинает перевешивать, либо чашки приходят в равновесие. При каких k это возможно? 10. 12 теннисистов участвовали в турнире. Известно, что каж- дые два теннисиста сыграли между собой ровно один раз и не было ни одного теннисиста, проигравшего все встречи. Дока- зать, что найдутся теннисисты Л, В, С такие, что А выиграл у В, В у С, С у А. (В теннисе ничьих не бывает.) IX класс 11. Вдоль прямой дороги располагались 113 дворцов, в каж- дом из которых жил король. Ежедневно один из них устраи- вал прием, на который утром съезжались все остальные, а вече- ром слуги развозили их по домам. Так они жили год и никуда больше не выезжали. Доказать, что самый большой путь за этот год проехал один из королей, живущих в крайних дворцах. 12. Какое максимальное количество черных шашек можно рас- ставить на шашечной доске 8X8 так, чтобы простая белая шаш- 111
ка могла взять их все за один ход, не попадая при этом в дамки? Шашки стоят на черных полях. 13. Дано 999-значное число. Известно, что если взять из него любые 50 последовательных цифр и вычеркнуть все осталь- ные, то полученное число будет делиться на 2 . (Оно может на- чинаться с нулей или просто быть нулем.) Доказать, что исходное число делится на 2"9. 14. Построить треугольник по радиусу описанной окружности и биссектрисе угла Л, если известно, что Z.B—Z.C=90°. 15. Мудрый таракан, который видит не дальше, чем на 1 см, решил отыскать Истину. Находится она в точке, расстояние до которой D см. Таракан может делать шаги, каждый длиной не более 1 см, и после каждого шага ему говорят, приблизился он к Истине или нет. Таракан может помнить все, в частности направ- ление своих шагов. Доказать, что он сможет отыскать Истину, з сделав не более —О-|-7 шагов. X класс 16. Масса каждой из 19 гирь не больше 70 г и равна целому числу граммов. Доказать, что из этих гирь нельзя составить более 1230 различных по массе наборов. 17. В угол АВС вписаны две непересекаюшиеся окружности Oi и Ог. М — точка касания Oi с BA, Р — точка касания О2 с ВС. Доказать, что окружности Oi и О2 высекают на прямой МР хорды равной длины. 18. У числа 21970 зачеркнули его первую цифру и прибавили ее к оставшемуся числу. С результатом проделали ту же опера- цию и т. д., до тех пор пока не получили 10-значное число. Доказать, что в этом числе есть две одинаковые цифры. 19. На плоскости даны 200 точек, никакие три из которых не лежат на одной прямой. Можно ли занумеровать эти точки номера- ми от 1 до 200 так, чтобы все сто прямых, проходящих через точки с номерами 1 и 101, 2 и 102, ... , 100 и 200, попарно пересекались? 20. В некоторых клетках таблицы 100 X Ю0 стоят крестики. Известно, что в каждой строке стоит хотя бы один крестик и в каждом столбце тоже. Доказать, что можно отметить 10 строк и 10 столбцов так, что если стереть все крестики в отмеченных строках и столбцах, то в каждой неотмеченной строке останется хотя бы один крестик и в каждом столбце тоже. 2-й тур VII класс 21. Доказать, что если целое положительное число k делится на 999 999 999 (9 девяток), то в его десятичной записи более 8 цифр отличны от нуля. 112
22. На окружности радиуса 1 отмечено 100 точек. Доказать, что на этой окружности можно найти такую точку, чтобы сумма расстояний от нее до всех отмеченных точек была больше 100. 23. В парке шесть узких аллей одинаковой длины, четыре из которых идут по сторонам квадрата и две по его средним линиям. По этим аллеям мальчик Коля убегает от папы и мамы. Смогут ли папа и мама поймать Колю, если он бегает втрое быстрее их (все трое все время видят друг друга)? 24. Квадратный лист бумаги разрезают по прямой на две части. Одну из этих частей снова разрезают по прямой на две части. Один из полученных трех листов бумаги разрезают на две части и т. д. Какое наименьшее число разрезов нужно сделать, чтобы получить 73 тридцатиугольника? 25. Король Людовик не доверяет некоторым своим придвор- ным. Он составил полный список придворных и приказал каждому из них следить за одним из остальных. При этом первый придворный следит за тем, кто следит за вторым, второй следит за тем, кто следит за третьим, и т. д., предпослед- ний следит за тем, кто следит за последним, последний следит за тем, кто следит за первым. Доказать, что у Людовика нечетное число придворных. VIII класс 26. Внутри круга радиуса 1 м расположены п точек. Дока- зать, что в круге или на его границе существует точка, сумма расстояний от которой до всех точек не меньше п метров. (Ср. с задачей 22.) 27. В маленьком зоопарке из клетки убежала обезьяна. Ее ло- вят два сторожа. И сторожа, и обезьяна бегают только по дорож- кам. Всего в зоопарке 6 прямолинейных дорожек: 3 длинные образуют правильный треугольник, 3 короткие соединяют середи- ны его сторон. В каждый момент времени обезьяна и сторожа ви- дят друг друга. Смогут ли сторожа поймать обезьяну, если обезьяна бегает в три раза быстрее сторожей? (Внача- ле оба сторожа находятся в одной вершине треугольника, а обезьяна в другой. Ср. с задачей 23.) 28. На участке земли квадратно-гнездовым способом посажено 10 000 деревьев: 100 рядов по 100 деревьев. Какое наибольшее число деревьев можно срубить, чтобы выполнялось следующее условие: если встать на любой пень, то за деревьями не будет видно ни одного другого пня? Деревья считать достаточно тонкими. 29. На ленте записаны подряд 80 ненулевых цифр. Разрезаем ленту на несколько кусков, длина каждого из которых больше, чем одна цифра. Числа, получившиеся на кусках, складываем. Доказать, что имеется по крайней мере два различных способа разрезания ленты, при которых получаются одинаковые суммы. из
30. См. задачу 32. Доказать, что если проволока имеет форму ломаной и расстояние между ее концами больше (2 + 2^) м, то по всему коридору ее протащить нельзя. IX класс 31. N деталей полотна игрушечной железной дороги имеют форму четверти окружности радиуса 10 см. Доказать, что, после- довательно присоединяя их концами так, чтобы они плавно переходили друг в друга, нельзя составить такой путь, чтобы его начало совпадало с концом, а первое и последнее звенья образовывали бы нулевой угол (рис. 5). 32. Плоский коридор шириной 1 м имеет форму буквы «Г» и бесконечен в обе стороны. Необходимо изготовить плоский ку- сок несгибаемой проволоки — не обязательно прямой — такой, чтобы его можно было протащить по всему коридору. Каково максимальное возможное расстояние между концами проволоки? 33. Во всех клетках таблицы 100X100 стоят плюсы. Раз- решается одновременно менять знаки во всех клетках одной строки или же во всех клетках одного столбца. Можно ли, пользуясь только этими операциями, получить 1970 минусов? 34. См. задачу 24; требуется получить сто 20-угольников. 35. По ребрам прозрачного куба ползают три паука и муха. Наибольшая скорость мухи в три раза больше наибольшей скорости пауков. Вначале пауки находились в одной вершине куба, муха — в противоположной. Смогут ли пауки поймать муху? (Пауки и муха все время видят друг друга.) X класс 36*. Около сферы радиуса 10 описан некоторый 19-гранник. Доказать, что на его поверхности найдутся две точки, расстоя- ние между которыми больше 21. 37. Доказать, что если целое положительное число k делится на 10 101 010 101, то в его десятичной записи по крайней мере 6 цифр отличны от нуля. 38*. См. задачу 35, где муху ловят 2 паука, а максимальные скорости паука и мухи совпадают. 39. Имеется натуральное число п >1970. Возьмем остатки от деления числа 2" на 2, 3, 4, ..., п. Доказать, что .сумма этих остатков больше 2л. 40. У Мерлина есть две таблицы 100X100; одна из них пустая, а на дру- гой, волшебной, написаны какие-то числа. Первая таблица прибита к скале у вхо- да в пещеру, а вторая — к стене внутри пещеры. Вы можете обвести на первой таб- лице какой-нибудь квадрат (размером 114
1X1» 2X2, ... или 100X100), расположенный в любом месте таблицы, и за шиллинг узнать у Мерлина сумму чисел, стоящих в клетках соответствующего квадрата на волшебной таблице. Какое наименьшее количество денег потребуется, чтобы узнать сумму чисел на диагонали волшебной таблицы? Дополнительный тур. День Пифагора VII класс 41. Число 1234567 ... 1000 умножили на какое-то целое число от I до 9 и вычеркнули из произведения все единицы. Оставшееся после этого число опять умножили на однозначное число и вычеркнули все единицы и так проделали много раз. Какое наименьшее число удастся таким образом получить? 42. Имеется зал размером 13X13 м, разбитый на метровые квадраты. Разрешается класть прямоугольные ковры произволь- ных размеров так, чтобы их стороны шли по сторонам решетки (в частности, по границе зала). Можно класть ковры так, чтобы они частично перекрывались и даже чтобы некоторые из них полностью перекрывались остальными, но не допускается, чтобы один ковер полностью лежал на другом или под другим (даже если между ними имеются еще ковры). Какое наибольшее количе- ство ковров вы можете положить, чтобы эти условия выполня- лись? 43. При обычной игре в домино кости выкладываются так, чтобы разность между числами на соседних костях равнялась 0. Можно ли выложить все 28 костей в замкнутую цепь так, чтобы все эти разности равнялись ±1? 44. Можно ли разбить числа 1, 2, 3, ..., 33 на 11 групп, по 3 числа в каждой, так, чтобы в каждой группе одно из чисел равнялось сумме двух других? 45. Али-Баба пытается проникнуть в пещеру. У входа в нее стоит барабан с четырьмя отверстиями по бокам. Около каждого отверстия внутри поставлен переключатель, имеющий 2 положе- ния: «верх», «низ». Разрешается засунуть руки в какие-либо 2 отверстия, пощупать, как стоят переключатели, и переключить их произвольным образом (в частности, можно не переключать). После этого барабан приходит в быстрое вращение, так что после его остановки уже нельзя установить, какие именно переключа- тели трогали в прошлый раз. Разрешается повторить эту опера- цию до 10 раз. Дверь в пещеру открывается в тот момент, когда все переключатели стоят одинаково. Доказать, что Али- Баба сумеет попасть в пещеру. 46. Известно, что в кадр фотоаппарата, расположенного в точке О, не могут попасть предметы А и В такие, что угол АОВ больше 179°. На плоскости поставлено 1000 таких фото- 115
аппаратов. Одновременно каждым фотоаппаратом делают по од- ному снимку. Доказать, что найдется снимок, на котором сфотографировано не больше 998 фотоаппаратов. XXXIV ОЛИМПИАДА (1971 г.) 1-й тур VIII класс 1. Город обнесен стеной, имеющей форму 1000-угольника (несамопересекающегося, но не обязательно выпуклого). На каж- дом углу с внешней стороны стоит часовой. Доказать, что найдется часовой, который видит менее пятисот других часовых. 2. Окружность пересекается с выпуклым пятиугольником ABCDE в точках Ль Лг, Bi, В2, ... , Е\. Е2 (рис. 6). Известно, что ЛЛ|=ЛЛг, ВВ|=ВВг, СС| = ССг, Ж = ОО2. Доказать, что ЕЕ\=ЕЕ2, 3. Во всесоюзном футбольном турнире участвовало 25 команд. После окончания турнира оказалось, что ни в одной встрече ни одна из команд не забила в ворота противника более четы- рех мячей. Какое самое низкое место в турнирной таблице могла занять команда города Тбилиси, забившая мячей больше, чем любая другая команда, и пропустившая меньше любой другой команды? 4. На клетчатой бумаге нарисован квадрат 100X100 клеток. Внутри каждой клетки поставили красную или синюю точку, так что в каждом столбце оказалось 50 синих и 50 красных точек и в каждой строке оказалось 50 синих и 50 красных точек. Соединим каждые две красные точки, расположенные в соседних квадратах (имеющих общую сторону), красным отрез- ком, а каждые две синие точки в соседних квадратах — синим отрезком. Доказать, что красных отрезков получится столь- ко же, сколько и синих. 5. Обозначим количество цифр в числе А череЬ k (Л). До- казать, что число k (5*09070*) — — fe^1090701) делится на 2. IX класс 6. В вершинах правильного 25-угольника расположены чис- ла О|, 02, Оз, ...» 025» причем О| =02= ... = О|3= 1, О|4 = О|5 = = ... = а25= —1. Над этими чис- лами произвели следующую операцию: к каждому числу прибавили ближайшее к нему Н6
о) б) Рис. 7 число по часовой стрелке. Например, к аг прибавили а^, к 6125 прибавили а\. Полу- ченные числа 6|, 6>, ..., 625 расставили в вершинах вместо чисел аь а*, аз, ..., 025 в том же порядке и произвели над ними ту же операцию и т. д. Всего эту опера- цию проделали 100 раз. Доказать, что одно из полученных чисел будет больше 1О20. 7. Пусть задан выпуклый fc-угольник P(fc>6), периметр кото- рого равен 2. Построим новый выпуклый fc-угольник Л4, верши- ны которого являются серединами сторон fc-угольника Р. Дока- зать, что периметр многоугольника М больше 1. 8. На плоскости проведено п прямых (п>2), причем никакие две из них не параллельны и никакие три не пересекаются в одной точке. Известно, что можно повернуть плоскость вокруг некоторой точки О на некоторый угол а <180° так, что каждая из проведенных прямых совместится с какой-нибудь другой про- веденной прямой. Указать все числа л, для которых это воз- можно. 9. Дано число 2*, где k — натуральное число, большее 3. Доказать, что никакое число, полученное из данного перестанов- кой его цифр, не равно 2Л, где п — любое натуральное число, большее k. 10. Доказать, что среди чисел [2**-\/2] бесконечно много составных. X класс 11. Дана замкнутая пространственная ломаная с вершинами Л1, Л2,... » Ап, причем каждое звено пересекает фиксирован- ную сферу в двух точках, а все вершины ломаной лежат вне сферы. Эти точки делят ломаную на Зп отрезков. Известно, что отрезки, прилегающие к вершине Ль равны между собой. То же самое верно и для вершин Лг, Аз, ...» Лл_1. Доказать, что отрезки, прилегающие к вершине Ля, также равны между собой. (Ср. с задачей 2.) 12. У Пети имеется набор «Юный паркетчик», который состоит из дощечек, уложенных в один слой в прямоугольную коробку так, что они покрывают всю ее площадь. Каждая дощечка имеет площадь 3 см2 и имеет форму либо прямоуголь- ника, либо уголка (рис. 7). Петя сказал, что он потерял дощечку в форме уголка, сделал вместо нее прямоугольную дощечку и уложил после этого все дощечки вместе с новой в один слой в коробку. Можно ли утверждать, что он лжет? 13. Про последовательность xi, Х2,... ,хп,... известно, что для любого п>\ выполнено равенство Зхп —хп-\=п. Кроме того, известно, что |xi | < 1971. Вычислить Х1971 с точностью до 0,000001. 117
14. п точек расположены в вершинах выпуклого п-угольника. Внутри этого n-угольника отметили k точек. Оказалось, что любые три из n + k точек не лежат на одной прямой и являются вершинами равнобедренного треугольника. Чему может быть рав- но число k? 15. Лежит кучка в 10 миллионов спичек. Двое играют в следую- щую игру. Ходят по очереди. За один ход играющий может взять из кучки спички в количестве рл, где р — простое число, n = 0, 1, 2, 3,... (например, первый берет 25 спичек, второй — 8, первый — 1, второй — 5, первый — 49 и т. д.). Выигрывает тот, кто берет последнюю спичку. Кто выигрывает при правиль- ной игре? 2-й т у р VII класс 16. Существует ли число, квадрат которого начинается с цифр 123456789 и кончается цифрами 987654321? 17. Дан квадрат ABCD и точка О внутри. Из точек Л, В, С, D опускаются перпендикуляры АН\, ВНг, СНз, DH4 соответственно на прямые ВО, СО, DO, АО. Доказать, что прямые, на которых лежат эти перпендикуляры, пересекаются в одной точке. 18. В колбе находится колония из п бактерий. В какой-то момент внутрь колбы попадает вирус. В первую минуту вирус уничтожает одну бактерию, и сразу же после этого и вирус, и оставшиеся бактерии делятся пополам. Во вторую минуту новые два вируса уничтожают две бактерии, а затем и вирусы, и остав- шиеся бактерии снова делятся пополам, и т. д. Наступит ли такой момент времени, когда не останется ни одной бактерии? 19. Имеется сетка, состоящая из квадратов размером 1X1. Каждый ее узел покрашен в один из четырех данных цветов так, что вершины любого квадрата IX1 покрашены в разные цвета. Доказать, что найдется прямая, принадлежащая сетке, такая, что узлы, лежащие на ней, покрашены в два цвета. 20. На плоскости расположены 7 точечных прожекторов. Каждый прожектор освещает угол в 90°. Если в квадранте, освещенном каким-либо прожектором, окажется другой прожек- тор, то от последнего упадет тень — темный бесконечный луч. Доказать, что эти 7 прожекторов можно расположить так, что от каждого из них будет падать тень длиной 7 км. VIII класс 21. Дано 29-значное число X — а\а?аз ... 028^29 (0^а*^9, Oi=/=0). Известно, что для всякого k цифра а* встречается в записи данного числа азо-л раз (например, если сю=7, то цифра а2о встречается 7 раз). Найти сумму цифр числа X. 118
22. Имеется картонный 1000-угольник (не обязательно выпук- лый). Этот многоугольник разрезали по прямой линии один раз. Он распался при этом на несколько новых многоугольников. Какое наибольшее число треугольников могло получиться среди этих многоугольников? 23. Доказать, что сумма цифр числа К не более чем в 8 раз превосходит сумму цифр числа 8К. 24. Разрешается взять любое число, состоящее из нулей и четверок, и проделать с ним следующие операции: поделить на 2, на 3 или на 5, если это деление возможно нацело, или же вставить 0 или 4 между цифрами этого числа, или приписать О или 4 к числу с любой стороны (справа или слева). С получен- ным числом можно проделывать те же операции и т. д. Можно ли таким способом получить любое натуральное число? 25. См. задачу 17. IX класс 26. Дан выпуклый 1971-угольник. Из каждой его вершины Ап каждая его сторона, не проходящая через ЛЛ, видна под одним и тем же углом аЛ. Доказать, что многоугольник правиль- ный. 27. См. задачу 21. 28. Можно ли каждую сторону квадрата так разделить на 100 частей, чтобы из полученных 400 отрезков нельзя было бы составить контура никакого прямоугольника, отличного от исход- ного квадрата? 29. Окружность разделена на п равных частей, и в точках деления написаны числа х\, *2, ... , хп, равные +1 или — 1. 360° Если повернуть окружность на угол ft——, перемножить числа в совместившихся точках и сложить полученные п произведений, то для всякого Л=0, 1,..., л — 1 эта сумма будет равной 0. Дока- зать, что число п является точным квадратом целого числа. 30. Доказать, что можно расставить в вершинах правильного л-угольника действительные числа хг...., хп. все отличные от 0, так, чтобы для любого правильного ^-угольника, все вершины которого являются вершинами исходного л-угольника, сумма чи- сел, стоящих в его вершинах, равнялась 0. X класс 31. См. задачу 29. Дополнительный вопрос: чему может равняться число л? 32. Даны два набора чисел: С|,..., ап и 61, ..., Ьп. Расположим числа ak в возрастающем порядке, а числа bk — в убывающем порядке. Получатся наборы ... 6*^ 119
...^6*. Доказать, что max (ai +6 ь 02 + ^2»...» + >max (аТ+бГ,... , a*+ft^). 33. Банкир и Игрок играют в следующую игру. Банкир называет 1000-значное число А\. Игрок, узнав это число, предла- гает Банкиру произвольное число В|. После этого Банкир по своему усмотрению вычитает из большего числа меньшее или складывает их, а результат сообщает Игроку — это число Лг. Затем Игрок предлагает Банкиру следующее число Вг- Банкир повторяет ту же операцию с числами Аг и Вг и т. д. Игра кончается, если у Банкира оказывается одно из следующих чисел: 1, 10, 100, 1000, ... . Доказать, что Игрок всегда может закончить игру, предложив Банкиру не более 20 своих чисел. 34. В пространстве даны точка О и п попарно непарал- лельных прямых. Точка О ортогонально проектируется на все данные прямые. Каждая из получившихся точек снова проекти- руется на все данные прямые и т. д. Существует ли шар, содержащий все точки, которые могут быть получены таким образом? 35* . Доказать, что сумма цифр числа W превосходит сумму цифр числа 55-М не более чем в 5 раз. (Ср. с задачей 23.) XXXV ОЛИМПИАДА (1972 г.) 1-й т у р VII класс 1. Дано 17 натуральных чисел: дь аг,... , Д|7. Известно, что а?2 = а23=д§4= ... =а^7=а^. Доказать, что д1=а2 = — ... =Д|7. 2. На конгресс приехали 1000 делегатов из разных стран. Каждый делегат знает несколько языков. Известно, что любые трое могут разговаривать между собой без помощи остальных. (При этом, возможно, одному из них придется переводить разговор двух других.) Доказать, что всех делегатов можно расселить в 500 комнатах так, чтобы в каждой комнате располагались 2 делегата и при этом они могли бы поговорить между собой. 3. Каждая вершина правильного 13-угольника покрашена ли- бо в черный, либо в белый цвет. Доказать, что существуют три точки одного цвета, лежащие в вершинах равнобедренного треугольника. 4. См. задачу 9; доказать, что АВ = ВС. VIII класс 5. В некоторых клетках квадратной таблицы пХп стоят звездочки. Известно, что если вычеркнуть любой набор строк (только не все), то найдется столбец ровно с одной невычерк- 120
нутой звездочкой. (В частности, если строки совсем не вычерки- вать, то столбец ровно с одной звездочкой существует.) Доказать, что если вычеркнуть любой набор столбцов (только не все), то найдется строка ровно с одной невычеркнутой звездочкой. 6. На плоскости лежат две одинаковые фигуры, имеющие форму буквы «Г». Концы коротких палочек у букв «Г» обозна- чим через А и А'. Длинные палочки разделены на п равных частей точками аь ..., ал_г, а[,... ,а£_| (точки деления ну- меруются от концов длинных палочек). Проводятся прямые Лаь Лаг, — , Лал_|; Л'а(, А'а^ ..., Д'ал_|. Точку пересечения прямых Aai и Л'а( обозначим через Хь прямых Лаг и Л'аг — через Хг и т. д. Доказать, что точки Xi, Хг,... ,ХЛ_| образуют выпуклый многоугольник. 7. Имеется 1000 монет, среди них 0, 1 или 2 фальшивые. Известно, что фальшивые монеты имеют одинаковую массу, отличную от массы нефальшивых монет. Можно ли за три взвешивания на чашечных весах без гирь определить, есть ли фальшивые монеты и легче они или тяжелее нормальных? (Коли- чество фальшивых монет определять не надо.) 8. Имеется набор натуральных чисел, причем сумма любых семи из них меньше 15, а сумма всех чисел из набора равна 100. Какое наименьшее количество чисел может быть в наборе? 9. В треугольнике АВС проведены медианы AD и BE. Углы CAD и СВЕ равны 30°. Доказать, что треугольник АВС правильный. IX класс 10. В треугольнике АВС угол С тупой. На стороне АВ отмечены точки Е и Я, на сторонах АС и ВС — точки К и М соответственно. Оказалось, что AH = ACt ЕВ — ВС, АЕ=АК, ВН = ВМ. Доказать, что точки Е, Н, К, М лежат на одной окружности. 11. В клетках шахматной доски размером иХи расставлены числа: на пересечении Л-й строки и m-го столбца стоит число а^/n. При любой расстановке на этой доске п ладей, при которой никакие две из них не бьют друг друга, сумма закрытых чисел равна 1972. Доказать, что существует два набора чисел xi, хг,..., хп и уи... ,ул, что при всех k и т выполняется равенство akm=xk+ут- 12. В некотором лесу расстояние между любыми двумя деревь- ями не превосходит разности их высот. Все деревья имеют высоту не более 100 м. Доказать, что лес можно обнести забором дли- ной 200 м. 13* . Натуральные числа тип взаимно просты и п<т. Какое число больше: [ 1 +[2-^-] + ... или [ 1 + +[2~] + ? 12!
14. Город X состоит из 10 бесконечных параллельных проспек- тов, пересекающих через равные интервалы поперечные улицы. Два полицейских, двигаясь вдоль проспектов и улиц, пытаются обнаружить гангстера, который может прятаться за домами. Если гангстер окажется на одном проспекте или на одной улице с каким-либо полицейским, он будет обнаружен. Скорость гангсте- ра не более чем в 10 раз превышает скорость полицейских, причем полицейским известно, что он в начальный момент времени находился от них на расстоянии не более 100 кварталов. Дока- зать, что полицейские смогут обнаружить гангстера. X класс 15. В городе «Многообразие» живут п жителей, любые два из которых либо дружат, либо враждуют между собой. Каждый день не более чем один житель может начать новую жизнь: перессориться со всеми своими друзьями и подружиться со всеми своими врагами. Доказать, что все жители могут подружиться. (Примечание. Если А — друг В, а В — друг С, то Л — также друг С.) 16. Дана бесконечная последовательность чисел аь аг,.... ал,..., причем каждое следующее число получено из предыдущего приписыванием справа любой цифры, кроме 9. Известно, что а\ — произвольное десятизначное число. Доказать, что в этой последовательности не менее двух составных чисел. 17. У тетраэдра ABCD все двугранные углы острые, а противоположные ребра попарно равны. Найти сумму косинусов всех двугранных углов тетраэдра. 18. Имеется несамопересекающийся невыпуклый п-угольникР. Рассмотрим множество Т его внутренних точек, из которых видны все вершины Р. Доказать, что Т — выпуклый много- угольник, число сторон которого не больше п. 19. См. задачу 14. 2-й т у р VII класс 20. Дан выпуклый четырехугольник ABCD, Точка пересечения диагоналей обозначена через О. Известно, что периметры треугольников ЛВО, ВСО, CDO, ADO равны между собой. Дока- зать, что ABCD — ромб. 21. На плоскости проведены четыре прямые a, b, с, d. Ника- кие две из них не параллельны и никакие три не пересекаются в одной точке. Известно, что прямая а параллельна одной из медиан треугольника, образованного прямыми fe, с, d. Доказать, что прямая Ь параллельна некоторой медиане треугольника, образованного прямыми а, с и d. 122
22. Даны 12 последовательных натуральных чисел. Доказать, что хотя бы одно из них меньше суммы своих делителей. 23* . В стране Мара расположено несколько замков. Из каждо- го замка ведут три дороги. Из какого-то замка выехал рыцарь. Странствуя по дорогам, он из каждого замка, стоящего на его пути, поворачивает либо направо, либо налево по отношению к дороге, по которой приехал. Рыцарь никогда не сворачивает в ту сторону, в которую он свернул перед этим. Доказать, что когда- нибудь он вернется в исходный замок. 24. Прямая пересекает стороны АВ и ВС треугольника АВС в точках М и К. Известно, что площадь треугольника МВ К равна площади четырехугольника АМКС. Доказать, что мв+вк 1 ЛМ+СЛ+КС 3 • VIII класс 25. См. задачу 20. 26. Числа a, b, с, d, е и f — натуральные. Известно, что af—be—l. Доказать, что d^b+f. 27. В городе Никитовка двустороннее движение. В течение двух лет в городе проходил ремонт всех дорог. Вследствие этого в первый год на некоторых дорогах было введено односторон- нее движение. На следующий год на этих дорогах было восста- новлено двустороннее движение, а на остальных дорогах введено одностороннее движение. Известно, что в любой момент ремонта можно проехать из любой точки города в любую другую. Доказать, что в Никитавке можно ввести одностороннее движение так, что из любой точки города удастся проехать в любую другую точку. 28. Пусть К (х) равно числу несократимых дробей таких, что а<.х и Ь<.х(а и Ь— натуральные числа). Например, /<(-|-)=3 (дроби 1; 2; -у-) . Вычислить сумму к(|<ю>+к(т)+к(1зг) + - +к(^+к@ 29. См. задачу 34 для 300 прямых и 100 треугольников. IX класс 30. В пятиугольнике длины всех сторон равны между собой, а величины всех углов меньше 120°. Доказать, что все углы пятиугольника тупые. 31. См. задачу 26. 32*. Улицы города «Л4» представляют собой правильную квадратную сетку размером 20 X 20 кварталов. На некоторых 123
перекрестках имеются станции метро. Известно, что, выйдя на улицу в любом месте, можно добраться до метро, пройдя не более двух кварталов по улице. Какое наименьшее число стан- ций метро могло быть в городе «Л4»? 33*. Существуют ли рациональные числа a, b, с, d, удовлет- воряющие равенству (а + bffi + (с + dy/2)2n =5 + 4^2 (где п— натуральное число)? 34*. На плоскости проведено 3000 прямых, причем никакие две из них не параллельны и никакие три не пересекаются в одной точке. По этим прямым плоскость разрезана на куски. Доказать, что среди кусков найдется не менее: а) 1000 треуголь- ников, б) 2000 треугольников. X класс 35*. В пространстве расположены плоскость П и треугольник АВС, не принадлежащий этой плоскости. Треугольник Л]В|С] является прямоугольной проекцией треугольника АВС на плоскость /7. Доказать, что треугольник Л1В1С] может быть накрыт полностью треугольником, равным треугольнику АВС. 36. Даны два набора чисел: %i, *2, ..., хл и у\, у?, ..., уп. Известно, что: а) %|>хг> ... >хЛ>0, у\>у2> ... >ул>0; б) xi > yi, xi+%2 > yi+f/2» ... , ^1+^2+ ... +*л > У\ + У? + + ... + ул. Доказать, что для любого натурального k справедливо нера- венство xt+%2+ •••+*£> + У2+ •••+<4- 37. На четырехстах карточках написаны числа I, 2, 3, ..., 400. Игроки А и В играют в следующую игру: первым ходом А берет себе любые 200 карточек, а остальные отдает В. Затем В берет 100 карточек из каждого набора, а остальные отдает Л; таким образом, у обоих игроков оказалось по 200 карто- чек. Следующим ходом А снова берет по 100 карточек из каждо- го набора, а остальные отдает В и т. д. После 200-го хода игрока В оба подсчитывают сумму чисел на своих' карточках: СА и Св—и А выплачивает игроку В разность Св — СА. Какую максимальную сумму может получить игрок В при правильной игре обоих? 38. Рассмотрим все рациональные числа между нулем и еди- ницей, знаменатели которых не превосходят п. Расположим их в порядке возрастания. Пусть -у- и ~--какие-то два соседних числа (дроби несократимы). Доказать, что \bc — ad| = l. 39*. На всех клетках шахматной доски 8X8 расставлены натуральные числа. Разрешается выделить любой квадрат разме- ром 3X3 или 4X4 и увеличить все числа в нем на 1. Мы хотим в результате нескольких таких операций добиться, чтобы числа во всех клетках делились на 10. Всегда ли это удастся сделать? 124
XXXVI ОЛИМПИАДА (1973 r.| 1-й т у р VIII класс 1. На острове, имеющем форму квадрата, расположено не- сколько стран. Можно ли разбить эти страны на меньшие так, чтобы не появилось новых точек пересечения границ и чтобы можно было раскрасить карту этого острова в два цвета? 2. Может ли число, составленное из 600 шестерок и некоторо- го количества нулей, быть квадратом целого числа? 3. На плоскости расположены пять точек так, что никакие три из них не лежат на одной прямой и никакие четыре — на одной окружности. Доказать, что среди этих точек можно выбрать такие две точки, которые расположены по разные стороны от окружности, проходящей через оставшиеся три точки. 4. Рассматриваются решения (л, у) уравнения ~i~-у~=~р~ ’ где х, у и р — натуральные числа, р > 1. Доказать, что при простом р это уравнение имеет ровно 3 решения; если же р — составное, то число решений больше трех. Замечание. Решения (а, Ь) и (6, а) при а Ф b считаются различными. 5. Игра «Чехарда». В трех вершинах квадрата находятся три кузнечика. Они играют в чехарду, т. е. прыгают друг через друга, причем если кузнечик А прыгает через кузнечика В, то после прыжка он оказывается от В на том же расстоянии, что и до прыжка, и, естественно, на той же прямой. Может ли после нескольких прыжков один из кузнечиков попасть в четвертую вершину квадрата? IX класс 6. На каждой стороне параллелограмма взято по точке. Пло- щадь четырехугольника с вершинами в этих точках равна поло- вине площади параллелограмма. Доказать, что хотя бы одна из диагоналей четырехугольника параллельна стороне парал- лелограмма. 7. Квадрат разбит на выпуклые многоугольники. Доказать, что их можно подразбить на меньшие выпуклые многоугольни- ки так, чтобы в новом разбиении квадрата каждый многоуголь- ник граничил с нечетным числом соседей (соседи — многоуголь- ники с общей стороной). 8. Дан многочлен с целыми коэффициентами. В трех целых точках он принимает значение 2. Доказать, что ни в какой целой точке он не принимает значение 3. 9. В городе X с любой станции метро можно проехать на любую другую. Доказать, что одну из станций можно закрыть на ремонт без права проезда через нее так, чтобы с любой из оставшихся станций можно было проехать на любую другую. 125
10. Грани кубика пронумерованы числами I, 2, ..., 6, при- чем сумма чисел на противоположных гранях равна 7. Имеется шахматная доска 50X50 клеток; клетки доски равны граням куби- ка. Кубик перекатывается через ребра из левого нижнего угла доски в правый верхний угол. При перекатывании он каждый раз перемещается только вправо или вверх (влево и вниз переме- щаться запрещено), и на каждой клетке по пути его движения отпечатывается то число, которое расположено на грани, соприка- сающейся с этой клеткой. Какое наибольшее значение может при- нимать сумма всех написанных чисел? А какое наименьшее? X класс 11. С натуральным числом k проделывается следующая опера- ция. Оно сначала раскладывается на простые множители: £ = =Pi-Р2- ... -рл-ирл, а затем находится сумма Р1+Р2+ - + +Рл-1 +рл+ 1. С полученным числом проделывается та же опера- ция, и т. д. Доказать, что получающаяся последовательность чисел, начиная с некоторого момента, будет периодической. 12. См. задачу 6. 13. Многочлен Р(х) с целыми коэффициентами при некоторых целых х принимает значения 1, 2 и 3. Доказать, что существует не более одного целого числа х, при котором значение этого многочлена равно 5. 14. Доказать, что у всякого выпуклого многогранника най- дутся две грани с одинаковым числом сторон. 15. На «черном ящике» (с неизвестным внутренним устрой- ством) находится табло с N лампочками и пульт из N пере- ключателей на два положения — тумблеров. При переборе всех возможных состояний пульта на табло последовательно заго- раются все возможные комбинации лампочек. Состояние табло однозначно определяется по состоянию пульта. Известно, что всег- да при переключении одного тумблера гаснет или загорается ровно одна лампочка. Доказать, что состояние каждой лампочки зависит от положения ровно одного переключателя (для каждой лампочки своего). 2-й т у р VII класс 16. Из некоторого четырехзначного числа вычитают число, составленное из тех же цифр, но расположенных в обратном порядке. Может ли получиться число 1008? 17. Дан остроугольный треугольник АВС. Построены круги, центры которых расположены в вершинах дЛВС, а радиусы равны высотам, проведенным из этих вершин. Доказать, что каждая точка треугольника покрыта хотя бы одним из кругов. 126
18. Лист клетчатой бумаги размером 100X100 раскрасили в 100 цветов. При этом каждую клетку либо закрасили одним из этих цветов, либо вовсе не закрасили. Раскраска называется правильной, если в каждой строке нет двух клеток оди- накового цвета. Можно ли закрасить лист правильным спосо- бом так, чтобы оказались закрашенными все клетки, если перво- начально были правильно закрашены: а) 1002—1; б) 1002 — 2; в) 100 клеток? 19. См. задачу 22а. VIЧ класс 20. На бумагу поставили кляксу. Для каждой точки кляксы определили наименьшее и наибольшее расстояние до границы кляксы. Среди всех наименьших расстояний выбрали наибольшее, а среди всех наибольших — наименьшее и сравнили полученные два числа. Какую форму имеет клякса, если эти два числа оказались равными между собой? 21. См. задачу 18, заменив число 100 на произвольное п. 22. В центре площади, имеющей форму квадрата, находится полицейский, а в одной из вершин — гангстер. Полицейский может бегать по всей площади, а гангстер — только по сторо- нам квадрата. Известно, что отношение максимальной скорости полицейского к максимальной скорости гангстера равно: а) б) 0,49; в) 0,34; г) —. Доказать, что в каждом из этих случаев «5 полицейский может бегать так, что в какой-то момент окажется на одной стороне с гангстером. 23. Доказать, что в выпуклый равносторонний (не обязательно правильный) пятиугольник можно поместить правильный тре- угольник с той же длиной стороны так, чтобы одна его сторона совпала со стороной пятиугольника, а весь треугольник оказался внутри пятиугольника. IX класс 24. Имеется 100-значное число, состоящее из единиц и двоек. Разрешается выбрать произвольно 10 идущих подряд цифр и поменять первые пять цифр с пятью следующими. Два таких числа называются похожими, если одно получается из другого несколькими описанными операциями. Какое наибольшее коли- чество попарно непохожих чисел можно выбрать? 25. На бесконечной шахматной доске с клетками размером 1 X 1 проведена замкнутая несамопересекающаяся ломаная, про- ходящая по сторонам клеток. Внутри ломаной оказалось k чер- ных клеток. Какую наибольшую площадь может иметь фигура, ограниченная этой ломаной? 26. См. задачу 29 для т = 5. 127
27*. На концах отрезка стоит по единице. Первым шагом между ними ставится их сумма — число 2. Следующим шагом между каждыми двумя соседними числами ставится их сумма, и т. д. 1 000 000 раз. (На втором шаге получается последователь- ность I 3 2 3 1.) Сколько раз в результате будет написано число 1973? 28*. В условиях задачи 22 максимальная скорость гангстера в 2,9 раза больше скорости полицейского. Сможет ли полицейский оказаться на одной стороне с гангстером? X класс 29. Пусть тип — натуральные числа, не меньшие 2. Дока- зать, что найдется такое натуральное число k, что 4\m__fc-f- д/fc7—4 k 2 ) 2 30. У трехгранного угла проведены биссектрисы всех плос- ких углов. Доказать, что попарные углы между биссектрисами одновременно либо все острые, либо все тупые, либо все прямые. 31. Двенадцать маляров живут в 12 красных и белых домах, расположенных по кольцевой дороге. Каждый месяц один из маляров, взяв с собой достаточное количество белой и крас- ной краски, выходит из дома и идет вдоль кольцевой дороги по часовой стрелке. Увидев красный дом, он перекрашивает его в белый цвет и идет дальше, а увидев белый дом, он перекрашивает его в красный цвет и идет мыть кисть. В течение года каждый маляр ровно один раз проделывает такое путешест- вие. Доказать, что в конце года каждый дом будем покрашен в первоначальный цвет, если в начале года хотя бы один дом был красным. 32. См. задачу 24. 33*. По арене круглого цирка радиуса 10 м бегает лев. Двигаясь по ломаной линии, он пробежал 30 км. Доказать, что сумма всех углов, на которые поворачивал лев, не меньше 2998 радиан. XXXVII ОЛИМПИАДА (1974 г.) 1-й т у р IX класс 1. Доказать, что число 100...001, в котором 219744~ 21000 — 1 нулей, составное. 2. Доказать, что в круг радиуса 1 нельзя поместить без наложений два треугольника, площадь каждого из которых больше 1. 3. Две одинаковые шестеренки имеют по 32 зубца. Их совмес- тили и спилили одновременно 6 пар зубцов. Доказать, что одну 128
шестеренку можно повернуть относительно другой так, что в местах сломанных зубцов одной шестеренки окажутся целые зубцы второй шестеренки. 4. Из отрезков, имеющих длины а, b и с, можно составить треугольник. Доказать, что из отрезков с длинами , также можно составить треугольник. 5. Выпуклый многоугольник обладает следующим свойством: если все прямые, на которых лежат его стороны, параллель- но перенести на расстояние 1 во внешнюю сторону, то полу- ченные прямые образуют многоугольник, подобный исходному, причем параллельные стороны окажутся пропорциональными. Доказать, что в данный многоугольник можно вписать окруж- ность. X класс 6. См. задачу 4. 7. Доказать, что для любого 13-угольника найдется прямая, содержащая ровно одну его сторону, однако при любом и >13 существует п-угольник, для которого это неверно. 8. См. задачу 3, где у шестеренок с 92 зубцами спилено 10 пар зубцов. 9. На кубе отмечены вершины и центры граней, а также проведены диагонали всех граней. Можно ли по отрезкам этих диагоналей обойти все отмеченные точки, побывав в каждой из них ровно по одному разу? 10. См. задачу 5. 2-й т у р VII класс 11. Точка О внутри правильного шестиугольника со стороной I соединена со всеми его вершинами. Доказать, что среди треугольников, на которые разбивается шестиугольник, найдутся два таких, что все их стороны будут не меньше I. 12. На прямой расположено 100 точек. Отметим середины всевозможных отрезков с концами в этих точках. Какое наимень- шее число отмеченных точек может получиться? 13. Сколько сторон может быть у выпуклого многоугольника, все диагонали которого имеют равную длину? 14. Несколько стеклянных шариков разложено в три кучки. Мальчик, располагающий неограниченным запасом шариков, мо- жет за один ход взять по одному шарику из каждой кучки или же добавить из своего запаса в одну из кучек столько 5 Заказ 247 129
шариков, сколько в ней уже есть. Доказать, что за несколько ходов мальчик может добиться того, что в каждой кучке не останется ни одного шарика. VIII класс 15. См. задачу 13. 16. См. задачу 12 для п точек на плоскости. 17. В клетках прямоугольной таблицы, имеющей 8 строк и 5 столбцов, расставлены натуральные числа. За один ход разре- шается одновременно удвоить все числа одной строки или же вычесть единицу из всех чисел одного столбца. Доказать, что за несколько ходов можно добиться того, чтобы все числа таблицы стали равными нулю. 18. Дан выпуклый пятиугольник, все углы которого тупые. Доказать, что в нем найдутся две такие диагонали, что круги, построенные на них как на диаметрах, полностью покроют пяти- угольник. 19. Сумма 100 натуральных чисел, каждое из которых не больше 100, равна 200. Доказать, что из них можно выбрать несколько чисел, сумма которых равна 100. IX класс 20. Существует ли такая последовательность натуральных чисел, чтобы любое натуральное число 1, 2, 3,... можно было представить единственным способом в виде разности двух чисел этой последовательности? 21. Доказать, что в произвольном 2п-угольнике найдется диагональ, не параллельная ни одной из его сторон. 22* . Имеется несколько гирь, масса каждой из которых равна целому числу. Известно, что их* можно разбить на k равных по массе групп. Доказать, что не менее чем k способами можно убрать одну гирю так, чтобы оставшиеся гири нельзя было разбить на k равных по массе групп. 23. Дан треугольник ABC. AD и BE — его биссектрисы. Известно, что АС>ВС. Доказать, что AE>DE>BD. 24* . Прямоугольный лист бумаги размером а%Ь см разрезан на прямоугольные полоски, каждая из которых имеет сторону 1 см. Линии разрезов параллельны сторонам исходного листа. Доказать, что хотя бы одно из чисел а или b целое. X класс 25. См. задачу 20. 26. Доказать, что в десятичной записи чисел 2Л + 1974Л и 1974л содержится одинаковое количество цифр. 130
27. Шарообразная планета окружена 37-ю точечными астеро- идами. Доказать, что в любой момент на поверхности планеты найдется точка, из которой астроном не сможет наблюдать более 17 астероидов. Примечание. Астероид, расположенный на линии горизонта, не виден. 28. На конгресс собрались ученые, среди которых есть друзья. Оказалось, что любые два из них, имеющие на конгрес- се равное число друзей, не имеют общих друзей. Доказать, что найдется ученый, который имеет ровно одного друга из числа участников конгресса. 29. См. задачу 24. XXXVIII ОЛИМПИАДА (1975 г.) 1-й т у р X класс 1. Найти все действительные решения уравнения с 4 неиз- вестными: x2+y2 + z2 + t2=x(i/ + z+t). 2. Точка А расположена на расстоянии 50 см от центра круга радиуса 1 см. Разрешается точку А отразить симметрич- но относительно произвольной прямой, пересекающей круг; по- лученную точку отразить симметрично относительно любой пря- мой, пересекающей круг, и т. д. Доказать, что: а) за 25 отражений точку А можно переместить внутрь круга; б) за 24 отражения этого сделать нельзя. 3. Натуральные числа а, &, с таковы, что числа р = а, q = ab + c, r = ca-}-b простые. Доказать, что два из чисел р, q, г равны между собой. 4. На шахматной доске размером 8X8 отмечены 64 точки — центры всех клеток. Можно ли отделить все точки друг от друга, проведя 13 прямых, не проходящих через эти точки? 5*. Можно ли разместить в пространстве четыре свинцовых шара и точечный источник света так, чтобы каждый исходящий из источника света луч пересекал хотя бы один из шаров? 2-й т у р VII класс 6. См. задачу 10 а) при п=100. 7. В окружность вписан выпуклый 7-угольник. Известно, что какие-то три его угла равны 120°. Доказать, что найдутся две его стороны, имеющие одинаковую длину. 8. Коля и Витя играют в следующую игру. На столе лежит куча из 31 камня. Мальчики делают ходы поочередно, а начи- нает Коля. Делая ход, играющий делит каждую кучку, в которой 5е 131
больше одного камня, на две меньшие кучки. Выигрывает тот; кто после своего хода оставляет кучки по одному камню в каж дой. Сможет ли Коля сделать так, чтобы выиграть при любой игре Вити? 9. В последовательности 19752... каждая цифра, начиная с пятой, равна последней цифре суммы предыдущих четырех цифр. Встретится ли в этой последовательности: а) набор цифр 1234; 3269; б) вторично набор 1975? VIII класс 10. Какое из двух чисел больше: ? л раз * У (л I) раз? б) З3 ) или 44 J 11. См. задачу 7. 12*. См. задачу 9 с добавлением пункта: в) встретится ли набор 8197? 13. Имеются две страны: Обычная и Зазеркалье. У каждого города в Обычной стране есть «двойник» в Зазеркалье, и наоборот. Однако если в Обычной стране какие-то два города соединены железной дорогой, то в Зазеркалье эти города не соединены, а любые два иссоедииенпых в Обычной стране города обязательно соединены железной дорогой в Зазеркалье. В Обыч- ной стране девочка Алиса не может проехать из города А в го- род В, сделав менее двух пересадок. Доказать, что Алиса в Зазеркалье сможет проехать из любого города в любой другой, сделав не более двух пересадок. 14. В футбольном турнире принимают участие п команд. Каждая команда встречается с каждой по одному разу, при этом выигравшей команде присуждается 2 очка, сыгравшей вничью — 1 очко, проигравшей — 0 очков. Какой максимальный разрыв в очках может быть между командами, занявшими соседние места? IX класс 15. См. задачу 10. 16. См. задачу 7. 17. См задачу 14. 132
18*. В государстве Мантисса города соединены дорогам». Длина любой дороги меньше 500 км, и из любого города в любой другой можно попасть, проехав по дорогам меньше 500 км. Когда одна дорога оказалась закрытой на ремонт, выяснилось, что из каждого города можно проехать по оставшимся дорогам в любой другой. Доказать, что при этом можно проехать меньше 1500 км. 19*. Можно ли какой-нибудь выпуклый многоугольник раз- резать на конечное число невыпуклых четырехугольников? X класс 20. См. задачу 10. 21. См. задачу 8 с заменой числа 31 на 100. 22. См. задачу 18. 23*. Арена цирка освещается п различными прожекторами Каждый прожектор освещает некоторую выпуклую фигуру. Известно, что если выключить один произвольный прожектор, то арена будет по-прежнему полностью освещена, а если выклю- чить произвольные 2 прожектора, то арена полностью освещена не будет. При каких значениях п это возможно? 24. См. задачу 19. XXXIX ОЛИМПИАДА (1976 г.) 1-й т у р X класс 1. Найти все положительные решения системы уравнений {Х|+*2=Хз. х24-х3 = х?. Хз+*4=Х5. Х4+^5=Х|, Х5+а*1 =Х2. 2. В остроугольном треугольнике АВС проведены медиана AM, биссектриса ВК и высота СН. Пусть М'К'Н' — тре- угольник с вершинами в точках пересечения трех проведен- ных отрезков. Может ли площадь полученного треугольника быть больше 0,499 площади треугольника АВС? 3. Каковы первые четыре цифры числа 11 + 22 + З3 4 5 * + .. + 4-9999"+1ООО1000? 4. Астрономический прожектор освещает октант (трехгран- ный угол, у которого все плоские углы прямые). Прожектор помещен в центр куба. Можно ли его повернуть таким образом, чтобы он не освещал ни одной вершины куба? 5. На бесконечном листе клетчатой бумаги (размер клетки 1X1) укладываются кости домино размером 1X2 так, что они 133
покрывают все клетки. Можно ли при этом добиться того, чтобы каждая прямая, идущая по линиям сетки, разрезала бы лишь конечное количество костей домино? 2-й т у р VII класс 6. Даны четыре одинаковых по виду шара массой 101 г, 102 г, 103 г и 104 г, а также чашечные весы со стрелкой, на которых можно взвесить произвольный груз. Сделав два взвешивания, определить массу каждого шара. 7. Может ли выпуклый неправильный пятиугольник иметь ровно 4 стороны одинаковой длины и ровно 4 диагонали одина- ковой длины? 8. Существует ли такое натуральное число л, что сумма цифр числа п2 равна 100? 9. Можно ли на плоскости расположить конечное число точек таким образом, чтобы у каждой точки было бы ровно три ближайших к ней точки? 10. В клетках таблицы размером 10X20 расставлено 200 различных чисел. В каждой строчке отмечены 2 наибольших числа красным цветом, а в каждом столбце отмечены 2 наиболь- ших числа синим цветом. Доказать, что не менее трех чисел от- мечены в таблице как красным, так и синим цветом. VIII класс II. См. задачу 8. 12. У квадрата ABCD длина стороны — целое число. Внут- ри квадрата размещены параллельно его сторонам отрезки, делящие квадрат на более мелкие квадраты, длины сторон ко- торых тоже целые. Длина каждого отрезка — целое число. До- казать, что сумма длин всех отрезков кратна 4. 13. См. задачу 17. 14. См. задачу 10. 15. См. задачу 24. IX класс 16. Может ли число п\ оканчиваться цифрами 1976000...000? 17. На сферическом Солнце обнаружено конечное число круг- лых пятен, каждое из которых занимает меньше половины поверх- ности Солнца. Эти пятна предполагаются замкнутыми (т. е. граница пятна принадлежит ему) и не пересекаются между собой. Доказать, что на Солнце найдутся две диаметрально противоположные точки, не покрытые пятнами. 134
18*. Доказать, что существует такое натуральное число п, большее 1000, что сумма цифр числа 2п больше суммы цифр числа 2л + 1. 19. Дано число N, в записи которого нет нулей. Если в нем стоят рядом две одинаковые цифры или два одинаковых двузнач- ных числа, то их разрешается вычеркнуть. Кроме того, разре- шается также в любое место вставить две одинаковые цифры или два одинаковых двузначных числа. Доказать, что, комбинируя эти операции, можно из числа N получить число, меньшее 1(г. 20*. На столе лежит большой лист клетчатой бумаги со стороной клетки 1 см и неограниченное количество пятикопеечных монет (пятаков); радиус пятака равен 1,3 см. Доказать, что лист можно покрыть пятаками, не налегающими друг на друга, так, чтобы все узлы листа оказались покрытыми. X класс 21. Существует ли такое натуральное число А, что если приписать его к самому себе справа, то полученное число ока- жется полным квадратом? 22. Существует ли такой выпуклый 1976-гранник, который обладал бы следующим свойством: при произвольной расста- новке стрелок на концах всех его ребер сумма полученных век- торов отлична от 0? 23. В клетках таблицы размером 10X20 расставлено 200 различных чисел. В каждой строчке отмечены 3 наибольших числа красным цветом, а в каждом столбце отмечены 3 наиболь- ших числа синим цветом. Доказать, что не менее 9 чисел отмечены в таблице как красным, так и синим цветом. (Ср. с задачей 10.) 24. На плоскости задано конечное множество точек. Дока- зать, что в нем найдется точка, у которой имеется не более трех ближайших к ней точек из этого же множества. (Ср. с задачей 9.) 25. Каждая точка пространства окрашена в один из фикси- рованных пяти цветов, причем имеется 5 точек, окрашенных в различные цвета. Доказать, что существуют прямая, все точки которой окрашены не менее чем в три цвета, и плоскость, все точки которой окрашены не менее чем в четыре цвета. XL ОЛИМПИАДА (1977 г.) 1-й тур X класс 1. Последовательность натуральных чисел (хп) строится сле- дующим образом: Х|=2, хп н =[1,5хп]. Доказать, что в этой последовательности бесконечно много: а) нечетных чисел; б) чет- ных чисел. 135
2. На столе расположено п картонных квадратов и п пласт- массовых квадратов. Никакие два картонных квадрата не имеют общих точек, в том числе и точек границы. То же самое верно и для пластмассовых квадратов. Оказалось, что множество вер- шин картонных квадратов совпадает с множеством вершин пласт- массовых квадратов. Обязательно ля каждый картонный квад- рат совпадает с некоторым пластмассовым? 3*. а) Двенадцать тонких твердых стержней длины 1 скре- пили так, что получился каркас куба. Можно ли сделать в плос- кости отверстие площади меньше 0,01, не разбивающее плоскость на куски, чтобы через него можно было бы протащить этот кар- кас? б) Тот же вопрос для каркаса тетраэдра с ребром 1. 4. Каждая точка числовой оси, координата которой — целое число, покрашена либо в красный, либо в синий цвет. Доказать, что найдется цвет со следующим свойством: для каждого нату- рального числа k имеется бесконечное количество точек этого цвета, координаты которых делятся на А. 2-й т у р VII класс 5. В каждой вершине выпуклого n-угольника находится охотник, вооруженный лазерным ружьем. Все охотники одно- временно стреляют в зайца, сидящего в точке О внутри этого n-угольника. В момент выстрела заяц пригибается и все охотники погибают. Доказать, что, кроме точки О, никакая другая точка не обладает тем же свойством. 6. Куб 3X3X3 составлен из 14 белых и 13 черных кубиков с ребром 1. Столбик — это три кубика, стоящие подряд вдоль од- ного направления: ширины, длины или высоты. Может ли быть так, что в каждом столбике нечетное количество: а) белых ку- биков; б) черных кубиков? 7. Доказать, что найдется более 1000 троек натуральных чисел (а, &, с) таких, что для них выполняется равенство а'* + Ь'ь = с'*. 8. В доске торчит 1977 гвоздей. Двое играющих делают ходы по очереди. Ход состоит в том, что играющий соединяет про- водом два гвоздя. Если в результате хода цепь замкнулась, то сделавший этот ход считается выигравшим. Кто выигрывает при правильной игре: первый игрок или второй? Замечание. Не разрешается соединять проводом два ранее соединенных гвоздя. 9. Найти такое наименьшее п, что любой выпуклый 100- угольнпк можно получить в виде пересечения п треугольников. Доказать, что для меньших п это можно сделать не с любым выпуклым 100-угольником. 136
VIII класс 10. См. задачу 5. 11. См. задачу 6. 12. См. задачу 7. 13. См. задачу 17 а). 14. См. задачу 9. IX класс 15. В пространстве расположено п отрезков, никакие три из которых не параллельны одной плоскости. Для любых двух отрезков прямая, соединяющая их середины, перпендикулярна обоим отрезкам. При каком наибольшем п это возможно? 16. а) Существуют ли такие 6 различных натуральных чисел, что для любых а и Ь из них сумма а-\~Ь делится на разность a —fr? б) Тот же вопрос для 1000 чисел. 17. а) После окончания волейбольного турнира оказалось, что для любых двух команд существует третья, которая выигра- ла у обеих. Доказать, что число команд в турнире не меньше 7. б) В другом волейбольном турнире для любых трех команд найдется команда, которая выиграла у этих трех. Доказать, что число команд не меньше 15. 18*. В пространстве расположен выпуклый многогранник, все вершины которого находятся в целочисленных точках (т. е. все три координаты каждой вершины — целые числа). Других целочисленных точек внутри, на гранях и на ребрах много- гранника нет. Доказать, что число вершин многогранника не превосходит 8. 19* . Дан многочлен Р (х) с целыми коэффициентами. Для каждого натурального п значение Р (п) больше п. Рассмотрим последовательность xi = l, x2 = P(xi), .... хп = Р .... Известно, что для любого натурального N найдется член после- довательности, делящийся на /V. Доказать, что справедливо ра- венство Р(лг) = х4-1. X класс 20. Можно ли на плоскости расположить бесконечное множе- ство одинаковых кругов так, чтобы любая прямая пересекала не более двух кругов? 21. См. задачу 16 для 15 чисел. 22. См. задачу 17 б). 23. Последовательность натуральных чисел (хп) строится по следующему правилу: xi=2, *2 = 3, ..., хп-ц =[1,5хп]. Доказать, что последовательность (уп), где уп=( — 1)Хя, непериодична. 24. См. задачу 19. 137
XLI ОЛИМПИАДА (1978 г.) VII класс 1. Найти все пары целых чисел (х, у\ удовлетворяющие уравнению 3-2*+l=t/2. 2*. На плоскости расположен пластмассовый треугольник. Если начать перекатывать его через стороны и если он в какой- то момент пересечется со своим первоначальным положением, то окажется, что он просто совпал со своим первоначальным положением. Какие треугольники удовлетворяют такому условию? Указать все виды таких треугольников. 3. Доказать, что в прямоугольник размером лХ2/л (л и т — целые) можно уложить в два слоя кости домино размером 1X2 так, чтобы каждый слой полностью покрывал прямо- угольник и чтобы никакие две кости из разных слоев не совпа- дали друг с другом. 4. См. задачу 16а). VIII класс 5. См. задачу 10. 6. См. задачу 2. 7. См. задачу 3. 8. См. задачу 16 а). 9. Дано некоторое 1000-значное число А. Про него известно, что любые его 10 идущих подряд цифр образуют число, крат- ное 210. Доказать, что число А делится на 2*000. IX класс 10. В л-угольнике расположено несколько точек таким обра- зом, что в произвольном треугольнике, образованном любыми тремя вершинами л-угольника, расположена по крайней мере одна точка. Какое наименьшее число точек могло быть? 11. Существует ли на плоскости конечный набор различных векторов ai, Л2» ...» On такой, что для любой пары различных векторов из этого набора найдется такая другая пара из этого набора, что суммы каждой из пар равны между собой? 12. См. задачу 16. 13. На плоскости расположено несколько прямых и точек. Доказать, что на плоскости найдется точка А, не совпадающая ни с одной из данных точек, расстояние от которой до любой из данных точек больше расстояния от нее до любой из данных прямых. 14. В поселке живут 100 жительниц. У каждой из них имеют- ся 3 знакомые жительницы. 1 января одна узнала интересную новость и сообщила ее трем своим знакомым; 2 января те 138
сообщили новость всем своим знакомым и т. д. Может ли случиться так, что 5 марта еще не все жительницы будут знать эту новость, а 19 марта — все? X класс 15. У белой сферы 12% ее площади окрашено в красный цвет. Доказать, что в сферу можно вписать параллелепипед, у которого все вершины белые. 16. Город имеет форму квадрата. В нем 6 улиц: 4 стороны квадрата и 2 его средние линии. Полицейский гоняется по улицам за гангстером. Если в какой-то момент полицейский и гангстер оказываются на одной улице, то гангстер сдается полицейскому. Доказать, что полицейский сможет поймать гангстера, если его скорость: а) в 3 раза; б)* в 2,1 раза больше скорости гангстера. 17. См. задачу 13. 18*. Доказать, что существует: а) одно; б) бесконечно много таких натуральных чисел л, что последние цифры числа 2Л образуют число л. 19* . Дано 8 действительных чисел: л, b. с, d, е, f, g, А. Доказать, что хотя бы одно из шести чисел ac-^-bd, ae-\-bf, ag + bht ce + dft cg + dh, eg+fh неотрицательно. XU I ОЛИМПИАДА (1979 г.) VII класс 1. На плоскости отмечена точка О. Можно ли так располо- жить на плоскости: а) 5 кругов; б) 4 круга, не покрывающих точку О, чтобы любой луч с началом в точке О пересекал не менее двух кругов? 2. Имеется несколько гирь, общая масса которых равна 1 кг. Каждой гире присвоен свой номер: 1, 2, 3, .... Доказать, что найдется такой номер л, что масса гири с номером л строго - 1 больше — кг. 3. Квадрат разрезан на прямоугольники. Доказать, что сумма площадей кругов, описанных около каждого прямоуголь- ника, не меньше площади круга, описанного около квадрата. 4. Коля и Витя играют в следующую игру на бесконечной клетчатой бумаге. Начиная с Коли, они по очереди отмечают узлы клетчатой бумаги — точки пересечения вертикальных и го- ризонтальных прямых. При этом каждый из них своим ходом должен отметить такой узел, чтобы после этого все отмеченные узлы лежали в вершинах выпуклого многоугольника (начиная со второго хода Коли). Тот из играющих, кто не сможет сделать очередного хода, считается проигравшим. Кто выигрывает при правильной игре? 139
VIII класс 5. На плоскости отмечена точка О. Можно ли так располо- жить на плоскости: а) 7 кругов; б) 6 Кругов, не покрывающих точку О, чтобы любой луч с началом в точке О пересекал не менее трех кругов? (Ср. с задачей 1.) 6. См. задачу 2. 7. Четырехугольник ABCD вписан в окружность с центром О. Диагонали АС и BD четырехугольника взаимно перпенди- кулярны. Доказать, что длина перпендикуляра О//, опущен- ного из центра окружности на сторону AD, вдвое меньше длины стороны ВС. 8. См. задачу 3. 9. На химической конференции присутствовало k ученых хи- миков и алхимиков, причем химиков было больше, чем алхими- ков Известно, что на любой вопрос химики всегда отвечают правду, а алхимики иногда говорят правду, а иногда лгут. Оказавшийся на конференции математик про каждого ученого хочет установить, химик тот или алхимик. Для этого он любому ученому может задать вопрос: «Кем является такой-то: хими- ком или алхимиком?» (В частности, может спросить, кем являет- ся сам этот ученый.) Доказать, что математик может установить это за: а) 46 вопросов; б) 26 — 2 вопросов. IX класс 10. Имеется несколько камней, масса каждого из которых не превосходит 2 кг, а общая масса равна 100 кг. Из них выби- рается несколько камней, суммарная масса которых отличается от 10 кг на наименьшее возможное для данного набора число d. Какое максимальное значение может принимать число d для всевозможных наборов камней? 11*. Можно ли все пространство представить в виде объе- динения бесконечного числа попарно скрещивающихся пря- мых? 12. а) Существует ли последовательность натуральных чисел Qi, 02, аз, ...» обладающая следующим свойством: ни один член последовательности не равен сумме нескольких других и ап^п при любом л? б) Тот же вопрос, если ап^п^/п при любом п. 13. См. задачу 7. 14*. См. задачу 9 при числе вопросов, равном 26 — 3. X класс 15. См. задачу 10. 16. На отрезке длины 1 отмечено несколько интервалов. Из- вестно, что расстояние между любыми двумя точками, принадле- 140
жащими одному или разным отмеченным интервалам, не равно 0,1. Доказать, что сумма длин отмеченных интервалов не пре- восходит 0,5. 17. Функция у = { (х) определена на отрезке [0; 1] и в каждой точке этого отрезка имеет первую и вторую производные. Извест- но, что f(O) = f(l) = O и что I/" (х)|<1 на всем отрезке. Какое наибольшее значение может принимать максимум функции f для всевозможных функций, удовлетворяющих этим условиям? 18. Объединение нескольких кругов имеет площадь 1. Дока- зать, что из них можно выбрать несколько попарно непересе- кающихся кругов, сумма площадей которых больше 19. См. задачу 14. XLIII ОЛИМПИАДА (1980 г.) VII класс 1. Найти наибольшее пятизначное число Д, у которого чет- вертая цифра больше пятой, третья больше суммы четвертой и пятой, вторая больше суммы третьей, четвертой и пятой и пер- вая цифра больше суммы остальных. 2. В каждой клетке прямоугольной таблицы записано одно из двух чисел: +1 или — 1. При этом количество -|-1 не меньше двух и количество — 1 не меньше двух. Доказать, что найдутся две строки и два столбца такие, что сумма четырех чисел, стоя- щих на их пересечении, равна 0. 3. Доказать, что максимальное количество сторон выпукло- го многоугольника, стороны которого лежат на диагоналях дан- ного выпуклого 100-угольника, не больше 100. 4. Три прямолинейных коридора одинаковой длины I образу- ют фигуру, изображенную на рисунке 8. По ним бегают гангстер и полицейский. Максимальная скорость полицейского в 2 раза больше максимальной скорости гангстера. Полицейский сможет увидеть гангстера, если он окажется от него на расстоянии, не большем г. Доказать, что полицейский всегда может поймать гангстера, если: а) г>^ б) • 5. Десять вершин правильного 20- угольника Д^гДз -.Дго покрашены в чер- ный цвет, а десять других — в белый. Рассматривается множество, состоящее из диагонали AtA4 и всех диагоналей, равных ей. Доказать, что в этом множе- стве количество диагоналей с двумя чер- ными концами равно количеству диаго- налей с двумя белыми концами. 141
VIII класс 6. Доказать, что если ai С Дг < Дз <... С Дю, то с» 4~ ••• 4'Сб 4“••• 4-аю 6 16 * 7. См. задачу 2. 8. На хорде АВ окружности К с центром в точке О взята точка С. D — вторая точка пересечения окружности К с окруж- ностью, описанной около ДДСО. Доказать, что CD = CB. 9. См. задачу 4. 10. См. задачу 5. IX класс И. Д|, Д2, Дз, .... ДЛ, ... — возрастающая последовательность натуральных чисел. Известно, что дл + 1<10ал при всех нату- ральных л. Доказать, что бесконечная десятичная дробь 0, л^гДз..., полученная приписыванием этих чисел друг к другу, непериодическая. 12. На пульте имеется несколько кнопок, с помощью которых осуществляется управление световым табло. После нажатия лю- бой кнопки некоторые лампочки на табло переключаются (для каждой кнопки есть свой набор лампочек, причем наборы могут пересекаться). Доказать, что число состояний, в которых может находиться табло, равно некоторой степени числа 2. 13- На прямоугольном листе клетчатой бумаги размером /лХл клеток расположено несколько квадратов, стороны которых идут по вертикальным и горизонтальным линиям бумаги. Извест- но, что никакие два квадрата не совпадают и никакой квадрат не содержит внутри себя другой квадрат. Каково Наибольшее число таких квадратов? 14. См. задачу 4 для случаев: а) г>4-; б)* г>4-. 15. См. задачу 8. X класс 16. См. задачу 11. 17. См. задачу 12. 18. См. задачу 14. 19. В каждой клетке таблицы 1980X1980 записано одно из чисел +1» —1,0. Сумма всех чисел равна 0. Доказать, что суще- ствуют две строки и два столбца такие, что сумма четырех чисел, стоящих на их пересечениях, равна 0. 20*. На сфере радиуса 1 расположено несколько дуг больших окружностей (большая окружность — это пересечение сферы с плоскостью, проходящей через ее центр). Сумма длин всех этих дуг меньше л. Доказать, что найдется плоскость, проходящая через центр сферы, которая не пересекается ни с одной из дуг. 142
XLIV ОЛИМПИАДА (1981 г.) VII класс 1. Натуральное число А при делении на 1981 дало в остатке 35, при делении на 1982 оно дало в остатке также 35. Каков остаток от деления числа А на 14? 2. См. задачу 11 для числа с 13 разрядами. 3. На двух равных круглых листах бумаги художник нарисо- вал одинаковых драконов. Оказалось, что на первом листе глаз дракона совпал с центром круга, а на втором — не совпал. Дока- зать, что второй лист бумаги можно разрезать на такие две час- ти, чтобы из них удалось сложить круг того же радиуса с тем же драконом, но чтобы его глаз уже находился в центре круга. 4. Дано число х, большее 1. Обязательно ли имеет место равенство 5. Имеется 5 гирь. Их массы равны 1000 г, 1001 г, 1002 г, 1004 г и 1007 г, но надписей на гирях нет и внешне они неотли- чимы. Имеются весы со стрелкой, которые показывают массу в граммах. Как с помощью трех взвешиваний определить гирю в 1000 г? VIII класс 6. В пятиугольнике проведены все диагонали. Какие 7 углов между двумя диагоналями или между диагоналями и сторонами надо отметить, чтобы из равенства этих углов друг другу следо- вало, что пятиугольник правильный? 7. См. задачу 2. 8. См. задачу 3. 9. См. задачу 4. 10. Дано 10 натуральных чисел: а|<а2<Дз<...<аю. До- казать, что их наименьшее общее кратное не меньше 10 ф. IX класс 11. Дано число, имеющее нечетное число разрядов. Дока- зать, что одну из его цифр можно вычеркнуть так, что в полу- ченном числе количество семерок на четных местах будет равно количеству семерок на нечетных местах. 12. Натуральные числа alt а2» .... ап таковы, что каждое не превышает своего номера и сумма всех чисел — четное число. Доказать, что одна из сумм at ±а2±аз±...±ал равна нулю. 13. X и У — два выпуклых многоугольника, причем много- угольник X содержится внутри У. Пусть S (X) и S (У) — площади этих многоугольников, а Р (X) и Р(У)— их периметры. Доказать, 41 Р(Х) P(Y) ’ 143
14*. Можно ли разбить множество натуральных чисел на бесконечное число бесконечных подмножеств, каждое из ко- торых получается из любого другого подмножества прибавле- нием одного и того же целого числа к каждому элементу? 15*. У правильного 1981-угольника отмечены 64 вершины. Доказать, что существует трапеция с вершинами в отмеченных точках. X класс 16. Рассматривается функция y—f(x)t определенная на всем множестве действительных чисел и удовлетворяющая для не- которого числа соотношению f (х4-Л)-(1 — /(х)) — 1 +f (х). Доказать, что f (х) — периодическая функция. 17. Дан многочлен Р (х) степени п со старшим коэффици- ентом, равным 1. Известно, что если х — целое число, то Р (х) — целое число, кратное р (р — натуральное число). Доказать, что и! делится на р. 18. Доказать, что последовательность xn = sin(n2) не стре- мится к нулю при и, стремящемся к бесконечности. 19. В квадрате со стороной длины 1 расположена ломаная без самопересечений, длина которой не меньше 200. Доказать, что найдется прямая, параллельная одной из сторон квадрата, пересекающая ломаную не менее чем в 101-й точке. 20. Радиус вписанной в треугольник окружности равен у, а длины высот треугольника — целые числа, сумма которых рав- на 13. Вычислить длины сторон треугольника. 21*. За круглым столом сидят п человек. Разрешается лю- бых двух людей, сидящих рядом, поменять местами. Какое наи- меньшее число таких перестановок необходимо сделать, чтобы в результате любые два соседа остались бы соседями, но сидели бы в обратном порядке? XLV ОЛИМПИАДА (1982 г.) VII класс 1. Петя купил в магазине «Машины Тьюринга и другие вы- числительные устройства» микрокалькулятор, который может выполнять следующие операции: по любым числам х и у он вы- числяет х-\-у, х — у и у (при х=/=0). Петя утверждает, что он может возвести любое положительное число в квадрат с по- мощью своего микрокалькулятора, сделав не более 6 операций. Л вы можете это сделать? Если да, то попробуйте перемножить любые два положительных числа, сделав не более 20 операций 144
(промежуточные результаты можно записывать» неоднократно используя их в вычислениях). 2. В квадрате ABCD находятся 5 точек. Доказать» что расстояние между какими-то двумя из них не превосходит ^АС. 3. Петя приобрел в магазине вычислительный автомат» ко- торый за 5 к. умножает любое введенное в него число на 3, а за 2 к. прибавляет к любому числу 4. Петя хочет» начиная с еди- ницы, которую можно ввести бесплатно, набрать на автома- те число 1981 и затратить наименьшую сумму денег. Во сколько обойдутся ему вычисления? А что будет, если он захочет набрать число 1982? 4. Какое наименьшее количество точек на плоскости надо взять, чтобы среди попарных расстояний между ними встретились числа 1, 2, 4, 8, 16, 32, 64? VIII класс 5*. Упростить выражение ________2________ д/4-3\/5+2л/5-У125 6. Прямоугольник разрезан на пять прямоугольников. Дока- зать, что среди полученных пяти найдутся два прямоугольника, один из которых полностью может располагаться внутри другого. 7. Числа 1, 2, 3, ..., 1982 возводятся в квадрат и запи- сываются подряд в некотором порядке. Может ли полученное многозначное число быть полным квадратом? 8. Каждая диагональ выпуклого пятиугольника параллельна одной из его сторон. Доказать, что отношение каждой диагонали к соответствующей стороне равно -уХл/5+1). 9*. Считая известной формулу 13 + 23 +... + л3 ==(П~П2^~^) , доказать» что для различных натуральных чисел at, .... ап справедливо неравенство (а] + «2 +... + ai)+(ai + >2 (a3+a3 + ...+a3)2. Возможно ли равенство для каких-нибудь различных натуральных чисел аь аг, .... ал? IX класс 10. Найти все натуральные числа л, для которых число л-2Л4-1 кратно трем. 11. Найти на плоскости точку, сумма расстояний от которой до четырех заданных точек минимальна. 12. На плоскости отмечены точки с целочисленными коорди- натами. Доказать» что найдется окружность, внутри которой ле- жат ровно 1982 отмеченные точки. 145
13. Число А =0,1 +0,024-0,003 + • • +л Ю~л+ ... записано в виде бесконечной десятичной дроби. Доказать, что в получен- ной записи не встретятся подряд идущие цифры 1982. 14. В выпуклом четырехугольнике две стороны равны I, а другие стороны и обе диагонали не больше I. Какое максималь- ное значение может принимать периметр четырехугольника? X класс 15. а) Доказать, что если из некоторой точки внутри пра- вильного тетраэдра все его ребра видны под одинаковыми углами, то эта точка — центр описанной около тетраэдра сферы. б) Существуют ли вне тетраэдра точки, из которых все его ребра видны под равными углами? Примечание. Если точка лежит на ребре или его про- должении, то считается, что из нее это ребро видно под углом л или 0 соответственно. 16. а) а, Ь, с — длины сторон треугольника. Доказать, что а4 + Ь4 + с4 — 2 (а2Ь2 + а2с2 + b2r)+a2bc + b2ac + c2ab > 0. б) Доказать неравенство из пункта а) для любых неотрица- тельных а, b и с, 17*. Петя приобрел в магазине «Машины Тьюринга и другие вычислительные устройства» микрокалькулятор, который может по любым действительным числам х и у вычислить хуху + 1 и не имеет других операций. Петя хочет написать «программу» для вычисления многочлена 1+х+х2 + ...+х1982. Под «про- граммой» он понимает последовательность многочленов fi (х), ..., fп (х) такую, что fj (х) = х и для любого 1 = 2, ..., п fi(x)=Ci или A (x)=fj (x)-fk (x) + fk(x)+fi (x)+l, где /<t, k<i, причем fn(x) = = 14-%+ ..4-х' . а) Помогите Пете написать «программу». б) Сумеете ли Вы написать «программу», е£ли калькулятор имеет только одну операцию ху + х-\-у? 18. Найти все такие натуральные и, для которых числа — и выражаются конечными десятичными дробями. 19*. Внутри правильного шестиугольника находится другой правильный шестиугольник с вдвое меньшей стороной. Доказать, что центр большого шестиугольника лежит внутри малого шести- угольника. XLVI ОЛИМПИАДА («М3 г.) VII класс 1. Найти все пары целых чисел (х, у), удовлетворяющих урав- нению хг=у24-2«/4-13. 146
2. Белая плоскость произвольным образом забрызгана чер- ной тушью. Доказать, что для любого положительного I сущест- вует отрезок длины /, у которого оба конца одного цвета. 3- Найти наименьшее натуральное число, начинающееся с цифры 4 и уменьшающееся в четыре раза от перестановки этой цифры в конец числа. 4. Двум друзьям необходимо попасть в соседний город. У них есть один велосипед, на котором может ехать только один человек. Каково минимальное время, за которое оба могут добраться до города (считая по последнему прибывшему), если скорости пешеходов и\ и 1/2, их скорости на велосипеде tn и у2, расстояние до города равно S (они могут возвращаться и остав- лять велосипед друг другу)? 5. Существует ли пятиугольник со сторонами 3, 4, 9, 11 и 13 см, в который можно вписать окружность? VIII класс 6. Доказать, что при любых х>д/2 и выполняется неравенство х4—х3у+х2у*—ху3+у4>х2 + у. 7. На сторонах треугольника АВС вне его построены пра- вильные треугольники ABCt, ВСА\ и САВ\. Доказать, что ААх+ВВх + СС\=^- 8. Может ли квадрат какого-либо натурального числа начи- наться с 1983-х девяток? 9. В вершинах правильного 1983-угольника расставлены чис- ла 1, 2, ..., 1983. Любая его ось симметрии делит числа, не ле- жащие на ней, на два множества. Назовем расстановку «хоро- шей» относительно данной оси симметрии, если каждое число одного множества больше симметричного ему числа. Существует ли расстановка, являющаяся «хорошей» относительно любой оси симметрии? 10. На окружности выбрано пять точек Д|, Дг, Дз» Д4» Н- Обозначим через Иц расстояние от точки Н до прямой Д,Д/. Доказать, что Л|2*Лз<=Л|4-Л2з. IX класс 11. Доказать, что при любой расстановке знаков « + » и « —» у нечетных степеней х выполнено неравенство х2Я±х2Я-,+х2п-2±х2Я-3+ _|_x4_fcx3 + je2_tJc+i>^_ (х—произвольное действительное число, а п — натуральное). 12. Три окружности радиусов 3, 4, 5 внешне касаются друг друга. Через точку касания окружностей радиусов 3 и 4 прове- дена их общая касательная. Найти длину отрезка этой касатель- ной, заключенной внутри окружности радиуса 5. 147
13. Доказать, что 1 l983 + 2l983+...-f- 19831983 делится на 1 + - -Ь 1983 14. Двадцать городов соединены 172-мя авиалиниями. До- казать, что, используя эти авиалинии, можно из любого города перелететь в любой другой (быть может, делая пересадки). X класс 15. Пусть Д|, Bi, Ci — точки, в которых окружность, вписан- ная в треугольник АВС, касается сторон ВС, АС и АВ соответст- венно. Известно, что ДЛ| = ВВ|=СС|. Доказать, что треуголь- ник АВС правильный. 16. Доказать, что 4т — 4Л делится на 3* + 1 тогда и только тогда, когда гп — п делится на 3*. Решить задачу: а) при Л= 1, 2, 3; б) при произвольном Л. 17. На доске после занятия осталась запись: «Вычислить '<°>-'(v)+'(v)~'(?)+-•+<(?)-'(?). ™ '«= = cos 5x + *cos 4x + *cos 3x + *cos 2x + *cos x-|-*». Увидев ее, сту- дент мехмата сказал товарищу, что оп может вычислить эту сум- му, даже не зная значений стертых с доски коэффициентов (вместо них в нашей записи ♦). Не ошибается ли он? 18. В пространстве выбрано 8 точек, никакие 4 из которых не лежат в одной плоскости. Проведено 17 отрезков, у каждого из которых оба конца лежат в упомянутых точках. Доказать, что: а) отрезки образуют хотя бы один треугольник; б)* треуголь- ников на самом деле не меньше четырех 19. За круглым столом сидят 13 богатырей из k городов, где 1 < k < 13 Каждый богатырь держит в руке золотой или серебряный кубок, причем золотых кубков тоже k. Князь повелел каждому богатырю передать свой кубок соседу справа и повто- рять это до тех пор, пока какие-нибудь два богатыря из одного города оба не получат золотые кубки. Доказать, что желание князя всегда будет исполнено XLVII ОЛИМПИАДА (1984 г.] VII класс 1. Назовем автобусный билет счастливым, если сумма цифр его номера делится на 7. Могут ли два билета подряд быть счастливыми? 2. Дорожки в зоопарке образуют равносторонний треуголь- ник, в котором проведены средние линии. Из клетки сбежала обезьянка. Ее ловят два сторожа. Смогут ли они поймать обезьян- ку, если все трое будут бегать только по дорожкам, скорость обезьянки и скорости сторожей равны и они видят друг друга? 148
3. Покупатель взял у продавца товара на 10 р. и дал 25 р. У продавца не нашлось сдачи, и он разменял деньги у соседа. Когда они расплатились и покупатель ушел, сосед обнаружил, что 25 р. фальшивые. Продавец вернул соседу 25 р. и задумался. Какой убыток понес продавец? 4. Из бумажного треугольника вырезали параллелограмм. Доказать, что площадь параллелограмма не превосходит поло- вины площади треугольника. 5. На шахматной доске 20X20 стоят 10 ладей и один король. Король не стоит под шахом и идет из левого угла в правый верх- ний по диагонали. Ходят по очереди: сначала король, потом одна из ладей. Доказать, что при любом начальном расположении ладей и любом способе маневрирования ими король попадет иод шах. VIII класс 6. Решить уравнение х2 —4 = 0. 7. Каждые две из шести ЭВМ соединены своим проводом. Ука- жите, как раскрасить каждый из этих проводов в один из пяти цветов, чтобы из каждой ЭВМ выходило пять проводов разного цвета. 8. Доказать, что сумма расстояний от центра правильного семиугольника до всех его вершин меньше, чем сумма расстояний до них от любой другой точки. 9. Сумма пяти неотрицательных чисел равна единице. Дока- зать, что эти числа можно расставить по окружности так, что сумма всех пяти попарных произведений соседних чисел будет 1 не больше и 10. Разрежьте квадрат на 8 остроугольных треугольников. 11. Является ли четным количество всех 64-значных нату- ральных чисел, не содержащих в записи нулей и делящихся на 101? IX класс 12. У треугольной пирамиды три боковые ребра равны и пло- щади трех боковых граней равны. Доказать, что в ее основании лежит равнобедренный треугольник. 13. Имеются провода двенадцати цветов. Можно ли 13 прибо- ров соединить попарно проводами так, чтобы от каждого из них выходило 12 проводов разных цветов? (Ср. с задачей 7.) 14. Какова минимальная ширина бесконечной полосы, из ко- торой можно вырезать любой треугольник площади 1? 149
15. По окружности расставлено п неотрицательных чисел, сумма которых равна 1. Докажите, что сумма п попарных произ- ведений соседних чисел не больше (Ср. с задачей 9.) 16* . В прямоугольнике размерами 3X4 расположены четыре точки. Докажите, что среди них найдутся две, расстояние между 25 которыми не превосходит -g-. 17. Существуют ли три отличные от нуля цифры, с помощью которых можно записать квадраты бесконечного числа различ- ных целых чисел? X класс 18. Доказать неравенство (не используя калькуляторов, таб- лиц и т. п.) sin 1 < Iog3 -^7. 19. Жюри олимпиады решило по ее результатам поставить в соответствие каждому участнику натуральное число таким обра- зом, чтобы по этому числу можно было однозначно восстановить баллы, полученные участником за каждую задачу, и чтобы из каж- дых двух школьников большее число соответствовало тому, кто набрал большую сумму баллов. Помогите жюри решить эту за- дачу! 20. Решить в целых числах уравнение 19х3—84г/1 2 = 1984. 21. В некотором царстве, в некотором государстве было вы- пущено неограниченное число монет достоинством в Л|, пг, пз, П4, ... копеек, где П|<П2<л3<П4<...— бесконечная после- довательность, состоящая из натуральных чисел. Доказать, что эту последовательность чисел можно оборвать: найдется такое число N, что любую сумму, которую можно уплатить без сдачи выпущенными монетами, на самом деле можно уплатить только монетами достоинством в n\t п2, n3t .... nN копеек. 22. Квадрат разрезан на остроугольные треугольники. Дока- зать, что их не меньше восьми. (Ср. с задачей 10.) 23* . Треугольное сечение куба касается вписанного в куб шара. Доказать, что площадь этого сечения меньше половины площади грани куба. XLVIII ОЛИМПИАДА (1985 г.) VII класс 1. Найти все значения х и у, для которых ху-\- \ =х-\-у. 2. Даны 5 различных положительных чисел, которые можно 150
разбить на две группы так, чтобы суммы чисел в этих группах были одинаковыми. Сколькими способами это можно сделать? 3. Длины а, Ь, с, d четырех отрезков удовлетворяют нера- венствам Q<a^.b^.c<d, d<a-\-b-\-c. Можно ли из этих от- резков сложить трапецию? 4. В центре квадрата сидит заяц, а в каждом из четырех углов по одному волку. Может ли заяц выбежать из квадрата, если волки могут бегать только по сторонам квадрата с макси- мальной скоростью, в 1,4 раза большей максимальной скорости зайца? 5. В магазин привезли цистерну молока. У продавца имеются чашечные весы без гирь (на чашки весов можно ставить фляги), а также три одинаковые фляги, две из которых пусты, а в третьей налит 1 л молока. Как отлить в одну флягу ровно 85 л молока, сделав не более 8 взвешиваний? (Предполагается, что во флягу помещается больше 85 л. Под взвешиванием понимается сле- дующая операция: на одну чашку весов ставится фляга с мо- локом, на вторую — пустая и в пустую флягу доливается столько молока, чтобы весы пришли в равновесие.) VIII класс 6. Найти все значения х, у, z, удовлетворяющие равенству (х—у 4-z)2=x2—у2 4-z2. 7. Числа at, аг, .... ai985 представляют собой переставленные в некотором порядке числа 1, 2, .... 1985. Каждое число а» умно- жается на его номер k, а затем среди всех полученных 1985-и произведений выбирается наибольшее. Доказать, что оно не меньше 9932. 8. На клетчатый лист бумаги положили бумажный квадрат, площадь которого равна учетверенной площади клетки. Какое наименьшее число узлов может накрывать этот квадрат? (Узел — это точка пересечения линий бумаги; если узел лежит на границе квадрата, он считается накрытым.) 9. За дядькой Черномором выстроилось чередой бесконечное число богатырей. Доказать, что он может приказать части из них выйти из строя так, чтобы в строю осталось бесконечно много богатырей и все они стояли по росту (не обязательно в порядке убывания). 10*. Доказать, что если длина каждой из биссектрис треуголь- ника больше 1, то его площадь больше —. Уз IX класс И. Найти все значения х, у, г, удовлетворяющие равенству >Jx—y+z=^x—y/y+-y/z. 151
12. В некоторой стране 1985 аэродромов. С каждого из них вылетел самолет и приземлился на самом удаленном от места старта аэродроме. Могло ли случиться так, что в результате все 1985 самолетов оказались на 50 аэродромах? (Землю можно считать плоской, а маршруты — прямыми. Все попарные расстоя- ния между аэродромами предполагаются различными.) 13. Сколько узлов клетчатой плоскости накрывает квадрат размером 2X2, если их не менее 7? (См. замечание к задаче 8; сторона клетки равна 1.) 14. Доказать, что в любой группе из 12 человек можно вы- брать двоих, а среди оставшихся 10 человек еще пятерых так, чтобы каждый из них удовлетворял следующему условию: либо он знаком с обоими выбранными вначале, либо не знаком ни с одним из них. 15*. (Задача Эйлера.) Доказать, что любое число 2Л при можно представить в виде 2п = 7х2+//2, где х и у — нечетные числа. X класс х —49 . х —50 49 . 50 16. Решить уравнение —777——пг~=----гтгН---ттг- Jr 50 49 х—50 х —49 17. См. задачу 3. 18. Назовем «сложностью* данного числа наименьшую длину числовой последовательности (если такая найдется), которая на- чинается с нуля и заканчивается этим числом, причем каждый следующий член последовательности либо равен половине преды- дущего, либо в сумме с предыдущим составляет I. Среди всех чисел вида , где m = I, 3, 5. 250— I, найти число с наиболь- шей «сложностью». 19. Даны 1985 множеств, каждое из которых состоит из 45 элементов, причем объединение любых двух множеств содержит ровно 89 элементов. Сколько элементов содержит объединение всех этих 1985 множеств? 20. Доказать, что если расстояния между скрещивающимися ребрами тетраэдра равны соответственно /и, /12. Лз, то объем тетраэдра не меньше 4-Л|ЛгЛз. О
ЧАСТЬ II РЕШЕНИЯ, УКАЗАНИЯ, ОТВЕТЫ 1.01. V=—L— м/с, £ = м. /2 —/| <2 — »1 1.02. Зафиксируем одну вершину А на первой (произвольной) из данных прямых и повернем вторую прямую относительно А на 90°. Точка В пересечения повернутой прямой с третьей прямой и будет второй вершиной квадрата. 1.03. Указание. Угол а при вершине удовлетворяет ра- а 1 венству sin — = — cos а. 2 V2 1.05. Из свойств вписанных углов вытекает, что точки, из которых диагональ видна под фиксированным углом, лежат на сфере. Для того чтобы этот угол был наименьшим, требуется, чтобы радиус сферы нельзя было увеличить, не «соскользнув» с куба. Отсюда ясно, что искомые точки — вершины куба, не принадлежащие рассматриваемой диагонали. 1.06. Одно, так как если х4-|/=2, то ху^Л. 1.07. Если Ьф 0, то решений нет. При Ь = 0 имеем х=у. 1.08. n2(2n2—1). 1.09. 30 способов. 1.10. Число способов равно (п —2)+(п —3)4-.„+I. Действи- тельно, если x+y+z=n, то х может быть равным 1, 2, 3,..., п —2; y-j-z=n—х и при фиксированном х у нас есть п—х— 1 возмож- ностей для значений у и z. 2.01. 7744 =882. 2.03. N = — log2 log2’yV...-\/2 (N радикалов). 2.04. Указание. Построить точку Р', центрально-сим- метричную Р относительно центра круга; на РР' как на хорде построить дугу величиной л —а (а — данный угол); точка пере- сечения этой дуги с окружностью — один из концов искомого диаметра. 2.06. Указание. Выразить х?+у5 через х+у и ху. 2.07. Указание. Докажите, что для всех треугольников пе- риметра 2р, построенных согласно условию задачи, будет вне- вписанной одна и та же окружность, вписанная в данный угол и касающаяся его сторон на расстоянии р от вершины. 2.09. 282=784 способа. Указание. Если 106=(2“'-5₽‘)Х Х(2^-51!2)-(2аз.5Рз), то а1 + а2 + аз = ₽1+₽2 + ₽з=6. 153
2.10. Число способов четно и заключено от 2 до 14. 3.01. (0, 0, а), (0, а, 0), (а, 0, 0). Указание. Воз- ведя в куб первое уравнение и вычтя из него третье, полу- чим: 3(x+y)(x + z)(y+z) = 0. Если х + у = 0, то 2 = а, и тог- да из второго уравнения полу- чаем: х = у = 0. Аналогично находятся остальные решения. 3.03. Обозначим отрезки че- рез АВ и CD соответственно. Построим параллелепипед на век- —>- —> —> >• торах ABt АС и AA' = CD. Тогда очевидно, что: а) высота параллелепипеда равна расстоянию между прямыми и потому не зависит от положения отрезков; б) все основания паралле- лепипедов при разных положениях отрезков равны; в) объем тетраэдра равен -^-объема параллелепипеда (см,, рис. 9). 3.05. 30 (число вершин додекаэдра + число плоскостей, пер- пендикулярных его большим диагоналям). 306 (n-l)(n-2)(n2-3n + 12)^ 4.01. Указание. Две последовательные центральные сим- метрии относительно точек X и Y есть параллельный перенос на вектор 2XY. 4.02. Указание. Если m— 1 плоскостей уже проведены, то пья плоскость добавляет столько частей, на сколько эта пос- ледняя разрезана прямыми ее пересечения с т—1 плоскостями. Тем самым наша задача сводится к аналогичной, но более прос- той: на сколько частей разделяют плоскость k прямых (k = 1, 2, ..., п-1)? 4.04. 686 чисел. Вычеркнем из 999 чисел, меньших 1000, числа, кратные 5: их [^J = 199. Далее вычеркнем числа, крат- ные 7, их = 142. Но среди чисел, кратных 7, имеется [999 1 — =28 чисел, одновременно кратных 5; они будут вычеркнуты дважды. Итого, нами вычеркнуто 1994-142 — 28 = 313 чисел; осталось 999 — 313=686 чисел. Замечание. Если требуется найти только приближенный ответ, его можно искать по формуле (попробуйте доказать ее сами): Nx 1000-( 1 —у) ( 1 —у) = |у 1000= 685.7... Это полезно, если нужно найти число чисел, нс делящихся на 3, 5, 7, II, 13.. 154
5.01. При b=0 решение одно: x—y=z=0. При 6#=0 решений нет. 5.04. Указание. Обозначьте у=-\/а+х-, тогда ~^а—у=х. Возведя эти уравнения в квадрат и сложив их, получим: (*+у)(л—у+1)=0. 5.05. Опишем окружность около данного треугольника. Про- должение биссектрисы пересекается с дугой окружности, стя- гиваемой основанием треугольника, в ее середине. Отсюда легко вытекает требуемое утверждение. 5.06. (а2-|-а+1)(а8—аг+а— а*+а3—а+1). Указание. Разложите сначала двумя способами на множители выражение а15 —1. 5.07. Указание. Замените четные коэффициенты на 0, а не- четные — на 1. 5.08. Указание. Предположив, что хорда CD уже прове- дена, проведите дополнительно хорду СК, параллельную прямой АВ, и найдите дугу KD. 5.09. Остаток 5. Указание. Если п—г кратно 6, то 10п—IO' кратно 7. 5.10. Искомая сумма отрезков равна сумме расстояний от точки Р до всех ребер основания, умноженной на тангенс угла наклона грани к основанию. Но сумма длин всех перпендикуля- ров, опушенных из точки Р на стороны правильного многоуголь- ника, вдвое превосходит отношение площади многоугольника к его стороне, т. е. постоянна. 5.11. На 30 частей. Указание. Сведите задачу к анало- гичной для окружностей на плоскости. Для п сфер: п^п ~£п+8) частей. 6.01. 3(Ь — с)(с—а)(а — Ь). 6.02. 35 суток. 6.03. 24 нуля (столько раз 5 встречается в качестве со- множителя в числе 100!). 6.06. Цифра 7. 6.07. 7 решений. Указание. Если данные четыре точки не лежат на одной окружности, то непременно часть из них будет лежать вне искомой окружности, а часть — внутри. Число реше- ний равно числу разбиений данных четырех точек на два непустых подмножества. ___ 6.09. Пусть а&с=а!4-&!4-с!. Поскольку 7! =5040> 1000, среди цифр а, Ь, с нет цифр больше 6. Среди них нет также цифры 6, так как в противном случае abc>& =720. Дальнейший разум- ный перебор показывает, что единственное подходящее число: 145=1! 4-4!+ 5!. 6.10. М= 121-64 = 7744. 6.12. В каждой точке паркета должны сходиться 4 четырех- угольника, повернутые разными своими углами. 6.13. 1-(1422—142)4-142=10153. 155
6.14. Число точек самопересечения равно максимальному целому nt для которого па<180°. 6.15. 300!>100jo°. 6.16. Указанно. Докажите, что высоты треугольника O1O2O3 — перпендикуляры к серединам сторон треугольника АВС. 6.17. -Lf ^4-^4-... = 1. л \ 02 Оз ап а\ ) У а2 а3 а, 6.18. 2857 чисел. Указание. Остатки от деления 2х — х2 на 7 периодически повторяются через 21. 7.02 . 523152 и 52365G. Указание. Разделите 524000 на 504 = 7-8-9 с остатком. 7.04 . Часть дуги окружности, построенной на ОР, как на диаметре. 7.05 . м (n + 1) (л + 2) (п + 3) + 1 = (п (п + 3) + 1 )2. 7.08 . Пусть Р(хо) = О. Если Хо четно, то разность Р(х0) —Р(0) также четна,— противоречие. Если же хо нечетно, то Р(х0) — — Р(1)= —Р(1) должно быть четным, а Р(1) нечетно,— снова противоречие. Следовательно, целых корней многочлен Р(х) не имеет. 7.10. х=—2 или х^2. 7.11. 31-2+1=63 корня. Указание. Постройте графики функций t/=sinx и у = -р~-. Затем воспользуйтесь приближен- но о. о иым равенством -^-«31,8... . 7.13. На три части. Указание. Из центра вписанной окружности опустить перпендикуляры на стороны треугольника. 7.15. а = 8 и а = 12. Указание. Сделать подстановки х = а и х= 10. 7.16. Указание. Простое число р>3 имеет вид 6л±Д, и его квадрат равен 12п (Зн2 ± 1)+ 1. 7.17. Указание. Точки //|, //2, Н$ лежат на описанной окру- жности. 7.19. Указание. Круг наименьшего радиуса, содержащий все точки, на своей границе содержит либо две диаметрально противоположные точки, либо три точки в вершинах остроуголь- ного треугольника. Поэтому г^1. 7.20. (а,&,с)=(3, 1, 2); (-3, -1, -2); (1,2, — |); (— 1, 1, -2). (См. 7.15.) 7.21. (0; 0); (1;0); (0; 1); (2; 1); (1;2); (2; 2). Указанйе. Преобразуйте уравнение к виду (х— 1)2 + (*/— 1)2 + (х — у)2 = 2. 7.22. Некоторая окружность, центром которой служит середи- на общего перпендикуляра к данным прямым. Указание. Исследуйте сначала случай пересекающихся прямых. 7.23. Указание. Сумма квадратов медиан к катетам в 5 раз больше квадрата гипотенузы. 156
8.01 и 8.06. Частное a—b. 8.02. Указание. Заменить каждое число на 2п 8.03. 29. 38. 47, 56, 65. 74, 83, 92. 8.07. 625 и 376. Указание. (N* - AQ ;(№ - Л/) -1 000. 8.08. Указание. Множество решений первого и второго уравнений есть соответственно пара прямых, пересекающихся в начале координат, и окружность, центр которой скользит (при изменении параметра а) вдоль осн абсцисс. 8.09. Длина этого отрезка равна разности между гипотенузой и меньшим катетом. 8.10. 1 769 580. Указание. Подсчитайте отдельно сумму единиц, десятков, сотен и тысяч. 8.11. 240 сторон. 8.12. Указание. Провести через точку Р прямую, парал лельную АВ. 8.14. (х,«)=(6, -21); (4. 15); (7, -12); (3.6); (8. -9); (2.3); 11,—6);. <-1.0); (14,-5); (-4,-1); (23.-4); (-13,-2). Указание. Переписать уравнение в виде (х —5)(t/-f-3) = — 18 8.16. Рассмотрите еще дополнительно треугольник АВ'С, центрально-симметричный треугольнику АВС относительно вер- шины А (окружность катится по стороне ВС). Тогда АВ’ и АС высекут на окружности дугу, равную дуге, высекаемой отрезками АВ и АС. Но угол А равен полусумме этих двух дуг, а с другой стороны, он равен 60е. 9.01. 3 угла. Указание. Сумма всех внешних углов рав- на 360е. 9.02. Указание. Сделать поворот на 60е по часовой стрел- ке вокруг точки В. 9.03. Число 1946. Указание. Частное от деления в обоих случаях должно равняться 112 — 98=14. 9.04. Xi = l, Хг = 2, Хз = 3, х4 = 4, хз= — 4, х6=—3, х?=—2, хв= — 1. Указание. Сложите все уравнения, получив утроен- ную сумму всех неизвестных. Далее сложите первое, четвертое и седьмое уравнения и получите xi = l. Остальные неизвестные находятся аналогично. 9.05. Указание. Данное произведение Р (х) удовлетворяет равенству Р(х) = Р( — х). 9.07. Указание. Проведите через А вторую прямую; из конца исходного отрезка проведите прямую, параллельную сто- роне рассматриваемого угла. 9.08. Указание. п24-3п4-5=(п + 7)(п — 4)+33. tex x-ysin2x 9.10. Производная функции у = ~ равна — и при хс( 0, у-) положительна, так как 2x>sin2x. 9.11. 7 или 14. 157
i, 9.12. Указание. Данное выраже- Jfa" ние преобразуйте к виду (х — 2у) (х —у)х /l Х(х + {/) (х4-2у) (х4-3{/). Полученные со- I множители, легко видеть, попарно раз- I личны. Но число 33 нельзя разложить Авл более чем на 4 различных сомножителя. лт А„ 9.13. х=±-(ОА +ОВ). Рис ю 9.14 и 9.20. 1°. Обозначим через а„ дорогу, выходящую из пункта Ап. Если машина т задерживается на перекрестке Рпт, то угол ап ближе к 90°, чем <хт (дорога ап идет круче, чем дорога ат\ рис. 10). 2°. Пусть ат — самая крутая из всех дорог, пересекающих о,. Если ат круче, чем ап, то машина т не может быть задержана раньше точки Рпт (доказывается от противного). Самая крутая дорога аи (см. условие), и поэтому машина № 14 нигде не будет задержана. Все машины, путь которых пересекается с а14, бу- дут задержаны (это машины № 1, 2, 3, 4, 6, 7, 8, 10, 12, 13, 18, 19, 22, 27, 28, 29, 30). Дальнейший аналогичный анализ показы- вает, что нигде не будут задержаны только машины № 14, 23, 24 и этот ответ не зависит от расстояний между соседними пунк- тами. 9.15 . Если общее число маршрутов равно Л/, а на маршруте п = 3 остановок, то N = n (п— 1)4- 1 =7. Пример из 7 маршрутов постройте сами. 9.16 . 10 очков. 9.17 . Найдется; первым из них будет член с номером 7501. Указание. Рассмотрите последовательность а*, составлен- ную из последних четырех цифр чисел исходной последователь- ности (а*^104), и воспользуйтесь принципом Дирихле. 9.18 . Указание. Воспользуйтесь тем, что множество 5 не изменится, если произвольным образом параллельно пере- нести отрезки АВ. CD и EF вдоль сторон треугольника PQR. 9.19 . 8 остановок. Указание. Докажите сначала, что каж- дый маршрут имеет одинаковое число п остановок и через каждую остановку проходит ровно п маршрутов. Получите отсюда урав- нение п (п — 1)4-1 = 57. Замечание. Эта задача возникла из нужд проективной геометрии. А именно пусть остановки — это точки на плоскости, а маршруты — прямые. Тогда условие 1) означает, что через любые две точки проходит прямая, а условие 2) — что любые две прямые пересекаются ровно в одной точке (что неверно в обычной геометрии, но верно в проективной). Теперь ясно, что основное отличие получающейся «геометрии» от обычной состоит в том, что число точек на плоскости не бесконечно, а равно 57. Оказывается, такая геометрия тоже существует; в ней каждая прямая имеет по восемь точек. 10.0 1. Остаток 6. Указание. Уменьшив каждое слагаемое на 1, получаем, что каждое новое слагаемое делится на х—1. 158
10.0 3. Член X18 будет отсутствовать, так как 18 нельзя представить в виде суммы пятерок и семерок. Коэффициент при х17 будет равен 20*^^=3420. 10.0 5. Эти окружности равны описанной окружности. 10.0 6. Свободный член полученного многочлена Р (х) равен Р (0)=((...((4 —2)2-2)2-...)2-2)2 = ... =(4 —2)2=4. Используя это и обозначив через Ak и В* коэффициенты при х и х2 соответствен- но, можно доказать, что Л* = 4Л*_| и В* = Л2_! 4-4В*_|. Так как Д1 = —4 (поскольку (х — 2)2=х2 —4x4-4), то отсюда легко сле- дует (по шагам), что Л*=—4*. Подставляя в формулу для В*, выраженную через Л*-1, Лл-2, ...» Д|, Bi, значения Bi = l, Л1==—4, ..., получаем: 10.0 7. Указание. Достаточно найти число, не имеющее с остальными пятнадцатью числами общих множителей 2, 3, 5, 7, 11 и 13. 10.0 8. 124-224-... + 82 = 204. 10.09 . Заменив х на (—х), получим, что коэффициенты при четных степенях х не изменятся. Значит, коэффициенты при х20 у данных многочленов те же, что и у многочленов (1 4~*2 + 4-х3)1006 и (1 — х2 —х3)1000. Ясно, что первый из новых много- членов имеет больший коэффициент при х20. 10.10. 0,89001. Указание. Можно ограничиться первыми пятью сомножителями. 10.11. Две касательные плоскости к цилиндру радиуса d и осью ЛВ, проходящие через точку М. 10.12. Указание. Сначала положите на чашку весов по одному кубику; затем, положив на одну чашку оба эти кубика, на вторую кладите поочередно по паре из оставшихся кубиков. 10.13. 31 цифра. Указание. Поскольку 2|0= 1024, то 2100 немного больше, чем 1ООО10, т. е. имеет 31 цифру. 10.15. Указание. Если многоугольник можно разрезать на параллелограммы, то для каждой его стороны найдется дру- гая, ей параллельная. Поэтому число сторон должно быть четным. 10.16. Указание. Среди выбранных чисел найдутся два таких, частное которых есть степень двойки. 10.18. Указание. Докажите, что в каждой строке четное число встречается уже на одном из первых четырех мест. 10.20. Указание. Рассмотрите наибольшие нечетные де- лители выбранных чисел. 10.21. Указание. Докажите, что скрещивающиеся ребра равны. 11.01. {2; 4; 4}, {2; 3; 6), {3; 3; 3}. 11.03 . Указание. Применим индукцию. Проведем еще одну прямую к имеющимся. Затем все цвета областей, 159
лежащих по одну сторону от нее, поменяем на противополож- ные — условие задачи будет по-прежнему выполнено. Тем самым индукционный шаг завершен. 11.04 . Если первое число равно т, то второе равно 2 (а'п“,4- + a'n“2-f-...-f-a+1), где а = 2Л. 11.05 . Указание. Воспользуйтесь тем, что пД0~3,16>я. 11.06 . Указание. Рассмотрите сначала частные случаи, когда точка А' лежит на ребре и когда она лежит на грани пира- миды ABCD. 11.07 . Длина пути из точки А в точку В, идущего по ука- занным в условии правилам, равна разности длин /л и tD каса- тельных AM и ВЛ4, проведенных к окружности в точках А и В. Поэтому длина общего пути по заданной ломаной будет равна (G — <в)+(«в — <с) + -+(<к—xL)+(lL — U=0. что и требовалось доказать. 11.08 . (2; 4), (4; 2). У к а з а н и е. Докажите сначала, что х и у — степени одного и того же числа. 11.10. Нет, так как точка, симметричная одному центру сим- метрии относительно другого, тоже центр симметрии. 11.11. Указание. Соедините середину диагонали с задан- ными серединами сторон и воспользуйтесь неравенством тре- угольника. р р+1 11.13. х=(-+ , у ===(Р^-") • Указание. Пусть y = kx (k — рациональное число), тогда из данного уравнения следует, । * что х = ЛЛ“\ y = kk~l. Пусть = тогда х , р+ч . Но так как х и у—рациональные, а р и q — взаимно простые, то из чисел р и p + q должен извлекаться корень 7-й степени. Поскольку при q^2 и р = пд справедливо нера- венство nq<Zp + q <. (я+ 1?» то 7=1. 11.14. Радиус наибольшей окружности Эта окружность вписана в максимальное по площади сечение куба, являющееся правильным шестиугольником. 11.15. 19 801 решение. 11.16. 4 луча (например, лучи, проведенные из центра тет- раэдра в его вершины). 11.17. Указание. Разложить вектор движения луча на три составляющие, перпендикулярные зеркалам. После каждого отражения соответствующая составляющая меняет знак. 12.01 . Указание. Разложите 26 460 на множители и до- кажите, что заданное выражение делится на 5-72 и на 22-33. 12.03 . Указание. Правая часть равенства всегда делится на более высокую степень двойки, чем левая. 12.04 . Пусть А — произвольная точка ломаной, а В — такая 160
точка ломаной, что обе части ломаной, соединяющие А и В, имеют длину -у-. Докажем, что середина отрезка АВ — искомый центр О круга радиуса покрывающего всю ломаную. Дейст- вительно, если М — произвольная точка ломаной, a М' — цент- рально-симметричная ей точка относительно О, то АМВМ' — па- раллелограмм. Сумма длин двух его соседних сторон не больше , следовательно, ММ'<С-^-. Отсюда что и требовалось доказать. 12.08 . Обозначим левую часть уравнения через у, правую — через yi, тогда x=y^+2ayi+-^, т. е. х выражается через yi точно так же, как у выражается через х. Отсюда следует, что параболы у=у(х) и yi=yi (х) симметричны относительно биссектрисы первого координатного угла. Точки пересечения этих двух графиков лежат на этой биссектрисе (докажите это!), откуда у=х=у]. Отсюда сразу находим оба действи- тельных корня (о<а<-0 : xt, 2=-Ц^±у(Ц^) — 12.11. Фишки неизбежно займут те же места. Указание. Если занумеровать 12 полей через 4 на пятое в порядке 1, 6, 11, 4, 9, 2, 7, 12, 5, 10, 3, 8, то одним ходом фишка сдвигается на соседнее место. 12.12. Точки Z в общем случае заполняют шестиугольник, стороны которого равны и параллельны сторонам треугольников АВС и DEF. 12.13. Указание. Докажите сначала, что разность масс двух любых гирь (в граммах) четна, а затем — что она делится на любую степень двойки. 12.14. Если расположить в вершинах шестиугольника гири равной массы, то точка пересечения медиан любого из образовав- шихся треугольников будет центром тяжести системы. 12.15. Какие-то два из чисел alt at+аг, ai+аг+аз, .... а(-)-...-J-аюо при делении на 100 дают одинаковые остатки. Раз- ность этих чисел и является искомой суммой. 12.17. Куб 2X2X2. 12.18. Наибольший центрально-симметричный многоугольник с данным центром О, вписанный в треугольник АВС, представ- ляет собой пересечение треугольника АВС и треугольника А’В'С', симметричного АВС относительно О. Таким образом, остается только выбрать точку О так, чтобы это пересечение было возможно ббльшим по площади; для этого надо, чтобы точка О совпала с точкой пересечения медиан треугольника АВС. 12.19. Указание. Докажите, что хотя бы одна из точек 1g 2, 21g 2, 31g 2, ... попадает в один из промежутков (lgAf+Л, 6 Заказ 247 161
1g(M4-1)4-Л); k — l, 2, .... а М — заданная комбинация цифр, с которой начинается степень двойки. 13.01. Радиус этой окружности равен а центр лежит в центре черной клетки. 13.02. Указание. Отделив последние 6 гирь, поделите оставшиеся на 61 группу по 9 гирь в каждой. В каждой из 62 групп гири разделить на 3 равные по массе группы теперь легко. 13.04. Против большей стороны треугольника лежит больший угол, поэтому (А —В) (а — 6)^0, (В — С) (b — с)^0, (С—Л)(с—а)^ ^0. Сложив эти неравенства, получаем искомое. 13.05. 25 мин. Указание. Докажите, что при передвижении на 2 км используется второй способ. 13.06. Всегда. 13.07. Может (см. рис. II). Угол А—октант (трехгранный угол с прямыми плоскими углами), AC = AD=l, АВ=3, А' выбрана достаточно близко к вершине В, например А'В=1. Тогда ЛВ4-ЛС4-ЛО = 5, A'B+A'C+A'D = 14-2>/5>5. Воз- можно и много других примеров. 13.08. Разложим сначала 9 гирь с массами л2, (п-)-1)2, ..., (л-|-8)2 на три группы так: 1-я группа: {п2, (п-}-5)2, (л 4-7)2}; 2-я группа: {(л-)-1)2. (л + 3)2, (л-|-8)2); 3-я группа: ((л4-2)2, (л-|-4)2, (л+6)2}. Масса первых двух групп одинакова, а третья группа легче на 18 г. Затем следующие за ними 9 гирь разложим так, чтобы первая и третья группы весили одинаково; наконец, после- дующие 9 гирь разложим так, чтобы вторая и третья группы име- ли одинаковую массу, а первая была легче на 18 г. Сгруппировав затем все первые, все вторые и все третьи группы, получим разло- жение любых 27 последовательных гирь на 3 кучи равной массы. Осталось эту процедуру повторить 3 раза, и тогда все гири окажутся разложенными требуемым способом. 13.09. 5С10. Указание. Выделить под радикалами полные квадраты. 13.11. 13 сторон. Очевидно, сторонами такого многоуголь- ника могут быть самое большее две диагонали, проведенные из данной вершины. Отсюда легко вывести, что число сторон не боль- ше 13. С другой стороны, если, например, исходный 13-угольник правильный, то среди многоугольников разбиения будет 13-уголь- ник (в центре). Аналогичен ответ для любого многоугольника с нечетным числом сторон. 13.12. Указание. Восполь- 1 2 3 зоваться тем, что —--г——X х о 4 V 98 991 'У х "’’99’100 100 \ 10/ ’ 13.14. Указание. Обозна- чим через kn число способов, ко- торыми можно соединить попарно 162
какие-либо 2п точки окружности п непересекающимися хордами. Докажите, что kn = kn-l + kikn-2+k‘ikn_3 + ...+kn-2ki+ka-i. Отсюда *2 = 2, *з = 5. *4=14. *5 = 42..*(0=16 796. Общий ответ для произвольного п таков: *я = 2^.^[ C"n+i. 13.16. Выберем сначала из данной последовательности ai, ..., йю! убывающую подпоследовательность следующим образом: первый ее член — at, второй — первое из чисел, меньших at, третий — ближайший член, меньший а2, и т. д. Затем выбро- сим из «большой» последовательности эти члены и построим тем же способом вторую убывающую подпоследовательность, затем третью и т. д., пока не исчерпаем все заданные числа. Если одна из наших последовательностей имеет больше 10 членов, то за- дача решена. В противном случае мы имеем не меньше 11 после- довательностей; докажите сами, что можно выбрать из каждой из них по одному члену так, чтобы они образовывали возра- стающую последовательность. (См. задачу 48.09.) 13.17. Указание. Пусть А, В, С и D — вершины данного четырехугольника; К, L, М, N — точки касания, причем К лежит на АВ, -L — на ВС и т. д. Докажите сначала, что KL лежит в одной плоскости с диагональю АС и что MN тоже лежит в одной (но отличной от первой) плоскости с АС, а затем, что либо KL и MN обе параллельны АС и, следовательно, параллельны между собой, либо пересекаются с АС в одной и той же точке. (См. также решение задачи 19.19.) 13.18. Можно. Рассмотрим, например, 10 прямых плоскости, никакие две из которых не параллельны и никакие три не пере- секаются в одной точке. Будем считать, что прямые — это авто- бусные маршруты, а их точки пересечения — остановки. При этом с каждой остановки можно проехать на любую другую: если остановки лежат на одной прямой, то без пересадки, а если нет, то с одной пересадкой. Далее, если даже отбросить в этой схеме одну прямую, то все еще останется возможность проехать с каждой остановки на любую другую, сделав в пути не больше одной пересадки. Однако если отбросить две прямые, то одна остановка (точка пересечения этих прямых) уже вовсе не будет обслуживаться оставшимися маршрутами и с нее будет невоз- можно проехать на какую-либо другую. 14.03 . Первое число меньше второго. 14.05 и 14.09. 3 звена; 4 звена. 14.06. 0,239. 14.07. Равносторонний треугольник со стороной а. 14.08. /?>2г. 14.10. Пусть х, у, г — «нормальные» шкалы на данных прямых. Положим теперь /=10* (это значит, что в точке с ко- ординатой, например, 2 мы ставим метку / = 100), /?= 10 2у и 1/=10~*. Если три точки на шкалах лежат на одной прямой, то x+z=2y, откуда /7=10'2= 10х- 10"2y=/-/?. 6* 163
14.11. Это число больше (1О50)3 и меньше (10so+1)3. 14.13. Указание. Докажите по индукции, что эта сумма равна (1+24-... +л)2- 14.16. Коэффициент при х14 равен (—1). 14.18. Количество чисел от 1 до т, кратных Ь, равно Из условия следует, что ни одно из чисел 1, 2, ..., 1951 не делится сразу на два из чисел ai, .... а„. Поэтому количество чисел из последовательности 1, 2, .... 1951, делящихся на одно из чисел ai, , ап, равно I -- I + ..+ —— и оно не превосходит 1951 „ /1951 .\ . ./1951 Л^..ЛС. 1951 , . 1951 <1951 + л <2-1951, и что и требовалось. 14.19. Проекция тетраэдра всегда совпадает с проекцией ка- ких-то двух его граней, например АВС и BCD. Пусть К, L, Л4, N — середины сторон АВ, ВС, CD и AD соответственно, тогда легко видеть, что KLMN — квадрат, площадь проекции которого ровно вдвое меньше, чем площадь проекции тетраэдра. Поэтому пло- скость искомой проекции должна быть параллельна плос- кости квадрата. 14.20. Кривая, отличная от окружности, которая обладает тре- буемым свойством, состоит из двух равных дуг окружности, со- держащих по 120°; радиус дуг равен стороне треугольника. По- думайте сами, как можно построить другие кривые с тем же свой- ством (их существует бесконечно много). 14.21. а) Докажем лемму: если в квадрате из 49 клеток отмечены крестиками 24 клетки, то можно из него вычеркнуть 4 строки П|, а?, аз, ад и 4 столбца Ь\, Ь%, Ь* так, что на пере- сечении строки at и столбца Ь, стоит крестик при всех / = 1, 2, 3, 4 Для доказательства нужно показать сначала, что можно вы- черкнуть одну строку и один столбец, на пересечении кото- рых стоит крестик (в дальнейшем о такой операции мы будем говорить коротко: «вычеркнуть крестик»), оставив при этом в таблице не менее 15 крестиков. Если в каждой строке и в каждом столбце стоит не более 5 крестиков, то это очевидно (можно вычеркнуть любой крестик); если же в какой-то строке стоит 6 крестиков, то в одном из соответствующих шести столбцов их не более 4 и можно вычеркнуть этот столбец вместе со строкой. Случай 7 крестиков в строке рассматривается аналогично. Далее следует показать, что из таблицы 6X6 с 15-ю крести- ками можно вычеркнуть крестик так, чтобы в квадрате 5X5 осталось не менее 8 крестиков, и затем в третий раз вычеркнуть строку и столбец, оставив хотя бы 1 крестик. Тем самым лемма будет доказана Вернемся теперь к задаче: между 7-й и 8-й остановками в автобусе едет н° более 25 человек. Составим квадрат 7X7, 164
занумеровав его строки от 1 до 7 («первые 7 остановок»), а столбцы — от 8 до 14. Точку, стоящую в t-й строке на /-м месте, отметим нуликом, если в автобусе есть пассажир, едущий от О/ до О, (1^7, остальные точки отметим крестика- м и. Поскольку в квадрате не более 25 нуликов, там есть 24 крестика, и по лемме в квадрате есть 4 строки и 4 столбца, пересечения которых отмечены крестиками. Их номера и будут номерами нужных восьми остановок. б) Предположим, что в автобусе ехало ровно по одному пас- сажиру от i-й до j-й остановки для всех i, j, 1 10 (а по- следние 4 остановки автобус ехал пустым). Нетрудно убедиться, что при этом условие задачи выполнено (больше всего народу в автобусе — 25 человек — ехало между 5-й и 6-й остановками) и не существует пяти пар остановок, между которыми бы никто не ехал. 15.01 . Указание. Углы LMN и ANL равны половине дуги LN, но ДЛМ£= ДЛ/ЛУ<90°. 15.04 и 15.06. Пешеход С должен выйти из W за — часа до выхода Л из М. 15.11. Указание. Из рисунка 57 к решению задачи 32.25 видно, что в рассматриваемых треугольниках ABD и BCD поме- щаются окружности, сумма радиусов которых равна R. 15.12. У к а з а н и е. Все числа последовательности, начи- ная самое позднее с аз, не превосходят 81+81+4 = 166. Для чисел от 1 до 166 нужно разумно организовать перебор. 15.14. Указание. Начните с равенства cosarcsinx = =sin arccos x=^j\ —x2. 15.15. Д о к а з а т e л ь с т в о по индукции: при п=2 неравенство очевидно; если оно верно для п, то (1—х)"+,+ +(1 +х)"+' <((1 -х)"+(1 +х)") ((1 -х)+(1 +х))<2"«2=2п+1. 15.16. Указание. Если Л, В, С — точки касания сферы с плоскостями угла, то треугольники ОЛ5, OBS, OCS равны. 15.17. Если а>0, то при очень большом р решений нет вообще. Если а<0, то при большом р меньший корень будет отрицательным (это видно из графика). Значит, а=0. 15.20. Указание. Пусть окружности, вписанные в тре- угольники АВС и ACD, касаются диагонали АС в точках Ki и Ki. Докажите, что расстояние между Ki и Ki равно -|-|(ЛД + ВС)—(ЛВ+СД)|. 15.21 н 15.23. Указание. Если 0<а<1, то а<-\/а<1. 15.22. Это множество изображено на рисунке 12. 15.24. Указание. Д NEA со д NDB и &NADco &NBF. 15.25. Если А и В — два таких числа Рис. 12. 165
того: хз = х5 и т. д.; аналогично л д__п и — — целое, то _ тоже целое и, как нетрудно пока- зать, делится на 9. Но по- д_____________п скольку —-— заведомо меньше 7, то Л —В = 0. 15.26. п =100 и и = 198. 15.27. Заметим, что все х,=/=0. Из первого и второго уравнений вытекает, что Х|=хз» из третьего и четвер- Л2=Х4 = .... Если п нечетно, то Х[=хп и из последнего уравнения следует, что Х|= ±1; тогда из первого уравнения вытекает х2=х\. Итак, для нечетного п справедливо = х2 = ...=xrt_i=xn = ± 1. Если п четно и Х| = ос, то х2=— и мы имеем: а 1 Х|=Х3 = ...=Хл~|=аэ Х2 = Х4 = ... = ХП = —. а 15.28. Оси цилиндров должны быть перпендикулярны друг другу (см. рис. 13). 15.30. Указание. Из вершины А проведите прямую AR под углом 60° к основанию АС (рис. 14). Она пересечет QC в точке S, которую соединим с точкой Р. Докажите, что Л QPS = = &QRP. Отсюда Z.PQS= Z.PQR, а так как их сумма равна 60°, то Z_PQC=30°. 15.31. Пусть А — самый низкий из высоких, а В — самый высокий из низких. Рассмотрим С, стоящего на пересечении продольного ряда, где стоит А, и поперечного ряда, где стоит В. Тогда А^С^В, т. е. А не ниже В. 2л 15.32. Обозначим данную функцию через / (х). Тогда J / (х) dx= =0. Отсюда следует, что если f(x) принимает положительные значения, то она принимает и отрицательные (из непрерывности f). Осталось показать, что f (х) не есть тождественный нуль. 16.02. Указание. Если число 11... 11 (п единиц) кратно 33...33 (100 троек), то п должно делиться на 3 и на 100. 16.03. Восставим перпендикуляр / к АВ в точке А- отметим на нем точки С и D. Восставим далее перпендикуляр Г к АВ в точке В, а затем — перпендикуляры к прямой I в точках С и О до их пересечения с Г соответственно в точках С и О'. Обозначим через X точку пересечения прямых AD' и BD и через Y — точку пересечения прямых АС' и ВС. Тогда прямая XY пересекает отрезок АВ в его середине. 16.04. п2+8п4-15=(п + 4)2- 1. 16.06. Указание. Обозначьте радикал, стоящий в знаме- нателе, через а. Тогда радикал в числителе равен -^2+а. Осталось доказать требуемое неравенство при 0<а<2. 166
16.10. Указание. OOt =—(AAi + BBt). 16.11. Пусть x200-y2W + l=f(x).g(y), f(0)=a, g(0)=b. Под- ставляя в написанное равенство сначала х=0 (у — произ- вольно), затем у=0 (х—произвольно) и наконец х=у=9, получим: a-g(y)=l, b-f(x)=l, ab = l. Отсюда f (х)Х Xg (у)= 1Ф14-x200i/2(® — противоречие с предположением. 16.12. BF\\CE и ДСЦЕГ, т. е. AFEC — параллелограмм (точ- нее, ромб). Следовательно, треугольники АСЕ и AFE равно- велики. Отрезая от треугольника АСЕ треугольники АЕ\Е и EEiDt и добавляя их к треугольнику AFE, мы одновременно к оставшемуся треугольнику ACDt добавляем равновеликие им треугольники C\D\D и DDiE и получаем многоугольник ABB\CCiDED^, равновеликий четырехугольнику ADiEF (рис. 15). 16.21 и 16.27. Все четные колеса вращаются в одну сторону, а все нечетные — в противоположную. Следовательно, чтобы система колес вращалась, число колес должно быть четным. 16.22. Сумма углов всех N треугольников, на которые раз- бивается 1000-угольник, равна сумме всех углов 1000-угольника и всех 500 полных углов, т. е. равна 998-180° 4* 500 «360°. Зна- чит, число треугольников равно 9984-2-500=1998. 16.24. Разобьем четырехугольник диагональю на два тре- ab cd угольника; их площади не превосходят -у и -у соответственно. Таким образом, S<-y(ab4-«0l аналогично доказывается, что S<-y(ad-|-frc)- Складывая эти неравенства, получаем искомое. 16.25. Пусть известно, что число A=aia2...an-ian(n=l953) делится на 27; докажем, что B=anai...an_i тоже делится на 27. Для этого достаточно доказать, что 10В4"О« при делении на 27 дает остаток ап. Но 10B4-an=an-10"4-4=an-99...94-7l4- 1953 4-ал, а так как 1953-значные числа 99...99Э и А делятся на 27, то все доказано. Даль- нейшее просто: надо перебрасывать по одной цифре из конца числа в начало. 16.26. В первой группе количество мно- гоугольников больше. Дело в том, что любой многоугольник второй группы мож- но «дополнить» вершиной Ai и получить многоугольник первой группы (обратная операция возможна не всегда, например для треугольника). 16.28. х*=1 при k четном; х*= — 1 при k нечетном. Указание. Вычтем из Рис. 15. 167
каждого уравнения предыдущее. Тогда из последнего уравнения находим: xioo=l, из предпоследнего: хд9 = = 1 — 2х|Оо= — 1 и т. д. 16.31. Значения трехчлена -|-х2-|- в точках Х| и х2 равны ( —|-xi) и -у-*2, следовательно, они име- ют разные знаки. Поэтому один из корней трехчлена расположен между xt и х2. 16.33. Искомыми будут четырехуголь- ные пирамиды АС'СВВ', АС'Ь'А'В' и ЛС'СОО' (рис. 16). 16.34. {1; 2; 3; ...; 4 16.36. Указание. Воспользуйтесь Х(хл-1—д^)2. Получите из него, что хл — V2<-|-(xn-i — д/2). Вто- рым соотношением воспользуйтесь до х30, а первым — с х3| до х36. 17.02. Да, будут равны. 17.03. Возможны два ответа: 1014 (при делении на 3 и на 2) и 1035 (при делении на 9 и на 5). 17.04. Не существуют, так как пг2—п2 нечетно или кратно 4, а 1954 — нет. 17.05. 100, так как |00а + |Ш,+с юо. а+Ь+с 17.06. Ответ показан на рисунке 17. 17.07. Если повернуть все построенные векторы на 90° и умножить их на 2, то они станут равны векторам, построенным на сторонах многоугольника, и поэтому их сумма будет равна 0. Но тогда и исходная сумма равна 0. 17.11. Условие задачи означает, что f (xo)=f' (х0)=0. Следо- вательно, х=хо — двукратный корень первого уравнения, и потому наш многочлен делится на (х—хо)2. 17.12. Искомое число равно 99999 78 596061...99100. Из двух чисел с одинаковым количеством цифр больше то, у которого больше первая (слева) несовпадающая цифра. Поэтому сна- чала надо вычеркнуть 84 цифры слева, оставляя все встре- чающиеся девятки: получим 99999 505152...9Э 100. Затем вычер- киваем 15 цифр 5, 0, 5, 1.5, 6, 5 — следующая цифра будет 7. Далее можно вычеркнуть только еще одну цифру 5 из числа 58. 17.13. Из данных неравенств вытекают следующие нера- венства: 168
at — «2^2 (a2 —аз), аг —аз ^2 (аз—а«),.... а1Оо — ai>2 (ai—аг). Сложив их все, получим 02>0, откуда легко вывести, что а> — а2==а2—аз=...=0. 17.15. Не существуют. Указание. Построим в пространстве треугольники АВС и BCD со сторонами 8, 10 и 13 (как указано в условии) и общей стороной ВС—13. Зафиксируем плоскость треугольника АВС и будем вращать треугольник BCD вокруг стороны ВС. Легко видеть, что AD максимальна, когда D лежит в плоскости треугольника АВС. Тогда ВС и AD — диа- гонали параллелограмма ABCD и легко подсчитать, что AD<C 13. Поэтому при любом расположении точки D, очевидно, ЛО< 13 и, следовательно, искомого расположения точек А, В, С, D в пространстве не существует. 17.16. У к а з а и и е. Дискриминант квадратного уравнения D=4 sin2 (ху)—4^0, отсюда |sin (xt/)| = 1 и х= —sin (ху). 17.21. Две. Эти буквы надо расставить в узлах сетки в шах- матном порядке. 17.22. Данная система обладает тем свойством, что коэф- фициент ал при Xk в k-м уравнении по модулю больше сум- мы модулей всех остальных коэффициентов. Воспользуемся этим: пусть хт — наибольшее по модулю из чисел хь.... х7. Тогда в m-м уравнении член amxm больше по модулю суммы всех осталь- ных. Но так как правая часть равна 0, то хт=0. Поскольку он наибольший по модулю, то имеется единственное решение: Х|=Х2 = ... =х7 = 0. 17.23. Ни одной, одна или 7 осей симметрии. Указание. При отражении одной оси симметрии относительно другой снова получается ось симметрии. 17.24. Указание. Если под делителем q написано ( + 1), то под делителем 2q будет стоять (—1), и наоборот. 17.25. Не смогут. Указание. Докажите, что на каждой прямой сетки возникает свое направление движения. 17.27. Указание. Принять точку О за начало прямоуголь- ной системы координат, а прямую mt— за ось Ох; если kt, kz, Лз — угловые коэффициенты остальных прямых, то (k3—*iV 1ОВ1 = V 2 7 1 (*з-М2 (*з-*1)2 4 (считаем |ОЛ|| = 1), что и требовалось доказать. 17.31. Окружность с центром на lt. 17.33. Указание. Заменив строгие неравенства в условии на нестрогие, докажите, что сумма трех максимальных чисел может быть равной 100 только в двух случаях: (1) когда максимальное число равно 100, а все остальные 99 чисел равны нулю и (2) когда какие-то 9 чисел равны , а остальные рав- ны нулю. 169
*6 17.34. Не существует. Указание. / \ Переставить числа в порядке возраста- / \ ния. / \ 17.35. Может. Указание. По- строив очередные 5 отрезков, мы увели- Л**^*^^**хХ чиваем число свободных концов на 4. _____,г \ 17.39. Легко видеть, что если |х|^1, то левые части обоих данных уравнений рис is положительны. Поэтому и их полусум- ма положительна, т. е. не обращает- ся в 0. 17.40. В первом классе должны содержаться все числа с нечетным числом двоек, во втором — с четным. 18.02 . Z_/MC = 60°, Z./1CB = 3O°. Указание. Докажите, что AB = BD, далее, BD = DC, Z_BDA=2 ZLBCA, АВАС = = 2 Л AC В. Ho Z.ВАС + /LACB =90°. 18.03 . Объединение высоты BH и дуги АС величиной 120°, которая построена на стороне АС как на хорде (рис. 18). 18.05 . Только прямоугольники размером аХ13а. 18.07 . Это множество точек заполняет параллелограмм. 18.09 . Плоскость, полуплоскость, а также полоса между двумя параллельными прямыми. 18.11. (fe-O + fe-1 +... + k (k-!))+(! 4-2 + ...4-fc)=^tll. Указание. Представить таблицу в виде суммы двух таблиц: у первой таблицы в /-й строке стоят числа £•(/—!), ...,£•(/—!), а у второй — числа 1, 2, 3, ..., /?(/=!, 2, ..., k). 18.12. Кольцо, заключенное между двумя концентрическими окружностями, центр которых — середина отрезка, соединяющего центры исходных окружностей, а радиусы равны полусумме и полуразности радиусов данных окружностей (при равенстве ра- диусов кольцо превращается в круг). 18.13. (1; 0); (0; 1). Указание. Из второго уравнения следует, что и z/^1. Но если х<1, тох3<1 и тогда из первого уравнения у^0\ аналогично х^0. Если 0<х<1, то х3>х4, y3^z/4 и уравнения системы противоречат друг другу. 18.14. Рассмотрим остатки от деления всех чисел а, на р. Их ровно р, и среди них нет нуля (иначе соответствующее сц не было бы простым числом); поэтому есть два числа су и ah дающие одинаковые остатки, так что а, —щ кратно р. Но 1а,— — п,| где d — разность прогрессии, k<Zp — целое. Отсюда следует, что d кратно р. 18.15. Указание. Пусть Н — точка пересечения АВ с про- должением CD. Поскольку точки А и В равноправны, можно считать, что точка Е находится на АН. Докажите, что разность EC — ED уменьшается, когда точка Е движется от Н в направ- лении А. 170
18.19. Можно. Для этого достаточно провести плоскость через точки А, В, С, равноудаленные от точки О. Тогда треуголь- ники ОАВ, ОАС и ОВС равнобедренные, поэтому все углы при основаниях АВ, ВС, АС острые. 18.20. x=y—z—0. Указание. Видно, что х3 четно, поэто- му х3 делится на 8. Сокращая на 2, видим, что у3 четно, поэтому у3 делится на 8. Снова сокращая на 2, получаем таким же образом, что z3 делится на 8. 18.21. Указание. Если f(х)=ах2 4-6х-|-с, то V/(*+1)— —V/ (*)< 1 при достаточно большом х. 18.23. 28 партий. Сначала по олимпийской системе за 25— 1 = =24 партии определяется чемпион. Второй по силе — тот, с кем играл чемпион; их может быть 5. По олимпийской системе еще за 4 партии выявляем чемпиона среди них. 18.24. Можно последовательно получать нужные разбиения для любого числа п прямоугольников, начиная с и=5, при- кладывая новые прямоугольники «по спирали», как показано на рисунке 19. 18.25. Положим f (х)=-\/ах? + Ьх+с и устремим х к -Ь<ю. Легко видеть, что а>0 и ZH-t-U 1W f(x+I)+f(x) f(x+|)+/(x) У Если x целое, то f (х) и f (х-|-1) тоже целые, а потому и d=-Ja целое. Более того, поскольку разность f (x-f-1)—f(x) целая, то при достаточно больших х она обязана равняться своему пре- делу: f (х-|-1)—f (x)=d. Пусть это соотношение верно, начиная с некоторого хо, и у=х0-Ьл; тогда f (y^f (x0)+nd, т. е. ay2 + by+c=(j (xo)+nd)2=(dy—dxo+f (х0))2. Отсюда уже легко следует утверждение задачи. 18.26 и 18.33. Указание. Сделать инверсию относительно окружности, проходящей через точку касания В первых двух б) Рис. !9. 17!
окружностей и имеющей центр в точке А пересечения внутрен- ней касательной с третьей окружностью. Докажите, что окруж- ность инверсии проходит через две точки М и N пересечения внешней касательной и третьей окружности. Отсюда AM=AN. 18.27. Просуммируем все номера на сторонах и на всех от- резках ОА/. Поскольку каждый отрезок ОА, учитывается дважды, эта сумма равна (1 4-.„4-n)4-2(1 4-.-4-п)=3п^+^ . Значит, сумма номеров на сторонах любого из п треугольников равна з у(п-|-1). Так как эта сумма — целое число, то п=£ 10 (пункт б). Пример для п=9 придумайте сами. 18.29. Будем считать, что а>0. Тогда все возникающие в задаче числа неотрицательны. Если а<-~1 , то |Л/а]<Л/ — 1, и потому среди чисел [а], .... [Л^а] есть совпадающие. Итак, . Но тогда и ~— (второе условие аналогично первому с заменой а на —, так что Легко видеть, что все числа из этого интервала годятся. Случай а<0 рассматривается аналогично. 18.31. Можно, например, выбрать систему отрезков: [1; 2], |^2-|-; 3-^-j , 4-|-J, ..., а в отрицательной части оси Ох симметрично относительно О (промежутки между отрезками об- разуют убывающую геометрическую прогрессию -g-, ...). Докажите самостоятельно, что не существует бесконечной арифметической прогрессии, все «мены которой попадают в «ще- ли» между отрезками указанной системы. 18.32. Пусть Х|, хг — два положительных корня, xi>X2. Раз- делим наше уравнение на х”, затем подставим вместо х оба корня по очереди и вычтем получившиеся равенства друг из друга; легко видеть, что получается противоречие. 18.34. Сыграно 5 партий; существует единственное распре- деление играющих (с точностью до перенумерации игроков и туров). 18.35. По теореме Безу f(x) делится на х—поэтому q-f(x) делится на qx — p, и в частном получается многочлен с целыми коэффициентами. А так как р и q взаимно просты, то f (Л) делится на qk—р. 18.38. С?95«. Указание. Следует брать все тройки, содер- жащие произвольно выбранную точку А. 19.02 . 18, 45, 90, 99. Указание. При умножении на 9 сумма цифр не меняется, поэтому исходное число кратно 9. Дальше — проверка умножением на оставшиеся цифры. 172
19.03. Через каждую точку самопересечения проходит 2 зве- на, и на каждом звене лежит ровно одна точка самопересечения. Поэтому число звеньев вдвое больше числа точек самопересече- ния. 19.04. 13. См. решение задачи 19.09. 19.05. 1304 точки. 19.07. Данное число обозначим через а, округленное — через ао и а = а—ао. Если а<0,01, то Оо=0 и — = 0. Если а>0,01, а то — =1—при этом Оо^0,01, а<0,01. Значит, 0<—<4- а а г а 2 и 1—2-^1. Докажите сами, что величина может при- нимать любое значение 0, удовлетворяющее неравенству 0^р^-|-. После округления числа могут получиться числа: 0; 0,50; 0,51; ...; 0,98; 0,99; 1,00. 19.09. Если числитель и знаменатель делятся на k, то на k делятся также a(cl+d) и c(al-t-b), а следовательно, и их раз- ность ad—be. 19.10. Каждое число, не стоящее с края, не превосходит наибольшее из своих соседей, т. е. не максимально. 19.11. Указание. Если масса четырехугольника сосредо- точена в его вершинах (скажем, если в вершинах размещены гирьки равной массы), то точка пересечения описанных пря- мых является центром тяжести четырехугольника. 19.14. Таких способов либо 8, либо 4, либо 2. Указание. Множество точек D есть пересечение каждой из двух параллель- ных плоскостей, удаленных от плоскости треугольника АВС на й с каждым из двух цилиндров с осями АС и ВС и радиусами — и — . АС вс 19.16. Квадрат, описанный около окружности радиуса г так, что одна его сторона лежит на основании треугольника, пере- секает обе боковые стороны треугольника; поэтому его сторона 2г больше стороны а рассматриваемого квадрата К. Квадрат, вписанный в указанную окружность, имеет сторону, меньшую сто- роны квадрата К, т. е. а>г^/2. 19.19. Пусть hi, Й2, .... йп—расстояния от точек At, .... Ап до данной плоскости. Тогда указанное в задаче произведение равно ftt Лг йз Ли-1 hn а • — • - I • . • * • *~ I е Аг Аз Л< hn hi Замечание. Тем же способом можно решить задач\ 13.17. а именно провести плоскость через три точки касания; аналогичным подсчетом доказать, что четвертая точка также лежит на этой же плоскости 173
19.22. Д(В1 = 2. Указание. -L(±-(1 +Д2В() + 1) + 4-... +1)4-1)=Д2£ь 19.25. Указание. Провести индукцию по числу сорванных листьев. 19.26. Будем распределять груз так. Грузим ящики на первую машину, пока их масса нс превысит 1,5 т; последний ящик сни- мем и положим рядом с машиной. После этого будем грузить, ящики на вторую машину, потом — на третью и так до тех пор, пока не загрузим 8 машин. Общая масса ящиков на этих 8 ма- шинах и ящиков рядом с ними больше 1,5-8=12 т. Поэтому масса оставшихся ящиков меньше 1,5 т и их сможет увезти девятая машина. Лежащие на земле 8 ящиков разобьем на две четверки; каждая четверка ящиков имеет массу меньше 4-350=1400 кг, и поэтому их можно увезти на двух машинах. 19.27. Указание. Пусть сумма чисел /-й строки 5^518; докажите, что сумма всех чисел в двух строках и двух столбцах с номерами i и 9—i не меньше 4S — 112^ 1960> 1956. 19.30. Предположим, что это не так. Занумеруем наши 9 прямоугольников произвольным образом и начнем закра- шивать их: сначала первый, потом второй и т. д. Закрашивая первый, мы закрасим площадь, равную 1. Закрашивая второй, мы закрасим площадь, большую -|- (площадь, меньшая воз- можно, уже закрашена, так как содержится в первом прямоуголь- нике). Подобным же образом, закрашивая третий прямоугольник, мы закрасим больше чем — и т. д. Получится, что в общем 8 7 1 закрашенная площадь больше чем 1+ —+ —+ ... + -§- = 5. Противоречие. 19.32. Можно. Указание. Плоскостями, параллельными граням куба, разобьем его на 133 = 2197 кубиков с ребром 1. Среди них, очевидно, имеется такой, внутри которого нет ни одной из 1956 точек. 19.33. Основные шаги решения: 1) числа х, у, z неотри- цательны; 2) среди них есть нуль; 3) среди чисел х\, yi, Zi также есть нуль, так что среди чисел х, у, z два совпадают. После этого остается проверить две тройки чисел: (0, 0, 1) и (0, 1, 1). 19.35. Указание. Докажите сначала, что если вершин кле- ток нет ни на сторонах, ни внутри треугольника, то его площадь равна Затем, воспользовавшись формулой Гсрона для пло- щади треугольника, докажите, что квадрат одной стороны тре- угольника не меньше суммы квадратов оставшихся. 19.36. Пусть ai, аг, ап — данные числа, а,— одно из под- черкнутых и k — наименьшее число такое, что сумма a<+a< + i + 174
положительна (если а,>0, то 6=0). Тогда легко видеть, что все числа а^+г, ...» ai+k подчеркнуты. Начнем теперь складывать все подчеркнутые числа, начиная с а\ (если оно подчеркнуто). Из приведенного рассуждения видно, что сум- ма нескольких первых подчеркнутых чисел положительна. Теперь можно отбросить начало последовательности и аналогично до- казать, что сумма нескольких следующих подчеркнутых чисел положительна, и т. д. Таким образом, сумма всех подчеркну- тых чисел, если их правильно сгруппировать, состоит из несколь- ких положительных слагаемых, т. е. положительна. (Решение задачи 29 аналогично.) 19.39. Указание. Рассмотреть развертку пирамиды. Она — треугольник, разделенный на 4 треугольника средними линиями. 20.01. Искомая трапеция отсекается от правильного 5-уголь- ника его диагональю. 20.02. Положив х = 0, получаем, что d кратно 5. Полагая да- лее х=1, х= — 1 и х = 2, легко показать, что 26, а + с и а —с кратны 5. Отсюда вытекает утверждение задачи. Замечание. Здесь существенны два обстоятельства: а) степень много- члена меньше 5; б) число 5 простое. Придумайте сами многочлен 5-й степени, который при всех х делится на 5, и многочлен 3-й степени, который при всех х делится на 4, причем у каждого многочлена и е все коэффициенты делятся на 5 и 4 соответственно. 20.03. Число горизонтальных звеньев, по которым улитка уда- лялась от пункта (звено — это отрезок, по которому улитка про- ползает за 15 мин), равно числу горизонтальных звеньев, по которым она приближалась к нему. Значит, число всех гори- зонтальных звеньев четно. По тем же соображениям четно и число вертикальных звеньев. Но числа вертикальных и гори- зонтальных звеньев одинаковы. Следовательно, общее число всех звеньев кратно 4. 20.05. 40 столбов. 20.06. Две концентрические окружности с тем же центром и радиусами R и yfiR2— г2. Указание. Сумма квадратов рас- стояний от любой точки О до концов одной диагонали равна сумме квадратов до концов другой. 20.08. Квадрат. Докажите сначала, что это прямоугольник. 20.09. Обозначим через S* сумму всех чисел Л-й строки (всего в таблице m строк), через S;— сумму чисел /-го столбца (всего столбцов п) и через А — сумму всех чисел таблицы. Тогда S1 + S2 + — 4-Sm = 21-I-X2-I- — + ?н=А. Так как каждое число в Л-й строке равно произведению S* на сумму чисел 2/ того столб- ца /, в котором оно стоит, то сумма всех чисел в Л-й строке равна Sk — Si-S*4-£2-3*4"...4*2n-S* = S* (St 4*••• 4-2n) = S* • A. Поэтому либо Д = 1, либо каждое S*=0, а значит, каждое число, стоящее в клетке (k, I), равно S*-S/=0. 20.12. х=^/4. Указание. [х]=х—(х), где 0^{х)< 1 — дроб- ная часть; х3—х4-{х)=3, откуда 2<х(х2—1)<3. 175
20.16. п четное (любое), нечетные п не подходят. 20.20. Можно. 20.21. Часть круга радиуса /, заключенная между окружностью радиуса / и границей квадрата, вершинами которого являются точки пересечения прямых ОА и ОВ с этой окружностью. 20.22. Можно. Указание. Достаточно занумеровать кон- такты лампы от 1 до 7 по часовой стрелке, а отверстия штеп- селя — против часовой стрелки. 20.23. Третья сторона должна быть равна большей из двух имеющихся. Указание. Зафиксируйте меньшую сторону b (Ь^а) и заставьте оставшуюся вершину треугольника пробегать окружность радиуса а с центром в одном из концов отрезка Ь. 20.24. 60°, 60°, 60°. Указание. Обозначив углы исходного треугольника через а, 0, у, вычислите углы второго и третьего треугольников. 20.25. Если S = a*+i + a*+2 + ..-+a*+8, то a*+9=a*+7 + + fl*+e<S<a* + io==S-|-a*+2, т. е. S не входит в последова- тельность. 20.28. Заметим, что каждое из трех слагаемых равно отно- шению перпендикуляров OKi и G//„ опущенных из точек О и G 'на одну и ту же z-ю сторону треугольника ABC(i = = 1, 2, 3). Но G//i = G//2=G//3 = a, a OKx + OKz + OK3=GHx + -|-G//2 +G//3 = 3a, откуда искомая сумма равна -^-=3. Замечание. Утверждение задачи верно для любого треугольника. 20.29. Указание. Легко видеть, что все три неизвестных обращаются в нуль лишь одновременно. Если же произведение всех неизвестных отлично от нуля, то, рассмотрев систему урав- нений, полученную из исходной заменой всех частей уравнений на им обратные, и сложив новые уравнения, получим равенство откуда каждое слагаемое равно нулю и поэтому справедливо равенство Х|=х2 = хз=1. 20.30. См. указание к задаче 20.24. 20.31. Указание. Любое поступательное движение раскла- дывается в сумму двух: горизонтального и вертикального; до- кажите утверждение для каждого движения отдельно. 20.33. Можно всегда, если число контактов четно. В самом деле, предположим противное. Тогда при каждом повороте на свое место попадает ровно один контакт. Пусть f-й контакт имеет номер ai на розетке и номер bi на штепселе. Тогда разности а, —bi должны принимать все значения 0, 1, ..., 19 (по модулю 20). Следовательно, сумма (а\ — + — 6г) +... + (020 — 620) сравнима по модулю 20 с суммой 1+2 + ... +19= 190. Но 176
ai + ...+fl20 И 61 + ...4-620 са- ми равны 190, и мы получаем противоречие. Замечание. Если число кон- тактов нечетно, аналогичные рас- суждения не приводят к противоре- чию (см. задачу 20.22). 20.34. Одно из слагаемых равно 164, а каждое из ос- тальных равно 163. Указа- ние. Числа должны быть при- мерно равными. 20.36. Указание. Если на трех сторонах ABCD лежат 3 вершины прямоугольника с заданным направлением сто- рон, то четвертые вершины прямоугольника образуют отрезок. 20.37. При четном п имеется 4 решения, для которых xi равно 0,1, ^2— и — соответственно (очевидно, х2, .... хп немед- ленно восстанавливаются по xi). Если п нечетно, первые два решения отсутствуют. Указание. Рассмотрим на плоскости точки с координатами (xi, хг) и (хг, хз). Обе они лежат на пара- боле у = \—х* (см. рис. 20). Чтобы попасть из первой точки во вторую, нужно пройти по двум стрелкам рисунка. Система будет удовлетворяться, если, пройдя по 2л стрелкам, мы вер- немся в исходную точку. Разберите сами, исходя из рисунка, почему при других xi это невозможно. Замечание. Точка А на рисунке (видно, что стрелки сходятся к ней) называется притягивающей, а точка В — отталкивающей. 20.38. Указание. Опустите перпендикуляры ОЛ} и GH, на все грани тетраэдра (t=l, 2, 3, 4). Далее см. решение задачи 20.28. 20.39. Указание. Возьмем произвольную цифру в числе из первой последовательности и справа от нее поставим 0 — по- лучим число из второй последовательности. Это позволяет уста- новить взаимно однозначное соответствие между цифрами первой последовательности и нулями второй. 20.40. Можно. Указание. Надо поместить два самых боль- ших числа, т. е. ап и an-i, в разные группы и затем размещать очередное число (an-2, аа-з и т. д.) в ту группу, сумма чисел ко- торой в данный момент меньше. 21.01 . Указание. Начинающий может, например, добиться того, что решением системы будут числа (0; 1; —1). (Для этого нужно, чтобы при у и z все время были равные коэффициенты.) 177
21.02 . Пусть О — центр круга. Тогда точки Р и Q лежат на окружности диаметра ОМ, а длина хорды PQ, опирающейся на угол POQ, постоянна. 21.03 . 12 + 22 + ...+192 = 2470. Указание. Выясните сна- чала, сколько имеется четырехзначных номеров, у которых сумма первых цифр и сумма последних цифр равна фиксированному k = 0, 1,..., 18. 21.04 . Нужно, чтобы точки А и В были противоположными вершинами квадрата. 21.05 . Указание. Сумма чисел в каждой строчке, начиная со второй, делится на 1958. 21.08 . Указание. Найдите тот наименьший диаметр каж- дой сосны, при котором два человека, обходя поляну, не видят друг друга. Покажите затем, что и три человека, двигаясь соглас- но условию задачи, нс увидят друг друга. 21.10. Указание. Используйте тот факт, что четыре поло- сы, попарно непараллельные, имеют максимальную площадь пере- сечения в случае, когда их оси пересекаются в одной точке (см. условие задачи 24.17). 21.11. Указание. гп растет как л, а 5Л— как л2. 21.12. Две непараллельные прямые 1\ и /2 в пространстве имеют 3 оси симметрии. Три непараллельные прямые имеют поэтому не более 9 осей симметрии. 21.13. xi = 1, Х2= 1, Xk = k при £ = 3, 4, ..., л. Указание. Заметьте, что правая часть больше откуда xi = l. Из уравне- ния 1—777—=-—— получаем: х=-~±—. Воспользовавшись да- 24-а !4-х J 14-е лее тем, что если два числа равны, то равны и их целые части, получаем ответ. 21.14. Указание. Перейдите в троичную систему счисле- ния. 21.16. Несложно проверить, что П 155979)2< 11551958+341958< <(1 155979+2)2. Число (1 155979+If явно не годится, так как оно четное. 21.18. S = 1+2 + ...+100. Указание. Все места равно- правны. 21.19. Докажем более общее утверждение: если по п попарно непараллельным пешеходным дорожкам с постоянными ско- ростями движутся п пешеходов, причем известно, что 1-й пешеход встречается со всеми остальными и 2-й пешеход встречается со всеми остальными, то: а) каждый пешеход встречается с к а ж д ы м; б) в каждый момент все пешеходы располагаются на одной прямой. Для доказательства к нашей плоскости, в которой располагаются все дорожки, добавим вертикальную пря- мую — ось времени и рассмотрим графики двжения («мировые линии») всех пешеходов. Эти графики — прямые линии, причем первая и вторая «мировые линии» пересекаются со всеми осталь- 178
ными «мировыми линиями». Но первая и вторая «мировые ли- нии» определяют в пространстве плоскость. Произвольная t-я «мировая линия» пересекается как с первой, так и со второй «мировой линией»; следовательно, она лежит в той же плоско- сти. Итак, все «мировые линии» лежат в одной плоскости. И если теперь предположить, что какие-то два пешехода не встретятся, т. е. их «мировые линии» не пересекутся, то эти «мировые линии», находясь в одной плоскости, обязаны быть параллельными. Но тогда их проекции на плоскость, в которой находятся дорожки, также параллельны, что противоречит усло- вию. Следовательно, любые два пешехода встретятся и пункт а) доказан. Рассмотрим момент t=0. Этот момент определяет- ся как момент начального положения всех пешеходов, с одной стороны, и как пересечение исходной плоскости с плоскостью «мировых линий». Следовательно, в начальный момент все пе- шеходы расположены на прямой пересечения этих плоскостей, а так как движение каждого из них равномерное, то и в каждый момент все пешеходы находятся на одной прямой, параллель- ной начальной. Второе решение. Перейдем в подвижную систему коор- динат, связанную с первым пешеходом. В ней 1-й стоит непод- вижно, следовательно, остальные проходят через точку, в которой он стоит. Из того, что 2-й встречается с 3-м и 4-м, следует, что все они идут по одной прямой. Поэтому 3-й встретится с 4-м. 21.20. Отметим внутри каждого многоугольника по точке и занумеруем их. Соединим первую точку со всеми остальными точками непересекающимися кривыми, проходящими через общие стороны многоугольников. То же проделаем и со второй точкой. Тогда, если общее число точек больше четырех, то одна из них окажется изолированной и ее нельзя будет соединить непересе- кающейся кривой со всеми остальными. 21.21. Указание. Докажите сначала, что можно изменить знаки ровно у двух произвольных чисел, а затем — что ровно у одного. 21.22. Можно. 21.23. (n!y=(l.n)-(2.(n-l))X... ...Х(Н«-*+ !))•-•(«♦ О- Но (Л—1) л>(Л—1)Л при n>k, откуда k (п—k +1 )> п. Итак, (п\у состоит из п сомножите- лей, каждый из которых боль- ше п (кроме двух крайних, рав- ных л), т. е. превосходит л". 21.24. Можно. Длина этой ломаной mn-{-m + n. 21.27. Проведем дополни- тельное построение, как на ри- сунке 21. В треугольнике OQP 179
высоты PD и QE пересекаются в точке С, откуда OCJ.PQ. Для решения задачи поэтому нужно доказать, что MN\\PQ. Построим на PQ как на диаметре окружность; очевидно, точки D и Е ле- жат на ней. Поэтому опирающиеся на дугу ЕР углы EDP и EQP равны. Аналогично, построив окружность на диаметре DE, мы видим, что Z-MND= Z.MED. Так как EM^PD, углы EDP и MED также равны; итак, Z.MND=Z_EQP Наконец, так как ND\\EQ, то и Af№||PQ. 21.28. В правой части столько же сомножителей, сколько и в левой, но все сомножители справа равны л, а слева есть сомножители, меньшие и. 21.29. а^Ь. Примеры а>Ь и а = Ь придумайте сами. Указание. Докажите, что а кругов с теми же центрами, но вдвое большего радиуса, покрывают полностью многоуголь- ник М. 21.31. Указание. Каждая прямая, проведенная через точ- ку О вдоль произвольного луча, должна делить плоскость на две полуплоскости, в каждой из которых лучей поровну (или их число отличается на 1). Например, можно расположить лучи так, чтобы между любыми двумя соседними угол равнялся 21.33. а^Ь, поскольку в каждом из а кругов находится не более одного центра круга из второго набора. 21.34. 100 (10 цепочек из 10 последовательно соединенных сопротивлений, соединенные параллельно, или цепочка из 10 зве- ньев по 10 параллельных соединений в каждом). 21.35. х = 3; у=\. Указание. Если у = 1, то х=3 (второй корень квадратного уравнения х= — 1 отрицателен). Пусть у> 1. Числа х и х-}-2 одной четности, поэтому (х-|-1) четно: х+ 1 = 2k. Получаем: (2k — \)2y+(2k)2y=(2k + \)2u, откуда неслож- но усмотреть (раскрыв скобки по биному Ньютона), что у кратно k при у> 1 Разделив теперь обе части уравнения на (2£)2\ по- лучаем: Отсюда y<.k, а потому у не может делиться на k. Значит, при у>1 решений нет. 21.36. Если отрезок АВ целиком содержится в многоуголь- нике, то прямая АВ делит его границу на две части, длина каждой из которых по условию больше 1; поэтому периметр больше 2. Пусть теперь отрезок АВ полностью в многоугольнике не содержится. Определим на прямой АВ точки А' и В' ее пересечения с границей многоугольника, лежащие на про- должениях отрезка АВ и ближайшие к Л и В соответствен- но; точно так же определим точки А" и В" как ближайшие к А и В (соответственно) точки пересечения отрезка АВ с грани- цей. Отрезки А'А" и В’В" разбивают весь многоугольник на 180
три области: границей одной является отрезок А'А" и часть границы многоуголь- ника, границей другой — отрезок В'В" и часть границы многоугольника и третья ограничена отрезками А'А", В'В", а также ломаными аир (рис. 22). Ломаные АА"аВ'В и АА'$В"В не выхо- дят за пределы многоугольника и имеют длины, большие 1 (по условию). Добав- ляя отброшенные области, получаем тре- буемое. 21.38. -г-. и 21.39. Указание. Если на види- мой стороне последней карточки написа- но число k, то на ее обороте может стоять либо k — 1, либо Л-|-1. Если человек за- хочет ответить «Л— 1», то он до этого Рис. 22. должен был либо увидеть карточку с числом fe-|-l и быть уверенным, что на ее обороте стоит k+2, либо увидеть две карточки с числами Л-|-1. То же относится и к числу А-|-1 (с естественными изменениями). Для k — 2 или k 4-2 рассуждение аналогично. 22.01. Указанная процедура есть умножение «в столбик» в двоичной системе счисления. 22.02. Разность 22195’ — 1 =42'95’— 1 кратна 4 —1=3, что и требовалось доказать. 22.03. Можно. Указание. Отметим 9 точек на плоскости и занумеруем их числами от 1 до 9. Проведем все отрезки, попарно соединяющие эти 9 точек. Каждому трехзначному числу abc, не оканчивающемуся на 0, сопоставим отрезок, ведущий из точки с номером а в точку с номером с. Теперь достаточно доказать, что существует путь, проходящий один раз по каждому отрезку, который начинается и кончается в одной и той же точке. Пройдя этот путь 10 раз (для d=0. 1....... 9), мы получим нужную последовательность. 22.04. Наименьшее число поворотов равно 6*2 +1 +1 = 14 (при движении ладьи по спирали из левого нижнего угла в центр доски). 22.05. Ответ показан на рисунке 23. 22.06. Нельзя. В результате всех переливаний будет перелито а=6(2—литров, где k и I — целые числа. Если I—ft=£0, то а иррационально; если же l=k, то a=2k — четное число (a^fel). 22.08. Проведем диагональ BD. Так как DK — медиана в треугольнике ADB, то S^AKD=-^-S^ABDt аналогично S^BMC = 181
Рис. 24. =-|-5двсо (Рис- 24). Проведя вторую диагональ АС, получим: $дВКс+5длмов =="2- Завсо' Значит, SABCD=S ^akd 4- 4-5 дВМС + 5 ^AMD + S д BKC — SabCD — —Smpko+Sabcp + Saaod (площади тре- угольников BCP и AOD в сумме учтены дважды). Значит, —5МрКо+5АвСр+5дЛОП=0, что и требова- лось доказать. 22.11. Возведем сумму а\ + аг + ...+01959 в степень 1000. Раскрыв скобки, получим сумму слагаемых, каждое из которых есть произведение тысячи из наших чисел, взятых в произволь- ном порядке. Выбрав среди этих произведений только нужные, получаем, что их сумма меньше 1. 22.13. Указание. Докажите, что общие хорды окружно- сти О и каждой из окружностей, проходящих через точки А и В, пересекаются в одной точке, лежащей на прямой АВ. Для этого воспользуйтесь теоремой о касательной и секущей, прове- денных из точки к окружности. 22.14. Из узла О мы можем выйти по одному из четырех звеньев. При попадании в очередной узел листа у нас остается только три возможности выбрать следующее звено ломаной. Итак, второе, третье, ..., k-e звено ломаной мы можем выбрать тремя способами каждое. Значит, различных непересекающихся ломаных не больше p*^4*3*_|=2*2*3*_|<2*3*. 22.15. Указание. Самое длинное ребро тетраэдра не может прилегать к тупому углу. 22.16. Пусть Тогда x*+y*=z*<2f/*.^i/f + fe-y*“l< < (y+tf. Отсюда y<z<y +1, т. е. z не целое. 22.19. На N2 — N. Указание. Докажите, что сумма чисел любой строки, не содержащей 1, на N больше, чем сумма чисел в столбце с тем же номером. 22.20. Допустим противное. Тогда, выбрав k чисел И|, аг,...» а*, получим: a*^-|-ai. Выбрав k чисел a*, a*+i, —, агл-i, получим: агл-i <4-а*<Х а>. И т. д.: an(*_i)+i <4rai. Рассмотрим сумму S1 =ai 4-о*4-ог*-i-Ь — 4-on(*-i)+i4- — ^oi04-у-4-^-4- ••• 4- 4-^г4--) =2в|. Следовательно, и S2=a24-o*+i 4-...4-an(*-i)+2-b 4-.-.^2oi, ..., S*_i — o*_14-02*—2-|-•••-|-on(*—1)+(*+1)-|-...<2o(. Просуммировав эти неравенства, получим: S=Si4-524-.-4- 4-5*_1^2(Л — 1)О1=2(Л—1)^-<1.Нопоусловию5=51 + ...+ 4-S*_i=oi4-a24-...4-o„4-...= 1. Противоречие. 182
22.21. Сгруппируем члены в левой и правой сумме по два с разных концов. Тогда разность этих сумм будет состоять из сумм скобок (akbk+an-k+ibn-k+i—akbn-k+i—an-k+ibk)=(ak — —ая_*+1) (bk — bn-k+\), которые положительны, так как а*>ап-»+1 и по условию. Значит, левая сумма боль- ше правой. 22.22. Такой точкой будет ортоцентр (точка пересечения вы- сот) треугольника. ________________________ 22.23. На число A=A<00 ... А|00 ... AjOO ... 0А|, где Ai = lll ... 1 222 ... 2 ... 777 ... 7 88 ... 8 9 9 9 9 8 выписывается произвольное число раз, а число нулей в проме- жутках равно 9. Указан ие. 999 999 999»А=А ООО 000 000— —А=111... 11, откуда А=А ООО 000 000—111 ... 11. Вычитая «в столбик», находим последние девять цифр числа А; под- ставляя их в число А00... О, продолжим вычитание и найдем следующие девять цифр числа А; снова подставим их в А00... О и так будем продолжать сколь угодно долго. 22.24. Переставим цифры так, чтобы выполнялись неравенст- ва ai^a4>a2>as>a3>ae- Тогда 0<(ai + аг-Ьсз)—(в4+ов+вв)^(в1-Ьс2-Ь°з)—(<&+аз+ов)= =ai—ae<9. 22.25. Половина слагаемых в данной сумме равна (+1), по- ловина— (— 1); отсюда n=2k. Но х<х<+1 =— 1 тогда и только тогда, когда сомножители разного знака. Значит, k есть число перемен знака в последовательности Xi, хг, .... х„, Xi, а так как начало и конец ее — одно и то же число, то k=21. Итак, n = 2k=4l. 22.28. Указание. Пусть О — центр вписанной в треуголь- ник АВС окружности, А', В', С'— вершины искомого треуголь- ника, лежащие против вершин А, В, С. Докажите, что на отрезках АА', ВВ' и СС' лежат центры Oi, Ог, Оз- 22.29. См. задачу 22.32. 22.31. Возьмем для начала числа хь хг.xso- Если их сумма отличается от больше чем на то пусть она, например, меньше тогда X51 + ... +хюо больше 4>~. Начнем «поднимать» числа нашего набора, заменив сначала Xso на Х51, затем xsi на Х52. затем, например, х« на xso и т. д. Поскольку мы, действуя так, в конце концов дойдем до набора {xsi,—, хюо), в какой-то момент сумма впервые станет больше Легко видеть, что в 183
этот самый момент сумма чисел набора будет отличаться от 1 1 — меньше чем на . 22.32. Угол между какими-то двумя отрезками не меньше Возьмем тот из двух вертикальных углов величиной не меньше сумма длин а и Ь двух сторон которого (ку- сочков данных отрезков длины I) не меньше 1. Нетрудно проверить, что длина третьей стороны треугольника со сторонами а, Ь и углом а^-~ между ними не меньше третьей стороны треугольника со сторонами ---, у- и углом у- между ними. п 22.33. Предположим, что вершины Л и В обе обладают указанным свойством. Тогда Z.САВ4- Z-DAB> 180° и Z. СВ А + Z. DBA> 180°, тогда как сумма всех шести углов тре- угольников САВ и DAB составляет всего лишь 180° + 180°. Проти- воречие. 22.34. Куб любого натурального числа имеет вид либо 9А, либо 9Л ± 1. Отсюда легко вывести, что числа вида 9Л 4- 4 и 9k 4-5 не представимы в виде суммы трех кубов. 22.35. Занумеруем поля доски (кроме центрального) в порядке обхода их шахматным конем (рис. 25, а); белые кони стоят на полях 1 и 3, черные — на полях 5 и 7. На рисунке 25, б указан- ные поля расположены по окружности; белый кружок отвечает белому коню, черный — черному. Так как после одного хода коня цвет поля меняется, каждый конь сделает четное число ходов. Покажите сами, используя рисунок 25, б, что никакой конь не может сделать ровно 2 хода. Следовательно, каждый конь сделал не меньше четырех ходов, откуда общее число ходов не меньше 16. 22.37. Указание. Точка пересечения отрезков К\Къ и КзК* будет центром тяжести набора гирек подходящих масс (выясните сами каких), если поместить эти гирьки в вершинах четырехугольника. 22.38. См. решение задачи 42.18. 22.39. Пусть точка z лежит вне многоугольника М= ... сп). Тогда многоугольник М виден из точки z под углом, меньшим л, т. е. точки 2 — С|,..., z — сп лежат внутри некоторого угла а<л с вершиной в нуле. Теперь заметим, что точка -у- получается из точки z симметрией относительно оси х 184
и умножением на Отсюда следует, что точки ~ ,.... также лежат внут- ри угла 0, симметричного углу а. Но тогда сумма этих комплексных чисел не может равняться нулю. 22.40. Предположим, что при любом расположении дисков части первого рода составляют меньше половины окружно- сти. Тогда общее число случаев, когда цве- 0 0 7 7 7 7 7 7 0 0 7 7 7 7 7 7 % 0 0 7 7 7 7 0 7 1 0 0 0 7 7 0 0 7 7 % 0 7 0 7 0 0 0 7 7 1 7 0 0 0 0_ 7 0 0 % % Уг 0 0 0 0 0 □ Рис. 26. та секторов не совпадают, меньше п-2п. Но если совершить полный оборот меньшего диска, то произволь- ный сектор большего диска будет п раз покрыт секторами того же цвета и п раз секторами другого цвета. Значит, общее число случаев, когда секторы обоих дисков окрашены по-разно- му, равно п-2п. Противоречие и доказывает утверждение задачи. 23.01 . Такому условию удовлетворяют все суммы, не меньшие 10 р., и только они. Указание. Действительно, если 3^10, то 3 = 14-14-... +1 (S раз) и 3 = 104-14-... 4-1 (3 — 10 еди- ниц). Четную же сумму, меньшую 10 р., нельзя представить не- четным числом билетов достоинством 1, 3 и 5 р. 23.02 . Указание. Соединить общую точку пересечения ок- ружностей с точками Ль Az, Аз. 23.03. 26 человек. 23.04. Точки Ai, N, Bz лежат на одной прямой (Z-AiNBz= = Z.A1NM+ Z.BzNM=n, так как вписанные углы AiNM и BzNM опираются на диаметры AiM и BzM). Z.AiBiM=-£- (опирается на диаметр Л|М) и zLBzAzM=-£- (опирается на диаметр BzM), поэтому Л|В| и BzAz— высоты треугольника Л1МВ2; MN также высота этого треугольника. Но высоты треугольника пересекают- ся в одной точке. 23.05. Если di — делитель п, то dz=-т~ также делитель п, а, и поэтому меньший из них не больше -у/п. Итак, все дели- тели можно разбить на пары (кроме -yfn, если п — точный квадрат); следовательно, их не более -\/п4--\^=2-\/л. 23.06. Данное число кратно 3 и не кратно 9. 23.07. 9 человек (на рис. 12б приведен пример соответствую- щей турнирной таблицы). 23.08. Проведем через вершину В прямую, параллельную диагонали АС; пусть Р — точка ее пересечения с продолже- нием стороны DC и Р — середина отрезка DP. Легко видеть, что прямая АР искомая. 23.09. 0, 2 или 4. Указание. Провести прямые ОА и ОВ и рассмотреть различные случаи расположения отрезка CD по 185
отношению к двум вертикальным углам с вершиной О, один из которых содержит отрезок АВ, 23.10. Таких чисел бесконечно много уже среди множества точных квадратов. Действительно, если гп2=р-|-и2*, то р=(/и— nk)(m + nk). Поэтому, поскольку р простое, tn — = 1 и р = гп4-л* = 2п*-|- 1. Но для бесконечного множества пар (и, А) число 2nk 4-1 не простое. 23.11. Пусть---правильная дробь. Найдем наименьшее целое число q\9 для которого qi-^-^l. Обозначив = <7i-g-1, найдем наименьшее целое <72, для которого 1. Введем дробь — 1 и определим тем же спосо- бом целое число 93, затем дробь и т. д. Заметим, что — » знаменатели этих дробей не превосходят Ь. Поэтому описанный процесс прервется через конечное число шагов, т. е. найдется такое л, что ал*г^ = 1. Легко ви- деть, что тогда Ь q\ q\q2 qiq?... qn * Замечание. Такое представление не единственно. Например, 1*=1+_L=±+1+_L. 50 4^100 5^17’50*17’ 23.13. Пусть А и В — точки пересечения прямой /, проходя- щей через точку О, с границей многоугольника (Л и В лежат на сторонах) и пусть ЛО=/=ОВ, например АО>ОВ, При повороте прямой / вокруг О на очень маленький угол а точки Л и В переходят в Л' и В' (соответственно), причем также выполняется неравенство Л'О>ОВ'. Но тогда 25лОааЮЛ| |ОЛ'| sin а> > |ОВ| I OB' I sin а«25лОВВ|, хотя из условия вытекает, что эти площади равны. Противоречие. Итак, ОА = ОВ и О — центр симметрии. ______ 23.14. Объединение окружностей радиусов -\J2R2 — ОА2 и ОА9 концентрических с данной окружностью с центром О и радиусом R (Л — вершина прямоугольника на этой окружности). См. ука- зание к задаче 20.06. 23.15. Ответом будет векторная сумма двух треугольников, являющихся ортогональными проекциями исходных на середин- ную плоскость. 23.16. Предположим, что дробь—целое число, т. е. п не- четно. Тогда дробь равна ап~' — an~2b-]-ari3b2— ... +bn~l (п слагаемых). Докажем, что любые два соседних слагаемых an~kbk и —+ 1 при делении на п дают одинаковый 186
остаток. Действительно, их разность, равная a"_*_,fr*(а+&), де- лится на п. Следовательно, сумма остатков от деления на л всех слагаемых делится на л. 23.18. Если А—точный квадрат, то его последняя цифра есть х2=1,4 или 9. А тогда ровно одно из чисел у/А+х и у[А—х кратно 5, а значит, и 5* (А — число нулей в А-|-1), и поэтому А—№>9*10*— противоречие. 23.19. Будем считать, что lai I I021 |а*|. Пусть |ai | = = ... = |а/| и |а<I > |а/+11. Разделив обе части данного ра- венства на ai, получим справа 0, а слева сумму: а) I слагаемых, каждое из которых равно (4-1) или ( — 1), и б) k — l сла- гаемых , i=l-]-1,..., k. Но при стремлении л к бесконечности сумма последних k—l слагаемых может быть сделана сколь угодно малой, откуда следует, что первые I слагаемых разбиваются в пары (+1, — 1). Отбрасывая их в левой части исходной суммы и проводя это рассуждение дальше, получаем требуемое. 23.20. Если точки А, В, С, D образуют выпуклый четырехугольник, то для любой точки М справедливы неравенства МА +МС^АС, MB-]-MDBD, складывая которые, убеждаемся, что искомой точкой является точка пересечения диагоналей АС и BD. Если же, например, точка D лежит в треугольнике АВС (внутри или на стороне), то можно показать, что для любой точки М выполняется условие AM-j-BM-]-СМ-f-DM^AD-]-BD-]-CD и поэтому искомой является точка D. Наконец, если А, В, С, D лежат на одной прямой (пусть С и D между точками А и В), то искомой точкой будет любая точка М отрезка CD. 23.21. Пусть a>6>c>d — стороны четырехугольника. Тогда можно построить треугольник со сторонами а—d, Ь, с (выпишите неравенства треугольника), который легко достраивается до тра- пеции присоединением параллелограмма со сторонами cud. (См. также 48.03.) 23.23. 31-е число, и только оно. 23.24. п=4. Указание. Все точки лежат на окружности, диаметром которой служит отрезок наибольшей длины с концами в этих точках. 23.27. Пусть ai — первый наблюдатель; az — последний из всех наблюдателей, начавших следить за улиткой до того, как за ней кончил следить ai; аз — последний из наблюдателей, начавших следить за улиткой до того, как за ней кончил следить az, и т. д. Нечетные промежутки наблюдения ai, аз, as,... между собой не пересекаются, так же как четные промежутки аг, 04, а6,... не пересекаются между собой (иначе один из наблюдателей а, был бы выбран неправильно). Но любой из этих промежутков равен 1 мин, а весь интервал наблюдения со- ставляет 6 мин. Поэтому как четных, так и нечетных проме- жутков не более 5. Отсюда число наблюдателей не больше 10, и, 187
значит, улитка проползла не более 10 м (она проползет ровно 10 м в такой, например, ситуации: ползет улитка только тогда, когда на нее смотрит ровно один наблюдатель; остальное время она стоит). 23.29. l<Zm и произведение Itn четно. 23.32. Криволинейный треугольник, образованный дугами полу- окружностей, построенных на средних линиях треугольника АВС как на диаметрах (рис. 27). 23.33. Рассмотрим фигуру Ф, состоящую из множества точек» удаленных от отрезка длины 10 не более чем на 1 (рис. 28). Разобьем весь квадрат 100X100 на 50 вертикальных полос шириной 2 каждая и во все эти полосы поместим по 8 непере- секающихся фигур, равных Ф. Тогда из условия следует, что в каждую из 400 фигур попадает по крайней мере один центр круга и поэтому кругов не меньше 400, что и требовалось доказать. 23.34. Предположим противное. Тогда заметим прежде всего, что среди чисел а\,...,а* не более 7 единиц. В самом деле, прибавляя остальные числа по одному, мы можем сделать сум- му меньше Л, но больше А —8 (до тех пор, пока она меньше, можно прибавить еще одно число). Поэтому, если есть 8 единиц, мы сможем сделать сумму равной А. Далее, среди а», ...» ak не более 7 двоек. В самом деле, будем рассуждать аналогично, прибавляя за оХин раз по одному четному числу или по два нечет- ных (чтобы сумма оставалась четной). В какой-то момент мы получим число, меньшее А, но большее Л —18 и притом четное, как и А. Добавив к нему нужное число двоек, мы получим А. Аналогичные рассуждения показывают, что среди наших слагаемых не очень много троек, четверок, ..., девяток. Но тогда сумма заведомо меньше 5040 = 2520-2 (2520— первое число,- делящееся на 1, 2,..., 9), что противоречит условию. 23.35. Если данное число А равно 10а 4-&, то полученное числоВ равно 106п-1 *Ь + а. Но тогда 10В — А=(106л — \)Ь кратно 7 (так как 999 999 кратно 7). А так как А делится на 7, то и В тоже. 23.36. П ред пол ож и м, Ai, А?, ..., Ат. Все эти что некто X имеет tn знакомых его знакомые по условию друг с другом незнакомы. Значит, для каждых двух человек (А/, А/) должен найтись еще один общий знакомый, отличный от X и не- знакомый с X (при этом раз- ным парам (А*, А/) отвечают разные люди). Наоборот, каж- дому из незнакомых с X соответ- ствует пара (Ai, А/) их общих с X знакомых. Значит, число людей, незнакомых с X, равно 188
числу пар Ст. Поэтому n = 14-/n + Cm (1—это сам X, т — число его знакомых, Ст — число не- знакомых с X). Отсюда т определяется однозначно, и оно, следовательно, одинаково для всех X (заметим, что п может принимать не все значения, а только 1, 2, 4, 7, 11, ...). 23.38. Отметим среди номеров 1, 2, 3, .... п те, которым отвечают отрезки, идущие вправо или вверх (С2« способов). Независимо от них можно выбрать п номеров, которым соответствуют отрезки, идущие вправо или вниз (тоже СХ спо- собов). Ясно, что этим выбором маршрут улитки однозначно определяется. Итак, общее число раз- личных маршрутов улитки равно (Сгп)2- 24.02. Легко видеть, что все числа, начиная с Gi&iCi, делятся на 9 и (если они не равны 0) их средняя цифра равна 9. Поэтому достаточно рас- смотреть десять чисел: 99, 198, 297, ..., 990, которые легко проверить непосредственно. 24.04. Эта окружность имеет центр в середине отрезка наи- большей длины с концами в двух из данных точек. 24.06. Указание. Треугольник М1М2М3 подобен треуголь- нику АВС с коэффициентом подобия k=—. 24.07. Достаточно одного вопроса, если второй задаст числа ai = l, О2=100, Оз=1002,.... an=102n-2 (или любой другой набор быстро растущих чисел). 24.10. Если из каждого отрезка натурального ряда вычесть первый член, то после перестановки чисел получатся такие наборы: (0,981, 1, 982, 2, .... 978, 1959, 979, 1960, 980) и (980, 0, 981, 1,..., 978, 1959, 979, 1960). Указание. Выясните сначала, с какого числа с должен начинаться искомый отрезок, если исходные отрезки начинались с чисел а и Ь. Для этого сложите все числа искомого от- резка. 24.14. Пусть K=4k. Так как у каждого пассажира нет монет мельче 10 к., то каждый из них обязан получить сдачу. Следовательно, 4k из 5k монет должны остаться у пассажиров. Вместе с тем стоимость проезда всех 4k пассажиров составляет 20Л копеек, на что нужно не менее k монет. Следова- тельно, в кассу было опущено не меньше k монет; общее число монет поэтому не меньше 5k. Требуемый пример правильной оплаты при наличии 5k монет такой. Разбиваем всех пассажиров на k групп, по 4 человека в каждой; 5 монет в каждой группе распределены так: у 1-го — 15 к., у 2-го — (104-10) к., у 189
3-го—15 к., у 4-го —20 к. Сообразите сами, как они должны расплатиться друг с другом. 24.15. Указайие. Рассмотрите выпуклую оболочку всех то- чек и докажите, что это п-угольник. 24.16. Применим индукцию. Пусть иы делится на 5в a U5*+i дает при делении на 5 произвольный остаток г^О. Тогда числа п5* + 2, п5*+з, п5* + 4, п5*+5 дают при делении на 5 остатки соответственно г, 2г, Зг, 5г = 0. 24.17. Все оси этих полос должны пересекаться в одной точке. 24.18. См. решение задачи 24.14. 24.19. Проведем из точки О касательные О А и ОВ к окруж- ности S. Пусть S — центр окружности, D — точка пересечения отрезков OS и АВ. Искомое множество центров — окружность, построенная на отрезке OD как на диаметре в плоскости, перпендикулярной данной, с выкинутой точкой О. Для дока- зательства достаточно заметить, что любая окружность касания сферы н конуса проходит через точки А и В, т. е. плоскость этой окружности вращается вокруг прямой АВ, a OD — перпен- дикуляр к этой плоскости. 24.21. Указание. Применить индукцию по числу проведен- ных диагоналей. 24.22. Указание. Рассмотреть отдельно случаи AP = AS и AP=^AS. 24.23. Пусть tn — очередное число, сумма цифр которого делится на 11; покажем, что среди следующих 39 чисел есть число с тем же свойством, а) Если последняя цифра пг не О, а предпоследняя цифра не 9, то сумма цифр числа пг-^-9 равна сумме цифр числа гп, т. е. т-|-9 — искомое число, б) Если пг кончается нулем, а предыдущая цифра не больше 7, то подходит число т-|-29 (его сумма цифр больше на 11). в) Если предпоследняя цифра числа пг равна 9, то, прибавив к пг не больше 10, мы получим число, оканчивающееся по меньшей мере двумя нулями. Но, прибавляя к такому числу 1, 2, 3,..., 18, 19, мы получим всевозможные остатки от деления суммы на 11, в частности одно из этих чисел дает остаток 0. Таким образом, подходящее число превышает пг не более чем на 104-19 = 29. г) Нерассмотренным остался только случай, когда пг= ... 80. Но здесь те же рассуждения, что и в случае в), показывают, что следующее подходящее число превосходит пг не более чем на 204-19 = 39. Пример т = 999 980 показывает, что эту оценку нельзя улучшить. 24.24. Если звездочек только 6, то всегда можно найти два столбца, в которых не меньше 4 звез- дочек, вычеркнуть их, а вычеркиванием строк унич- тожить две оставшиеся звездочки. Пример для 7 звездочек показан на рисунке 29. Рис. 29. 24.25. Так как abcd — a=a {bed— 1)= 1961, то * * * * * * * 190
а нечетно. Из остальных равенств следует, что b, с, d тоже нечетные. Но тогда число abed нечетное и поэтому разность (abed—а) четна. Противоречие. 24.26. Предположим, что такую ломаную можно провести; се можно считать незамкнутой. Ломаная входит в каждую область и выходит из нее; при этом для данной области каждому входу отвечает выход, кроме того случая, когда ломаная войдет последний раз в область и больше из нее не выйдет. Так как верхние две области (рис. 3) состоят из пяти (н е- четного числа) отрезков, то один конец ломаной должен лежать в одной из этих областей, а второй — в другой. Но нижняя средняя область (рис. 3) тоже состоит из пяти отрезков, поэтому какой-то конец ломаной должен содержаться и в ней. Однако третьего конца у ломаной нет. 24.27. Опустим из О, точки пересечения диагоналей прямо- угольника, перпендикуляр ОК на общую внешнюю касательную к окружностям 1 и 3. Проведя радиусы л и к этой каса- тельной, получим прямоугольную трапецию, в которой ОК — средняя линия. Значит, ОК—Г'^Г*. Такую же длину имеют и остальные перпендикуляры, опущенные из О на общие касатель- ные к окружностям 1, 3 и 2, 4 (так как rt +г3 = Г1+г4). Зна- чит, О — центр вписанной в образовавшийся четырехугольник окружности. 24.30. Пусть какая-то четверка совпадает с исходной. Решение распадается на следующие шаги. 1) Доказать, что abcd=l. В противном случае произведение получающихся чисел никогда не совпадает с произведением исходных. 2) Доказать, что ас=1 и, следовательно, bd=\. Это следует из того, что для всех четверок, начиная со второй, такое равенство будет выпол- няться. 3) Исследовать, что происходит с четверкой чисел а, Ь, у- в результате наших преобразований. 24.31. Центр этой окружности — вершина равностороннего треугольника с основанием О|Ог- 24.32. Будем менять знаки каждой строки и каждого столбца, у которых суммы чисел отрицательны. При этом, возможно, «испортятся» другие строки или столбцы, но сумма всех чисел таблицй S при каждой такой замене возрастает. При этом величина, на которую возрастает S, не стремится к нулю (легко сообразить, что эта величина может принимать лишь конечное число значений) и поэтому в какой-то момент она достигнет максимума. Но если ее нельзя увеличить, то сумма чисел любой строки и любого столбца неотрицательна. 24.33. Указание. Докажите сначала, что существует точка, из которой выходит ровно один отрезок. Для этого пройдитесь по графу из любой точки, пока идти станет невозможным Далее — индукция по числу точек. 191
Рис. 30. 24.34. Обозначим наибольший общий- делитель чисел b и р — а через d. Отсю- да b = kd и р — a=ld, где k и I взаимно просты. Но тогда ak-\-bl=a--^--\-b-^^-= ^p = kp. 24.35. Самый выгодный для Коли — первый способ, самый невыгодный — третий. 24.36. Указание. Докажите снача- ла, что из любой бесконечной последова- тельности натуральных чисел можно выбрать неубывающую подпоследовательность. Дальнейшее просто: из последователь- ности дь д?, ... выбираем неубывающую подпоследовательность <21^02^ ...; из отвечающей ей (по индексам) подпоследователь- ности b\. Ь2, ... выбираем неубывающую Ь"^.Ь2^. ...; из отве- чающей ей подпоследовательности с", с", ... выбираем неубы- вающую . Тогда а"' ^а2" <...; bi" ^.b2" ... . 24.37. Для того чтобы круг лежал в прямоугольнике, не пересекаясь с квадратами, нужно, чтобы его центр лежал в прямоугольнике размером 19X24 и отстоял от каждого квадрата более чем на у, т. е. не попадал в фигуру, изображенную на рисунке 30. Но сумма площадей ста двадцати таких фигур равна 120-^3+-^ < 19-24, так что место для центра круга диаметра 1 найдется. (Ср. с решением задачи 23.33.) 24.39. PC = 5. Проведем луч ВМ так, чтобы Z_CBM= Z.ABP и Z_PBM=60°. Отложим на нем отрезок ВР' = ВР и соединим Р' с точками С и Р. Тогда треугольник РВР' равносторонний, откуда РР'==РВ=3; ДВСР'=Д4ВР, откуда Р'С = Р4 = 2. Следовательно, PC^iPP' -|-Р'С = 5. Равенство достигается, если РР'С — прямая. 24.40. Легко доказать по индукции, что через 2Р шагов получится набор ai^+i, а2а2р+2 «... ,a2ka2P+2k. В частности, через 2* шагов набор будет такой: af, al. al,..., ofy, т. е. будет состоять из одних единиц. 25.01. Можно ограничиться случаем, когда точки М и N расположены в одной полуплоскости от прямой АВ. Тогда М — центр вписанной в треугольник ABN окружности и \AB — BN\ равно модулю разности двух отрезков, на которые делится отрезок АВ. 25.02. Указание. Рассмотрите бесконечную треугольную сетку, один из треугольников которой — наш. Сторону нашего треугольника ориентируйте стрелкой. Эта стрелка задаст ориента- цию самого треугольника, а также всех треугольников сетки. 192
25.04. Если у двух чисел суммы цифр Xr***4. \ равны, то разность этих чисел кратна 9. Поэтому 2а—а —а делится на 9. /у Ч 25.05. Указание. Докажите, что най- I / дутся такие числа alt .... ait k^n, и та- V кие k карточек, что на первой из них на- К писана пара (ai, аг), на второй — пара 'Л > (а2, Аз), ..., на (k— 1)-й— пара (а*-ь / на fc-й — пара (aA, ai). Отбросив их, про- / ведите это рассуждение несколько раз для остальных карточек. Рис. 31. 25.06. Указание. Отрезки этих пря- мых разбивают треугольник АВС на четыре равновеликих тре- угольника. 25.08. Преобразовав наше соотношение так, чтобы выразить п, мы видим, что п=т2— . Поэтому, положив m=d-\-2, мы найдем целое п, удовлетворяющее условию при </=#= — 1. Если d= — 1, то следует взять т=1, а п произвольное. 25.12. Задача разрешима только при четных п. 25.14. 1; . Указание. Домножьте второе I-д/5 I+V5 уравнение на Xi, третье на Х|Хг и т. д. и воспользуйтесь пер- вым уравнением. 25.15. Для этого достаточно, чтобы радиусы были очень ма- ленькими, например в 4000 раз меньше длины меньшей из сто- рон прямоугольника. 25.16. MN минимально, если точки М и N равно отстоят от точки пересечения данных лучей. 25.17. Указание. Предположив, что разбиение на паралле- лограммы произведено, рассмотрите два соседних параллелограм- ма, имеющих общую вершину квадрата, и докажите, что они обя- заны пересекаться. При доказательстве этого воспользуйтесь тем, что никакой треугольник нельзя разбить на параллелограммы. 25.18. Указание. Если п — любое натуральное число и Огп^^П <^am4-|, ТО П—am<Zam-i. 25.21. Нужно пустить шар так, чтобы траектория его движе- ния отсркла от всех углов бильярда равнобедренные треуголь- ники, как показано на рисунке 31. Равенство длин всех траекторий шара вытекает из рассмотрения «выпрямленных» траекторий. 25.22. Указание. Пусть О — центр окружности МВС', О| — центр окружности АВС\ N — середина ВМ\ L — точка пересече- ния ON и ВН. Докажите, что треугольник OOiL равнобедренный. 25.23. а) Площадь уголка равна 3, но 1961 и 1963 на 3 не делятся, б) Прямоугольник 1963 X1965 разобьем на три прямо- угольника: 1965X1958, 1956 X 5 и 5X9. У первого одна сторона делится на 3, а вторая — на 2; следовательно, его можно раз- бить на прямоугольники размером 2X3, каждый из которых разби- 7 Заказ 247
вается на два уголка. Второй пря- моугольник разобьем на прямо- угольники 5X6 и каждый из них — на 10 уголков (рис?182, а) Третий разобьем на 15 уголков (рис. 32, б) Рис. 32 25.24. Поскольку 1962 делит- ся на 3, данное число есть сумма кубов: /V = (10654)3+ I3- Оно делится на 10654 +1. 25.25. Возьмем две наиболее далекие друг от друга точки А и В и построим два круга радиуса 1 с центрами в них. Для произвольной точки С возьмем ту из точек А и В, которая ближе к ней; тогда расстояние до нее меньше 1 (в про- тивном случае получили бы противоречие с условием). Итак, все 25 точек лежат в построенных двух кругах, а тогда в одном из них лежит не менее 13 точек. 25.26. Указание. Следует доказать более сильное утвержде- ние; найдутся две несоседние вершины, из которых не проведено ни одной диагонали. Это легко доказывается индукцией по числу проведенных диагоналей. 25.27. См. решение задачи 25.32. 25.28. Пусть п — простое число. Рассмотрим вершину А и пусть Д' — ближайшая вершина, в которую может перейти вершина А при повороте на угол q = ka вокруг центра О (k — любое натуральное число). Покажем, что внутри угла АОА' нет других вершин многоугольника. Предположим противное, т. е. допустим, что внутри угла АОА' есть вершины Bi, 82, ..., Вр. Тогда внутри угла А'ОА", где А" — вершина, в которую переходит Д' при повороте вокруг О на угол <р, находятся вершины В(, В'>, ... , Вр, в которые переходят при этом повороте соответственно вершины Bi, В2, .., Вр. Число вершин многоугольника равняется п = = (₽+•)•—» т. е. не равно простому числу п при р^=0. Значит, 0. Итак, очевидно, что при последовательных поворотах на угол ср вокруг О вершина А может перейти в любую другую вершину многоугольника (при некотором k), откуда следует, что п угольник правильный. Но по условию он неправиль- н ы й. Отсюда вытекает, что наше предположение неверно и п -составное число. 25.29. Заметим, во-первых, что а\ + •• + «ю = 4 (xi + ... +хб)« Таким образом, мы знаем сумму S=xi+x2 + ... +*5 Пусть а\ наибольшее из чисел ... , аю, а ою — н а и м е н ь- ш е е. Тогда, очевидно, а\ — сумма двух наибольших из чисел %1, %2, .... хб, а Дю — сумма двух наименьших. Поэтому S—Ci — — аю = хз, где хз— среднее по величине из чисел %i, ..., х$. Теперь уже нетрудно найти все остальные неизвестные: ’Напри- мер, если а? — второе по величине число, то Х\=а2 — Хз- 25.30. Указание. Опустите перпендикуляры из центров 194
Oi и Oj на хорды AM и ВМ до их пере- сечения в точке R. Докажите затем, что RP.LMP. Тогда MR — RH и, поскольку AR=tylR, и MR = RB (по построению точки.-г-/?), получаем, что R — центр искомой описанной около МАМ В окруж- ности. 25.31. Пусть аь ai, ал — число за- дач, решенных школьником за некото- рые идущие подряд 8 дней. По усло- вию oi + a2+...-|-a?=25, а2 + а3+...+ 4-ав = 25, откуда ai=a%. Итак, число задач, решаемых школьником за день. должно повторяться через каждые 7 дней и школьнику надо разработать план (at, а2, .... ai) только на одну неделю. Если бы школьник решал все 25 задач в понедельник, то он затратил бы на все задачи в течение года время S|. Точно так же опреде- лены числа S2, S3, ..., Si. Решая по понедельникам не 25, а at за- дач, школьник за все понедельники потратит время ^tSi; анало- гично за все вторники — время S2 и т. д. Общее затраченное время равно S=^-(aiSi4-0757). Выбирая из чисел Si, S2, .... S7 наименьшее S*, полагаем а* = 25. а все остальные а<=0; тог- да S будет минимальным. 25.32. Сумма в условии равна ±(ai — a2)±(a2 —a3)±...± ±(a25 — 01). Если раскрыть скобки, то в полученной алгебраиче- ской сумме 25 слагаемых будут стоять со знаком плюс и 25 — со знаком минус. Эта сумма, следовательно, не больше 2&2s + 2624 4- -|-...4-2&i4-|-6i3 —613—26|2 —... —2&|, где Ь\, .... 62g — наши чис- ла, расположенные по возрастанию. Для того чтобы получить наибольшую возможную сумму, достаточно расположить числа в следующем порядке: 625, Ь\, Ьц, Ь2, ..., blt, bl2, bt3. 25.33. Из рисунка 33 легко видеть, что площадь многоуголь- ника увеличится больше чем на 12-{-я>15. 25.36. Указание. Точка В по точке С строится так: про- водится СС' ±/ (С' лежит на окружности) и берется точка пере- сечения касательной в точке С' с прямой I. 25.39. Параллелепипед должен располагаться так, чтобы плоскость, проходящая через вторые концы А, В, С его ребер, Дыходящих из одной вершины, была горизонтальна. Указа- ние. Площадь ДАВС всегда равна половине площади всей проекции параллелепипеда, а площадь проекции любого тре- угольника на горизонтальную плоскость равна площади самого треугольника, умноженной на косинус угла между плоскостями. 25.40. Доказательство проведем по индукции. Для k=2 утверждение очевидно. Если для k=n участники си, .... ап распо- 195
ложены в порядке возрастания, то (л4-1)-го ста- вим в этот ряд перед первым из тех, кому он не проиграл. 26.02. См. решение задачи 26.06. 26.05. Нельзя. Прямая, пересекающая 50 клеток, должна пересечь не менее 49 разде- ляющих их линий, а на листе таких линий все- го 48 (19 вертикальных и 29 горизонтальных). 26.06 . 0=(ai 4-а2 4-... + ап)2=а2 4- а2 4-... 4-а2 4- 2S, откуда 2S = — (а2 4~ ^2 4~ ••• 4" а%) 0. 26.07 . Отрезки, соединяющие середины сторон четырехуголь- ника ABCD, образуют параллелограмм П, и легко видеть, что SH=~S. С другой стороны, они являются средними линиями в четырех треугольниках с общей вершиной М, на которые де- лится четырехугольник PQRS, откуда SPQ/?s=4Sn. Окончатель- но Spq/?s = 2S. 26.08 . Можно вначале найти решения только в натуральных числах, так как если (х0, уо. ?о) — решение, то, изменив знак у любых двух чисел этой тройки, снова получим решение. Данное уравнение умножим на 2xyz и воспользуемся неравенством а2 4; b2 2ab; 6xyz — 2х2у2 4- 2x2z2 4- 2y2z2 =(х2у2 4" *Qz2) + (*?у2 + 4- y2z2) 4- (x2z2 4-у2z2) 2x2yz 4- 2y2xz 4- 2z2xy — 2xyz (x 4- у 4" z), от- куда x 4~У 4~ 2^3. Но x, у, z — натуральные, поэтому x=y = z = = 1 —единственное решение в натуральных числах. Ос- тальные решения исходного уравнения таковы: (—1, —1, 1), (1, -1, -1), (— 1, 1, -1). 26.09 . Перенесем все прямые параллельно самим себе так, чтобы все они пересекались в одной точке. Углы между ними от этого не изменятся. Но после этого они делят полный угол в 360° на 14 частей, и если бы каждая часть составляла не меньше 26°, то в сумме они дали бы по крайней мере 14*26° =364° >360°. Получили противоречие. 26.10. При п=4£. Указание. На рисунке 34 показано, как заполнить карточками прямоугольник размером 4X5, а из таких прямоугольников можно составить прямоугольник 4^X5. Если п нечетно, то карточками нельзя заполнить прямоугольник, а если пделится на 2, но не на 4, то прямоугольник придется заполнить нечетным числом карто- чек и потому произведение всех чисел в таблице будет отри- цательно, откуда ясно, что не может быть положительным произведение в каждой строке. 26.11. Указание. Если разность прогрессии меньше 10*, то в прогрессии найдется число, состоящее не менее чем из /?4~1 цифр, начинающееся с любой цифры, в том числе и с 9. 26.13. Пусть x = a + b, у — Ь-^с, z — a + c\ тогда левая часть данного неравенства примет вид: S=-y(x-*~y 4~ + 196
+^±г2Д)=т(-+-+-+-+-+--3)’ а так х / Z\y'x'z'x z'y / —+-^>2, —+—>2. -^-+—^2, то S>4-(6-3)=4-- i/ x z’x^ z ' у 2 4 ' 2 26.14. См. решение задачи 26.17. 26.15. Нельзя, так как 26.17. Предположим противное. Рассмотрим наименьший вы- пуклый многоугольник, внутри которого расположены все точ- ки, и возьмем произвольную вершину А этого выпуклого многоугольника. Соединим ее со всеми оставшимися точками. Получим 4 угла, больших 30°, откуда угол А больше 120°. Но углы при всех вершинах выпуклого многоугольника не могут превосходить 120° — противоречие. 26.18. m-\-n — 1. См. решение задачи 5. 26.19. Если все числа а, 6, с положительны, то неравенство очевидно. Если одно из них (например, а) положительно, а два отрицательны (6<0, с<0), то а> |6| + Id и потому ап> |6|Л + |с|л. Это равносильно нужному неравенству. 26.20. Окружность, описанная около треугольника АВС. Дей- ствительно, если М' — точка, в которой пересекаются перпенди- куляры к прямым AM, ВМ и СМ, то ММ' — диаметр описанной окружности. 26.21. Тангенс суммы S трех арктангенсов равен дроби, у кото- рой числитель %+y+z— xyz, а знаменатель 1 — xy — xz — yz. При сама дробь и ее знаменатель положительны, а при ----отрицательны. Поэтому числитель всегда положителен. 26.22. Если бы такая ломаная существовала, то, сориентировав ее звенья против часовой стрелки, получили бы, что сумма 25-и векторов равна 0. Рассмотрим эти векторы с общим началом в точке А и ортогонально спроектируем их на прямую, перпенди- кулярную I: сумма векторов-проекций не равна 0 из-за равенства их длин и нечетности числа 25. Следовательно, наше допуще- ние неверно. (Ср. с решением 39.22.) 26.24. Указание. Каждая из цифр 1, 2, ..., 7 встречается на каждом из мест 720 раз, а 720 делится на 9. 26.25. На 15 частей: 11 тетраэдров (1 при вершине, 3 в сред- нем слое, 7 в нижнем) и 4 октаэдра (1 в среднем слое и 3 в ’нижнем). 26.26. 8 штук. Указание. Красные шарики разбивают си- ние на три группы, и при выполнении условий задачи легко видеть, что ответ равен числу разбиений числа 7 на три слагаемых (быть может, нулевых). Вот эти разбиения: 7+0 + 0, 6+1+0, 5 + 2 + 0, 5+1 + 1, 4 + 3 + 0, 4+2+1, 3 + 3+1, 3+2 + 2 26.28. Указание. Все прямые проходят через точку пересе- чения медиан дЛВС. 197
Рнс. 36. I 26.29. 75 чисел. Указание. Нужно -------—1 взять все нечетные числа, делящиеся -------------- Ог на 13. ________ о, 26.31. Составим диаграмму (она на- I зывается диаграммой Юнга), как показа- а но на рисунке 35, где в t-й строчке распо- 5 ложено О/ квадратов. Тогда Ь, — число Рис 35 квадратов в /-м столбце. Отсюда сразу вытекает требуемое. 26.32. Предположим противное. Тогда, , если путь из одной клетки в другую тре- бует k шагов, то разность чисел, стоящих 2 в этих клетках, не превосходит 4k. Но раз- 3 ность чисел 1 и 64 равна 63, а число 4 шагов между клетками, в которых стоят 5 эти числа, не больше 14. Так как 4-14<63, 6 получаем противоречие. (Решение зада- 7 чи 26.39 аналогично.) 26.33. Круг с центром О и радиусом R (без границы), где О — центр дан- и ной окружности радиуса R. Указание. Рассмотреть предельный случай прямо- угольных треугольников. 26.34. 655 чисел. Указание. Рассмотрите числа вида 3fe+ 1, /?=0, 1, 2, ...» 654 (разность любых двух из них кратна 3, а сумма — нет). Большего количества чисел выбрать нельзя, так как иначе нашлись бы два числа, отличающиеся на 1 или на 2, и их сумма делилась бы на их разность. 26.35. Движение джентльменов периодично с периодом 12 мин: через каждые 12 мин любой из них находится в первоначаль- ной точке на аллее и движется по ней в т у ж е сторону. Поэтому ось времени t естественно представлять намотанной на окруж- ность длины 12 мин, а эту окружность считать трехслойной: первый слой образуют две дуги, по 6 мин каждая, второй — 4 дуги, по 3 мни каждая, третий — 6 дуг, по 2 мин каждая. Дуги каждого слоя покрасим в два цвета, черный и белый (цвет определяется направлением движения данного джентльме- на по аллее), так, чтобы эти цвета в одном слое чередовались. Рассмотрим пересечение одноцветных дуг первого и второго слоев, получим по крайней мере две общие черные дуги а и 0, по 2 мин каждая, и две белые дуги у и б, также по 2 мин. 6 дуг третьего слоя делятся на 3 черные и 3 центрально-симметричные им белые дуги. Поэтому, если пересечение дуг а и 0 с тремя черными дугами третьего слоя составляет дугу меньше 1 мин, то пересечение дуг у и 6 с белыми дугами составляет дугу, большую 1 мин, и утвержде- ние задачи доказано. 26.37. 17 точек. Пример такой ломаной на рисунке 36. 198
Указание. Замкнутая 14-звенная ломаная имеет 7 вертикаль- ных и 7 горизонтальных звеньев, так как из каждой ее вершины вы- ходит одно горизонтальное и одно вертикальное звено. Зануме руйте горизонтальные звенья от 1 до 7 и выясните, сколько то- чек самопересечения может лежать на каждом из них. 26.38. Указание. Описать вокруг 10-угольника окруж- ность; для каждого образовавшегося треугольника измерять его углы в единицах . После этого задача сведется к комбинаторной задаче по подсчету числа дуг на окружности, суммы которых п о три одинаковы, а сами дуги различны. 26.41. В самом деле, если п нечетно, то z"=x"-f-«z" делится на х+у. Но тогда z делится на x-j-y и, следовательно, z^x-)-«/, т. е. 26.42. (и!)* ломаных. Указание. Если it, it, .... in — произ- вольная нумерация горизонтальных прямых (таких нумераций и!), а /I. /2, /л — произвольная нумерация вертикальных прямых (их также п!), то.ломаная со звеньями (й, ji), (о, /г). (in. jn) по этой нумерации строится однозначно. 26.43. Нужно взять любые 12 векторов подряд. Указание. Если OS—сумма максимальной длины (О — центр), а I — прямая, проходящая через О перпендикулярно OS, то в сумму —► OS входят все векторы, лежащие в той же полуплоскости от /, что и OS. 26.44. Пусть А', В1 — середины сторон АВ и ВС соответ- ственно. Очевидно, и аналогичная формула имеет место для других треугольников. Поэтому сумма площадей треугольников ABC, BCD, CDE, DEA, ЕАВ в 4 раза больше суммы площадей малых треугольников, дополняющих A'B'C'D'E’ до ABCDE. С другой стороны, эти пять треугольников покры- вают пятиугольник ABCDE меньше 2 раз, так что сумма их площадей превосходит площадь ABCDE меньше чем в 2 раза. Отсюда легко следует нужное неравенство. 26.45. Указание. Докажите по индукции, что ап=4ап- । On—2- 26.46. Указание. Если, например, х — наибольшее из этих четырех чисел, то х* >хг-(-х/>z* 26.47. Указание. Расположите все десятичные дроби друг под другом и рассмотрите бесконечное число столбцов (разря- дов) высоты 11: в каждом столбце не меньше двух одинаковых цифр, откуда и вытекает утверждение. 26.48. Р(х)=ах(х— 1)(х—2)...(х—25). Указание. Дока- жите последовательно, что Р (х) делится на х, на х— 1, на х—2. на х—25. Отсюда Р(х)=х(х—1)(х—2)...(х—25)«Q(x). Затем до- кажите, что Q(x— l)=Q(x), откуда Q(x)=const. 199
Рис. 37. Рис. 38. 26.50. Предположим противное. Занумеруем эти дуги римски- ми цифрами I, II и 111 и дополним их до окружностей с теми же номерами. Обозначим диаметрально противоположные точки пере- сечения окружностей I и II через А и Лг, II и III — через В и Вг, I и III — через С и С|. Эти точки пересечения принадлежат хотя бы одной дуге; пусть А\ и С\ лежат на дуге I, В\ — на дуге II, тогда В и С принадлежат дуге III, А —дуге II. Обозна- чим через а, р, у плоские углы трехгранных углов, как показано на рисунке 37 (О — центр сферы). Поскольку дуга 1 не содержит точек А и С, должно выполняться неравенство 360° — р>300°. Аналогично дуга II не содержит точек В и откуда 180° +а>300°, и, наконец, для дуги III имеем: 360° — у>300°. Отсюда получаем: Р<60°, а >120°, у <60°, значит, а>р + у — противоречие с теоремой о плоских углах трехгранного угла. Замечание. Оценка 300° для трех дуг на сфере точная 27.01. 90°, 45° и 45°. 27.02. Пусть О — центр окружности, К — точка пересечения касательной в точке С с касательной ВМ. Тогда КС^КВ (как отрезки касательных, проведенных из одной точки). И если Z.G4B = a, то 21АС0 = а, а так как ZLOCA = 90°, то = =90°—а. Из треугольника AM В Z.KMC = 90° — а, откуда КС=КМ. Поэтому КВ = КМ=±-ВМ. 27.03. Число с суммой цифр 5 имеет вид 9k + 5 и, следо- вательно, при делении на 3 дает остаток 2. Но любой квадрат при делении на 3 дает остаток 0 или 1. 27.04. 41 звено. Звенья следует стирать через одно. 27.05. Указание. Докажите (по индукции), что все члены последовательности, кроме первых двух, дают при делении на 4 остаток 2 или 3. 27.07. Числа вида р и 2р, где р — простое, и числа 8 и 9. 27.08. Если х=/=0, то число вида %+V* никогда не является полным квадратом, так как x+V* лежит между (-у/х)2 и (V*+ О2- Поэтому у=1. Отсюда х = п2, г = п, где п — произвольное число. Если же х=0, то z=0, а у — произвольное число. 27.10. Единиц на одну больше (за счет числа 106). Указа- ние. Единицы получаются из чисел вида 9^+1, а двойки — из чисел вида 9k + 2. 200
27.11. x—y=z= 1. Указание. Докажите сначала, что либо все три числа больше 1, либо все три меньше 1. Затем дока- жите, что все они равны между собой и xyz=i. 27.12. Если т(т + 1)=а*. то, поскольку m и m-f-1 взаимно просты, каждое из этих чисел также является Л-й степенью, что невозможно. 27.13. Обозначим 27 — k = m, тогда а-^=а^2Г + 3.272 _ 3 . 27т + т2 Следовательно, -—— целое при всех целых т, что возможно только при а=273. 27.15. Если в треугольнике провести две высоты, то их основания и оставшаяся вершина треугольника являются верши- нами подобного треугольника. Следовательно, каждый из четырех треугольников, на которые разбивается диагоналями четырех- угольник MNPQ, подобен соответствующему треугольнику из четырех, на которые разбивается диагоналями четырехугольник ABCD; значит, и сами четырехугольники подобны, что и требова- лось доказать. 27.16. Лг= 1681. Указание. Зачеркнем две последние циф- ры числа N и заменим их нулями. Очевидно, получится тоже точный квадрат некоторого числа п. Тогда так что (« +1)2—n2s^99. Отсюда n^49. С другой стороны, п, очевидно, кончается нулем. Таким образом, максимальное подходящее п=40, т. е. W=412. 27.18. а=27,9С4. См. решение задачи 27.13. 27.20. На 5 тетраэдров. Указание. К каждой из двух проти- воположных граней куба примыкают не менее 2 тетраэдров, и все эти 4 тетраэдра различны. Легко видеть, что ббъем каждого из них не больше 4- а3. Следовательно, эти 4 тетраэдра не О заполнят всего объема куба и нужен пятый тетраэдр. Пример разбиения на 5 тетраэдров изо- бражен на рисунке 38. 27.21. Обозначим радиусы проведенных окружностей через Гь р», г. Рассмотрим треугольни- ки OiRO и OzSO (рис. 39). У них О|/?=ОО2=Г1, OiO—O2S—r2 и Z.OOiR= Z-OO2S (поскольку OxR параллельно O2S). Следо- вательно, эти треугольники рав- ны по двум сторонам и углу меж- ду ними, откуда OR —OS. Опу- ская перпендикуляр ОН на хорду 201
PQ, получаем: RH=SH и PH=QH; значит, PR = PH — RH = = QH — SH—QS. 27.22. Если все знакомы, то задача решена. Пусть теперь Л и В незнакомы. Осталось 2и —2 человека, причем как с Л, так и с В знакомы более чем п человек. Значит, более чем (п 4- п) — (2п — 2) = 2 человека являются общими знакомыми для Л и В. Посадим теперь Л и В друг напротив друга, а между ними — их общих знакомых. 27.23. Будем проводить всевозможные отрезки, соединяющие данные точки друг с другом и с вершинами квадрата и попарно непересекающиеся, до тех пор, пока это возможно. В тот момент, когда это станет невозможным, квадрат окажется разбит на треугольники (иначе в одном из оставшихся многоугольников можно было бы провести еще одну диагональ). Треугольников с вершинами в данных точках будет не меньше 100 (докажите это сами индукцией по числу точек), и их суммарная площадь меньше 1. Поэтому площадь хотя бы одного из них мень- ше 0,01. 27.25. а = 2. Указание. Положим /=-^-, тогда t — ра- X г циональное число, являющееся корнем уравнения t2 — al 4-1=0. Но тогда . Число ~^а2 — 4 при целом а может быть рациональным только при д= ±2. 27.26. Только при n = 2fc. Указание. Пусть п — произволь- ное число и всю воду удалось за т переливаний перелить в один стакан; докажем, что п = 2к. Очевидно, перед последним переливанием было два стакана, в которых было поровну воды. Таким образом, если принять общее количество воды за 1, то на (т —1)-м шаге распределение воды по непустым стаканам былому-, . Сделаем теперь индукционное предположение, что на (ш —/?)-м шаге воды в стаканах было(-^-, , ..., . Что было на предыдущем шаге? Поскольку нумерация стаканов произволь- на, мы можем считать, что вода на этом шаге переливалась из вто- рого стакана в первый. Есть две возможности: либо второй стакан опустел, тогда на предыдущем шаге воды было ^-г, ..., либо в нем что-то осталось, тогда предыдущее рас- пределение было gr+2i4........... • В обоих случаях мы видим, что все знаменатели имеют вид 2*. В частности, именно та- кими были знаменатели перед первым переливанием. Но это и зна- чит, что n=2k. 27.27. Указание. Докажите, что в четырехугольнике О\О^ООз (пусть С лежит между Л и В) Z. Оз = 180°— А.ОСВ, Z. О2= 180°—Z. ОСИ, откуда АО24-АО3= 180°. 202
27.28. Указание. Первый игрок должен делать ходы, сим- метричные ходам второго, пока не представится случай занять угловую клетку. 27.29. Указание. Соеди- нить точку О со всеми вер- шинами и опустить из О все перпендикуляры на стороны; се- миугольник разобьется на 14 прямоугольных треугольников. Для каждой пары треугольников с общей гипотенузой выразить квадрат ее длины через сумму квадратов катетов (длину сторо- ны 7-угольника положить равной 1). Сложить 7 полученных равенств и получить отсюда, что сумма отрезков, стоящих в ле- вой части указанного в условии равенства, равна . 27.33. Расположим всевозможные пары целых неотрицатель- ных чисел в таблицу и будем их нумеровать в порядке «змейки», показанной на рисунке 40. Тогда легко убедиться, что пара (х, у) имеет номер -|-((х-|-у)2-f-3x-|-y). Но это означает, что функция f (*» задает взаимно однозначное соответствие между целыми числами, с одной стороны, и парами чисел — с другой. Отсюда сразу вытекает утверждение задачи. 27.35. Введем систему координат с началом в узле (оси на- правлены по вертикальной и горизонтальной линиям бумаги). Пусть Xi и у, — проекции отрезков ломаной на оси координат (с учетом знака). Тогда xi4-Х2 4-...4-хя=0, yt +yt+...+y*=:Q. rf+yi=c (1 Рассмотрим отдельно три случая: 1) с де- лится на 4; 2) с имеет вид 4Л4-2; 3) с нечетно. В случае 1) в каждой паре (xit у,) оба числа четные; пусть 2* — наибольшая степень двойки, на которую все х, и у, одновременно делятся. Выписанные выше равенства для сумм координат сокра- тим на 2* и попадем в один из случаев 2) или 3). В случае 2) xt и yi всегда нечетны, и имеем п нечетных слагаемых с ну- левой суммой. Следовательно, п четно и все доказано. В случае 3) числа Xi и {А разной четности. Пусть имеется m нечет- ных чисел xt (остальные четные); тогда п — m нечетных чисел у, (остальные четные). Из равенств сумм нулю следует, что m и п-m одновременно четные. Значит, и п тоже четно. 27.36. Можно. А именно легко видеть, что здесь применима индукция и мы вправе считать, что в (п — 1) мензурках уже составлены равномерные смеси из (п—1)-й жидкости, одна мен- зурка пуста и еще одна «стоит в стороне» — в нее налита та же жидкость, что и с самого начала. Теперь, пользуясь делениями. 203
разольем жидкость из первых п — 1 мензурок поровну по п мен- зуркам (в которых она была и пустой, количество жидкости в каждой считаем равным 1). Тог- да в каждой из них жидкости станет 1 —— (включая бывшую пустую мензурку). Долив их 27.37. Указание, ной точка О — центр 27.40. Указание. доверху (на из последней мензурки, мы получим требуемые равномерные смеси. При таком движении остается неподвиж- тяжести данных точек. Каждое натуральное число п > 1 встре- чается на карточках столько раз, сколько существует несократи- мых правильных дробей 0<^-<1 со знаменателем и, другими словами, сколько имеется натуральных чисел, меньших п и взаим- но простых с ним. 27.41. Сумма длин проекций всех векторов на координатные оси не меньше 4 (поскольку сумма двух длин проекций одного вектора не меньше его длины). Рассмотрим те векторы, сумма длин проекций которых на одну из четырех координатных полу- осей (лучей с началом О) наибольшая — она не меньше 1. Если она больше 1, то эти векторы искомые, если же равна 1, то суще- ствует вектор (какой?), добавление которого к выбранным векто- рам дает искомую систему. 27.44. Проведем построение, как указано на рисунке 41. Легко видеть, что если О — центр пирога, К и L — середины сторон АВ и ВС, то окружность, построенная на ВО как на диаметре, про- ходит через К п £. Пусть Р — точка, в которой пересекаются раз- резы (или их продолжения), проведенные из точек К и £. Если точка Р лежит внутри круга OKBL, то КР^ВО = \ и £Р^1, так что в точке Р пересекаются не продолжения, а сами разрезы, и от пирога, таким образом, отрезан кусок KPLB. Пусть теперь Р лежит вне круга. Тогда Z_KPL<Z_KOL, откуда Z_OKP< AOLP, т. е. а<0. Остается заметить, что так как точки /(, L, М, ... идут по кругу, то не могут одновременно выполняться неравенства а < 0, 0 < у,..., б < а. Это и означает, что хотя бы один кусок будет отрезан. Замечание. Из решения видно, что если длины разрезов меньше /, то при п>3 всегда можно провести их так, чтобы не отрезать от пирога ни одного куска. 27.45. Рассадим для начала рыцарей произвольным образом. Пусть при этом какие-то два врага — рыцари А и В — оказались 204
рядом, причем В сидит справа от А. Найдем теперь среди друзей рыцаря А такого С, что его правый сосед D—друг В (он найдется, поскольку у А есть п друзей, а число врагов у В, не считая А, всего лишь п — 2). Теперь «развернем» весь .участок стола BDE ... FGC от В до С в обратную сторону: CGF ... EDB. Легко видеть, что при этом число пар соседей-врагов уменьшится. Действуя так и дальше, мы в конце концов придем к требуемой рассадке. 28.01 . Проводим окружность с диаметром ОМ, находим точку N ее пересечения с прямой а (если она есть внутри круга О) и соединяем М с N. MN — искомая прямая. Решений может быть 0, 1 или 2. Указание. ON_LMN, так как ON делит пополам хорду круга О, лежащую на MN. 28.02 . Указание. Воспользоваться тем± что 999 делится на 37, и представить число abcOOO в виде а6с-999-|-а6с. 28.03 . Пусть О — точка пересечения прямых АВ и CD. Отложим от точки О на этих прямых отрезки ОХ—АВ и OY=CD, а затем построим параллелограмм OXYZ. Точка пересечения прямых OZ и а является искомой. Задача имеет два решения. (Почему?) 28.04 . Предположим противное. Пусть си — число матчей, сыгранных t-й командой; очевидно, что 0^а,^29. Поэтому, если все числа а, разные, то одно из них равно 0, одно — 1,.... одно — 29. Но команда, сыгравшая 29 матчей, сыграла со всеми командами, в том числе и с той, которая сыграла 0 матчей. Противоречие. (Решение задачи 28.08 аналогично.) 28.05 . См. указание к задаче 28.02. 28.06. Точка О лежит на пересечении прямых, параллельных сторонам треугольника и отстоящих от них на -±~, и -i- соот- и «3 х ветствующеи высоты. 28.07. По свойству биссектрисы rrs=37>4b=r^ - Отсюда Mb оС Ab I\d легко следует, что точка М отстоит от прямой АС дальше, чем К. Поэтому прямые МК и АС пересекаются в точке Р, расположенной правее С (рис. 42). А тогда Л. MCA как внешний угол треуголь- ника СМР больше его внутреннего угла СМР, т. е. р > <р, поэтому из &МКС видно, что МК>КС. Тдчно так же угол МКА больше угла КАР, т. е. ib>a, отсюда АМ>МК. Итак, АМ>МК>КС, что и требовалось доказать. 28.09. Указание. Докажите, одного куска ковра, то ни в одном месте коридора не лежит сразу 3 ковра. 28.10. Проведем к этим окруж- ностям касательные, параллель- ные сторонам треугольника, так, чтобы каждая из окружностей оказалась вписанной в треуголь- что если нельзя убрать ни Рис. 42. 205
ник, отсекаемый соответствующей касательной от исходного. Эти два отсекаемых треугольника подобны исходному с коэффициен- тами Л|<1 и Л2<I, поэтому r\=k\r и г2 = £2Г. Но так как они пересекаются, то откуда fi + f2>G что и требова- лось доказать. Замечание. Тем же способом можно решить и задачу 15.11. 28.12. Отрезок АВ считаем неподвижным, а двигать будем угол вокруг него. Вершина угла опишет тогда дугу АВ окружности, в которой отрезок АВ — хорда. Перпендикуляр к середине X хорды АВ пересекается с биссектрисой угла в се- редине У дуги а, дополняющей дугу АВ до полной окружности, следовательно, длина отрезка XY постоянна при движении вер- шины угла. 28.13. Указа ки е. Докажите, что наибольшее из всех чисел, находящихся в карманах, строго больше суммы остальных «кар- манных» чисел. (Ср. с решением задачи 28.15.) 28.14. 3932 треугольника и 5896 разрезов. Указание. Найдите сумму углов всех треугольников. (Ср. с задачей 16.22.) 28.15. Если |х|1>2, то IXяI > 1 + кп-2|+... +1. Поэтому f (х) при х£[ — 2; 2] имеет тот же знак, что и хп, и, следовательно, не равно 0. 28.16. Указание. Касательные к окружностям в точках Л, В, С пересекаются в одной точке D — центре описанной вокруг треугольника АВС окружности. 28.17. Очевидно, н а и б о л ь ш е е значение, которое может 1 I -х / 1 I « 1 Ч-л/б принимать Х = 2 V 4 Равно V 2”' Пусть теперь у — корень уравнения, удовлетворяющего усло- вию задачи, и |а| 1. Легко видеть, что ау — корень уравнения у2 4-pay + (/а2=0, которое также удовлетворяет условию. По- этому искомое множество значений есть множество чисел вида |а| 1, т. е. отрезок J • 28.18. Окружность, являющаяся пересечением сферы, по- строенной на АВ как на диаметре, с конусом, вершина которого — точка В, а основание — данная окружность. 28.19. Нельзя. Указание. Если между двумя карточками, на которых написано число а, лежит одна карточка с числом 6, то между карточками с числом Ь лежит ровно одна из карточек с а. Отсюда легко следует, что при произвольном расположении карточек количество карточек, лежащих между парами, бу- дет четным. Но согласно условию задачи оно должно рав- няться 1 4-2+ ... + 9=45. Таким образом, искомого расположения не существует. 28.20. Указание. Пусть ap=ak. Докажите по индукции, что для любых /, /, таких, что /4"/=Р + *, выполняется равенствЪ ai=aj. 28.22. Сделаем все 1965 поворотов. Суммарное число совпа- 206
дений красных секторов будет, очевидно, равно 200*200 = 40000. Предположим теперь, что при всех, кроме 59, положениях совпа- дает не менее 21 красных секторов; тогда общее число совпа дений не меньше.59*04-(1965— 59)-21 =40В26> 40 000. 28.23. Гангстер должен двигаться навстречу полицейским со скоростью 2v или -у и. Указание. Допустим, что появля- ется не 1 гангстер, а много — по одному напротив каждого по- лицейского. Поскольку все они находятся в одинаковом положе- нии, то если ни один полицейский не видит первого гангстера, то он не видит ни* одного гангстера. Соответственно ни один гангстер не видит ни одного полицейского. Но это значит, что гангстеров и полицейских можно поменять местами, так что если t»i = av — решение задачи, то и vt==~ v тоже решение. 28.28. Построим на сторонах данного треугольника внутрь него дуги по 120° каждая и отметим их середины А, В, С. Искомое множество центров описанных равносторонних треугольников вокруг данного — описанная вокруг ДДВС окружность. 28.29. За 2 взвешивания. Первое взвешивание. На одну чашу весов кладем по одной монете из 10 мешков, на дру- гую— 10 монет из оставшегося мешка. Второе взвеши- вание. На первую чашу кладем одну монету из первого мешка, 2 монеты из второго, 3 — из третьего, .... 10 монет из десятого. На другую — 55 монет из последнего (того же, что и в первый раз) мешка. Подумайте сами, как, сравнивая разности масс при взвешиваниях, установить, какой из одиннадцати мешков содер- жит фальшивые монеты. 28.30. Предположим, что этот процесс бесконечен. Тогда най- дутся такие i и /, что i-й слой совпадает с /-м, a (t'4- 1)-й совпа- дает с (/+1)-м. Тогда из условия укладки кубиков легко заме- тить, что (i—1)-й слой совпадает с (/—1)-м, (<—2)-й с (/—2)-м и т. д. В частности, второй слой совпадает с (/—<-|-2)-м, а пер- вый— с (у —Л + 1)-м. Но тогда нетрудно видеть, что процесс можно было остановить на (у—<)-м слое. 28.31. Предположим, что шарик попал в одну из угловых луз, пройдя перед этим путь длиной I. Где же он был, пройдя путь ? Из соображений симметрии по отношению к началу и концу пути эта точка должна находиться на равном расстоянии от начальной и конечной точек и, мало того, прямая его движения должна располагаться симметрично по отношению к этим точкам. Отсюда легко следует, что шарик должен в этот момент находиться либо в центре бильярда, но это невозможно, так как* на биль- ярде размером pX2q шар вообще никогда не попадает в центр (почему?) .либо на середине борта, т. е. свалится в среднюю лузу, что и требовалось доказать. 28.32. Если бы -это было не так, то суммы давали бы остатки 207
1, 2, 3, ...» 2п. Но тогда сумма всех полученных чисел при делении на 2п давала бы тот же остаток, что и 14-2 4-.-. 4-2л, т. е. п. Это, как легко видеть, неверно. 28.33. Назовем самый большой ящик ящиком ранга п9 следующие по величине два ящика — ящиками ранга п — 1 и т. д. до ящиков ранга 1, в которых лежат монеты. Разность числа гербов и решек в каком-нибудь ящике назовем дефектом этого ящика. Дефект самого большого ящика обозначим через d. Докажем, что всегда найдется ящик, при переворачивании которо- го общий дефект уменьшается по крайней мере вдвое; тогда задача будет решена, так как |d|^2n и d всегда четно. Предпо- ложим противное, и пусть d>0. При переворачивании всех монет в ящике с произвольным дефектом di общий дефект d меняется на 2d|. По предположению имеем: |d — 2dJ >-у |d|, либо di<--, либо d\>~ Дефект d' любого ящика ранга 1 меньше 1 — |d'|, так как d чет- но) , т. е. d' <-у-. Отсюда следует, что дефект любого ящика ран- га 2 не больше , а значит, меньше -у-; отсюда в свою очередь получится, что дефект любого ящика ранга 3 меньше -р и т. д. В конце концов получим, что дефект ящика ранга п меньше ---, хотя он просто равен d, и d>0— противоречие. (Отметим, что для того, чтобы все монеты стали «смотреть» гербом вверх, необходимо, вообще говоря, 2я переворачиваний.) 28.34. Ясно, что р< 19. Кроме того, если р нечетно, то //-j-l делится на 2. Мало того, если р имеет нечетный делитель, например р = 5Л, то рр 4-1 = (р*)5 4- 15 и делится на р* 4" 1, а потому не являет- ся простым. Итак, остается проверить р=|, 2, 4, 8. 16. Но 16,6==264 = (2,0)6- 16, а так как 2,0> 1000, то 260>Ю'8, откуда видно, что 161* имеет больше 19 цифр. Если р = 8, то 884-1 = ==2244- I =(284- 1)’(216 —284-1) составное. Остаются р=1, 2, 4. Прямая проверка показывает, что 2, 5, 257 — искомые простые числа. 28.35. Указание. Докажите, что последние цифры чисел пп и (л4-20)”+2° совпадают (проверьте это для различных последних цифр числа п). Периодически повторяется набор цифр: 0, 1, 4, 7, 6, 5, 6, 3, 6, 9, 0, 1, 6, 3, 6, 5, 6, 7, 4, 9. 28.36. Указание. Если М — точка пересечения отрезка АВ с плоскостью Р, ЛЛ4=а и ВМ~Ь, то проведенный через М диаметр высекаемого на плоскости круга делится точкой М на такие отрезки х и у, что ху — аЬ. Поэтому этот диаметр мини- мален при x=y = ^Jab. 208
28.38. Рассмотрим все строки и все Г”| Г столбцы и выберем ту строку (или стол- —------------------- бец), где нулей больше всего. Пусть в этой----------------- строке k нулей и I единиц, k + l=m. Тог-----------------— - да в А столбцах, соответствующих нулям_______*________*____ этой строки, по меньшей мере k единиц (это_____•________• следует из условия), а в остальных I столб- • • • цах не менее I единиц (это следует из вы- • • Т-- бора исходной строки). Таким образом,---------------------- общее число единиц не меньше Л2-|-/2^ ——————*- ^k2+(т — . Придумайте сами при- Рис. 43. мер таблицы, для которой это неравенство превращается в равенство. 29.01. Отрезок, соединяющий середину основания треуголь- ника АВС с серединой высоты, опущенной на основание. 29.02. Все числа, кроме делящихся на 20. 29.05. Нельзя, потому что в цепи одинаковые цифры идут парами, а осталось нечетное число экземпляров каждой цифры 29.06. Сложив уравнения, после тождественных преобразо- ваний получим равенство (х—1)(у—l)-|-(z—1)(/ —1)=2. По- скольку оба слагаемых в левой части неотрицательны, есть только два случая: а) оба они равны I, тогда, очевидно, x=y=z=t=2; б) одно равно 2, другое 0. В этом случае мы получаем решение х=3, у=2, z=5, 1 = 1 и еще 7 решений, получаемых из него перестановкой неизвестных. Всего задача имеет 9 решений. 29.07. fe=65. Указание. Очевидно, при переходе от А к (fc+1) знаменатель увеличивается в (Л 4-1) раз. А числитель? Его слагаемые умножаются на 19 и 66 соответственно. Поэтому он умножается на некоторую громоздкую дробь, которая заведомо меньше 66, но от 66 отличается ненамного (первое слагаемое много меньше второго и потому не играет особой роли). Легко показать, что если k не слишком мало, например если k>5, то числитель увеличивается более чем в 65 раз. Таким обра- зом, Ah возрастает, пока k <65, и начинает убывать после этого; Лб5 максимально. 29.08. Рассмотрим концы радиусов, соединяющих центр ок- ружности с * вершинами пятиугольника. Каждая сторона пяти- угольника с вершинами в этих концах не меньше соответствующей стороны данного пятиугольника. А в любом вписанном пятиуголь- нике найдется сторона, не большая стороны правильного пяти- угольника, так как имеется сторона, стягивающая угол . 29.09. Числа k образуют арифметическую прогрессию ЗОр 4- 29, р=0, 1, ... . Указание. Для каждого /г = 1, 2, ... рассмотрите отдельно периоды повторения остатков от деления числа А* 4- 1 па 2, 3 и на 5 (длины периодов равны соответственно 2, 6-и 20). 209
Рис. 44. 29.10. 16 дамок. На рисунке 43 показана нужная расстановка. Докажем, что расставить 17 дамок невозможно. Предполо- жим, что на доске стоят 17 дамок; очевидно, ставить дамку на край нельзя, так что они заполняют весь средний квадрат 6X6, за исключением одного поля. Но тогда в центральном квадрате 4X4 находится не менее 7 дамок. Для того чтобы их можно было бить» все они должны соседствовать с единственным свободным полем в квадрате 6 X 6. Но рядом с этим полем есть только 4 места. 29.11. Решение этой задачи приведено на рисунке 44 (ра- диус окружности № 7 равен , а окружности №8----------------. О о J 29.12. аюоо— 495. Указание. Проверьте, что начальные члены последовательности аь аг, .... ал, ... выглядят так: 1, 1, 1, 1, 2, 2, 2,3, 3, 4,4,4, 5, 5, 6,6,.... Докажите, что в этой последователь- ности каждое натуральное число встречается дважды, за исклю- чением единицы, которая встречается 4 раза, и чисел вида 2й, ко- торые встречаются по 3 раза каждое. Отсюда а»ооо находим простым подсчетом. 29.14. 10 линий. Указание. Из условия вытекает, что с данной линии метро можно пересесть максимум на три другие ли- нии, с каждой из этих — еще на две. Поэтому число линий не пре- вышает 14-34-2-3=10. На рисунке 45 показана схема пересадок для десяти линий, удовлетворяющая условию. Замечание. Можно доказать, что если 9 из линии, соединяющих точки на рисунке 45, прямы е, то и десятая будет прямой. Этот чертеж соот- ветствует теореме Дезарга в проективной геометрии (см., например: Мацуо Комацу. Многообразие геометрии, чертеж 34, с. 78). 29.15. Рассмотрим число N= 10001000... (количество нулей в числе N мы заладим позже). Выясним, как меняются первые 4 цифры числа fe! при k — N, #4-1, ... . Если k\ = dbcde ... , то М- 10Ю0100 ...=abcx ... , где х равно либо d, либо d+1, в зависи- мости от того, случается ли перенос от суммы а + е в следующий 210
разряд при умножении «в столбик». Так как при этом пятая цифра указанного произведения увеличивается на а, то не реже чем один раз в 10 шагов этот перенос будет происходить и число abed будет увеличиваться на 1. Пусть теперь А = 1 000 100 000. На протяжении 100 000 шагов число abed будет увеличиваться не бо- лее чем на 1 при каждом умножении, но и не реже чем через 10 умножений действительно будет увеличиваться на 1. Таким об- разом, четыре первые цифры числа Л1 последовательно пробегут 10 000 значений, среди которых встретится и 1966. 29.13. и 29.18. Сформулируем вначале общие принципы, при- меняемые при решении подобных задач. 1°. Если из Т (или меньшего числа) шаров радиоактивен один, то его можно найти за п испытаний: первым шагом испытать половину шаров и таким образом узнать, в какой половине находится активный, вторым шагом взять половину от этой половины и т. д. 2°. Если шаров больше чем 2“. то за п шагов нельзя обеспечить отыскание одного активного шара. Предположим противное. Сделаем п испытаний, отмечая наличие радиоактивности плюсом, а ее отсутствие — минусом. По предположению, зная возникшую последователь- ность знаков, мы сможем указать, какой из шаров активен. Но разных последовательностей существует всего лишь 2". Указав по каждой из них активный шар, мы придем к нелепому выводу, что те шары, которые не соответствуют никакой последователь- ности, вообще не могут быть активными. 3°. Если из m шаров активны 2, то имеется Ст ———- вариантов, смотря по тому, какая пара шаров активна. Аналогично, если то за п испытаний не удастся наверняка найти активную пару. 4°. Если из т шаров мы первым шагом испытываем k, то ре- зультат «—» соответствует Cm-ь вариантам (оба активных шара находятся среди т — k оставшихся), а результат « + » — осталь- ным Ст — Ст-л вариантам. Если в нашем распоряжении осталось / испытаний, то ни одно из этих чисел не должно превышать 2f. Естественно, наилучшая стратегия состоит в том, чтобы эти числа были по возможности одинаковыми. Решение задачи 18. Положим, что на первую проверку взято 2 шара и получен отрицательный результат («минус»). Теперь нужно из 9 шаров выделить 2 за 5 проверок. Но так как Cl=36 > 2s, сделать этого нельзя (п. 4°). Пусть теперь на первую проверку взято 4 шара. Если был получен « + *• то число остав- шихся вариантов равно Си — Су=34 > 2s, и задача вновь нераз- решима. Итак, для первой проверки необходимо брать 3 шара. Предположим, что получен « —». Нам остается из 8 шаров выделить 2 за 5 проверок. Аналогичный подсчет показывает, что на вторую проверку надо брать больше одного, но меньше трех шаров. Итак, берем на II проверку 2 шара. Предположим, что опять получен « — » и остается из 6 шаров выделить 2 за 4 провер- 211
ки. Но теперь если мы возьмем 1 шар, то в случае результата « —» останется С&=10>23 ва- риантов; если же на III проверку взято 2 шара, то в случае « + > вариантов останется Cl — С*» = 9>23. Итак, выделить 2 шара из 11 за 6 проверок можно лишь при удачном стечении обстоя- тельств. Сделать это за 7 про- верок нетрудно. Решение задачи 13. Возьмем на I проверку 5 шаров. Предположим, что получен « + ». Тогда на II проверку берем 8 шаров из 14 оставшихся. Если вновь получен « + », то один активный шар — в первой пятерке, а другой — во второй восьмерке, и каждый из них можно най- ти за 3 испытания (п. 1°). Если же вторая проверка дала « —», то на третью проверку берем 4 шара из оставшихся. В случае « + » за 3 испытания находим один шар из пяти и за 2 — один из четырех; случай « —» разберите сами. Пусть теперь первая проверка дала « —». Остается за 7 проверок найти 2 шара из 14. Теперь следует взять на проверку 4 шара. Дальнейшие рассуж- дения, аналогичные предыдущим, проведите сами. 29.19. Набор гирь: 26, 25, 24, 22, 19 и 11 удовлетворяет усло- вию. Пусть теперь выбраны какие-то 7 гирь. Заметим сразу, что нельзя брать одновременно гири 26, 25, 24, 23, а потому сумма масс любых четырех гирь из этих семи будет меньше 98. Но нетрудно подсчитать, что существует 98 разных наборов из 7 гирь по 1, 2, 3 и 4 гири. Поэтому массы каких-то двух наборов будут совпадать. 29.20. 14. Указание. Переставляя строки и столбцы, всегда можно перейти к расположению, которое аналогично по- казанному на рисунке 46. Далее остается найти число спосо- бов разбиения числа 11 на слагаемые, каждое из которых не меньше 2. 30.01. Существуют. Наименьшая такая пара состоит из чисел N и /V—1, где /V = 8999...9900...0 (13 девяток, 14 нулей). 30.02. Искомая точка М — основание высоты СМ треугольника АВС. Действительно, по теореме синусов « + ^=4<^+ec).-J5ii7S!±(ZC+B4. 30.03. Такого способа не существует, потому что слова, у которых первые 8 знаков (обозначим их звездочкой) совпадают, должны по условию попадать в разные группы, а таких слов имеется четыре: (♦••), (♦ —), (♦—•), (♦-)• 212
30.04. При АВфВС искомых точек М —14; при АВ = ВС искомое множество состоит из окружности радиуса ВА с центром В и еще 6 точек. 30.05. 37-28-2 = 2 072. 30.06 . Указание. Пешеходы неизбежно встретятся на самой длинной окружности, поскольку ее длина больше суммы длин всех остальных окружностей. 30.07 . Нельзя, потому что суммарная площадь пяти частей, заштрихованных на рисунке 47, равна — (т. е. отлична от 30.09 . Максимальная разность равна 1006 — 993=13. 30.10. Пусть первые 12 цифр полученных 120-и чисел образуют числа Л|, Л2, Л120. а оставшиеся 108 цифр каждого числа — число В. Тогда надо доказать, что сумма (Л| + ...+Л|2о)-10l08 + 4-120*В кратна 120. Но + - +^120 делится на 3 (так как остатки при делении на 3 каждого Л4 одинаковы, а их 120), а Ю|03делится на 40. 30.11. На ft-f-1 треугольников при любом k. Пример, для которого k треугольников не хватает, такой: все k точек распо- лагаются на одной диагонали с одной стороны от центра квадрата. 30.12. Решение аналогично решению задачи 29.08. 30.13. Указание. Покажите, что левая часть никогда не дает остаток 5 при делении на 9. 30.14. Указание. Рассмотрите куб, содержащий № изю- минок, и докажите, что любая плоскость разрезает в этом кубе порядка N2 изюминок. 30.15. Указание. 3 +1 =(2+1)-(3+!)• ... -(р* +1). 30.16. Указание. Докажите, что треугольник АВС равно- сторонний. 30.17. Наименьшая высота равна половине длины диагонали квадрата. Действительно, 4 одинаковых круга радиуса г<~ № не могут вместе покрыть 4 вершин квадрата со стороной 1 и его центр. 30.18. Сложим несколько чисел вида 21000- 10*', где ki =0, а остальные ki подбираются так, чтобы сумма (она, очевидно, делится на 21000) не имела нулей на последних tn местах. Легко видеть, что это можно сделать для любого tn. Таким образом, существует число, делящееся на 21000 и не имеющее нулей на последних 1000 местах спра- ва. Теперь остается заметить, что все осталь- ные цифры можно просто отбросить (дели- мость на 2,00° от них не зависит). Второе решение: докажите, что если число из k цифр делится на 2*, то можно к нему слева приписать цифру 1 или 2 так, чтобы полученное число делилось на 2*+1. По- этому существует даже число, делящееся на 2'1 Рис. 47. 213
и состоящее из единиц и двоек (такая задача предлагалась на Всесоюзной олимпиаде 1972 г.). Замечание. Наша задача в действительности является леммой, помо- гающей доказать более общее утверждение: если число m не оканчивается нулем, то существует N, кратное m и не имеющее в своей записи ни одного нуля. Решите ее сами. 30.19. Указание. Докажите сначала, что если сумма двух целых чисел равна 999 ... 99, то при сложении ни в одном раз- ряде не произошло переноса в следующий разряд. 30.20. Возьмем два «наиболее северных» прожектора (если несколько прожекторов находятся на одной параллели, то вы- берем «наиболее северные» произвольно). Легко видеть, что, на- правив их на юго-запад и юго-восток, мы осветим всю полу- плоскость к югу от них. Теперь легко направить остальные два прожектора так, чтобы вся плоскость была освещена. Замечание. Основная идея этого решения состоит в том, чтобы, разделив множество прожекторов пополам, на «северные» и «южные», свести задачу на плоскости к аналогичной задаче на прямой (осветить всю прямую двумя прожекторами, каждый из которых может осветить полупрямую), кото- рая тривиальна. Аналогично можно свести пространственную задачу 30.34 к ре- шению данной задачи. 30.22. Пусть q — делитель N. Очевидно, если q не делится на р, то q также делитель d (А/), в противном случае -2- является делителем d (N). Отсюда легко вывести, что d (N)^2d (d (N)). Обозначим d(N)=b. Поскольку b не может делиться на числа за единственным исключением с = Ь, то из неравенства d (b)следует, что b делится на все числа, меньшие кроме, быть может, одного. Отсюда легко вывести, что ft С {12, 8, 6, 4, 3, 2}, a N=pb. Остается перебрать числа вида pft, что мы предо- ставляем проделать читателю. 30.24. См. решение задачи 47.05. 30.25. Для того чтобы выполнялось утверждение задачи, школьникам необходимо посетить 8 сеансов в одном из кинотеат- ров и, как минимум, по 2 сеанса в каждом из остальных шести ки- нотеатров — всего 20 «сеансопосещений». Но за один раз они посещают только 2 сеанса (шестеро — один, седьмой — другой), т. е. за один день они посетят лишь 16 сеансов. 30.26. Указание. Докажите сначала, что при сложении не могло случиться так, чтобы перенос единицы произошел во всех 200 разрядах. 30.27. Указание. Самая длинная последовательность по- лучается При Xi = 1, Х2 = М. 30.28. Указание. Выразить радиус через катеты и гипо- тенузу. 214
30.29. Заметим, что нули в конце числа N можно отбрасывать (если М=Л»10', то N=A кратно К, а поэтому и А=А крат- но К). Пусть N=iPia2... On-ta„ кратно К и а„^\. Вычи- тая из /У»10"+2 число N «в столбик», получим разность М = а,а2 ... а„~1 (а„— 1)-99 (9 —а„) (9 —a„~i)... (9 — а2) (10 — ai). Сложив теперь «в столбик» М с М, получим число П —1 Л — I У= 10 999 ...99 98 900?.. 000, кратное К. Если бы мы проделали те же действия, отправляясь от чисел ДМ0"+3 и N, то получили бы число Х = 10 999 ...99 998 900 ... 000, п — I _________ п - I также кратное К. Но тогда (X—Y)=99 кратно К, что и требо- валось доказать. 30.30. Занумеруем портреты по порядку: 1, 2. п. Оче- видно, достаточно доказать, что из любого исходного рас- положения портретов можно получить порядок 1, 2, .... п. Чтобы его получить, подгоним сначала 1-й портрет ко 2-му (очевидно, это можно сделать без помех, так как 1-й портрет можно пере- ставить с любым другим), затем поведем портреты 1 и 2 вместе до тех пор, пока не подгоним их к 3-му, и т. д. 30.31. Очевидно, достаточно доказать, что ни одно из чисел не вернется на свое место во 2-й, 3-й, .... п-й строках. Замечание. Легко проверить на примерах, что (л 4- 1)-я стро- ка, если написать ее по тому же правилу, совпадает с 1-й. Просле- дим за судьбой какого-нибудь числа /. При переходе в сле- дующую строку оно сдвигается вправо на одно из (положи- тельных или отрицательных) чисел n — k, n — m—k, —m. Пусть после нескольких шагов оно х раз побывало в I группе (на одном из мест от 1 до k), у раз во 11 и г раз в 111; тогда оно сдвинулось вправо на N=x(n — k)+y(n— m—fc)+z(—m) мест. Пусть оно, ни разу не побывав дважды на одном месте, нако- нец вернулось на исходное место. Тогда М=0, причем O^x^fe, 0^y^m — k, 0^z^n — m. Уравнение /V=0 имеет два очевидных решения: х=0, y=m, z=n — m — k и х=1, у=—J. z=l. Представим эти тройки чисел как координаты векторов щ =(0, т, n — m — k) и и2=(1, —1, 1); каждый из векторов перпендикулярен вектору v=(n — k, п — т — k, —т), откуда легко следует, что любой вектор й—(х, у, z), являющийся решением. уравнения /У=0, будет их линейной комбинацией: u=aiiii +а2и2. Остается показать, что вектор atUt -f-a2u2 не может иметь целых координат х, у, z, удовлетворяющих неравенствам O^x^k, Q^y^m — k, — m. Сделайте это сами. 215
0 а -а 0 -а 0 0 а о 0 0 -а 0 -о а 0 Рис. 48. 30.33. Из условия вытекает, что числа 1, 2, 3, 10, 11, 12 не могут стоять рядом. Следовательно, они стоят через одно по окружности. Но тогда располо- жить число 4 согласно условию уже не удастся. 30.35. Указание. Докажите сначала, что есть два числа, различающиеся только одной циф- рой, к которым приписаны разные цифры. Пусть, например, к числу 11... 1 приписана 1, а к числу 2Н ... 1 — цифра 2; тогда неизбежно к числу 33 ... 3 при- писана цифра 3, а к числу 22 ... 2 — цифра 2, Тогда нетрудно доказать, что к каждому числу приписывается его первая цифра (докажите это сначала для чисел, начинающихся с 3 и состоящих из двоек и троек) .В общем случае аналогично доказы- вается, что ко всем числам, у которых с-я цифра равна 1, припи- сывается одна и та же цифра а (а равна 1, 2 или 3); к тем, у которых г-я цифра равна 2, приписывается цифра Ь, а к осталь- ным — цифра с. 31.01 . 4 и 1. 31.02 . Единственно возможный вариант показан на рисунке 48. 31.03 . Следует выбрать простое число, на которое первые три не делятся. 31.04 . Ровно один город должен быть пересадочным; а по- скольку в связном графе с 50 ю вершинами число ребер не мень- ше 49, то 49— наименьшее число авиалиний. 31.05 . 66—12 = 54 ничейных партий. Указание. Каждый нз участников турнира выиграл только по одной партии. 31.06 . После четвертого вычеркивания должно остаться толь- ко одно число. Очевидно, необходимо оставить число 44. Но с другой стороны, поскольку сумма данных чисел делится на 11, легко видеть, что 44 надо было вычеркнуть на первом этапе. Противоречие. 31.07 . Нельзя, потому что сумма дуг Дь42Дз, Л3Л4Л5 и A^A^Ai равна 120°+120°+И0° >360°. 31.08 . Запишем данную алгебраическую сумму в виде S=X| +( — Л2-Ь*з)-Н — *4 + *5)+...+(—*98+*99)~*100. Чтобы 5 было максимальным, неположительный член (—хюо) должен быть равен нулю: хюо=О- Далее, поскольку x* + i — Xk^Xk, имеем: 5 -Сл'| +Х2 + Х4 + -. -Ь*96-Ь*98. Равенство будет, если положить л3 = 2х2, xs = 2x4, . *99 = 2x93. Очевидно, что для получения максимального S надо положить также x2 = 2xi, х< = 2хз, .... Поэтому ответ такой: xi = 1, х2 = 2, хз=4, ..., Х99 = 298, хюо=О. 31.09 . Выберем систему координат так, чтобы координатные оси не были параллельны ни одному из отрезков. Тогда конец одного из отрезков, имеющий наибольшую абсциссу (или орди- нату), не может упираться в другой отрезок. 31.10. Существует. Например, условию удовлетворяет произ- вольная трапеция с основаниями 1 и ^2. Докажем это. Пред- положим. что площади всех четырех треугольников рацио- 216
нальны. Высота указанной трапеции площади 1 равна 2 I+V2 ’ Высоты треугольников, примыкающих к основаниям трапеции, равны а и где аи₽ рациональны. Итак, а 4-^ = ^^- Отсюда следует, что число V2 является корнем квадратного уравнения ajr 4-(Р4-а—2) х + Р=0, что невозможно. 31.12. Поскольку длина дорожек в 10 раз больше длины коридора, то либо весь коридор застелен десятикратно (и тогда незастеленных кусков нет вообще), либо есть какая-то точка коридора, застеленная не менее чем 11 кусками до- рожки. Но тогда эти 11 кусков застилают некоторый сплошной промежуток, так что имеется не более 10 застеленных и соот- ветственно не более 11 незастеленных кусков. Пример, в котором незастеленных кусков действительно 11, постройте сами. 31.13. Можно. Простейшее решение — выбрать те шестизнач- ные номера, у которых сумма цифр делится на 10. 31.14. У к а з а н и е. p4 + qp=(pp + qp)+(pf>+2—pl>y, докажи- те, что оба слагаемых делятся на (p + q). 31.15. Указание. Перелет надо совершать так, чтобы в каждый момент времени в воздухе находилось как можно меньше самолетов, т. е. сажать самолет, как только в баках остальных освободилось достаточно места, чтобы принять все его горючее. 31.16. Первым ходом начинающий должен съесть кучку в 33 конфеты, а другую разделить на кучки в 17 и 18 конфет. В даль- нейшем он должен играть так, чтобы оставлять партнеру в обеих кучках 564-2 или 56 4-3 конфет (проверьте, что он сможет этого добиться). Таким образом, его партнер в конце концов вынужден будет делить кучку в 2 или в 3 конфеты и проиграет. 31.17. Нельзя. Максимальное возможное число классов в данной задаче равно 2,0= 1024< 1968. Указание. Докажите сначала, что два числа, отличающиеся друг от друга лишь по- рядком цифр, принадлежат одному классу. 31.18. Указание. Докажите по индукции, что если |х| > 1, то |/л+1(х)|> |f„(x)|. 31.19. Множеством центров искомых параллелограммов яв- ляется объединение трех прямых, каждая из которых есть пере- сечение двух «серединных» плоскостей для соответствующих пар скрещивающихся прямых. По центру О точки {Д} уже строятся однозначно центральной симметрией прямых {/J. 31.20. Начинающий должен первым ходом провести диаго- наль через центр 1968-угольника, а в дальнейшем поддерживать своими ходами симметрию относительно этой диагонали. 31.21. Указание. Все получающиеся точки будут лежать на одной окружности. 31.22. 49 переходов (дальтоник должен чередовать свои цве- та, а красить подряд, начиная с первого участка). 217
31.23. x = 4, f/ = 6. 31.24. Указание. Рас- смотрим систему координат (л, /), где х — координата точки на траектории пули, / — время. Траектория пули на этой плоскости есть пря- мая, и лопасть вертилятора попадает на нее,- если отре- зок х = at, t С Г h; 4- , L оО ti + П^q0'5] (это один из изо- браженных на рисунке 49 вертикальных отрезков) пе- ресекается с ней. Итак, тре- буется доказать, что, како- вы бы ни были /|, /2, /з, /4 (они соответствуют углам поворота дисков друг отно- сительно друга), существует прямая, пересекающая 4 из нарисованных отрезков. До- кажите это сами. 31.25. Можно. Для это- го нужно в каждой груп- пе брать все числа, кроме последнего, по порядку, на- чиная с 1 и исключая уже взятые, а последнее — вида (/и7 — ai — а2 —... — аД 31.26. Пусть кузнечик сделал 2 прыжка вдоль ло- маной ЛВС, как показано на рисунке 50. Построим точ- ку С', симметричную С относительно ОВ. Легко ви- деть, что /LABC' — 180° — 2а. Отсюда легко получить, что если мы будем отражать путь кузнечика сначала относительно ОВ, затем относительно ОС' и т. д., то точки А, В, С'9 D', Е’ и т. д. попадут на одну окружность и будут отсекать от нее равные дуги. Отсюда уже легко вывести нужное утверждение. 31.27. На рисунке 51 показан путь некоторой точки пирога после нескольких операций. Из него видно, что через некоторое время эта точка займет место, симметричное относительно &L тому, которое она занимала вначале. Отсюда уже нетрудно получить нужный результат. 31.28. 990. Указание. Такова разность между числами 218
909 909 и 908 919. Чтобы дока* As. зать, что искомая разность не может быть больше 990, вое- пользуйтесь тем, что среди пер- \\ У] вого миллиона чисел с интерва- \ / лом 1001 встречаются числа abcabc, удовлетворяющие уело- / \____/ вию. \\W^ \/ ^W// 31.30. Искомое множество по- \ / казано на рисунке 52. Y 31.31. Не более трех. У к а- А з а и и е. Пусть последняя точка dj968 находится между Ak и 4/. г~—\ Тогда А1967 находится между рис. 52. Д*_| и Д/_| (за исключением случаев Л=1 или /=1, которые нужно рассмотреть отдельно), и потому дуги, возникшие в результате 1968-го разбиения, имеют такие же длины, что и некоторые из возникших раньше. Замечание. Эта задача имеет отношение к одной из теорий современной математики — эргодической теории. В этой теории, в частности, имеется раздел под названием «Перекладывания», в котором изучаются преобразования Т (пе- рекладывания), переставляющие между собой конечное число отрезков (за под- робностями отсылаем к книге: К о р н ф е л ь д И. П., С и и а й Я. Г., Ф о м и н С. В. Эргодическая теория.— М.: Наука, 1980). Любая степень 7” перекладывания Т заданных г отрезков также является перекладыванием, но более мелких отрез- ков и в большем количестве. Сколько же различных длин может встретиться среди всех длин этих мелких отрезков? Оказывается, их число не зависит от показателя степени п, а зависит только от неходкого числа г пере- кладываемых отрезков: оно не превосходит 3(r—1). Это утверждение получено в 1983 г. при доказательстве следующего очень важного и интересного математического результата: почти все перекладывания эргодичны. Конструкция, рассматриваемая в задаче 31.31, отвечает перекладыванию отрезков (делая циркулем первую засечку, мы разбиваем окружность иа 2 «отрезка»); здесь л = 1968, а число различных длин не превосходит 3(г—1)= =3(2-1)=3. Таким образом, в последние годы получено очень далекое обобщение ут- верждения задачи 31.31, имеющее важные применения во многих разделах современной математики. 31.32. Выигрывают белые. Для выигрыша они должны вна- чале двигать короля по маршруту al—61—с!—... до тех пор, пока черный король не сойдет с восьмой горизонтали (если он не сойдет с нее, то белые выиграют на восьмом ходу). После того как черный король сойдет с восьмой горизонтали, белый ко- роль должен немедленно занять такое положение, чтобы между королями было четное число горизонталей и вертикалей (напри- мер: 1. Кр al — 61 Лр й8—Й7. 2. Кр Ы — с2) и затем держаться такой симметрии, по возможности приближаясь к черному ко- ролю. Тогда легко видеть, что рано или поздно короли займут 219
одно из положений, показанных на рисунках 53, а\ 53, б; 53, в при ходе черных. После этого черным придется ходить вверх или вправо — белый король «потащит черного на буксире» и выиграет. 31.34. Предположим сначала, что в числе N поров- ну единиц и восьмерок, двоек и семерок и т. д. Тогда первое разбиение можно произвести так: ,55 ... 5 66 ... 6 77 ... 7 88 ... 8 99 ... 9 ' 44 ... 4 33 ... 3 22 ... 2.11 ... 1 00 ... О 99 ... 9 99 ... 9 99 ... 9 99 ... 9 99 ... 9‘ Тогда вторым сложением можно получить число 99 ... 9 + 9= 100 ... 08 и третьим шагом задача будет ре- шена. Таким образом, дело сводится к тому, чтобы получить из исходного числа N число с равным (или почти равным) коли- чеством цифр, дополняющих друг друга до 9. Покажем, как это можно сделать, на примере двоек и семерок. Пусть, например, семерок много больше, чем двоек; сделаем следующие три сло- жения: 1) 2... 2 7... 7 7... 7 ф 7... 7 0... 0 2. .23 5. .54 7... 7 2) ,342... 2 5... 5 7... 7 + 5... 5 0... 0 342 .23 I.. 10 7 ... 7 3) 334 2.. 2 1 ... 1 7... 7 ф 1 ... 1 0... 0 334 2... 2 2... 2 7...7 Если количество складываемых семерок вначале подобрано пра- вильно, то в итоге число двоек и семерок будет отличать- ся не более чем на 7. Таким образом, четвертым сло- жением мы уничтожим все цифры, кроме нескольких (не бо- лее 10), превратив их в девятки. Дальнейшее несложно. 31.35. В общем случае окружность радиуса R касается трех сторон Л1, продолжения которых образуют треугольник, все вы- соты которого не меньше 1. А тогда площадь S этого треуголь- но
ника не меньше его полупериметра, а с другой стороны, S = Rp. Отсюда (равенство достигается для равносто- 'роннего треугольника с высотой I). 31.37. Решим сначала следующую систему уравнений с тре- м я неизвестными: ( Х\ + Х2 4- Хз =0, I + Тройка (I, —2, I) — решение первого уравнения; подстав- ляя во второе, получаем слева ( — 6). Следовательно, уменьшив эти числа в раз, получаем решение (xi, хг, х3) выписан- ной системы (при этом Х| = хз, хг = — 2xi). Учитывая вышеизложенное, решим следующую систему трех уравнений с девятью неизвестными: Xi + хг + ••• + *9 = 0, *1 + *2 + ••• + *9 = 0, *? + *2 + ••• + *В = I • Первая и последняя тройки неизвестных совпадают с найден- ным решением (xi, Х2, *з), вторая тройка получается из этого решения умножением на ( — ^2). Тогда первые два уравнения выполняются автоматически, а левая часть последнего равна не- которому числу к. Уменьшая все 9 неизвестных в раз, полу- чаем значения неизвестных (xi, хг, .... хе, хэ)— решение этой системы. Затем поступаем аналогично: увеличиваем на 1 число уравнений и утраиваем число неизвестных; при этом все 27 не- известных делятся на три группы по 9; первая и третья группы совпадают с найденным решением, а средняя полу- чается из них умножением на — д/2- Тогда сумма этих 27 чисел, сумма их кубов и сумма их пятых степеней равны 0, а сумма седьмых степеней равна некоторому числу А,; деля все числа на получаем решение этой системы. Аналогичными добав- лениями уравнений по одному и утраиванием числа неизвестных получим систему из 11 уравнений с З10 не- известными, которая имеет решение (попробуйте подсчитать, сколько времени понадобится, чтобы выписать полностью от- вет). 31.38. Может. Сначала отсекаем у треугольника 3 уголка, потом у шестиугольника — 6 уголков, у 12-угольника — 12 угол- ков и т. д., всего 19 раз. В результате получится 1+3 + 3*2 + +3-22 + ... + 3*2,8> 106 многоугольников. Любая прямая пере- секает не более двух треугольников каждого ранга, а всего не более 2* 19 + 1 <40 многоугольников. 31.39. Предположим противное. Легко видеть прежде всего, что мелом отмечены либо все 8 вершин, либо ни одной. Таким образом, помимо вершин, отмечены 100 или 92 точки. Но легко 221
видеть, что вместе с каждой точкой не в вершине придется добавлять на других гранях 5 (для точки в центре грани), 11 (для точки на середине ребра) или 23 точки (в остальных случаях) так, что их общее число будет делиться на 6. Но ни 100, ни 92 на 6 не делятся. 32.01. Ладья должна двигаться по средней линии доски, отставая от слона на ход коня. 32.02. Указание. Провести в АЕКМ медианы MF, EL и 7VK. 32.03. Еще 2 команды, набравшие по 4 и 3 очка. Указа- н и е. Среднее число очков, приходящихся на одну команду, на I меньше числа команд. 32.04. Никакое число, оканчивающееся четырьмя одинаковы- ми цифрами, не делится на 16. 32.05 и 32.07. См. решение задачи 32.15. 32.08. Ладья должна двигаться по средней линии доски, каждый раз становясь по диагонали (на ход слона) от коня. 32.09. Если ВМ и АН — равные медиана и высота, то, опустив из точки М перпендикуляр МК на сторону ВС, полу- чаем: МК=±-АН=~-ВМ. И тогда из треугольника МКВ сле- дует, что Z_A1B/< = 300 (рис. 54, о). Отразив симметрично вер- шину В относительно точки М, получим точку В', причем Z_AB'В=30°. Геометрическое место (множество) точек В' есть дуга окружности в 30°, построенная на АВ как на хорде. Теперь геометрическое место (множество) точек С находится просто: оно получается перемещением множества точек Bz на вектор АВ = В'С (рис. 54,6). 32.10. Можно. Например: —а, 6, 6, —-а, 6, 6, ..., —а, 6, Ь, —а, 6, где а и Ь таковы, что 2Ь—а>0 и 136—7а<0. Одно из решений: а=25, 6 = 13. 222
32.13. Нельзя. В самом деле, пусть сц, .... а$о — такие числа. Тогда ai4-a24-...4-a34>0, a ai+aj-j-... -|-азо<О. Следова- тельно, аз14-032+азз+аз4>0. Аналогично докажем, что аз5+озб+азт + аз8>0, аз9+а<о+а«1+а«2>0 и т. д. Складывая эти неравенства, получим, что аз| + оз2 + -.-+а5о>-0, что, как лег- ко видеть, противоречит второму условию. 32.15. Пусть стена образует некий многоугольник ABCD ...Е. Заметьте, что каждый из углов ABC, BCD и т. д. содержит внутри себя угол 37-угольника, описанного вокруг одного из городов, и потому не меньше, чем угол правильного 37-угольника, т. е. не меньше л—|у-. Поскольку сумма внешних углов в любом мно- гоугольнике равна 2л, а каждый внешний угол не превосходит , то число углов не меньше 37. 32.17. Докажем, что последовательность, указанная в задаче, может быть бесконечной только при ai+a2 + ... + aioo=0. Дейст- вительно, если ai +...+аюо= — е<0 и Ne>at для некоторого натурального N, то <2i<xw4-i =Oi — Л/е<0 — противо- речие. Отсюда — aioo —пээСО и laiool = — aiooCa99 = lawl- 32.18. Достаточно показать, что последовательность из пяти перфокарт всегда можно укоротить. Из четырех лежащих спра- ва перфокарт могут совпадать только первая и четвертая, иначе сразу выбрасываются две. Пусть они действительно совпадают по цвету, т. е. цвета идут в порядке ...abca. Тогда достаточно подложить справа карту цвета с, чтобы можно было выбросить две карты (с и а). Остается рассмотреть случай, когда 4 край- ние различны, а пятая совпадает с одной из них. Тогда есть две возможности: ... dabcd и ... cabcd. В первом случае нужно подложить справа цвета cbc, а во втором — цвета bd. 32.19. Существует. Например, Л=т^- Указание. При 1969я любом п дробная часть числа 77^7- равна 1Уио 32.20. Ответ показан на рисунке 55. 32.21. Указание. т(-—|- 4—h \ а b —+у-) =a+6 + -.. + fe. 32.23. Сможет. Один из способов сделать это показан на рисунке 56. См. 32.37. 32.24. Первым ходом первый игрок вычеркивает 9 чисел от 47 до 55. Остальные числа разбиваются на две группы: от 1 до 46 и от 56 до 101. Для любого k, вычеркнутого вторым игро- ком, первый вычеркивает |55—А|; раз- ность оставшихся двух чисел равна 55. 223
Рис. 56. 32.25. Чтобы найти жемчужину за 33 разреза, д о ст а точ- н о сделать их параллельно друг другу на равных расстояниях. Доказательство неразрешимости задачи за 32 разреза основано на следующем утверждении: как бы ни бы- ли проведены k разрезов, в получившиеся k +1 частей можно вписать не более k +1 кругов, суммарный радиус которых ра- вен 10 см. Применим индукцию. При /г--0 это очевидно. Пусть это верно для k разрезов и еде чан еще один разрез. Если он не затрагивает вписанных кругов, то они удовлетворяют усло- вию; если же один из кругов оказался разрезанным, то заменим его двумя, как показано на рисунке 57. При этом суммарный радиус кругов не меняется, и утверждение доказано. Таким обра- зом, после 32-х разрезов можно будет построить 33 круга сум- марного радиуса 10 см. Но тогда радиус хотя бы одного из них больше 3 мм, и если жемчужина находится в нем, она останется необнаруженной. Замечание. Если бы требовалось сделать сразу все разрезы, то для обнаружения жемчужины понадобились бы те же 33 разреза. Однако для этого случая решение коренным образом меняется (при до- казательстве специально рассмотрите поверхность шара). 32.27. Указание. Как бы ни играли партнеры, тот, за кем очередь, всегда смо- жет сделать свой ход, если есть еще хоть одна незамазанная клетка. Поскольку все- го клеток на доске 63 (не считая клетки al), выигрывает независимо от хода игры начинающий. 32.28. Указание. Все четыре указан- ные прямые пересекаются во второй точке пересечения окружностей, описанных около этих пятиугольников. 224
32.29. Предположим (это допустимо, если применить индук- цию), что для любого /<1969 число последовательностей, кон- чающихся на 19 меньше I. Рассмотрим все последовательности, кон- чающиеся на 1969, и отбросим в них последний член. Тогда все эти последовательности кончаются на одно из чисел 1, 2, 3, .... [д/1969]. Для каждого из этих чисел количество последовательно- стей, кончающихся на него, меньше -\Г1969. Поэтому общее их число меньше чем [д/1969] • V1969 < 1969. 32.30. Начнем с правого края. Самый правый кубик черный. Действительно, если бы он был белым, то по условию в любом на- боре из одного кубика было бы не менее одного белого, т. е. все кубики были бы белыми. Аналогичное рассуждение показыва- ет, что первые 4 кубика справа черные (иначе среди любых че- тырех кубиков был бы белый и число белых кубиков было бы не меньше 25). Пятый кубик белый: в противном случае в любой пятерке было бы не более одного белого кубика, т. е. число белых кубиков не превосходило бы 20. Рассуждая так и далее, нетрудно доказать, что среди первых k кубиков справа белых ровно [тбб”] ’ Н° отсюда следует, что цвет каждого кубика определяется однозначно. (См. задачу 32.40.) 32.32. Докажите самостоятельно лемму: если через точку О провести три различные окружности одинакового радиуса г, то остальные три точки их попарного пересечения — вершины тре- угольника, радиус описанной окружности вокруг которого ра- вен г9 а точка О является точкой пересечения его высот. (Для доказательства леммы воспользуйтесь тем, что этот треуголь- ник равен треугольнику с вершинами в центрах трех проведен- ных окружностей.) Теперь для построения по заданным точ- кам А, В9 С четвертой вершины параллелограмма ABCD доста- точно провести пять окружностей. Сначала проведем две окруж- ности через точки А и В и третью — через точки В и С; пусть ее центр О| и она пересекает первые две окружности в точках Е и F. Проведем теперь еще по одной окружности через точки С и £ и точки С и F — они и пересекаются в искомой точке D. ABCD — параллелограмм, так как точки А, £, F, D лежат на одной окружности (с центром О2) и AB±£F, CDA.EF, AB = CD = OiO2. 32.33. Если дана последовательность х длины п9 то обратной к ней х назовем такую последовательность длины п9 в которой нули стоят на местах единиц у х9 а единицы — на местах нулей. Произведением ху двух последовательностей х и у на- зовем последовательность z, полученную поразрядным умноже- нием х и у. Доказательство утверждения задачи вытекает из следующих лемм. Лемма 1. Среди наших 2я ~| последовательностей содер- жится ровно одна из двух последовательностей: х или х(х — про- извольная последовательность длины п). Действительно, пред- 8 Заказ 247 225
положив противное и добавив к х и х произвольную последова- тельность у, получаем: x-x-z/==0 — противоречие с условием. В частности, последовательность 0..000..0 не принадлежит данной системе. Лемма 2. Если х и у — две последовательности из нашей системы, то последовательность ху также принадлежит нашей системе. Доказательство аналогично доказательству леммы 1 и проводится с ее использованием. Лемма 3. Пусть xi, х2......х^~ ’ ~ данные в условии после- довательности. Тогда их произведение — последовательность Х=х\ ... хг~' состоит из одной единицы и (п — 1)-го нуля. Дейст- вительно, Х^О по предыдущим леммам. Если X содержит не меньше двух единиц, то в этих двух разрядах каждая последо- вательность Xi содержит единицу и тогда, расставляя в остав- шихся (л —2)-х разрядах нули и единицы произвольным обра- зом, получим только 2Л-2 различных последовательностей, а не 2Л-1, как требует условие.- Лемма 3 тем самым доказывает, что все последовательности х{ имеют единицу в том единственном разряде, в котором стоит 1 У X. 32.34. Сможет. В самом деле, сторонников президента име- ется 200 000. Пусть самые мелкие группы составляют по 5 че- ловек, тогда для победы президента нужно, чтобы в группе было 3 его сторонника. Таким образом, 200 000 сторонников дадут (если их правильно распределить) 66 666 «выборщиков первого порядка» из общего числа 4 миллиона. Вновь разобьем эти 4 миллиона на группы по 4 человека. В них для большинства нужно также 3 человека, и Мирафлорес сможет получить 22 222-х «выборщиков второго порядка» из общего числа 1 мил- лион. Далее, деля на группы то по 5, то по 4 человека (порядок безразличен), он получит: 7407 выборщиков из 200 000; 2469 вы- борщиков из 50 000; 823 выборщика из 10 000; 274 выборщика из 2000; 91-го выборщика из 500; 30 выборщиков из 100; 10 вы- борщиков из 25; 3 выборщика из 4, обеспечив, таким образом, себе победу. (Подумайте, мог бы победить Мирафлорес, если бы число его сторонников было равно 38-52= 164 025.) 32.35. Опишем вокруг 1060-угольника окружность; его вер- шины разбивают окружность на 1000 одинаковых дуг, в кото- рых будем измерять длины хорд, опирающихся на эти дуги. Рас- смотрим треугольник, содержащий центр окружности. Одна из его сторон имеет длину, не меньшую —-—| +1 “334. К ней при- мыкает треугольник, у которого из трех сторон средняя по 334 величине сторона имеет длину, не меньшую —к ней при- мыкает треугольник, из трех сторон которого длина средней 334 стороны не меньше и т. д.; у восьмого треугольника сред- 226
няя по величине сторона не 334 меньше ——> 1. Легко по- 2е казать, что указанные 8 диа- гоналей различны по длине. 32.36. Пусть сначала первый мудрец вычеркнет все нечетные числа, за- тем второй — все числа, большие 512 (их оста- лось как раз 256), затем пер- вый — все числа, не де- Рис. 58. лящиеся на 4, а вто- рой — все числа, большие 256, и т. д. Тогда в итоге второй уплатит первому, как нетрудно видеть, 32. Придумайте сами обобщение этой стратегии для первого мудреца на случай, ког- да второй играет произвольным образом, чтобы доказать, что первый всегда может добиться разности не меньше 32, и об- общение стратегии второго мудреца на случай, когда произ- вольным образом играет первый, чтобы доказать, что он может добиться разности не больше 32. 32.37. Да. На рисунке 58 показано, как перегнуть треуголь- ник так, чтобы получить из него замкнутый отрезок, пройден- ный дважды. Аналогичным путем легко из отрезка получить шестиугольник. (Ср. с задачей 32.23.) 32.38. См. решение задачи 32.25. 32.39. Можно. Указание. Начните с крайнего ободка до- ски. Те клетки, которые неясно, как заполнить, проще всего за- полнять нулями (при этом в дальнейшем не возникнет никаких трудностей). 32.40. Возьмем k белых кубиков и 1969 — k черных и расставим их в ряд, располагая белые на местах (считая справа) с номерами —, 2--9", 3- , ... (нецелые числа следует округлять в сторону увеличения). Нетрудно показать, что такое расположение удовлетворяет условию. Полагая /?=0, 1, 2, .... 1969, мы получим 1970 расстановок. Можно показать, что других расстановок, удовлетворяющих условию, нет (см. задачу 32.30). 33.01 . Нельзя, так как белая шашка при взятии черной сдви- гается всегда на четное число вертикалей. Поэтому если она взяла одну шашку, то она всегда будет отстоять от второй на четное число вертикалей и не сможет ее взять. 33.02 . Воспользуйтесь тем, что среди чисел 1, 2, .... 99 нечет- ных больше, чем четных. 33.03. 53°, 63°, 64° (на 60° меньше углов из условия). 33.04. Указание. Класть гири парами (по одной на чаш- ку), причем класть более легкую гирю на перевешивающую чаш- ку весов. 8* 227
тельных, что 33.05. Указание. Занумеруем кот- теджи так, чтобы человек из первого переехал во второй, человек из второ- го — в третий и т. д. Найдется такое й, что человек из Л-го коттеджа переезжает в первый, и на этом цепь замкнется. Те- перь докажите утверждение задачи для первых k коттеджей, а затем рассмотрите остальные. 33.07. Пусть L и М — точки касания АЕ и DE с окружностью (см. рис. 59). Легко видеть, сравнивая отрезки каса- 4£ = KC + DM = 1. Отсюда EL — BK= = АЕ—\—число целое и ЕМ —КС—тоже целое. Поскольку EL = EM, отсюда вытекает, что В/( = КС = 0,5. 33.09 . Только при Л=1 или k = 2. Действительно, если иа левой чашке лежат гири ai, ..., ak общей массой S, а на пра- вой — гири frj, ..., bk общей массой S, то из условия следует, что 0<S —3^2(а/—-6/) при каждом./=1, ...» Л. Складывая k таких неравенств, получаем: k (S —-3)^2 (S — S), откуда Л^2. 33.10. Возьмем в качестве А того, кто проиграл больше всего партий, а в качестве В — того, у которого А выиграл. Докажите сами, что тогда удастся найти С, удовлетворяющего условию. 33.11. Зафиксируем произвольного короля и предположим для определенности, что он чаще ездил влево, чем вправо. Тогда нетрудно видеть, что любой король, живущий правее его, проехал за год больший путь. Отсюда сразу следует, что король, живущий не на краю, проделал не самый большой путь. 33.13. Докажем, что все цифры числа, кроме, быть может, первых 50-ти,— нули. Для этого возьмем 51-значное число А и заменим его последнюю цифру а нулем — получим число Д'. Числа А н Д', а значит, и их разность а кратны 250, откуда а=0. Следовательно, исходное число кратно 2999. 33.15. Указание. Не более чем за 7 шагов таракан вы- ясняет, в каком из четырех квадрантов находится Истина, а за- тем начинает двигаться параллельно сторонам этого квадранта и делает не более D шагов. 33.16. Указание. Воспользуйтесь тем, что суммарная мас- са гирь во всяком случае не больше 19*70=1330 г, и докажите, что существует не менее 100 различных по массе наборов, ко- торые составить не удастся. 33.17. Пусть МК и РЕ — эти хорды (см. рис. 60). Тогда Р/(.РЛ1 = (РЛ1,)2=(Л1Р,)2 = М£*Л1Р, т. е. PK = ML. Поэтому PL—MK. 33.18. Если бы все цифры этого числа были различны, то их сумма равнялась бы 0+1 +2+ ... + 9=45, т. е. это число де- лилось бы на 9. Однако ни одно число в цепочке на 9 не делится. 228
д A 8 о) 5) Рис. 61. 33.19. Проведем ориентированную прямую I, по разные сто- роны которой расположено по 100 точек (считаем эту прямую вертикальной). Все точки слева от нее покрасим в красный цвет, все точки справа — в синий. Проведем прямую tn, пер- пендикулярную I, и спроектируем на нее наши точки. Начнем теперь непрерывно поворачивать прямую I так, чтобы в каждый момент слева и справа от нее (по отношению к ориентации на /) было по 100 точек, и перекрашивать точки так, чтобы всегда слева от / они были красными, а справа — синими; вместе с I движется и перпендикулярная ей прямая m со спроектированны- ми на нее точками. В первый момент, когда в проекции на m сольются синяя и красная точки, дадим соответствующим точкам номера 1 и 101 и отбросим их. Затем отметим номерами 2 и 102 следующие две разноцветные точки, проекции которых на m сольются раньше всех, и т. д. Докажите, используя тот факт, что после поворота прямой I на 180° исходные красные точки станут синими, а синие — красными, что прямые (1, 101), (2, 102), .... (100, 200) попарно пересекаются. 33.20. Указание. Отметьте для начала одну строку (или столбец), а именно ту, в которой число крестиков минимально среди всех строк и столбцов. 33.21. Существует признак делимости числа на 999...99, аналогичный признаку делимости на 9: число делится на 999...9Э, в котором k девяток, тогда и только тогда, когда сумма его граней по k цифр (грани идут справа налево) делит- ся на 999...99. Например, 24 354 делится на 99, потому что 2-|-43-|-54 делится на 99. (Докажите этот признак сами.) Но легко видеть, что если у числа не более 8 цифр отличны от нуля, то сумма граней не может делиться на 999 999 999. 33.22. Возьмем две произвольные диаметрально противо- положные точки А и В. Очевидно, сумма расстояний от них до любой из отмеченных точек больше 2. Поэтому сумма расстоя- ний от одной из них до всех отмеченных точек больше 100. 33.23. Смогут. Для этого папа должен контролировать до- рожку АВ так, чтобы Коля не смог проскочить ни через узел А, ни через В (см. рис. 61, а). Тогда мама будет бежать за Колей 229
Рис. 62. С . Рис. 63. фактически по фигуре, изображенной на рисунке 61,6, и, ко- нечно же, поймает его. 33.24. 1970 разрезов. Указание. Сумма углов квадрата равна 2л, и легко сообразить, что каждый разрез увеличивает общую сумму углов всех частей еще на 2л. Таким образом, пос- ле k разрезов получится (А + 1) многоугольников, а сумма углов будет равна 2л (Л 4-1). У всех 30-угольииков сумма углов сос- тавляет 73 «28л. Остальные многоугольники, число которых равно (Л 4-1)—73, имеют сумму углов, не меньшую чем (Л —72)л. Отсюда получаем: 73-28 л 4-(&—72) л^2л (&4~ 1)> откуда /г>73-27—1 = 1970. Легко видеть, что эта оценка точна. 33.25. Расположив придворных по окружности в том порядке, в каком они следят друг за другом, и предположив, что их коли- „ л чество п четно, получаем, что придворный с номером — сле- дит за тем, кто следит за первым придворным, хотя им должен быть придворный с номером п. (На рисунке 62 изображен слу- чай л = 8.) 33.26. См. задачу 33.22. 33.27. Сторожа поймают обезьяну, если примут следующий план. Вначале они должны занять вершины А и В (см. рис. 63); очевидно, можно считать, что обезьяна находится в нижней ча- сти рисунка. Теперь один сторож должен спускаться вдоль АС, а второй контролировать отрезок АВ, чтобы обезьяна не проско- чила через А или В (убедитесь сами, что это возможно). Даль- нейшее просто. 33.28. 2500 деревьев. 33.29. Будем рассматривать только те способы, при которых получается 4 трехзначных числа и (80—4-3): 2=34 двузначных. Все такие суммы меньше чем 40004-3400 = 7400. Докажите, что существует более 10Q00 разрезаний указанным способом. 33.30. Указание. Предположив, что ломаную протащить можно, рассмотрите тот момент, когда отрезок, соединяющий концы ломаной, окажется под углом 45° к сторонам угла. 33.31. Пусть из деталей составлен некоторый замкнутый или незамкнутый путь. Очевидно, плоскость можно разбить на квад- 230
Рис. 64. Рис. 65. раты так, чтобы детали были вписаны в них, как показано на рисунке 64. Закрасим все квадраты в белый и черный цвет в шахматном порядке. Заметим теперь, что поезд, едущий по этой дороге, будет обходить все белые квадраты по часовой стрелке, а все черные — против часовой (или наобо- рот). Отсюда сразу следует, что требуемого условием пути не существует. 33.32. (2 2-у/2) м (см. задачу 33.30). Подходящим куском проволоки будет, например, сегмент окружности ширины I, стя- гивающий хорду длины (2-|-2 -^2) м. 33.33. Заметьте, что порядок, в котором мы меняем знаки, безразличен (проверьте это на примере одной строки и одного столбца). Поэтому можно считать, что мы вначале переменили знак в k строках, а затем в I столбцах. Но после этого число минусов будет равно (k + fyXOO—kl. Приравнивая его к числу 1970, мы видим, что нужно решить в целых числах, не превы- шающих 100, уравнение kl —100(Л4-04-1970 =0 или (100 —А)Х X (100—/)=8030, которое, легко видеть, неразрешимо. 33.34. 1699 разрезов (см. решение задачи 33.24). 33.35. Смогут. Для этого один паук должен контролировать ребро АВ, а другой — ребро СС так, чтобы муха не могла пройти ни через одну из вершин А, В, С, С (рис. 65, а). Тогда на оставшейся части куба нет ни одного замкнутого пути (рис. 65,6), так что муха не сможет уйти от третьего паука. 33.36. Площадь данной сферы равна 400л. Поэтому площадь описанного 19-гранника больше 400л, а площадь некоторой его грани больше — Пусть А — точка касания грани со сферой нВ — такая точка грани, что АВ>-$А (такая точка существу- ет, так как иначе вся грань лежала бы внутри круга радиу- са и имела бы площадь меньше 21л<^-л). Если О — центр шара, то ВО>V121 = Н. Отсюда легко следует, что дли- на отрезка, проведенного из В через О до пересечения с мно- гогранником, больше 21. (Можно доказать, что эта длина боль- ше чем 22.) 231
33.37. Пусть 10 101 010 101 = =Л, k=A-B. Если fe<1012, то В ^99. и утверждение очевидно. Пусть теперь fe>1012; перенесем первую его значащую цифру на 12 разрядов вправо. Это означа- ет, что из k вычтено число, про- порциональное 999...99 (12 девя- ток), так что оно не перестало делиться на Л. С другой сторо- ны, легко видеть, что число зна- чащих цифр при этом не увели- чивается: Отсюда сразу следует утверждение задачи. 33.38. Смогут, Разберитесь в этой задаче сами (ср. с зада- чами 23, 27, 35). 33.39. Воспользуйтесь тем, что остаток от деления на число вида (2fe + l) всегда не меньше 1, от деления на (4fe + 2)— не меньше 2, от деления на (8fe4-4)— не меньше 4 и т. д. 33.40. Достаточно 100 шиллингов (можно просто обводить квадратики на диагонали таблицы). Указание. Докажем, что 99 шиллингов не хватит. Предположим, что волшебная таб- лица заполнена какими-то числами хь хг....хюо так, как пока- зано на рисунке 66. Тогда, какой бы квадрат мы ни обвели, задав один вопрос, можно узнать только одно из чисел xi (то, которое стоит в последнем столбце обведенного квадрата). Поэтому после 99 вопросов нам будет известно только 99 чисел и сумма чисел по диагонали будет неизвестна. 33.41. Число 0. Указание. Если умножить любое число п на однозначное а так, что 10*, а (а —1)п< 10*, то, вычерк- нув первую единицу, мы получим число, меньшее исходного. Таким образом, описанной операцией можно уменьшить любое наперед заданное число. 33.42. На олимпиаде выше оценивались те работы, в кото- рых предлагалось размещение большего количества ковров. Наибольший известный ответ: 680 ковров. 33.43. Для того чтобы расположить кости согласно условию, нужно всегда располагать четное число рядом с нечетным. Но на костях домино 32 четных числа и только 24 нечетных. По- этому задача неразрешима. 33.44. Нельзя, так как сумма 1+2 + 3+... + 33 нечетна, а сумма чисел в каждой группе должна быть четной. 33.45. Легко поставить не менее трех переключателей в положение «верх» (сначала поставить вверх пару соседних, за- тем — пару диагональных). Если дверь в пещеру не открылась, значит, 4-й переключатель стоит в положении «низ». Тогда Али- Баба должен вставить руки по диагонали; если нащупан «низ», 232
его следует переставить в поло- жение «верх» и войти в пещеру, если же оба переключателя стоят в положении «верх», один из них ставится в положение «низ». После этого, очевидно, два со- седних переключателя стоят в по- ложении «верх» и два переключа- теля — «низ». Зате^ Али-Баба вставляет ру- ки по стороне квадрата: если оба переключателя в одном и том же положении, он щел- кает ими и открывает вход. Если же онй в разных положе- ниях, то он щелкает ими, а затем Рис. 67. вставляет руки по диагонали и щелкает обоими переключателями в последний раз. 33.46. Соединив точки, в которых расположены фотоаппара- ты, получим 1000-угольник (иначе сразу найдется фотоаппарат, из которого видны не все остальные). Но сумма всех углов 1000-угольника равна 180°-999 > 179° • 1000, т. е. каждый аппа- рат не может «обозревать» сразу все остальные. 34.01 . Пусть А и В — концы диаметра города. Соединим А, В ломаной, проходящей внутри города; город разобьется на две части. Никакой часовой не видит ни одного часового, находя- щегося в другой части города. Остается взять часового, нахо- дящегося в той части города, в которой не более 500 часовых. 34.02 . См. решение задачи 34.11. 34.03 . Последнее место. На рисунке 67 указана соответствую- щая таблица турнира. 34.04 . Проведем из каждой точки по 4 отрезка в 4 соседние точки (3 и 2 отрезка, если точка стоит на краю или в углу). Легко видеть, что число отрезков, проведенных из красных точек, равно числу отрезков из с и н и х точек. Теперь мы долж- ны выбросить отрезки, соединяющие красную точку с синей. Очевидно, при этом равенство не нарушится. 34.05 . Указание. Докажите сначала, что k(5,09070,)4- 4-Л (2,WOT01) четно. 34.06 . После каждой операции сумма всех чисел 25-уголь- ника удваивается. Сумма всех исходных чисел равна единице. За 100 операций, следовательно, эта сумма станет равной 2100. Поэтому одно из 25 чисел будет не меньше Ю2®> Ю20. 34.07 . Стороны многоугольника М являются средними ли- ниями треугольников, образованных соседними сторонами и ма- лыми диагоналями многоугольника Р. Поэтому периметр М ра- вен полусумме длин всех малых диагоналей и больше полупе- 233
Рис. 69. риметра «звезды» (рис. 68), со- ставленной из кусочков малых диагоналей. Периметр «звезды» в свою очередь превосходит пе- риметр Р; следовательно, пери- метр многоугольника М больше 24='- 34.08. Возможны все значе- ния п, отличные от 2* (fe^2). 34.09. Справедливо утверж- дение: разность двух чисел, составленных из одних и тех же цифр, кратна 9. Пусть из числа 2* переста- новкой цифр получилось число 2я и n>k. Тогда разность 2я —2* кратна 9, откуда и 2я-*—I делится на 9. Но 2я-*=|г<9, поскольку при перестановке цифр число не может увеличиться в 9 или более раз. Получили проти- воречие. 34.10. Первое решение. 2". 72=[2яа, 0<а<1. Если а<-£-, то [2я+1 -72]— чет- ное число. Если же а>—, то 2п+'-л/2 = 2[2п-^2]+2а и [2" + 1 -^/2] — нечетное число, причем {2а}<а. Умножая 2я-^2 на последовательные степени двойки, мы будем каждый раз уменьшать дробную часть результата, пока эта дробная часть не станет меньшей -i-. Второе решение. Докажем, что в по- следовательности а* бесконечно много четных чисел. Дейст- вительно, гели в этой последовательности с какого то места идут только нечетные числа, то их двоичная запись оканчивается на 1, а тогда в двоичной записи а* с какого-то места идут одни единицы. Это противоречит иррациональности -yj2. 34.11. Пусть отрезок Л1Л2 пересекает сферу в точках af, а2; отрезок А2Аз — в точках а2, а"; отрезок Л3Л4 — в точках аз, сП; ...; отрезок ЛП|ЛП — в точках aj-i, а", и, наконец, отрезок A-Ai— в точках ah. а" (рис. 69). Нам надо доказать, что из равенств Atal-Aia", А2и2 = А2а?, .... An-iah-i—An-iaZ-i следует, что А„йп ===АпОп. Рассмотрим плоскости треугольников OAiA2, ОАгАз,... и OAaAi 234
и в них — равнобедренные треугольники Oafa", ... , Oa»-ia?, Оа£ а". Все они равны между собой, откуда, в частности, следует равенство a"an-i=a,na,f. Применим теперь теорему о том, что если из точки X вне сферы проведен луч, пересекающий ее в двух точках Y и Z (считаем XY <.XZ), то произведение XY~XZ есть постоянная величина, не зависящая от того, какой луч проведен (а зависящая только от выбора точки X). Следовательно, ЛпаАХ XAncff =АпаХ >Ляап-1, откуда ЛяПя (АпОпЧ"а£а\)—АпО% (АпО%-j-OnOn— i). Если теперь предположить, например, что Апап<Апа", то левая часть последнего равенства будет меньше правой и получаем противоречие. Точно так же не может выполняться противопо- ложное неравенство. Тем самым утверждение задачи доказано. 34.12. Утверждать, что Петя лжет, нельзя. Простейший при- мер приведен на рисунке 70. Основная идея построения подобных примеров состоит в том, чтобы оказалось нечетное число уголков. Тогда при утере одного уголка оставшиеся уголки можно разбить на пары и каждую пару поместить в прямоугольник 2X3. 34.13. Положим yk=-^-k—1-; очевидно, Зуп— yn_t=n. Легко видеть, что |х*—у*| =-|-| и потому при больших k «5 условие Xkf^yit выполняется с достаточно высокой точностью. В частности, xim fsymi =985,250000. 34.14. fe=l. Указание. Отрезая несколько раз малыми диагоналями неостроугольные треугольники, получите остро- угольный треугольник со всеми k точками внутри него. 34.15. При правильной игре выигрывает первый игрок. Его стратегия: поскольку своим ходом он может взять I, 2, 3, 4 или 5 спичек, то каждым своим ходом независимо от хода второго игрока он оставляет ему число спичек, кратное 6. Поэтому через конечное число ходов он оставит второму игроку ровно 6 спичек и затем, после хода второго, возьмет ос- тавшиеся спички, чем закончит игру. 34.16. Существует. Нетрудно сообразить, а потом проверить умножением <в столбик», что таким числом является 111 111 111. 34.17. Повернем квадрат ABCD вокруг его центра на 90°. Тогда повернутый квадрат совме- стится с исходным, а прямые AHi, ВНг, СПз и DH4 перейдут в прямые АО, ВО, СО, DO, которые пересекаются по условию. 34.18. Число бактерий через k минут равно (« — £)• 2*. Поэто- му через п минут бактерий не останется. 34.19. Пусть на горизонталь- 235
ной прямой / три идущих подряд узла закрашены в разные цвета а, Ь, с (если таких трех узлов нет, то прямая I — искомая). Тогда легко видеть, что три узла над ними должны быть закрашены в цве- та с, d, а соответственно, а следующие три — опять а, Ь, с (см. рис. 71) и т. д. Теперь ясно, что любая из прямых I, II, III удовлетворяет условию за- дачи. Рис. 71. 34.20. Решение приведено на рисунке 72. Сначала размещаем прожекторы 1, 2, 3 и 4 требуемым в условии способом, а затем добавляем к ним нужное количество других прожекторов (5, 6, 7, ...). 34.21. Забудем на время, что наше число 29-значное, и оп- ределим, какое наибольшее число цифр оно могло бы иметь. Пусть k (Л) — число цифр, равных А. Из условия следует, что если цифры В и С симметричны относительно середины числа, то k(B)=C и k(C)=B. Но тогда эти цифры всюду должны стоять на симметричных местах; поэтому k(B)=k(C) и В=С. Отсюда сразу следует, что всегда й(Л)=Л, причем цифра, стоя- щая на 15-м (среднем) месте, встречается нечетное число раз, а все остальные цифры — четное число раз. Но тогда число цифр числа X не превосходит k (2)+fe (4)+fc (6) + fe (8)4- +fe (9)=2 + 4+6 + 8+9=29. По условию их ровно 29. Значит, в нашем числе ровно 2 двойки, 4 четверки, 6 шестерок, 8 вось- мерок и 9 девяток. Сумма его цифр равна 22 + 42+6* +82+92= = 201. 34.22. 501 (см. рис. 73). 34.23 и 34.35. Обозначим сумму цифр числа X через S(X). Несложно доказать, что для любых натуральных А и В выпол- няется неравенство S (Л+В)^5 (Л)+5 (В). Выведем из него не- равенство S (ЛВ)<5 (Л)*5 (В). Пусть A =aia2 ... ап. Умножая А на В «в столбик», мы должны сложить п «сдвинутых» на один разряд друг относительно друга чисел ап*В, ап_|*В, ..., ai*B. Поэтому Рис. 72. 236
S (ДВ)< S (2 а,В)< (2 cuY S (B)= S (Д). S (B). ii«I i = 1 Из этого неравенства следует, что: I) S(K)=S(1000K)=S(125-8/0<S(125).S(8/()=8-S(8K); 2) S(JV)=S(105^=5(2s-55A0<S(32)-S(55A0=5-S(5SAZ). Улучшить эти оценки нельзя, так как S(125)=8-S(1000), 5(32)=5-S(10s). 34.24. Можно. Будем производить противоположные опера- ции. Берем любое п. Если п оканчивается на 1, 2, 3, 5, 6, 7, 8 или 9, умножим его соответственно на 4, 5, 8, 2, 4, 2, 5, 6. Получим число Л1<10л, оканчивающееся нулем или четверкой, и тогда последнюю цифру можно будет откинуть. После этого число станет меньше п (проведите детальные рассуждения). В результате нескольких таких операций мы придем к однозначно- му числу — 0 или 4. С него-то и следует начать наш прямой про- цесс. 34.26. Указание. Рассмотрите произвольный треугольник, отсекаемый малой диагональю, и докажите, что углы при его основании равны между собой. 34.28. Можно. Указание. Разделим каждую сторону квад- рата произвольно (например, на равные части) и будем изменять длины частей следующим образом. Возьмем какие-то 2 отрезка на одной стороне и один удлиним, а второй укоротим с сохране- нием их суммарной длины. Это можно сделать таким образом, что в любом прямоугольнике, который составим из имеющихся 400 частей, эти 2 должны будут попасть на одну сторону (до- кажите, что есть лишь конечное значение длин первого отрезка, при котором наши 2 отрезка «имеют право» попасть на разные стороны). Затем берем другую пару отрезков и делаем то же самое. В конце концов окажется, что любая пара отрезков, принадлежащая одной стороне квадра- та, должна будет попадать на одну сто- рону прямоугольника. /\ 34.29 и 34.31. Число п равно квадро- / \ ту суммы всех заданных чисел. Действи- / \ тельно, так как по условию 2 х(Х;=0 и / \ п п п / \ 2 xf=n, то n=2 х?+2«2 лус)=(2 ж,)2. ~т^~К—ЦГ'лХ <=| <-1 i<j «=i Х/'-’чх/ Можно доказать, что п кратно 4 (см. по * » * • этому поводу задачу 22.25) и что п =/=16. * ’ Неизвестно, может ли п быть отличным 50tf от 4. Рис. 73. 237
34.30. Проведем через центр л-угольника прямую, не парал- лельную ни одной стороне и не проходящую ни через одну его вершину. Будем считать эту прямую числовой осью Ох; в каждой вершине поставим число х,9 равное по модулю расстоянию от вершины до оси Ох. Ни одно из xi не равно 0, все х4 различны, а сумма чисел в вершинах любого правильного 6-угольника рав- на нулю, как вторые координаты векторов, сумма которых равна 0. 34.32. Можно заранее считать набор чисел Ь\9 Ьъ9 ..., Ьп упорядоченным по убыванию; надо доказать, что макси- мальная сумма bk + aik принимает наименьшее значение, когда числа {a4J расположены в возрастающем порядке. Если для какого-то расположения чисел {a4J этот максимум достигается на сумме bs + ais и при этом o4s>a4/ для некоторого l>s9 то переставим ais и а4/ местами. Тогда обе суммы 6s+a<z и bt+ais не будут превосходить bs + ais, т. е. максимум сумм {Ьл + a/J после перестановки не увеличится. Выбирая этот новый максимум, проводим для него те же рассуждения и в конце концов получаем указанное расположение чисел {а4). 34.33. Нули на конце числа всегда можно не принимать во внимание. Игрок должен называть следующие числа. Сначала он должен число At удвоить и ответ 2А| = ai ... ал... ая (ал — средняя цифра) представить в виде суммы xi+Уь где *i = = ai ... алО ... 0, у! = ал4-1 ... ап, а затем назвать число В\ = =у-(Х|— у\). Тогда Л14-В1=Х|, Д1— Bi=yi9 и, таким образом, оба раза получается число с примерно вдвое меньшим числом цифр. Аналогично получаем числа (х2, {/2), (*з, Уз) и т. д., до тех пор пока не получится однозначное число. А однозначное число не более чем за 5 вопросов можно превратить в 1 или 10 (проделайте это сами). Итак, нам понадобится не более (log2 10004-3)4-5^18 вопросов. 34.34. Существует. Указание. В случае двух прямых проек- ции исходных точек стремятся к концам кратчайшего отрезка, соединяющего пары точек на прямых. 35.01 . Предположим, что не все числа равны между собой — пусть ai>ci2 (случай ai<a2 рассматривается аналогично). Воспользуемся тем, что если степени равны, а основания раз- личны, то больший показатель имеет меньшее основание; тогда О| >» О2 О2 <С ОзГ=> Сз S> 04 => ... => 016 <С 017 => О|7 > О1 => fll <С а2. Первое неравенство в этой цепочке противоречит последнему, стало быть, исходное предположение неправомерно, откуда at = = ... =О|7- Замечание. При четном количестве чисел утверждение задачи неверно. Придумайте контрпример. 35.02 . Из любых трех делегатов всегда найдутся два, кото- 238
рые могут поговорить друг с дру- гом. Отбросим их, тогда для оставшихся делегатов справед- ливо это же утверждение. После нескольких таких операций мы придем к случаю четырех де- легатов, для которого утвержде- ние легко проверяется. 35.03 . Так как 13 — нечет- ное число, найдутся две соседние вершины одного цвета. Зануме- руем вершины по часовой стрел- ке от 1 до 13. Пусть выделенные вершины одного цвета имеют при такой нумерации номера 2 и 3. Тогда какие-то три из вершин 1, 2, 3, 4 и 9 лежат в вершинах равнобедренного треугольника и имеют одинаковый цвет. 35.06. Очевидно, что точки Х\, Хг, Хз, .... Хп_| расположены на одной окружности (АА' — хорда этой окружности), поскольку углы между соответственными прямыми равны между собой (рис. 74). 35.07. Разобьем монеты на две группы по 500 монет и срав- ним их массы (I взвешивание). Возможны 2 случая: А. Одна из чашек весов (скажем, правая) перевесила. Следо- вательно, фальшивые монеты есть (их одна или две) и все они лежат на одной чашке (если бы они лежали по одной на каждой, то весы остались бы в равновесии). Разобьем теперь более тяжелую группу на две по 250 монет и сравним их (II в з в е- шивание). Если одна из чашек перевесила, то фальшивые монеты находятся среди этих пятисот и, следовательно, т я- ж е л е е — в этом случае III взвешивание уже не нужно. Если же чашки в равновесии, то либо фальшивые монеты среди остав- шихся пятисот (и легче настоящих), либо они разделились: по одной среди групп по 250. Чтобы выяснить, какой из этих случаев имеет место, одну из групп в 250 монет делим пополам и сравниваем их (III взвешивание). Если весы в равнове- сии, имеет место первый случай, если нет — второй. Б. При I взвешивании чашки в равновесии. Значит, фаль- шивых монет имеется 0 или 2 (по одной на каждой чашке). Опять разбиваем одну из групп на две по 250 монет (II взве- шивание). Если чашки опять в равновесии, то на чашках четное число фальшивых монет, а так как их не более од- ной, значит, их чет вовсе. Если же одна из чашек перевесила, то всего фальшивых монет 2 и ровно одна из них лежит на одной из чашек. Разделив более тяжелую группу поползи, мы третьим взвешиванием легко узнаем, тяжелее ли фальшивая монета или легче. Замечания. 1. Существенно для решения, что число монет делится на 8; в противном c/iyiae трех взвешиваний не хватило бы. 2. Если бы потребовалось 239
в условиях задачи узнать, сколько имеется фальшивых монет, то понадобилось бы ие менее 8 взвешиваний. 3. Легко заметить, что три взвешивания, описан- ные выше, можно было бы осуществлять, не обращая по ходу дела внимание на то, какая чашка перевешивает: разложить монеты по 500, потом половину их — по 250 и, наконец, разложить половину половины. Мы раскладывали более тя- желую группу только для определенности. 35.08 . Ясно, что 50 чисел (2, 2, .... 2) удовлетворяют условию задачи. С другой стороны, 49 чисел (или меньшее количество) можно было бы разбить на 7 групп по 7 чисел; сумма каждой группы была бы не больше 14, а общая сумма S^7-14=98. 35.09 . Треугольники ADC и ВЕС подобны по двум углам. Если Л—коэффициент подобия, то АС = "К'ВС и —BC=DC~ 1 =к-ЕС =--'к-АС, откуда Л=1 и АС=ВС. Тогда в треуголь- нике ВЕС сторона ЕС, лежащая против угла в 30°, равна половине основания ВС. Поэтому Z. ВЕС=90° и Z.C=60°. Этого уже достаточно. 35.10. Надо доказывать более сильное утвержде- ние: пять точек К., Е, Н, М, С лежат на одной окружности. Так как КСНЕ и СМНЕ — равнобочные трапеции, то обе они вписаны в свои окружности. Точки С, Е, и Н, лежащие на ка- кой-то третьей (заранее неизвестной) окружности, лежат и на первых двух. Значит, все три окружности совпадают. 35.11. Указание. Докажите, что a*m = a*i +(aim—ап). 35.12. Расположим высоты деревьев в порядке убывания и в этом же порядке соединим основания деревьев. Длина получен- ной (быть может, самопересекающейся) ломаной не превосходит суммы разностей высот соседних деревьев, а значит, и высоты максимального дерева. Поэтому достаточно обнести с двух сто- рон полученную ломаную забором, длина которого не будет пре- восходить 1004-100=200 м. 35.13. Первая сумма больше второй на m-п. Действительно, докажем, что после отбрасывания одного крайнего слагаемого в каждой сумме (т. е. чисел /пил) оставшиеся суммы Si и $2 уже будут равны. Рассмотрим для этого все целые точки в пря- моугольнике /пХл, левый нижний угол которого расположен в начале координат О:1^х^/п—1, l^y^n—1. Всего их (m — 1) (л — 1), причем ровно половина — над диагональю, выходящей из О, и ровно половина — под ней (на диаго- нали нет целых точек в силу взаимной простоты т и л). Но количество целых точек с абсциссой k равно [ft(то же верно и для ординаты у). Следовательно, S|=S2=-|-(/n—1)(л—1). 35.14. Занумеруем проспекты от 1 до 10 сверху вниз. Пусть первый полицейский находится на первом перекрестке, второй — на втором, ипредположим, что гангстер находится в верхней полосе. Тогда первый полицейский, пройдя по своему проспекту не более 100 кварталов, обнаружит гангстера. Если же этого не 240
произошло, то, следовательно, гангстер находится не в первой по- лосе и полицейские могут спуститься на одну полосу вниз. Контро- лируя теперь вторую и третью полосы, они гарантируют, что ганг- стер не попадет на первую полосу. Действуя, как и раньше, и опускаясь каждый раз (если это нужно) на одну полосу вниз, полицейские в какой-то момент обнаружат гангстера. 35.15. Все жители разбиваются на два множества, причем любые двое жителей из одного множества дружат между собой, а из разных — не дружат. Затем, ссорясь в одном множестве, каждый житель будет переходить в другое множество. Это будет происходить до тех пор, пока все жители не станут друзьями. 35.16. Предположим, что в последовательности не более од- ного составного числа. Следовательно, все числа, начиная с не- которого а, простые. Что же приписывается к числу а? При- писывать четные цифры или цифру 5 нельзя, а приписать 1 или 7 можно не более одного раза, так как приписывание этих цифр увеличивает на 1 остаток от деления на 3. Значит, с какого-то места будет приписываться только цифра 3. Если до этого места было получено простое число р, то, приписав к нему не более р троек, получим число, кратное р, так как из чисел 3, 33, ..., 33... 3 (р троек) хотя бы одно кратно р. 35.17. Очевидно, все грани тетраэдра равны; обозначим че- рез S площадь грани. Примем одну из них за основание и спроецируем на нее три остальные — они покроют ее. Отсюда S =S cos a-f-S cos 0 + S cos y; cos a + cos 0+cos y = 1, где a, 0, у — двугранные углы при основании. Отсюда легко следует, что сумма косинусов всех двугранных углов равна 2. 35.18. Указание. Множество Т есть пересечение п полу- плоскостей, ограниченных сторонами л-угольника. 35.20. Докажем сначала, что в точке О диагонали делятся пополам. Если a, 6, с, d — длины четырех отрезков ОА, ОВ, ОС. OD и с^а. d^b. то, отложив на отрезке ОС отрезок ОМ длины а, а на отрезке OD отрезок ON длины 6, получим параллелограмм ABMN. разделенный на 4 треугольника того же самого пери- метра — противоречие. Итак, ABCD — параллелограмм, а из сравнения периметров двух смежных треугольников следует, что он ромб. 35.21. Указание. Проведите через вершину треугольника, образованного прямыми 6, с, d, прямую, параллельную прямой а. Воспользуйтесь тем, что в треугольнике средняя линия парал- лельна основанию. 35.22. Одно из 12 последовательных натуральных чисел крат- но 12. Сумма делителей этого числа превосходит это число. 35.23. Предположим, что рыцарь странствует настолько дол- го, что по какой-то дороге он проехал не менее 6 раз. Тогда по ней он проехал не менее 3 раз в одном направлении, следо- вательно, по одной из двух дорог, на которые можно затем свернуть, он двигался дважды в одну сторону. Но тогда весь 9 Заказ 247 241
его дальнейший путь будет совпадать с уже пройденным (до вступления на эту дорогу) и будет проходить через тот замок, из которого рыцарь выехал в первый раз. 35.24. Обозначим МВ = а, BK = bt КС = с, CA=d, АМ = е. Так как площадь треугольника ВМК больше площади треуголь- ника МКС, а площадь треугольника ВМК больше площади треуголь- ника AM К. то Ь>с, а>е. Допустим, что ---------<4-, т. е. J c+d±e 3 За + ЗЬ+ d + + b + Тогда 2a-|-2b<d, откуда я + е + + b+c<2a + 2b<d и АВ + ВС<АС, противоречие с нера- венством треугольника. 35.26. d = d (af - be) = adf-bed=(adf - bcf)+(bcf - bed) = — / (ad~bc) + b (cf — ed)^f* 1 + b-1 =f + b, что и требовалось доказать. 35.27. Фиксируем первую часть дорог и заданное односто- роннее движение на них (т. е. данную в условии первую раз- метку); дороги покрасим в красный цвет и сориентируем их соответствующими стрелками. Задача решается в два этапа. На первом будем увели- чивать число красных дорог в разметке, добавляя (указанным ниже способом) новые красные дороги со стрелками так, что- бы при каждом новом добавлении выполнялось условие задачи. Мы прекратим добавлять новые красные дороги, когда достиг- нем следующего: для любых двух перекрестков А и В, непосред- ственно соединенных красной стрелкой, идущей из Л в В, су- ществует также и путь из В в А, идущий только по красным стрелкам (цикличность красной разметки). На втором эта- пе мы укажем искомую ориентацию дорог, удовлетворяющую утверждению задачи, чем и завершим доказательство. Первый этап. Пусть перекрестки А и В соединяются красной стрелкой Л-^В, но из В в А нет красного пути, т. е. на любом пути, ведущем из В в Л, не все стрелки красные, имеются и «пустые» дороги. Возьмем любой такой путь из В в Л. Все красные дороги, содержащиеся в этом пути, имеют согласованную ориентацию со стрелкой Л-^В (так как из В в Л есть проезд по выбранному пути); на «пустых» дорогах введем такую же (согла- сованную с Л-^6) ориентацию и покрасим их в красный цвет. Тогда из В в Л можно проехать по красным стрелкам. В силу конечности числа дорог в городе мы за несколько шагов добьем- ся цикличности красной разметки, т. е. получим «островки» из красных циклов. Второй этап. Дороги, не попавшие в красные циклы, закрасим зеленым цветом; на них пока что двойная ориентация. Оставим на всех зеленых дорогах по одной стрелке, отвечающей второй разметке, о которой сказано в условии задачи. Таким образом, те- перь уже во всем городе введено одностороннее движение. Докажем, что, двигаясь по дорогам согласно введенной ориен- тации, можно из любого перекрестка Л попасть в любой другой 242
перекресток В. Действительно, возьмем тот путь из А в В, ко- торый отвечает второй разметке из условия задачи. Теперь он состоит из красных и зеленых стрелок. По зеленым стрелкам мы всегда будем двигаться в нужном направлении, а по красным — не всегда. Если на дороге ХУ, входящей в рассматриваемый путь А ... XY ... В, отмечена красная стрелка из У в X (т. е. дви- гаться по дороге XY нельзя), то пойдем по пути из красных стрелок, ведущем из X в У и дополняющем стрелку У->Х до цикла согласно построениям первого этапа. Полученный путь из Л в В — искомый. 35.28. Рассмотрим все дроби, сократимые и несократимые, у которых числитель и знаменатель не превосходят 100, их 1002. Если HOD (a, Ь) = п, то дробь у- сократима на п, при этом сц 100 полученная несократимая дробь — такова, что а\ и , 100 г, / юо \ ~ « , т. е. в количестве > несократимых дробей мы уч- тем и эту дробь. Следовательно, искомая сумма равна количеству всех рассмотренных вначале дробей, т. е. равна 10 000. 35.30. Пусть в пятиугольнике ABCDE угол С не тупой, а дли- ны всех сторон равны 1. Проведем диагонали АС и СЕ, которые разобьют пятиугольник на три треугольника, причем два из них, ДЛВС и &CDE, равнобедренные с углами при вер- шине, меньшими 120°. Значит, третий оставшийся треуголь- ник АСЕ остроугольный, причем угол при его вершине С меньше 90° — 2 *30° = 30°, а боковые стороны (они одновремен- но являются основаниями треугольников АВС и CDE) имеют длины, меньшие -\/3. Но тогда из теоремы косинусов легко пока- зать, что его основание АЕ имеет длину меньше 1, что противоре- чит равенству ЛЕ=1. Значит, угол С пятиугольника тупой. 35.32. 60 станций. Указание, ресекающиеся фигуры, изобра- женные на рисунке 75, а, так, что- бы они заполнили весь квадрат 20X20. Таких фигур не меньше 60, и в каждой из них находится по крайней мере одна станция. Значит, станций не меньше 60 (ровно 60 можно расставить с по- мощью той же конструкции; рис. 75, б). 35.33. Если выполнено равен- ство, данное в условии, а числа а, Ь, с, d рациональные, то выпол- нено и равенство: (a — b^j2)2n4- + (c-d ^2fn = 5—4-fi. Расставить по спирали непе- Рис. 75. 9* 243
Но 5-4л/2<0, а левая часть положительная. Противоречие. Следовательно, исходного равенства быть не может. 35.34. а) Возьмем прямую а и ближайшую к ней точку А пересечения двух других прямых b и с. Треугольник, образован- ный прямыми а, Ь, с, не пересекает никакая другая прямая, ина- че нашлась бы более близкая к прямой а точка пересечения пря- мых, чем точка А. Следовательно, к любой прямой примыкает треугольник, а так как каждый из них при подсчете общего числа учитывается трижды, то таких треугольников не менее 3000X х4-=1000. б) Имеется не более двух таких прямых, что о все точки пересечения остальных прямых лежат с одной стороны от каждой из них. Действительно, если таких прямых больше, выберем три из них. Они разобьют плоскость на 7 частей, и все точки пересечения остальных прямых должны лежать только в одной из этих частей, чего не может быть (четвертая прямая пересекает две области). Итак, у 3000 — 2 = 2998 прямых мы дополнительно к пункту а) насчитаем еще по треуголь- ник у, т. е. насчитаем 2998 + 3000 треугольников. Однако каж- дый из них будет просчитан трижды, откуда — «5 2 —Поскольку N целое, то /V^2000. <5 35.35. Длины сторон и высот спроектированного треугольника Л|Й|С1 не превосходят длин сторон и высот соответственно проеци- руемого на плоскость П треугольника АВС. Теперь следует рас- смотреть (сделайте это сами) два случая: а) какие-то два угла ДАВС не меньше соответствующих углов 2\Л|В|Сг, б) наоборот, два угла &АВС не больше соответствующих углов £±А\В\С\. 35.36. Докажем, что если имеется набор невозраста- ющих чисел а\ ^ач^аз^ - ^ап^0 и выполнены все условия задачи, то a\Xi + a2X2 + ..- + anxn>aiyi + а>у2 + ...+апуЛ. Действительно, существуют такие неотрицательные „1 числа Ь|, ..., ЬПу что an~b\, art-i = bi+b2t ...» а2 —2j bi9 aj = п /=1 =S bt. Имеем: • aixi + ... +апхп = & b,) ai+(S bi) x2 + ... + fr|Xn = bi (S x,)+ i=l i=l /-I П- I П •! + 62 (X Ai)+ ... + ЬЛХ| ^b\ (2jy <)+••.+ bnyi =(21 bi) y< + -.• + «—i i=i ii + b iyn = aiiji +... + anyn. Доказанное утверждение применим k раз. 1) Полагаем ai=Xi~\ Тогда x^ +*2 +... +Ллх\ !yi+...+xn !уп 244
2) Полагаем а,=х? 2у,, получаем: j4 *!Zi+—2«/? + k) Наконец, положив a, = t/f_|, получим: xiy* 1 + ••• -f-JM/n 1 4“ ••• "bf/n- Объединяя все k неравенств в одну длинную цепочку, по- п п лучасм утверждение задачи: i = I с — I 35.37. 20 000. Если Х1^...^хгоо—числа первого набора, а ^^2оо— второго, и при своем ходе игрок В берет карточки xi, ...» xioo и £/|, ..., £/юо» то он обеспечит себе (xi —Xioi) + ...+ +(хюо — X2oo)+(f/i — f/ioi)+ .+О/100 — £/2oo)>2O 000. Но игрок Л, положив первым ходом числа 1, 2, ..., 100, 201, ..., 300 в первый набор, а остальные — во второй, после каждого хода В будет возвращаться к этой же ситуации; при этом Cfi —Сл =20 000. 35.38. Индукция по л. Для п = 3 все такие числа легко 0 112 3 выписать: —, —, —, —, а затем проверить справедливость утверждения. Пусть утверждение верно для п — L Новый набор из п чисел получится из предыдущего набора добавлением не- которых чисел. Если соседние числа в новом наборе являются соседними и в старом, то для них все доказано. Если же дробь k о а с ~ — оказалась соседней с дробями — и — старого набора, то до- кажем, что A—kb — ар=\ и В —ср —kd= I • Предположим противное: пусть максимальное из чисел А и В больше 1. Тогда b + d<b-B + d'A=p (bc—ad)=p\ b-\-d<.p. Следовательно, во всяком случае, £44=/=—. но b+d р а»_а + с__с —<b+d<-'d' и поэтому ----не соседняя дробь с -у- и у----противоречие. Замечание. Эту задачу можно решить с помощью числовых решеток на .плоскости; такое доказательство имеется в книге Г. С. Кокстера «Введение в геометрию» (М., Наука, 1966 г., с. 303 — 304). 35.39. Не всегда. Будем рассматривать числа по моду- лю 10 (т. е. рассматривать последние цифры этих чисел). Возьмем набор «все нули» и подсчитаем, сколько наборов мы смо- жем получить, исходя из него с помощью указанных в условии операций. Клеток размером 3X3 и 4x4 имеется (8 —3+1)24- 245
Рис. 76. Рис. 77. -Ь(8 — 4-|-1)2 = 61, т. е. искомых наборов не больше 10г>|. Однако всевозможных различных наборов существует 10Ъ4. Сле- довательно, существует набор, который с помощью указанных операций из нулевого набора получить не удастся. Именно этот набор и следует принять за исходный. 36.01. Достаточно «раздвоить» границы всех стран, как это показано на рисунке 76. 36.02. Не может. Если число является точным квадратом, то оно оканчивается четным числом нулей и их можно не рас- сматривать. Оставшееся число имеет вид 2А, где А — число из 600 троек и некоторого числа нулей, оканчивающееся на 3. Итак, А нечетно, значит, 2А — не точный квадрат. 36.03. Среди пяти точек существуют такие две (обозначим их А и Я), что оставшиеся три точки С|, С2, С3 лежат по одну сторону от прямой АВ. Без ограничения общности будем считать, что /LACiB< Z~AC2B< ААСзВ. Тогда точка С\ лежит вне окружности, проходящей через точки А, В и С2, а точка Сз— внутри нее. 36.04. Умножив данное уравнение на хур, получим: рх+ру = =ху или (х — р) (у — р) = р2. При простом р отсюда следует, что: 1) х — р=1; у — р = р2, или 2) х—р = р; у — р=р, или , 3) х — р = р2; у — р = 1. Если же р составное, то р2 раскла- дывается на множители и другими способами. 36.05. Не может. 36.07. Сначала все многоугольники разобьем на треуголь- ники так, чтобы никакая вершина треугольника не лежала внут- ри стороны другого треугольника. При этом все внутренние треугольники будут граничить с тремя треугольниками (т. е. с не- четным числом). Остался случай, когда одна из сторон тре- угольника лежит на стороне квадрата. В этом случае произво- дим разбиение, указанное на рисунке 77. 36.08. Если а^Ь — целые числа, то ак — Ьк делится на а — Ь; поэтому Р (а) — Р (Ь) делится на а — Ь. Пусть Р (<21) = Р (аг) = = Р(а3) = 2, Р (Ь) = 3. Тогда Р (b) — P (at) — 1 делится на b — at (/=!, 2, 3), откуда |а< — Ь| = 1. Но это равенство не может вы- полняться для трех различных чисел а/. 36.09. Пусть А — произвольная станция метро. Тогда следует закрыть такую станцию метро, до которой от А ехать дальше всего (по числу остановок). 246
36.10. Средняя сумма отпечатанных на доске чисел равна 3,5*99 = 346,5. Если на пути движения кубика два раза встре- чается число а, то между ними обязательно встречается и число 7—а. Чисел вида а и 7—а может быть либо поровну, либо число их может отличаться на 1. Следовательно, сумма отпеча- танных чисел может отличаться от средней суммы этих чисел на 4,5 в ту или другую сторону. Поэтому максимальное значение суммы равно 351, а минимальное — 342. 36.11. Воспользуемся тем, что ab^a+b при а^2, 6^2. Обозначим через f(k) число, получающееся из числа k одной операцией. Тогда при любом k. При этом если k — четное и больше 7, то f (Л)=24-Д24-—4-Р«4-1 С34-р2*...-ря= =3-|—|-<Л. Следовательно, для любого Л^7, т. е. рассматриваемая последовательность принимает значения толь- ко из отрезка [1, Л4-1]. Значит, она принимает какое-то значение дважды, а потому является периодической. 36.14. Пусть существует выпуклый многогранник, все грани которого — многоугольники с р а з н ы м числом сторон. К грани с максимальным числом сторон п примыкает п граней с числом сторон от 3 до л—1. Противоречие. 36.15. а) В любом положении тумблеров состояние любой фиксированной лампочки можно изменить переключением ка- кого-либо одного тумблера, поскольку переключениям разных тумблеров соответствует изменение состояний у разных лампочек. б) Приведем тумблеры в такое состояние, чтобы все лампоч- ки погасли. Это состояние каждого тумблера назовем «выклю- ченным», а противоположное ему — «включенным». Если теперь включено k тублеров, то горят лампочек. И если r<.k, то можно переключить так г тумблеров, чтобы все лампочки по- гасли (см. а), хотя при этом не все тумблеры выключены. Противоречие приводит нас к равенству г=Л. в) Занумеруем тумблеры числами 1, 2, ..., п, а затем зану- меруем лампочки так, что если включен <-й тумблер, а осталь- ные выключены, то горит i-я лампочка. Докажем, что если вклю- чены тумблеры it, .... ik, то горят лампочки с теми же но- мерами. Из б) следует, что горят k лампочек. Пусть одна из них имеет номер /, отличный от it, .... 4- С помощью k — 1 переключений можно погасить остальные k—1 лампочек (см. а). В результате включен один из тумблеров it,.... ik, а горит лампоч- ка /. Это противоречит выбору /, и утверждение доказано. 36.16. Не может. Пусть 01020304— искомое четырехзначное число и <11020304 — 04030201 = 1008. Начиная вычитать с конца, мы видим, что at = 04+2, аз=аг 4-1. Но тогда разность должна иметь цифру сотен, равную 9, а не 0. 36.17. Пусть О — точка пересечения высот треугольника АВС. Тогда треугольник АВС разбивается на 3 четырехугольника, 247
ъ 7 ъ % Рнс. 78. вообще гово- каждый из которых покрыт соответствую- щим кругом (даже кругом меньшего ра- диуса). 36.18. а) Можно; б), в) ря, нельзя. 36.20. Пусть наименьшее расстояние от произвольной точки кляксы до ее границы принимает наиболь- шее значение л в точке Л, а наибольшее расстояние принимает наименьшее значение г2 в точке В. Тогда круг радиуса л с цент- ром А лежит внутри кляксы, а круг радиуса г2 с центром В содержит кляксу. По условию Г| = г2 = г, значит, точки А и В совпадают и клякса имеет форму круга радиуса г. 36.25. 4fe4~l (см. рис. 78). Указание. К каждой черной клетке примыкает не более четырех белых. Докажите, что не менее k—1 белых клеток примыкает сразу к двум черным и, сле- довательно, были только что учтены дважды. 36.27. 1972 раза. Указание. Любое число п будет выписа- но столько раз, сколько имеется чисел, меньших п и взаимно простых с п (так называемая «функция Эйлера» <р(л)). Но 1973 — простое число.___ 36.29. Число х = ~4 удовлетворяет уравнению х + ±-= =п. Положим xm+(^ = fem, тогда fem + ! = fem^x+~-^ — fem-l = = n-km — fem-1- Поэтому раз целое, то и fe = xm+^r це- лое при любом ш. Но тогда xrn=* + ^~~4 . 36.30. Пусть дан трехгранный угол. Отложим на его ребрах векторы а, В, с единичной длины. Тогда векторы a-^Bt с + а направлены по биссектрисам плоских углов. Скалярные произведения (a+S)-(b + c)9 (5 + с)-(с+а), (с+а)-(а+5) равны между собой и равны 1+ Это и означает, что углы между биссектрисами будут одновременно острыми, прямыми или же тупыми. 36.31. Указание. Проверьте сначала, что окончательная раскраска не зависит от порядка выхода маляров (достаточно проверить это для двух маляров). 36.33. Мысленно «выпрямим» траекторию льва, вращая арену цирка последовательно относительно точек поворота льва. При таком выпрямлении центр О арены каждый раз перемещается на отрезок, не больший произведения соответствующего угла по- ворота льва (выраженного в радианах) на 10 м (так как центр отстоит от льва не далее 10 м). Всего точка О переместится на величину, не превосходящую сумму всех углов поворота льва, умноженную на 10 м. Но нетрудно видеть, что длина отрезка, 248
соединяющего начальное положение точки О с конечным, не меньше 29 980 м (так как 30 000 — 20 = 29 980), откуда и следует утверждение задачи. 37.01 . Обозначим Ю2'000 через а, а 29744~1 — через л. Тогда наше число равно ап +1 и оно кратно а4-1. 37.02. Докажем, что если площадь треугольника больше 1 и он расположен в круге радиуса 1, то центр круга находится строго внутри треугольника. Действительно, все высоты треуголь- ника больше 1, поскольку каждая сторона не больше 2 (диаметра круга), а площадь треугольника больше I. Следовательно, дан- ный треугольник является пересечением трех полос, ширина каж- дой из которых строго больше 1, а поэтому содержит внутри себя центр круга. Если же в круг радиуса 1 помещены 2 тре- угольника, площади которых строго больше 1, то они оба содер- жат внутри себя центр круга, а стало быть, пересекаются друг с другом. 37.03 и 37.08. Обозначим через п число спиленных пар зу- бцов (л = 6 или л = 10). Тогда у каждой шестеренки п2 — л-|-2 зубцов. Всего существует л2 — л 4~ 1 таких поворотов верхней шестеренки относительно нижней, при которых все зубцы обеих шестеренок оказываются совмещенными. Назовем дыркой то место шестеренки, где отсутствует зубец. Рассмотрим произволь- ную дырку нижней шестеренки. При л—1 положениях верхней шестеренки (кроме исходного) над этой дыркой оказывается дыр- ка верхней шестеренки. Но дырок на нижней шестеренке л, поэто- му из л2 —л 4-1 поворотов верхней шестеренки только при л (л—1) из них наблюдаются совпадения дырок обеих шестере- нок. Поскольку (л2—л 4" О—л (л— 1) = 1, найдется такой поворот верхней шестеренки, когда совпадения дырок не будет. Этот поворот искомый. 37.04. По условию числа а, Ь, с удовлетворяют неравенствам а + Ь>с9 Ь + Оа9 с+а>Ь. Очевидно, —|—> , , *7-. , а+с (а + Ь)+(а + Ь) Т"?—> / . хл Г/ , л » отсюда следует, что —!—|--г-5—>—гг- Ь-]-с (а4-Ь)+(а-|-Ь) J а+с 1 b+с а + Ь Аналогично доказываются остальные два неравенства треуголь- ника. 37.05. Разобьем полоску ши- риной 1 вокруг многоугольника на прямоугольники, построенные на- его сторонах, и «уголки» около вершин (рис. 79). Из «уголков» составим многоуголь- ник, подобный данному: его сто- роны равны разностям сторон первоначального и полученного многоугольников, а углы равны 249
Пятнадцати- угольник Семнадцати- угольник б) б) Рис. 80. соответственным углам исходного многоугольника. Но в послед- ний многоугольник, очевидно, можно вписать окружность (ра- диуса 1). Из этого следует утверждение задачи. 37.07 . Предположим, что на каждой прямой, проходящей че- рез стороны 13-угольника, лежит не менее двух сторон 13- угольника. Тогда количество этих прямых не превосходит 6. Каждая из них пересекается поэтому не более чем с 5-ю другими, и, следовательно, на каждой прямой лежит не более 5-и вершин 13-угольника. Но стороны, лежащие на произвольной прямой, не пересекаются и не имеют общих вершин. Значит, их не более двух на этой прямой и тогда сторон должно быть не более 12. Однако их 13 — противоречие. Отсюда следует, что на одной из прямых расположена только одна сторона 13-угольника. Построение п-угольников при четном п с требуемым свойством видно на рисунке 80, а, а построение п-угольников при нечетном п можно начатье 15-угольника (рис. 80,6), а затем отрезать четное число углов так, как это показано на рисунке 80, в, 37.09 . Нельзя. Двигаясь по отрезкам диагоналей, мы прохо- дим поочередно через вершины и центры граней. Но у куба 8 вершин и только 6 граней. 37.11. Центр шестиугольника принадлежит одному из 6 имею- щихся треугольников, и у этого треугольника все стороны не меньше 1, поскольку каждый его угол при единичном основании не меньше 60°. Несложно показать, что один из двух треуголь- ников, имеющих общую сторону с рассмотренным треугольником, также имеет стороны, не меньшие 1. 37.12. 197 точек. См. решение задачи 37.16. 37.13. 4 или 5. Указание. Известно, что в квадрате и пра- вильном пятиугольнике все диагонали равны. Предположив, что в п-угольнике, где все диагонали равны, возьмем его сторону АВ и пересекающиеся диагонали AD и ВС (вершины С и D несоседние с А и В). Тогда AD + BC>AC + BD, что противоречит равенству AC = CB = BD = DA. Итак, случай невозможен. 37.16. Обозначим через А и В точки, расстояние между ко- торыми максимально. Соединим точку А со всеми точками, 250
кроме В. Середины полученных п —2 отрезков не совпала- ю т (иначе совпадали бы вторые концы отрезков) и лежат в круге радиуса -~-АВ с центром в точке А. Аналогичные рассуждения, проведенные для точки В, дают еще п —2 середины, лежащие в круге радиуса -±-АВ с центром в В. Построенные два круга с центрами А и В имеют ровно одну общую точку — середину отрезка АВ. Итак, мы явно указали (я — 2)+(п — 2)-|- + 1 =2/i — 3 середины отрезков. Пример (2п—3)-х середин: все точки располагаются на одной прямой на равных расстояниях друг от друга. Значит, наименьшее число середин всевозможных отрезков равно 2/1 — 3. 37.17. Рассмотрим сначала один столбец. Несколькими вы- читаниями единицы из всех чисел этого столбца добьемся того, чтобы наименьшее число в нем стало равным 1. Теперь строки, содержащие 1, удвоим, а затем из всех чисел нашего столбца вычтем по единице. Тем самым мы смогли единицы столбца не изменить, а все числа, большие 1, уменьшить на 1. Действуя таким образом и далее, мы вскоре сумеем сделать все числа столб- ца равными 1, а потом вычитанием 1 получить 0 в каждой его клетке. Аналогичную процедуру проделаем и со всеми осталь- ными столбцами. 37.18. В пятиугольнике ABCDE зафиксируем вершину А, про- ведем диагонали АС и AD и построим на них как на диа- метрах круги с центрами в точках Oi и Оъ Эти круги покрывают треугольники АВС и AED, поскольку углы АВС и AED тупые. Если один из углов ACD или ADC тупой, то один из кругов с центром Oi или Ог покрывает треугольник ACD. Если же углы ACD и ADC не тупые, то круг с центром Oi покры- вает треугольник АНС* а круг с центром О2— треугольник AHD, где Н — основание высоты АН в треугольнике ACD (точка Н ле- жит на отрезке СО). Тем самым круги с центрами О\ и Ог полностью покрывают весь пятиугольник. 37.19. Для равных чисел все очевидно. Пусть теперь среди данных 100 чисел ai, ..., пюо имеются хотя бы два различных, например си и аг. Рассмотрим 100 следующих чисел: ai. аг. а»4-аг, сп+аг + аз» ...» ai + ... +099. Докажем, что одно из них или же разность двух из них делится на 100; поскольку это число больше 0 и меньше 200, оно в точности равно 100. Для доказательства рассмотрим две последние цифры чисел исходной последовательности. Среди получившихся двузначных чисел либо встретится 00, и тогда делимость на 100 доказана, либо найдет- ся пара одинаковых чисел. Во втором случае это не пара ai, аг. Взяв разность чисел из этой пары, получим кратную 100 сумму a*+t + a*+2-|-... +ял. 37.20. Существует. Числа в последовательности будем выпи- сывать парами Первая пара: ai = 1, аг = 2 Пусть мы уже смогли 251
построить k пар; построим (й-|-1)-ю лару.Рассмотрим всевозмож- ные разности, реализуемые среди имеющихся k пар чисел, обозначим через d наименьшую разность, которая еще не реали- зована. Положив теперь 02*4-1=202*+!, 02*4-2=02*4 1+*Л по- лучаем требуемую последовательность. 37.21. У выпуклого 2п-угольника п (2п—-3) диагоналей. Легко видеть, что диагоналей, параллельных данной стороне, не более п—2 (любые 2 такие диагонали соединяют разные пары вершин). Сторон имеется 2п, так что число диагоналей, параллельных од- ной из них, не больше 2п(п —2), т. е. меньше общего числа диагоналей. 37.22. Назовем гирю существенной, если, убрав ее, мы не сможем разбить оставшиеся гири на k равных по массе групп. Все прочие гири будем называть несущественными. Предположим, что утверждение задачи неверно, т. е. что сущест- венных гирь меньше k. Мы тогда докажем, что при любом на- туральном k масса любой несущественной гири кратна knt т. е. делится на сколь угодно большое число, а значит, равна н у л ю,— противоречие. Доказательство этого проведем по индук- ции. Для п = 1 оно верно. В самом деле, масса всех гирь делится на k и, убрав произвольную несущественную гирю, можно ос- тавшиеся гири разбить на k равных по массе групп (по опреде- лению несущественной гири). Следовательно, масса убранной гири кратна Л1. Предположим теперь, что масса любой несущест- венной гири кратна Ля, и докажем, что она кратна йя+1. По пред- положению существенных гирь меньше fe, значит, существует группа, все гири в которой несущественные (в противном случае несущественных гирь было бы не меньше, чем групп). Масса всей группы из несущественных гирь кратна йя, всего групп k, а масса любых двух таких групп одинакова. Отсюда следует, что масса всех гирь кратна Ля+1 как до того, как убрана несущественная гиря, так и после. Следовательно, масса несущественной гири кратна йя+1. 37.23. См. решение задачи 28.07. 37.24. Указание. Утверждение задачи можно доказать ин- дукцией по сумме целых частей а и Ь. Для [а]+[6]=1 все оче- видно. Предположение индукции: если [о]+[6] = п, то одно из чисел — а или b — целое; надо доказать это же утверждение для [a] + [6] = n+ 1. Для доказательства следует рассмотреть два слу- чая. Первый: к нижней стороне листа бумаги все прямо- угольники примыкают своими сторонами, длины которых отличны от 1. Второй, более сложный случай, состоит в том, что предлагается существование прямоугольника, примыкающего к нижней стороне листа своей единичной стороной. (Подробное решение в ж. «Квант», № 4, 1975 г.) 37.26. Поскольку 1974я> 103я, то количество цифр в числе 1974я не меньше Зп. Обозначим это количество через ^предпо- ложим, что у 1974я+2я больше цифр, чем у \§14п. Тогда 252
1974я + 2я>10*. Легко видеть, что 987я<2*“я-5*^987я +1, по- скольку 10* делится на 2я. Следовательно, 2*~я-5* = 987я-|-1, k — n^2n. Если п>2, то 2*~я-5* делится на 8, но 987я + 1 на 8 не делится (дает в остатке 2, 4 или 6). Поэтому получен- ное равенство не имеет места. Утверждение доказано. 37.27. Проведем через центр планеты и произвольную пару астероидов плоскость П (будем считать ее экваториальной). Проведем ось планеты, перпендикулярную плоскости П. Она пе- ресекает поверхность планеты в «полюсах» Л и В. Тогда легко видеть, что наблюдатели в точках А и В видят вместе лишь 37 — 2 = 35 астероидов, а потому один из них видит менее 18 астероидов. 37.28. Рассмотрим ученого с максимальным количе- ством друзей, равным п. По условию все его друзья имеют различное число друзей, отличное от нуля, но не большее п. Таких возможностей ровно п: один друг, два друга, п друзей; следовательно, все случаи реализуются. Поэтому существует, в частности, ученый, имеющий ровно одного друга. 38.01 . x=y = z = / = 0. Указание. Замените уравнение равносильным ±-х?+(±.Х-у) =0. 38.02 . Обозначим буквой О центр круга. Проведем произ- вольную прямую /, пересекающую круг, и отразим относительно нее точки Л и О, получив точки Л' и О' Из треугольника ОАО' следует что длина стороны 00' не меньше О А — — О'Л|, а так как О'А = ОА'. то |ОЛ' — ОЛ| не превосходит удвоенного расстояния от точки О до прямой /, т. е 2 см. Следовательно, за 24 отражения точка Л сможет приблизиться к центру круга не более чем на 48 см и поэтому еще не попадет в круг, за 25 же раз загнать ее внутрь круга можно. 38.03 . Два из чисел а, Ь. с одной четности; пусть ими будут числа а и Ь. Так как Ьс той же четности, что и Ь, то р = Ьс-\- а чет- ное. Но р — простое число, стало быть, р = 2, откуда а = 6=1 А тогда (?=16-|-с = сп-|-1=г, что и требовалось. 38.04 . Нельзя. Для доказательства соединим центры 28 край- них клеток доски. Получим замкнутый контур квадрата Г. Каж- дая из 13-и прямых пересекает контур Г не более чем в двух точках, откуда общее число точек пересечения с Г не больше 26, и они не могут отделить друг от друга даже эти 28 центров, а тем более все центры. 38.05 . Можно. Пусть источник света располагается в точке О. Построим правильный тетраэдр ABCD с центром О Рассмотрим далее 4 круговых конуса, содержащих строго внутри себя пира- миды OBCD. OACD. ОАВС. OABD и имеющие общую вершину О (конусы бесконечные). Эти конусы частично пересекаются, так чтг. любой луч, исходящий из источника света, содержится внутри одного из конусов Впишем теперь в каждый конус по одному шару так, чтобы эти шарм не пересекались (для этого
радиусы шаров должны сильно отличаться друг от друга, напри- мер они могут равняться 10, 104, 107 и Ю10). Очевидно, что любой луч ОХ, исходящий из источника, пересекает хотя бы один из шаров. 38.07 . Указание. Докажите сначала, что два из трех углов по 120° примыкают к одной стороне семиугольника (ср. с реше- нием задачи 31.07). Затем докажите, что если многоугольник вписанный, то из равенства углов АВС и BCD следует АВ = = CD. 38.08 и 38.21. Указание. Коля проигрывает, если число камней равно 2*— 1; в остальных случаях он выигрывает, каждый раз загоняя Витю в положение, где число камней в наибольшей куче имеет вид 2Z—1 (например, если /г = 100, первым ходом Коля должен разделить кучу на 63 и 37 камней). При и = 31 выигрывает Витя. 38.09 и 38.12. а) Заменим все четные цифры в последова- тельности на 0, а все нечетные — на 1. Получим последователь- ность, в которой периодически повторяются цифры 11110. Набо- рам 1234 и 3269 отвечают четверки цифр 1010 и 1001, которые в последовательности не встречаются, б) Имеется лишь конеч- ное число четырехзначных чисел (в них включаются и числа, начинающиеся с нулей). Поскольку вся последовательность бес- конечна, найдутся два одинаковых четырехзначных числа: 1975 ... А ... А .... Докажем, что в промежутке между этими равными числами расположено число 1975. Для этого заметим, что нашу последовательность можно однозначно продолжить влево с сохранением условий задачи (т. е. зная 4 идущие подряд цифры, можно найти не только цифру, идущую за ними, но и цифру, стоящую перед ними). Следовательно, зная лишь один кусок последовательности (Я ... Л), м ы можем восста- новить бесконечную в обе стороны последо- вательность. И если бы в промежутке между Л и Л не встречалось число 1975, то оно не встречалось бы и во всей последовательности, что противоречит условию. Следовательно, набор 1975 встретится вторично, в) Заметим, что перед набором 1975 стоит цифра 8. Следовательно, мы нашли набор 8197 в бесконечной в обе стороны последовательности. Стало быть, он имеется ив исходной последовательности. 38.10. а) При всех л2>3 правое число больше (доказательство по индукции), хотя при п=2 верно обратное, б) Обозначим че- рез Ап левое число (степени из п троек), через Вп-\—правое (степени из и —1 четверок). Докажем по индукции, что Ля + |> >2ВЯ. Предполагая, что Ап >2Вп-1, имеем: Ля +1 = 3Ап > 32Вп~1 = =9fl" =(2,25)йя"’ • 4fi"“‘ > 2 • 4В-' = 2ВЯ. 38.13. В Обычной стране города А и В не соединены доро- гой (иначе из Л в В можно было бы попасть без пересадок), поэтому они соединены дорогой в Зазеркалье. Теперь пусть С и D — любые два города. В Обычной стране С не мог соединяться 254
с А и В одновременно (иначе из Л в В можно было бы проехать с одной пересадкой), так что в Зазеркалье из С ведет дорога в А или В. То же верно для D. Поэтому Алиса в Зазеркалье может проехать из С в О (через города Лив), сделав не более двух пересадок. 38.14. п очков. Например, первая команда выигрывает у всех остальных, набирая 2п — 2 очка, а остальные команды играют друг с другом вничью, набирая по и — 2 очка каждая. 38.18. Пусть X и Y — два города таких, что после закры- тия дороги АВ кратчайший путь I из X в Y и е меньше 1500 км. Легко видеть, что на пути / должен быть город Л19 от- стоящий не менее чем на 500 км как от Xt так и от Y. Отсюда сле- дует, что до закрытия дороги АВ кратчайший путь из X в Y проходил через дорогу АВ (имел вид ХАВМ) и кратчайший путь из Y в М тоже имел вид либо YABM, либо YBAM. В пер- вом случае после закрытия дороги АВ остались открытыми пу- ти ХА и YA и путь XAY не превосходит 1000 км, что противоре- чит предположению, что кратчайший путь из X в Y не меньше 1500 км. Во втором случае пути ХА, ВМ, YB и /1А1 по-прежнему открыты, ХА-|-ВМ < 500 и УВ-|-АМ <500. Поэтому путь XAMBY также короче 1000 км. Замечание. Улучшить оценку 1500 км нельзя, что видно из следующего примера: пусть в Мантиссе только 4 города А, В, С и D, причем ДВ=1 км, АС = CD — DB = 498 км. Тогда после закрытия дороги АВ кратчайший путь из А в В — почти 1500 км. 38.19. Предположим, что выпуклый многоугольник М разре- зан на п невыпуклых четырехугольников; сумма их углов S = =360°п. Вершину четырехугольника, угол при которой больше 180°, будем называть «невыпуклой». Невыпуклая вершина X не может лежать на границе многоугольника М, и никакие два четырехугольника не могут иметь общую невыпуклую вершину (иначе они пересекались бы). Отсюда число невыпуклых вершин равно числу четырехугольников, причем все невыпуклые вершины расположены строго внутри М. Определим сумму углов с верши- нами в точках Xi, ..., ХЛ, являющихся невыпуклыми вершинами четырехугольников. Она равна 360°п. Но тогда S не меньше, чем эта сумма плюс сумма всех углов многоугольника М. Про- тиворечие. Замечание. Сферу можно разбить на конечное число невыпуклых четырех- угольников (разбиение придумайте сами). 38.23. Искомое освещение возможно для произвольно- г о числа прожекторов. Занумеруем их числами 1,2,..., п. Заметив, что число пар (£, /) равно -—и (и — 1), построим следующую кон- струкцию. Расположим над ареной п круглых дисков того же радиуса, что и арена. В каждый диск впишем правильный 255
Л=4; Рис. 81. Рис. 82. -i-/? (n —1)-угольник одинаковым образом. Отрежем теперь от всех дисков, кроме выбранных двух, один и тот же маленький сегмент, образованный стороной вписан* ного многоугольника и опирающейся на нее дугой окружности (рис. 81). Потом от- кинем другую пару дисков и у остальных отрежем один и тот же сегмент и т. д., всего -~п{п — 1) раз. В результате у каж- дого диска будет отрезано /г — 1 сегментов. Очевидно, что полученная система про- жекторов удовлетворяет заданным усло- виям. 39.01. Обозначим через X и Y соответ- ственно наибольшее и наименьшее числа среди Х|, ..., *5. Тогда из соответствующих уравнений имеем: Х2^2Х и У2^2Y. По- скольку Х>0, У>0, получаем: 2^У^ ^Х^2. Следовательно, система имеет единственное решение: Xi =х2 = ... = х5 = 2. 39.02. Может. Для этого угол АВС дол- жен быть почти прямым, сторона АВ (к которой проведена высота СН) — очень маленькой, а сторона ВС (к которой про- ведена медиана AM) — очень большой. В предельном положении точка Н совпа- дает с в и площадь треугольника М'1ГК' (в пересечении) станет равной Сле- довательно, в близком к предельному по- ложению площадь этого треугольника мо- жет быть больше 0,499. 39.03. Первые четыре цифры числа бу- дут 1000. Указание. Оценив сумму первых 999-ти членов геометрической про- грессии 999-|-9992-|-... -J-9999", докажите, что при сложении с 1ООО1000 она не влияет на четвертую цифру слева. 39.04. Можно. Поместим в центр куба 8 прожекторов, осве- щающих все пространство, так, чтобы один из них освещал ровно две вершины куба (это можно сделать, поскольку угол между большими диагоналями куба острый). Тогда один из прожекторов, по принципу Дирихле, не освещает ни одной вершины данного куба. 39.05. Можно. Для этого следует, например, выложить костя- ми домино указанные на рисунке 82 бесконечные уголки. 256
39.07 . Может. Для этого следует сделать общей вершиной точку С у равных тупоугольных треугольников АВС и CDE (ту- пой угол отличен от 108°; см. рис. 83), а затем расположить их так, чтобы длины диагоналей АС, СЕ, ЕВ,- AD были равны друг другу и отличны от длины BD-, длина стороны АЕ не равна 1. 39.08 . Существует. Например, число 10 111 111 111. 39.09 . Можно. Указанное расположение изображено на ри- сунке 84 (каждая точка соединена с тремя ближайшими к ней). 39.12. Указание. Вставляя перегородки длины 1, разбейте все квадраты на единичные. 39.16. Не может. Сократив л! на максимальную степень 10, получим число, кратное большой степени 2. Однако 1976 не делится даже на 16. 39.17. Рассмотрим пятно наибольшего радиуса и про- ведем окружность чуть большего радиуса с тем же центром (она содержит рассматриваемое пятно), не пересекающуюся с пятнами. Отразим центрально-симметрично (относительно центра сферы) все пятна. Легко видеть, что отраженные пятна не будут покрывать всю окружность. Любая непокрытая точка окружности и ей диаметрально противоположная искомые. 39.18. Указание. Предположив противное, докажите, что для любых натуральных А и k справедливо неравенство S (Л •26ft)>S (Л) + 27А>, где S (А) — сумма цифр числа А. Но S (Л-26*)<9 1g А -|- 18Лг, откуда легко получается противоречие. 39.19. Д о ка ж е м, что в числе N любые две соседние цифры можно переставить местами. Обозначим эти цифры че- рез о и & и добавим слева к ab число ЪЬ, справа число аа, т. е. получим ....bbabaa... . Вычеркнув теперь два стоящих под- ряд числа Ьа, получим искомую перестановку ab -*-* Ьа. Отсюда вытекает, что всегда можно одновременно вычеркивать две оди- наковые цифры, даже не обязательно стоящие рядом. Сделав все такие вычеркивания, получим число с попарно различными цифрами; оно меньше 109. Замечание. В математике это задача о структуре группы, о которой квадрат любого элемента равен единице (соответствующий «сомножитель» можно вычеркнуть или вписать). ас=ао^ев=ес Рис. 83. 257
39.20. На клетчатой плоско- сти можно так расположить оди- наковые пятиугольники с верши- нами в узлах («домики»), чтобы они покрывали все узлы (рис. 85). Можно покрыть каждый «домик» пятаком так, чтобы пятаки не соприкасались. 39.21. Существует. Наимень- шее число, обладающее этим свойством, равно Д=(10и4-1)Х Х-^г= 13 223 140 496. Указа- н и е. Если число АА —точный квадрат, где А — n-значное чис- ло, то 10Л-|-1 должно делиться на квадрат простого числа р. Следует положить р=11, а п выбирать нечетными. 39.22. Существует. Например, произвольная пирамида, в ос- новании которой расположен выпуклый 1975-угольник. Указа- ние. Расставим произвольно стрелки на ее ребрах и спроеци- руем полученные векторы на прямую, содержащую высоту пира- миды. Сумма проекций отлична от нуля. 39.23. Указание. Заменив 10, 20 и 200 на ft, г и Лг, решите задачу в общем виде индукцией по сумме k + r. 39.24. Введем на плоскости такую прямоугольную систему координат, чтобы все точки оказались в первом квадранте. Среди всех рассматриваемых точек выберем множество таких, расстоя- ния от которых до ближайших к ним минимально возможные (все эти расстояния одинаковы). Рассмотрим точку А из этого мно- жества, являющуюся в выбранной системе координат самой правой точкой из всех самых нижних точек по отноше- нию к остальным точкам этого множества. Соединим А со все- ми ближайшими к ней точками Bi, В2, Вз, ... . Тогда все углы /.Й|ЛВ2, Z-B2AB3, ... не меньше 60°. Следовательно, у точки А не более трех ближайших соседей. 39.25. Будем считать, что имеющиеся по условию 5 разноцвет- ных точек таковы, что никакие 4 из них не лежат в одной плос- кости (противоположный случай почти очевиден). Соединим по- парно эти 5 точек 10-ю прямыми и предположим, что каждая из них окрашена только в два цвета. Возьмем плоскость, не параллельную ни одной из проведенных прямых. Тогда среди всех точек пересечения этой плоскости с указанными прямыми найдутся по крайней мере 4 точки разного цвета. Далее, взяв эти 4 разноцветные точки, проведем аналогичное рассуждение для плоскости, получаем прямую, окрашенную не менее чем в 3 цвета. 40.01 . Заметим, что если хп ч е т н о и равно произведению некоторой степени двойки на нечетное число, xn=2fem, то xn+k н е- 258
четно. Если же хп нечетно, xn=2* (2/n -J-1) -J- 1, то таким же рассуждением получаем, что Хп+л четно. Следовательно, нечет- ных чисел, так же как и четных, в данной последовательности бесконечно много. 40.02. Обязательно. Указание. Рассмотрите выпуклую оболочку всех 2п квадратов. 40.03. Можно в обоих случаях. Д.пя протаскивания прово- лочного куба годится щель в форме буквы «П» с единичной длиной перекладины и достаточно длинными вертикальными «ножками». Для протаскивания тетраэдра годится щель в виде буквы «Т» с длинными прямолинейными частями. 40.04. Пусть А и В — множества соответственно синих и красных точек. Если бы множество А содержало лишь конеч- ное число точек с координатами, кратными а, а множество В содержало лишь конечное число точек с координатами, крат- ными Ь, то оказалось бы, что в ряду натуральных чисел имеется лишь конечное количество чисел, кратных ab. 40.06. а) Не может быть. Иначе в вертикальных столбиках находилось бы нечетное число белых кубиков, а их 14. б) Мо- жет. 12 черных кубиков можно расположить вдоль шести ре- бер АВ, ВС, СС', C'D', D'A', А'А большого куба ABCDA'B'C'D' и еще один черный кубик надо поместить в центр. Тогда в каждом столбце будет или 1, или 3 черных кубика. 40.07. Исходное равенство равносильно такому: ег+(т),!- Выбрав произвольные числа п и А, положим -2-=л, — = fe. Тогда с = п1б-|-А,б> после чего а и b определятся однозначно: а = п (n,6 + fc16)» b = k (nI64-fe15). 40.08. Второй. 40.09. л =50. Любой выпуклый 100-угольник можно получить в виде пересечения 50-и треугольников, беря вершины 100-уголь- ника через одну (рис. 86, а). 100-угольник с одной очень длинной стороной и очень маленькими остальными не представляется в виде пересечения меньшего числа треугольников (рис. 86,6). Рис. 86. 259
40.16. Существуют (во всех рассматриваемых случаях). Ес- ли п искомых чисел сщ а2» .... <зл-ь ап уже построены, то новые п+ 1 чисел строим так: полагаем Ьъ = НОК (аь а2, аД bi = fli + fe2 = G2“|“6o, ^л = Ол4“6о- 40.18. Предположим, что вершин у многогранника боль- ше 8, и рассмотрим произвольные 9 его вершин. Из них по крайней мере 5 имеют первую координату одной четности (или все 5 четные, или все 5 нечетные). Вторые координаты по крайней мере трех из этих 5 точек также имеют одинаковую четность. Наконец, третьи координаты каких-то двух из этих трех точек также имеют одинаковую четность. Следовательно, мы нашли такие две вершины многогранника, у которых совпа- дают четности каждой из трех координат. Но тогда середина отрезка, соединяющего эти две вершины, имеет целые коорди- наты, т. е. является узлом решетки. А в силу условия выпуклости многогранника она принадлежит многограннику. Получили про- тиворечие с условием. 40.20. Можно, Например, круги можно расположить далеко друг от друга так, чтобы их центры лежали на параболе у=х2. 40.23. Пусть последовательность уп периодична с периодом Т начиная с некоторого nG. Тогда хп четно при всех л>ло» с другой стороны, оно равно ^-|-^п”Ло-(хЛо+г —хЛо). При боль- шом п последнее число нечетно. Противоречие. 41.01. (0, 2); (0, -2); (3, 5); (3, -5); (4, 7); (4, -7). 41.02. Треугольники с углами: а> Т’ Т’ б) Т’ Т’ Т’ в) г> Т’Т’Т- 41.09. См. решение задачи 33.13. 41.10. л —2 точки. Соединим произвольную вершину л-уголь- ника с остальными, получим п — 2 треугольника. Значит, мини- мальное число точек не меньше л—-2. Докажем, что л —2 точки можно расставить требуемым способом. Для этого выделим произвольную сторону АВ и рассмотрим все треуголь- ники с основанием АВ (их л —2). В каж- дом полученном треугольнике очень близ- ко от его вершины, противолежащей сто- у j \\ роне АВ, поместим по точке так, чтобы в кажД°м треугольнике, образованном I \ малой диагональю и двумя сторонами I I многоугольника, содержалась точка. До- рхД | кажем теперь, что в треугольнике с про- ---1извольно выбранными вершинами XYZ ч / / содержится хотя бы одна точка. Дсйст- ’// вительно, один из углов дХУ/ содержит некоторый Z.AX*B (где %* совпадает с X, Y или Z), а потому содержит точку, распо- Рис. 87 ложенную вблизи Xk (рис. 87). 260
8 Рис. 88. 41.11. Не существует. Указание. Рассмотреть все век- торы с одной и той же максимальной л-координатой и вы- брать из них два вектора, имеющие наибольшие (/-координаты. 41.13. Выберем в качестве оси произвольную прямую /, не перпендикулярную ни одной из данных прямых и достаточно удаленную от множества S точек пересечений данных прямых друг с другом. Тогда от точки А на оси /, имеющей достаточно большую координату R, расстояния до всех точек множества S превосходят все расстояния до данных прямых (если первые расстояния «/?, то расстояние от А до произвольной прямой sin <р, где <р — угол между этой прямой и осью /). 41.14. Может. На рисунке 88 указан граф сообщений, со- держащий 22 ромба и 100 вершин; в каждой вершине стоит число, равное количеству дней, необходимых для попадания в нее новости из вершины 0. Вершине 64 отвечает 5 марта; еще через 8 дней (13 марта) новость становится известной всем. 41.15. Заметим сразу, что 12% в условии задачи можно заме- нить на (50 —е) %, где е>0 — произвольное положительное, но сколь угодно малое число. Действительно, отразив центрально- симметрично сферу, получим, что не вся сфера красная: на ней имеется полностью белый участок. Центрально-симметрич- ный ему участок на сфере также белый. Выбрав по 4 вершины квадрата в каждом из этих белых участков (квадраты центрально- симметричны относительно центра сферы), получим искомые 8 вершин параллелепипеда. (Другое решение состоит в том, что при трех отражениях сферы относительно трех взаимно перпендикулярных плоскостей, проходящих через центр сферы, у полученной сферы будет закрашено красным не более 8* 12% =96% площади; следовательно, найдется белая точка, ко- торая в совокупности с точками, попавшими в нее при отраже- ниях, дают искомые белые вершины параллелепипеда. На этом пути можно доказать, что существует куб с белыми верши- нами.) 261
41.18. Наименьшее л, при котором 2Л = ...л, равно 36: 236 = 7/736. _____ Далее нужно применить индукцию: пусть 2n=...[a]/ij где а — цифра, стоящая слева от л; докажите, что = Да]л. 41.19. Рассмотрим четыре вектора t?j=(a, 6), t?2 = (c, d), ^з = = (е, f), v4 = (g9 h). Указанные шесть чисел равны попарным скаляр- ным произведениям этих векторов: ac^bd = v\*V2. ae^bf = v\>vz9 ..., eg + fh = V3*v4. Но один из углов между введенными векто- рами не превосходит поэтому одно из шести скалярных произведений неотрицательно. 42.01 . а) 5 кругов искомым образом расположить можно. Например, можно разбить плоскость на 5 одинаковых углов ои, а2, аз» а^ as (а, = 72°) и вписать по одной окружности в углы ai(Ja2, агЬаз, аз11«4, ailjas, asUai. б) Нельзя. Рас- смотрим углы с вершиной в точке А, в которые вписаны окружно- сти. Величина каждого из них меньше 180°, и, следовательно, сумма всех углов меньше 4* 180° = 720°. Однако, если бы произ- вольный луч с вершиной А пересекал не менее двух из этих четырех кругов, эта сумма была бы не меньше 2-360° = 720°. Противоречие. 42.02 . Пусть гирь п штук и р* — масса гири с номером k. Пусть pi ^4“’ тогда общая масса всех гирь 1 1 п не превосходит 1 Противоречие с условием. 42.03. Среди всех прямоугольников, вписанных в данный круг, наибольшую площадь имеет квадрат (докажите сами). Значит, площадь S (П) прямоугольника П, вписанного в круг X, и площадь S (К) круга К удовлетворяют условию ~ S (П) S (X). Пусть квадрат Xх площадью S (X') разбит на прямоуголь- ники Пь Пг, Пз, ...» Пп. Тогда, учитывая, что площадь круга, описанного около квадрата, равна -^-*5 (Xх), и используя выпи- санное выше неравенство, получаем: y-«S(Xx)^^-(S(II1)-|-...+ 4-S (Пл))<£ (Xi)4-..-4-S (Хл), где X,— круг, описанный вокруг прямоугольника Ц. Требуемое неравенство доказано. 42.04. Витя всегда выигрывает. Для этого он может посту- пать так: выбрать центр какой-то клетки и отмечать точки сим- метрично точкам Коли относительно этого центра. Легко видеть, что Вите всегда найдется ход (выпуклый многоугольник после хода Вити остается выпуклым). После 6 ходов Коля может отме- чать только узлы бумаги, расположенные в шести треугольных областях, образованных каждой стороной шестиугольника и про- должениями соседних с ней сторон, т. е. у Коли остается лишь 262
Рис. 89. конечное число возможностей. Следова- тельно, игра кончится в пользу Вити через конечное число ходов. (Ср. с решением задачи 31.20.) 42.05. См. решение задачи 42.01. 42.07. Проведем хорду АЕ, перпенди- кулярную DA. Дуга АЕ равна дуге СВ и величина каждой из них равна 180°—^>DA. Из прямоугольного треугольника ADE (рис. 89) следует, что ОН=-^-АЕ. 42.10. Пусть D — максимальное значение числа d. Рассмотрев набор из 55 камней одинаковой массы -j-p, мы видим, что Докажем, что D=jy. Предположим, что D >• рр, т. е. существует набор камней с общей массой 100 кг, масса любого поднабора которого удовлетворяет неравенству |М —10|>~. Рассмотрим тогда такой поднабор, для которого М> 10, но после выкидывания любого камня масса уже становится меньше 10 (очевидно, такой поднабор существует). Пусть xi, .... хь— массы камней этого поднабора, М =xi-f-...По предположению Af> 10-|--рр, но М—х<<10—-рр при любом I. Это означает, что все массы камней Xi, 1=1, 2, ..., k, больше ^р. По условию Х|^2, ..., х*^2, откуда следует, что число камней k >5. Но теперь ясно, что M'=xi 4-Х2+Х34-Х44-Х5 удовлетворяет неравенствам 5*рр< <ЛГ<10, т. е. масса первых пяти камней отличается от 10 кг меньше чем на 10—рр-=-|р-. Противоречие. 42.11. Можно. Например, поступим так. Возьмем прямую а и через две точки на ней проведем перпендикулярные к ней непа- раллельные между собой прямые b и с. Рассмотрите далее мно- жество плоскостей, параллельных друг другу и прямой а; проведем прямые через точки пересечения прямых b и с с каждой из этих плоскостей. И наконец, совершим всевозможные повороты пространства вокруг оси а. Образы построенных прямых и обла- дают нужным свойством. Замечание. Другое (неэлементарное) решение состоит в том, что все пространство есть объединение софокусных однополостных гиперболоидов, каждый из которых «соткан» из попарно скрещивающихся прямых (так называемая линейчатая структура однополостного гиперболоида или подвиж 263
ная стержневая модель гиперболоида). Соответствующие конструкции см. в книге: Гильберт Д.» Кон-Фоссен С. Наглядная геометрия.— М.; Наука. 1981. 42.12. а) Существует. Строим последовательность кусками. Нулевой кусок состоит из одного члена: ао = 1, п-й кусок состоит из членов, равных ±-bn + bn, -^-Ьп + 2Ьа, .... Ьп, где Ьп = 2у . Сумма членов n-го куска равна (т Ьп + *) = ТЬ" +тЬ,‘ Ь" ~ Ьп = Ьп+' “ Ьп- Значит, сумма членов предыдущих кусков не больше Ьп — Ь0, т. е. меньше bnt откуда следует, что никакой член n-го куска не равен сумме нескольких других. Оценим теперь n-й член последователь- ности. В (п— 1)-м куске число членов равно + 1, следова- тельно, номер любого члена в n-м куске больше 2зЛ”2“|, а его величина не превосходит aN < b2 = 22 зП-* =64 .(23"-2-')6 <64 • №. Итак, aN<64N* для любого N, откуда aN<N7 при Л/^64. Для всех N, меньших 64, неравенство aN<N7 проверяется непосредственно, б) Не существует. Убедитесь сами, что уже число а5 не удастся выбрать так, чтобы выполнялось условие задачи. 42.09, 42.14, 42.19. Поставим всех ученых в одну шеренгу и занумеруем их слева направо. Затем начнем спрашивать каж- дого в них, начиная с 1-го, о его правом соседе. Пусть п — минимальное число, при котором n-й сказал о своем соседе (т. е. об (п + 1)-м), что тот алхимик; до этого все говорили, что их соседи — химики. Тогда пару (п, п+1) выводим из цепочки и спрашиваем (п —1)-го о его новом правом соседе и т. д. В кон- це концов, выбросив несколько пар, мы дойдем до конца цепочки. Теперь в цепочке каждый сказал о своем правом соседе, что тот химик, а в каждой паре один сказал о другом, что тот ал- химик. Поэтому: 1°. В каждой из пар не меньше одного алхимика; поэтому 2°. В оставшейся цепочке химиков больше, чем алхимиков, в частности в ней есть хоть один химик. 3°. Если l-й человек в цепочке — химик, то и все последующие — химики. В частности, последний человек в цепочке всегда хи- мик, а если в цепочке больше одного человека, то двое послед*- них — химики. До сих пор было задано Л —1 вопросов. Поэтому если в це- почке больше одного человека, то достаточно теперь задать последнему в цепочке Л —2 вопроса о всех остальных, кроме предпоследнего в цепочке, общее число вопросов равно (&—1)4- + (Л — 2)—2& — 3. Если же оставшаяся цепочка состоит из одного 264
человека (и он химик), то в каждой из отброшенных пар ровно по одному алхимику (иначе алхимиков было бы больше, чем химиков). Теперь достаточно задать по вопросу про одного из ученых выкинутой пары; число вопросов не превосходит k— 1 4- +4-=^-L Замечание. Московский математик П. М. Блехер (победитель 26— 28-й Московских олимпиад) доказал, что точная оценка в этой задаче равна 1)|. (Доказательство опубликовано в его статье «On a Logical Problem», Discrete Math. 43 (1983), с. 107—НО.) 42.16. Разрежем отрезок [0, 1] на 10 равных частей и про- нумеруем их. Затем все части с нечетными номерами сдвинем вправо на 0,1, а все с четными — влево на 0,1. Множество отмеченных интервалов перейдет в множество интервалов, не пере- секающихся с исходными (это следует из условия), а общая длина отмеченных и полученных интервалов будет не больше 1. Значит, сумма длин исходных интервалов не больше 0,5, что и требовалось. 42.17. Наибольшее значение равно оно принимается функцией g(x)=±-x(l —х) в точке х=-^~. Действительно, предположим, что нашлась такая функция [, что f (0)=f (1)=0, If" (х)| 1 для всех хб [0, 1] и в некоторой точке а6(0,1) f(a)>~. Положим ft(x)=f (х)—£^-g(x). Так как g"(x)= —1, f(a)>-|->g(a)>0, то Л (0)=Л (!)= 0, Л" (х)= =f" (х)4-^->0. Кроме того, ft(a)=0. Из условия Л"(х)>0 следует, что Л' (х) монотонно возрастает. Значит, на одном из отрезков [0; а] или [а; 1] она не меняет знака. Но тогда либо а I Л(0)=—J h' (x)dx=#0, либо Л (l)=J/t'(х)с/х^£=0, что приводит 0 а оба раза к противоречию. 42.18. Возьмем круг наибольшего радиуса и рассмотрим но- вый круг с тем же центром втрое большего радиуса. Затем выбросим все круги, оказавшиеся полностью внутри нового круга: оставшиеся круги уже не пересекаются с исходным. Возьмем среди них максимальный круг и проделаем с ним ту же процедуру. Так делаем до тех пор, пока не получим несколь- ко «раздутых» кругов, площадь объединения которых больше 1. Отвечающие им исходные (втрое меньшего радиуса) круги не пересекаются, и их общая площадь превосходит 43.01 . 95 210. 265
43.03. Ориентируем все стороны отме- ченного внутреннего многоугольника про- тив часовой стрелки. Каждой стороне по- ставим в соответствие вершину исходного многоугольника, в которую направлен ко- нец стрелки на этой стороне. В силу выпуклости многоугольников разным сто- ронам отвечают разные вершины. Поэто- му число сторон внутреннего многоуголь- ника не превосходит числа сторон исход- ного, т. е. не больше 100. 43.05 . Повернем 20-угольник вокруг его центра на угол, рав- ный по величине дуге окружности, концы которой совпадают с концами черной диагонали. При этом одна черная точка перей- дет в другую черную точку (в другой конец дуги). Следова- тельно, какая-то белая точка перешла опять в белую. Эти две белые точки соединяются диагональю той же длины, что и чер- ная диагональ. 43.08 . Достаточно доказать, что р = 2а (рис. 90), отсюда сразу следует, что треугольник BCD равнобедренный. Для доказательства соединим центр О окружности с точками А и D. Углы AOD и ACD опираются на одну и ту же дугу AnD, поэтому Z_AOD = fi. Но угол AOD центральный в окружности О, а угол ABD вписанный в нее, поэтому Z_ABD =^~Z-AOD* т. е. 0 = 2а. 43.11. Указание. Используйте тот факт, что в числе Ok либо столько же цифр, сколько в а*-ь либо на одну больше. Поэтому если наша дробь периодична, то найдется такое А, что число ak в точности совпадает с периодом. После этого рассмотрение числа быстро приводит к противоречию. 43.12. Д о к а з а т е л ь с т в о проведем по индук- ции. Пусть число состояний табло равно 2*, когда на пульте п кнопок (для п=1 имеется ровно два состояния, отличающиеся нажатием кнопки). Добавим еще одну кнопку. Если она «ба- зисная», т. е. при ее нажатии появляется новое состояние табло, отличное от всех 2* предыдущих, то каждому из состояний отвечает противоположное (когда зажженной лампочке отвечает незажженная и наоборот). Всего получаем 2-2* = 2*+l состояний табло для пульта из (п + 1)-й кнопки. Если же кнопка не «ба- зисная», то при ее нажатии загорается одно из уже встречав- шихся 2* состояний табло. Итак, в любом случае получаем степень двойки. 43.13. тп квадратов. У каждого квадрата отметим его пра- вую верхнюю вершину. Никакие две отмеченные вершины у раз- ных квадратов не совпадают, что вытекает из условия задачи. Следовательно, количество квадратов на листе не больше числа узлов бумаги, не лежащих на левой и нижней сторонах листа, 266
Рис. 91 Рис. 92. которых в точности тп. (тп квадратов размещаются требуемым способом однозначно. Докажите!) 43.20. Пусть аь а2, ... — указанные дуги, причем по условию ai+a2 + -..<n. Рассмотрим произвольную дугу а, и выберем точку S на ней, а также полярную к S окружность Os (поляр- ной окружностью к точке S называется большая окружность, все точки которой равноудалены от точки S — полюса). Очевидно, что полярная окружность к произвольно выбранной точке на окружности Os проходит через точку S. Заставим теперь S пробегать всю дугу а/, при этом полярная окружность Os «за- метет» заштрихованную на рисунке 91 область. Площадь этой области составляет ^-=—-ю долю площади всей сферы. 2л л Окружности, полярные к любой точке этой области, и только они, пересекают дугу а(. Построим аналогичным образом такие же области для всех дуг ait а2, .... Они не покрывают полностью сферу в силу условия Zj — <1. Следовательно, найдется не- л с покрытая этими областями точка. Плоскость полярной к ней окружности искомая. 44.01. Остаток равен 7 (остатку от деления 35 на 14). 44.03. Наложим второй круглый лист на первый так, чтобы драконы совпали; тогда глаз нарисованного на втором листе дра- кона совпадает с глазом первого дракона» т. е. находится в его центре О. Разрежем второй лист по дуге АпВ (см. рис. 92) и наложим отрезанную часть на соответствующую часть первого круга; глаз дракона окажется в центре. 44.04. Обязательно. Указание. Если [VV*J—то <х<(п + I)4. Теперь легко доказать, что [д/[7*|] = п. 44.05 . Легко проверить, что, зная сумму масс любых двух гирь, можно определить массу каждой гири в отдельности. Поэ- тому поступаем так. Выбираем произвольные две пары гирь и взвешиваем каждую пару. Тем самым мы определяем, есть ли хотя бы в одной из них гиря массой 1000 г. Если есть, то третьим взвешиванием одной из гирь соответствующей пары опреде- 267
ляем ее, если нет, то оставшаяся пятая гиря имеет массу 1000 г. 44.10. Обозначим наименьшее общее кратное всех чисел через А. Тогда A = = ... = Люаю, где fcj.— натуральные, и из условия получаем: k\ >Л2>...> Л9>Лю. Значит, минимальное значение kx равно 10, т. е. А = k\aK 10«|. 44.12. Нетрудно видеть, что если а и b — натуральные числа, причем a^k и b^fe, то справедливо хотя бы одно из неравенств: |a + b|<b-l, la-b\^k-\. Поэтому найдутся наборы знаков « + » и « —» такие, что одно- временно будут выполнены неравенства 1п//±Нл-11 — 1, la/. + an-i —2. I Ort ± о>п~ । ± ... ± Oj I I • Но число ап ±оЛ-1 ±...±О| четно, поскольку сумма си + .-.+Яп четна. Значит, последняя сумма равна нулю. 44.13. Указание. Докажите сначала следующее утвержде- ние: если М — выпуклый многоугольник, S — его площадь, Р — периметр, г = г(М)—радиус наибольшего расположенного внутри М круга, то у-г<^-<г. Отсюда |^<г(Х)^г (У)<2у^-. 44.14. Можно. Сначала рассмотрим два множества натураль- ных чисел Л и В: в Л входят все натуральные числа, у которых на четных местах в их десятичной записи, считая с правой последней цифры, стоят нули, а в В входят все натуральные числа, у которых нули стоят на нечетных местах. Каждое натуральное число п единственным способом представляется в виде п — a-j-b. а£А, Ь£В. Искомое разбиение натурального ряда на подмножества Ль Л2, Аз, ... таково: Л|=Л, а каждое Ak (Л = 2, 3, ...) получается из множества Л прибавлением ко всем его элементам числа bk£B, т. е. Л2, Лз, ... — сдвиги множества А на соответствующие элементы множества В. 44.15. Справедливо общее утверждение: если у правильного п-угольника (п^4) отмечено + + l вершин, то существует трапеция с вершинами в отмеченных точках. Для и =1981 получаем Л = 64. 44.16. Из условия следует равенство f (x + k) = \ . Тогда /(х + 2Л)=—f (x + 4fe)= -f M+l2~k=f (4 Значит, f(x) — периодическая функция с периодом 4Л. 44.17. Рассмотрим многочлены Pi (х) = Р (х)—Р (х — I), Рг (х)= = Р1(х)-Р1(х-1), Р;(х) = Р*_, (х)-Р,_, (х-1) Рп(х) = = Pn_i (х)—Р„_| (х — 1) (многочлен Р, (х) называется i-й раз- ностью многочлена Р (х)). Нетрудно проверить, что степень много- члена Р, (х) равна п —i и что Р, (х) при любом целом х делится на р. Но Р„(х) = п!, поэтому п! кратно р. 268
44.18. Воспользуемся следующим тригонометрическим нера- венством: |sin (а —р)| < |sin а| + |sin р|. Пусть sin(£2)->0. Выберем е< <-|-|sin2| и такое N, что |sin (п2)| <е при любом n>N. Ис- пользуя тригонометрическое неравенство дважды, получаем: Isin «л + I)2 —л2)| = |sin (2л 4- 1)| < |sin (л 4- 1)2| + Isin (л2)! <2е, Isin ((2л 4-3) —(2л + i))| = Isin 2| < |sin (2л 4-3)1 4- Isin (2л 4- 1)1 < <2е4~2е = 4е, откуда следует противоречивое неравенство: Isin 2| <4e<4.1^1=4-|sin 2|. о 2 ал ол п ______ *^2 _ ___ 24 ___ 16 44.20. а\ » ^2 =“т—, а.з =—— • /15 44.21. Если n = 2k. то наименьшее число перестановок равно 2»Cl = k (Л—1); если и = 2А4“1, то оно равно Cl+Ck+\=k2. Указание. Наименьшее число перестановок получается, если всех людей разделить на две одинаковые (в четном случае) или отличающиеся на единицу (в нечетном случае) группы, разделив круглый стол по диаметру, а затем в каждой поло- вине сделать сначала перестановку одного человека со всеми людьми этой половины, затем перестановку второго со всеми (кроме первого), затем — третьего и т. д. 45.02. Проведя через центр квадрата прямые, параллельные его сторонам, разрежем квадрат на 4 одинаковых квадрата. Какие-то 2 из 5 точек лежат в одном из этих квадратов, и рас- стояние между ними не превосходит длины диагонали этого квадрата. 45.03. Любое вычисление числа а состоит из вычислений цепочки чисел: ао = 1, «ь аг, ...» fln-i, ал, где либо ak = Sak-\, либо ak = ak-14-4. Очевидно, вычислимы только нечетные числа, так как если ak нечетно, то числа За* и а* 4-4 тоже не- четны. В частности, 1982 невычислимо, а 1981 вычисли- мо. Стоимость набора числа 1981 равна 42 к. 45.04. 8 точек. Очевидно, что 8 точек Ль Л .. Ав, распо- ложенных на расстояниях Л|Лг = 1, Л2Л3=2, .... ЛтЛв=64, удовлетворяют условию. Покажем, что меньшего числа точек на плоскости расположить нельзя. Для каждого k=0, 1, .... 6 выберем пару точек, между которыми расстояние равно 2*, и соединим их отрезками. Из неравенства треугольника следует, что полученные 7 отрезков (и никакая часть из них) не обра- зуют замкнутого многоугольника. Следовательно, число точек должно по крайней мере на 1 превосходить число этих отрезков, т. е. быть не меньшим 7 + 1=8. 45.05. 1 +\/5. 45.07. Не может. Указание. Полученное число дает при делении на 3 остаток 2 и потому не является полным квадратом. 269
45.08. Из условия следует, что EDCD' и DCBC'— параллелограммы (рис. 93). Поэтому ЕС' = ЕВ — С' В = ЕВ — DC= = EB-ED' = D'B. &ЕС'В'со дДС'В, aED'Cco £±AD'B, откуда J АВ С'В ED' ЕС _ АВ АВ _ \ ЕВ' + В'С ~ ЕВ' + ЛВ ~ ЕВ' ‘ 1±АВ Обозначив х=^-, получаем уравнение корень которого х = Л^~1 (отрицательный корень отбрасываем). Значит, .££-= 1 1 4-^L-L=-Li3^. в в силу симметрии остальные отношения равны тому же числу. Замечание. Любой такой пятиугольник можно получить из правиль- ного пятиугольника некоторым аффинным преобразованием (например, сжа- тием вдоль произвольно выбранного направления). 45.09 . Доказательство проведем по индукции. Для п = \ вер- но ai 4-ai (ai — 1)2^0. Предположив, что при я=Л неравенство справедливо, дока- жем его для п = £4-1. Пусть ai<...<a*+i — произвольные нату- ральные числа. По предположению индукции + +(af 4~... 4~ajQ^2 (а?4-... 4"я2)2- Но 2al+i 4"4a2-|-i + 2a£_|_14~4a*_|_| (l34-23 +... + a?+ + я*+ ..• + (<**+! — 1)3) = ^2a* + i 4-4q3+j + i — l)2a*+i =a*+i+a* + i (здесь мы суще- ственно использовали, что все сц различны). Сложив эти нера- венства получим: (а{ +... + a7k +1)+(а? +... + al +1)2 (a? +... + 4-a* + i)\ что и требовалось доказать. Равенство справедливо при ai = l, fl2 = 2, ..., ап = п. 45.10. Все числа вида 6Л-Н и 6Л4-2, Л = 0, 1, 2, .... 45.11. См. решение задачи 23.20. 45.12. Легко видеть, что точка д(\/2;-^ удалена от всех узлов квадратной решетки на различные расстоя- ния. Измерим расстояния от А до всех узлов решетки и рас- положим их по порядку: Ri — < j?i982</?i983<-.. • Тог- да окружность с центром в точке А и произвольным радиусом R, где Rigw<.R<. Я|98з, содержит внутри себя ровно 1982 узла решетки. 45.13. Указание. Наше число является бесконечной перио- дической дробью ^-=0,(123456790) и поэтому не содержит даже цифры 8. (Подробнее см. в книге [24], задача 92.) 270
45.14. P = 2-J-4 sin 15°. Указание. Сумма диагоналей четырехугольника ABCD больше суммы противоположных сто- рон. и поэтому противоположные стороны не могут равнять- ся 1. Следовательно, можно считать, что АВ = ВС=1, причем Х.АВС^60° (поскольку ЛС^1) и точка D лежит в секторе АВС круга радиуса 1 с центром В. Теперь нетрудно показать, что пери- метр будет наибольшим, если D лежит на окружности, притом на равных расстояниях от Л и С, и X-АВС=60°. 45.16. Обозначив левую часть неравенства через f (а, Ь, с), заметим, что f (a, b, c)=(a-f-b+c) (abc—(Ь4-с—а) (а+с—Ь)Х X(b-f-a— с)). Среди чисел Ь-\-с—а, а+с — b, b-j-a— с не более одного отрицательного (если а-\-Ь — с<0,6-}-с — а<0,то 26 < 0). Если отрицательно ровно одно из этих чисел, то их произведение неположительно и поэтому f (а, Ь, с)^0. Если же они все неотрицательны, то а2Ь2с2^(а2—(Ь — с)2)^2— —(а —с)2) (c2-(a — b)2)=(b + c — af-(a + c — b)2 (b-\-a — cf=> =$-abc—(64-c—a) (a-}-c — b) (6-}-a — c)^0, откуда f (a, b, c)^0 при любых неотрицательных a, b, c. 45.17. а) Обозначим <p(x, i/)=xi/4-*4-1/+ 1. Многочлен x+a при любом аУ=1 вычисляется по такой программе Р(х+а): х; а-2;Ф(х, ; ч>(Ф(х, , а-2). При а=1 программа Р(х-|-1) такова: х; 0, <р(х, 0)=х-{-1. Для любого п программа P(fn) вычисления многочлена fn (х)= = 1 +х+... -{-Xя строится индукционно с помощью «схемы Горнера»: P(f„-i); P(fn-’t — 1); Р(х— 1); <р(fn_t — 1, х— 1); P(<p(fn-i — 1, х—1)4-1) (так как fn (x)=x-fn--i (х)4-1 =<р (fn_| — 1, х—1)4-1)- б) Пусть gi, g2, ... gn — произвольная программа. Индук- цией по п докажем, что если gn^feconst, т о gn (—1)= = — 1. Для и=1 это очевидно: gi (х)=х. Далее, gn=gigj+gi+ +gj(i. j<ti\ и, например, gt^const. Тогда g< (—!)= — ! (пред- положение индукции) и £«( —1)=—£/(—1)—14-gj(—1)= —1. что и требовалось доказать. Но gn=fn по условию; однако f„(—1)=0 или fn(—1)=1, a g„(—1)= — 1. Полученное противоречие доказывает, что для fn (х) искомой программы (с указанной операцией калькулятора) не существует. Замечание. Все многочлены, вычислимые иа калькуляторе с операцией <р(х, у)—xy-hx-hi/, имеют вид Fn (х)=А (х-J- 1)я— >• и любой такой многочлен вычислим. 45.18. п = \ или и=4. Указание. Покажите, что л = 2в*5* и п +1 =2C‘5J (а, 6, с, d — целые числа). 45.19. Пусть центр О большого шестиугольника Q не лежит в малом 6-угольнике q. Обозначим через d радиус вписанного в Q круга (одновременно это диаметр круга, вписанного в q, т. е. расстояние между его противоположными сторонами). Опустим 271
из О перпендикуляр ОМ на самую дальнюю сторону q\ легко видеть, что |0М| >d. Поэтому точка Н на пересечении ОМ и вписанной в Q окружности лежит ближе к О, чем М. Проведем через Н касательную к этой окружности до пересечения с Q в точках А и В; очевидно, отрезок АВ параллелен стороне малого 6-угольника и длиннее ее. Но это немедленно ведет к противоре- чию: легко показать, что отрезок касательной АВ к окружности, вписанной в правильный многоугольник со стороной а, имеет длину меньше Замечание. Приведенное рассуждение годится для произвольного пра- вильного л угольника, где п — четное число. 46.01 . (4, I); (4, -3); (-4, 1); (-4, -3). Указание. Пред- ставим правую часть уравнения в виде (у-\-1)2+12; перенеся G/+1)2 влево, получим, что (х — (/— 1) (х+у +1) = 12. Заметив, что каждая скобка — четное число, получаем 4 возможности: одна из скобок равна ±2, вторая + 6, откуда легко следует ответ. 46.02 . Две вершины равностороннего треугольника со сто- роной /, имеющие одинаковый цвет, искомые. 46.03 .-Обозначим искомое число через Х = 4аЬ...с = 4- Юл4-Л, где А=аЬ...с (Л — n-значное). После перестановки первой цифры в конец получаем число У=а6...с4 = ЮЛ-J-4. По условию 4- 10Л4-Л =4 (ЮЛ 4-4), откуда 39Л = 4-99...96 или 13Л = 4-33...332. Производя деление Л — I л— I на 13 «в столбик», находим наименьшее значение Л: Л = 4 - = = 10 256. Следовательно, наименьшее значение X равно 410 256. 46.0 5. Нет. Указание. Докажите сначала, что стороны длин 11 и 13 должны лежать рядом. Воспользуйтесь тем, что отрезки сторон от любой вершины до точек касания равны между собой. 46.0 6. Левая часть равна Хх^_Уу • Поскольку л>д/2 и у>у[2, то х54-</5>2 (х3+у3) и * >2 х = 2 (х2 — ху + у2)^х2-\-у2. * + У х+у 46.07. Заметим, что АА\=АС 4-СЛ|, ВВ\=ВА-\-АВ^ СС| = — = СВ -\-ВС\ (таким образом, стороны треугольника АВС ориен- тированы против часовой стрелки). Сложив эти равенства, получим: ААх + BBi + СС, —(АС + СВ + ВА)+(АВ, + САХ + ВСх). Сумма векторов в первой скобке равна 0, а каждый вектор во второй скобке получается из соответствующего вектора первой 272
скобки поворотом его на 60° по часовой стрелке; отсюда следует, что и вторая сумма равна 0. 46.08. Может, Одним из таких чисел является N = 999...995 (1983 девятки). Действительно, /V2 = (101984 — 5)2 = 1021984 - 10- 1019844*25 = 99...9900...025 (1983 девятки и 1983 нуля). Вообще, годится любое число, у ко- торого первые 1983 цифры — девятки, а 1984-я цифра не меньше 5. (Ср. с задачей 15.23.) 46.09 . Существует, и притом единственная. Найдем эту рас- становку. Проведем ось симметрии через вершину с номером 1983 и будем считать, что она вертикальна, а вершина 1983 лежит на верхней полуокружности. Ось симметрии разбивает окруж- ность на левую и правую половины. Номера вершин на левой половине обозначим (отсчитывая от вершины 1983 против ча- совой стрелки) через аь аг, ...» аээо, аээь а номера симметричных им вершин — буквами bi с теми же индексами. Пусть aj>&i, Q2>bi, .... а99о>6э9о, «991>б991. Проведем теперь ось симмет- рии через вершину 6991 (она пройдет между вершинами 1983 и ai). Так как расстановка «хорошая» и относительно этой оси, а 1983 >О|, ТО ..., ^989 990, ^990 ^>#991- ОбЪ- еднняя эти и предыдущие неравенства в одну цепочку, получаем: 1983>Ц| >&| >П2>^2>.-.>а990>^990>«991- ОТСЮДЗ ВСЯ рЗС- становка полностью определяется: «| = 1982, Ь\ = 1981, ..., «990 = 3, 6990 = 2, д99| = 1. Простая проверка показывает, что эта расста- новка является «хорошей» относительно любой оси симметрии. 46.10. Указание. Докажите, что треугольники /7/7|2/7ц и подобны, где — основание перпендикуляра, опущен- ного из точки И на -прямую АД. 46.11. Достаточно доказать, что f (х)=х2п—х2п~ ] х2п~2— —... — х3 4-х2 — %+ 1 >у- Для любого положительного х. Но f (Х) = Ц“Т—; если U то, очевидно, f(x)^l; если же х<1, 1 -|-Х то знаменатель не превосходит 2 и опять дробь больше 46.12. . 46.13. Указание. Докажите общее утверждение: S= ln4* 4-2"4-—4-л'1 при нечетном п делится на 1 +24--.. + **=” (т. е. 2S делится на п и на п 4*1)- Для этого просуммируйте по- парно крайние члены в сумме S, а затем отбросьте член п" и просуммируйте таким же образом оставшуюся сумму. 46.14. Рассмотрим произвольный город А. Предположим, что из него можно добраться только в 20 городов. Множество из п городов, в которые можно добраться из А обозначим че- рез Х\ множество оставшихся 20— п городов обозначим через 10 Заказ 24 7 2 73
У. Тогда ни из одного города множества X нельзя попасть ни в один город множества У, поскольку в противном случае из го- рода А можно было бы попасть в какой-то город множества У. Итак, среди С2о=19О возможных прямых авиалиний отсутствуют л (20 — п) авиалиний. Но п (20 — 19 при любом 0<п<20, и тогда 20 городов оказываются соединенными менее чем 172-мя авиалиниями, что противоречит условию. Стало быть, наше предположение неверно и утверждение задачи доказано. 1 46.16. Достаточно доказать, что из делимости 4m“n —1 на 3*+1 СЛедует, что (т — п) делится на 3*. Это следует из того, что наименьшее а такое, что 4°—1 делится на 3*+1 равно 3*. 46.17. Студент прав. Сумма равна 10. Указание, cos 0-f- 4-cosр...4-cos ^- = cos }-cos cos ^- = 0. <) Э Э D Э 46.18. а) Выберем точку А, из которой выходит максималь- ное число отрезков; обозначим это число через п. Если бы отрезки не образовывали ни одного треугольника, то вторые кон- цы отрезков, выходящих из Л, не были бы соединены между собой. Из каждой из оставшихся 7 —л точек выходит не более п отрезков, поэтому общее количество отрезков не превосходит л+(7 —л) л = л (8 —л)^ 16, а их по условию 17. Противоречие. б) По доказанному в пункте а) существует треугольник ЛВС, составленный из наших отрезков. Сколько к нему может примыкать (общей стороной) треугольников? Если лл, пв и пс — количества отрезков, выходящих из вершин Л, В, С (не считая сторон треугольника ЛВС), то таких треугольников не мень- ше пА +пв + пс — 5. Если пА 4-Лд + лс^8, то вместе с треуголь- ником АВС уже имеются 4 треугольника, составленных из наших отрезков. Пусть теперь лл+лв4-лс^7. Без ограничения общ- ности можно считать, что пА+пв^4. Выкинем точки Л и В и все выходящие из них отрезки (всего лл4-лв4-3 отрезков). Останется 6 точек и не менее 17 — 7=10 отрезков. Остается проверить, что они образуют 3 треугольника. Один треугольник есть в силу рассуждений пункта а). Далее рассуждениями, ана- логичными приведенным, сводим задачу к проверке того, что 5 отрезков между 4-мя точками образуют 2 треугольника. Но это уже очевидно. 46.19. Пусть В|, ..., В/—богатыри из некоторого города М. Предположим, что кубки обошли полный круг. Тогда у каждого из богатырей В, побывал каждый из золотых кубков, следова- тельно, они держали золотые кубки в руках kl раз. Поскольку \<Zk<Z 13, kl=£ 13. Если kl> 13, то в какой-то момент у богаты- рей из М было 2 золотых кубка одновременно, и все доказано. Остается случай kl <13. Но тогда в некоторый момент у богатырей из М не было ни одного золотого кубка. Поскольку число золотых кубков равно числу городов, в этот момент 2 зо- лотых кубка были у богатырей какого-то другого города N, что и требуется. 274
47.01. Могут. Например, годятся би- леты 159 999 и 160 000. Вообще, годится любая пара номеров, первый из которых имеет вид а&£Г999, если a-f-b равно б или 13. Можно доказать (сделайте это сами), что других подходящих пар нет. ТСм. 30.01 и 08). 47.02. См. решение задачи 33.27. 47.03. 25 р. (одолженные и возвра- щенные соседу деньги можно не прини- мать во внимание). 47.04. Пусть сначала все стороны па- раллелограмма попарно параллельны сто- ронам треугольника. Очевидно, наиболь- ший из таких параллелограммов распо- ложен так, как на рисунке 94, а. Если S — площадь ДЛК£, Si и S2- площа- ди £±ВКС и &DCL, то Si=X2S, S2=(1-X)2S, где Л=^, 1-Л=^- коэффициенты подобия, и Si4-S2 = -g—}- 2 4-2^Х—0 j S не меньше чем -i-S ( равенство достигается при м-)- Если же одна из сторон параллелограмма не параллельна стороне треугольника, то можно через одну из вершин треуголь- ника провести прямую так, как на рисунке 94, б. Она разрезает треугольник и параллелограмм на два меньших, и в каждом из малых треугольников одна из сторон параллелограмма уже па- раллельна стороне треугольника. Проделав такую операцию один или два раза, мы сведем общую задачу к уже рассмотрен- ному случаю. 47.05. Если предположить, что король ни разу не встал под шах, то каждая ладья должна была по ходу игры сменить как свою первоначальную вертикаль, так и свою первоначальную горизонталь, т. е. сделать не менее двух ходов. Отсюда общее число ходов всех ладей не меньше 20. Но это число долж- но совпадать с числом ходов короля, т. е. быть равным 19. Противоречие. 47.06. Из данного уравнения следует, что л3 = (4 — х2) д/4—х~ = (4 — х2)2 . Возводя обе части в квадрат и извлекая кубический корень, получаем: 4 —х2 = х2, откуда х2 = 2 и х= Ч=л/2- Проверка показывает, что отрицательный корень посторонний. Поэтому х=^2 — корень данного уравнения. №♦ 275
Рис. 95. 47.07. Расположим 6 точек (ЭВМ) в вершинах правильного шестиугольника. Его стороны закрасим через одну цвета- ми 1 и 2, а диагонали — цветами 3, 4 и 5 (одним цветом закрашиваются 2 па- раллельные малые диагонали и перпенди- кулярная к ним большая). , । 47.08. Пусть Ai — вершины данного 7-угольника, О — его центр, В\ — произ- вольная точка, отличная от О. Повернем хСХ 2л г, 2л ~ 2л вектор ОВ\ на углы —, 2~ ..., 6-—, получим 7 точек В2 ...» В7 в вершинах правильного 7-угольника. Легко видеть, что сумма длин векторов Д|В|, ..., Л1В7 равна сумме S длин векторов Bi/h, ..., ВИИрис. 95). Но ^-(Л^В14-... + Л^В7)= = Л|О, откуда по неравенству треуголь- ника 7- |ЛГО | С M’.Bi | + ... + I АхВ7| =Х. 47.09. Обозначим данные числа через х..... хг>. Из очевидного неравенства (*|~4") + —+(*s—4) следует, что + ... + .V5) — 5-^- =-i-. о zo э Так как lM*i+"+*s)2 = *i+” + *| + 2((xlX2-b*2X3-|-X3X4-l- +х4х54-х5Х|)-Ь(х|Хз-Ь хзх5-Ь х5х24- Х2Х4 -Ь x4xi)), то, обозначив две суммы в круглых скобках через и S2, получаем: 2(21-Ь2г)^ | 4 2 —5” ““5“’ откуда Следовательно, меньшая из сумм Si и S2 не больше -i-. О Замечание. Можно доказать общее утверждение: п неотри- цательных чисел с суммой / можно расставить по окружности так, чтобы сумча всех п попарных произведений соседних чисел была не больше — • Для этого надо рассмотреть суммы Zi, X?,... попарных произведений соседних чисел но всевозможным расстановкам их на окружности. Каждые два слагаемых х,х и Xk*t будут во всех этих суммах встречаться равное число разь Из того, что х»Ц-.. и (xt4-...-|-ли)‘= I следует, что среднее арифме- тическое всех сумм Xi, S2, --- не больше — , значит, одна из этих сумм не I превосходит —• 47.10. Решение показано на рисунке 96. 276
47.11. He является. Указание. Введем новую «цифру» ♦, обозначающую число 10, и обозначим А =***...** = 111...110.. 64 64 Число Л, как нетрудно видеть, кратно 101. Для каждого 64-знач- ного числа а = а|а2-..Дбзвб4, удовлетворяющего условию задачи, рассмотрим «симметричное» ему число р = (* — аО — о^). Оно тоже удовлетворяет условию, поскольку кратно 101 и каждая его цифра (♦—а,) отлична от нуля. Могут ли совпасть два сим- метричных. числа? Если а = р, то а, = * — at при всех i и тогда вес а< = 5: такое число единственное и равно 555...55. Остальные 64 числа разбиваются на пары (а, Р) неравных между собой чисел. 47.12. Обозначим через а, р, у плоские углы при вершине пирамиды. Выражая площадь каждой грани через длины ребер и синус угла между ними, получаем равенство sin a = sin p = sin у. Отсюда следует, что по крайней мере два из трех углов a, р, у равны между собой, а тогда две из боковых граней — равные треугольники и поэтому их основания равны между собой. 47.13. Нельзя, поскольку число проводов одного (произвола 13 ного) цвета в таком соединении должно быть равным —, т. е. не целому числу. 47.14. Равносторонний треугольник площади 1 имеет сторону а=— и высоту ft=V3; легко видеть, что он не поместится V3 в полосу, уже, чем h. Докажем теперь, что любой другой тре- угольник площади 1 поместится в полосу ширины Л. Предпо- ложим противное; тогда любая из его высот больше h и потому любая его сторона меньше а. Но треугольник, у которого 1 9 все три стороны меньше а, имеет площадь меньше — a sin а, где а — любой из его углов, и так как можно взять угол a ^60°, его площадь меньше 1, что противоречит выбору треугольника. 47.15. Пусть а, 6, с, d — четыре идущие подряд на окру ж ности числа, причем а^Ь\ тогда, очевидно, ab +bc-\-cd^ (^ + с)+(64-с) d. Поэтому, выбросив числа Ь и с и заменив их одним числом &4-с, мы не нарушим условия задачи и заменим сумму Sn на большую Sn-i. Проведя такую процедуру несколько раз, мы сведем дело к четырем числам на окружности; если х, у, z, t — эти числа, то s4—xi/4-t/z4-z/4-/x=(x4-z) (1 — (х4-г)Х~-±-=-^- 47.16. Указание. Докажите, что если наименьшее рас- стояние между четырьмя точками принимает максимально воз- можное значение, то они расположены в вершинах ромба со 277
стороной две противоположные вершины которого совпадают с противоположными вершинами заданного прямоугольника, а остальные две лежат на больших сторонах прямоугольника. 47.17. Существуют. Примеры: 333 ... 3342 = 11 ... 155 ... 56; 666 ... 6672 = 44 ... 488... 89. 47.18. Данное неравенство вытекает из цепочки: sin 1 <sin -5-=-^ <7-<log3 у/7 о Z О (последнее неравенство следует из того, что 37<74). 47.19. Можно, например, каждую задачу оценивать по 6-балльной системе (число баллов меняется от 0 до 5). Считая, что количество решаемых участником олимпиады задач равно 6» поставим в соответствие ему такое 8-значное число, две первые цифры которого выражают сумму набранных учеником баллов, а каждая из остальных шести цифр равна числу баллов за соот- ветствующую задачу. 47.20. Уравнение не имеет решений в целых числах. Ука- зание. Перепишем его так: 19 (х3 — 100) = 84 (1 +«/2); правая часть кратна 7, поэтому х3 —2 кратно 7. Но кубы чисел при деле- нии на 7 не дают в остатке 2. 47.21. Обозначим через d наибольший общий делитель всех данных чисел (т. е. d — наибольшее натуральное число, на которое делятся одновременно все числа пь п2, из, .-) и через ds — наибольший общий делитель чисел П|, п2> .... ns. Посколь- ку di>d2>x/3> ...» то = = ... = d для некоторого 6>2. Возьмем тогда числа ..., tik и рассмотрим все суммы, которые можно уплатить этими монетами. Эти суммы, расположенные в порядке возрастания, с какого-то места образуют арифмети- ческую прогрессию с разностью = эту же арифметическую прогрессию образуют и суммы, полученные из первоначального бесконечного набора монет {nJ, l^Ci<oo. Поэтому, добавив к монетам ...» nk необходимое количество монет n*+i, ...» nN. не попадающих в указанную арифметическую прогрессию, получаем искомый конечный набор монет п>. .... nv, которыми можно уплатить все возможные суммы, образованные из исходного набора монет ль л2, л3, ... . 47.22. Пусть А и В — две вершины треугольников, лежащие внутри квадрата. Очевидно, к каждой из них прилегает не менее 5 треугольников. Итак, мы уже насчитали 10 треугольников, из которых только 2, быть может, сосчитаны дважды (прилегающие к стороне Л В, если такая сторона есть). Следовательно, число треугольников не меньше 8. Остается разобрать случай, когда внутри квадрата лежит не более одной вершины, а осталь- ные лежат на сторонах квадрата или на сторонах других треуголь- ников. Этот случай разберите сами. 47.23. Пусть АВС — треугольное сечение куба с ребром 2 (площадь грани куба равна 4), касающееся вписанной в куб 278
сферы в точке Н. Обозначим через С', В', А' точки касания сферы с теми граня- ми куба, на которых лежат отрезки АВ, АС и ВС соответственно (см. рис. 97, а). Легко видеть, что &АС'В= &АНВ, &ВА'С=&ВНС, &СВ'А = &СНА. Сле- довательно, площадь треугольника АВС равна сумме площадей треугольников АС'В, ВА'С, СВ'А. Выразим площадь каждого из трех последних треугольников через величины х, у, z — расстояния меж- ду каждой из точек Л, В, С и серединой того ребра куба, на котором она лежит (расстояния берутся со знаком; см. рис. 97,6). Эти три площади равны -i-(l — ху), yz), zx}, и нетрудно дока- зать, что если |х| < 1, |у|<1, |z| < 1, то |-(1 -ZJf)<2- Рис. 97. 48.01. Одно из чисел равно 1, а дру- гое произвольно. 48.02 . Ровно одним способом. 48.03 . Можно. Указание. См. ре- шение задачи 23.27. 48.04 . Может. Для этого заяц должен придерживаться такой стратегии. Сначала он выбирает произвольную вершину А ква- драта и бежит к ней по диагонали с мак- симальной скоростью до тех пор, пока не окажется от А на расстоянии, меньшем -i—(д/2—1,4) (например, на расстоянии 0,005; сторону квадрата полагаем рав- ной 1). Затем он, не меняя скорости, сво- рачивает на 90° и движется перпендику- лярно диагонали к той стороне квадрата, на которой находится только один волк риваемый момент в А находится волк, то (если в рассмат- заяц сворачивает на 90° в произвольную сторону; рис. 98). Нетрудно видеть, что в момент, когда заяц пересечет сторону квадрата, ни один волк не сможет оказаться в той же точке этой стороны. 3 а- мечание. Если скорость волка в д/2 раз больше скорости зайца, то волки уже ловят зайца: они в каждый момент ока- зываются в концах «креста» с центром «заяц», отрезки которого параллельны диагоналям квадрата. 279
48.05. Указание. Сначала за 5 взвешиваний следует получить в одной фляге 17 л молока, а две другие* сделать пустыми. Затем за 2 взвешивания наполнить 17-ю литрами оставшиеся пустые фляги, получить сливанием 34 л (без взвешивания), восьмым взвешиванием еще 34 л и, наконец, слить вместе 34 л, 34 л и 17 л. Замечание. Аналогичным способом можно отлить (2я* 4-1)(2Л2 + 1) ••• (2Л*+1) л молока за JV —(и, 4-1)-}-... + (nk4-1) взвешиваний. У нас 85=(224-1) (244-1) и N —8. 48.06. у=х или y—z. 48.07. Чисел akf не меньших 993, в точности 993. Поэтому хотя бы одно из них имеет номер, не меньший 993, и для него Л-а^9932. 48.08. Два узла (рис. 99, а). Действительно, уже впи- санный в квадрат круг накрывает не менее двух узлов. Для доказательства этого возьмем клетку, в которой находится центр круга, разобьем ее диагоналями на 4 треугольника и возьмем тот из них, в котором лежит центр. Тогда круг целиком накры- вает выбранный треугольник (рис. 99,6), что и требовалось. 48.09. Пусть в череде богатырей не существует богатыря наименьшего роста. Это означает, что для каждого богатыря найдется богатырь меньшего роста и тогда искомая цепочка легко строится последовательным выбором все меньших и меньших по росту богатырей. Если же имеется богатырь А\ наименьшего роста, то отбрасываем его и среди оставшихся выбираем бога- тыря Лг с наименьшим ростом (если же такого Лг нет, то для оставшихся проходит описанное выше рассуждение и утвержде- ние задачи доказано). Далее, отбрасывая А\ и Л2, из остальных 280
богатырей выбираем Аз с наименьшим ростом (если такого Аз нет, то все доказано), потом At, Аз и т. д. В результате получаем цепочку стоящих по росту богатырей At, Л2, Аз. 48.10. Пусть Z-Д ^60° — наибольший в треугольнике АВС угол, AD> 1 — биссектриса угла А. Проведем через точку D все- возможные прямые, пересекающие стороны угла А, и выберем треугольник АВС минимальной площади. Этот треугольник равнобедренный (сравните площади заштрихованных на рисунке 100 треугольников BMD и CND), и его площадь больше — . V3 48.11. Заменами х=х?, у=у\, z=z? и возведением обеих частей уравнения в квадрат приходим к задаче 48.06. 48.12. Могло случиться. Расположим сначала 50 аэродромов в вершинах правильного 50-угольника, а остальные аэродромы — в его центре. Тогда все самолеты из центра прилетят в вершины, а из вершин полетят в диаметрально противоположные вершины. Таким образом, все самолеты соберутся во всех 50 вершинах 50-угольника. Учитывая, что попарные расстояния между аэро- дромами должны быть различными, немного пошевелим описан- ную конструкцию, добившись выполнения и этого условия. 48.13. Девять узлов. Указание. Если квадрат 2X2 со- держит два узла на расстоянии 2^/2, то он, очевидно, накрывает все 9 узлов. Остается случай, когда квадрат содержит «домик»: 6 узлов по 3 в два ряда и один узел посредине в третьем ряду (рис. 99, в). Докажите, что проекция «домика» на одно из двух произвольно выбранных перпендикулярных направлений со- держит точки, удаленные друг от друга не менее чем на 2. (Мож- но также доказывать, что равнобедренный треугольник с основа- нием 2 и боковой стороной -у/5 единственным способом помещается в квадрат 2X2.) 48.14. Доказательство проведем от противного. Выб- рав любых двух человек А и В, среди оставшихся десятерых выберем таких людей Ct, Сз, ..., С*, каждый из которых знает ровно одного в этой паре. В силу предположения имеем Л>6, так как при Л^5 в рассмотренной группе 10 — k^5 че- ловек удовлетворяют условию задачи. Подсчитаем число N троек {Л, В, С,} двумя способами. Все- го имеется 12-11=66 пар {Л, В), и каждой паре отвечает не менее 6 человек С„ поэтому Л/^6-66. С другой стороны, С, можно фиксировать и искать для него такие пары {Л, В), в кото- рых он знает ровно одного чело- века. Если у С, есть п знакомых, то число искомых пар {Л, В} равно 281
n(ll-n)<30. Выбрать же Ct можно 12 способами, откуда N (5 • 6) • 12. Итак, 6 • 66 N 360 — противоречие, доказы- вающее утверждение. 48.15. Доказательство по индукции. Для п=3 утверждение верно; пусть оно верно для n = kt 2к = 7х2-]-у2, х и у нечетны. Рассмотрим две пары чисел: {-|-(х —у)\ -|-(7х+у)| и (х+</); -|-(7х—у)}. Для каждой пары усемеренный квадрат первого числа плюс квадрат второго дает 2*+1. Остается заме- тить, что в каждой паре стоят числа одной четности, а в разных — разной четности, поэтому числа одной из пар нечетны. 48.16. {О; 99; 49+gj. 48.18. Максимальную «сложность», равную 101, имеет число 2 I 1 / 1 \50 Г? т\ Т/ * *казание- Если аЛ=— имеет максималь- ную «сложность» среди чисел со знаменателем 2Л, то среди чисел со знаменателем 2л+1 наибольшую «сложность» имеет число an + i=(^\-, где аи2=2л+1 — т{. 48.19. По условию любые два множества пересекаются по’ одному элементу. Докажем, что все множества пересекают- ся по одному элементу. Предположим противное. Возьмем мно- жество № 1. В нем найдется элемент Л, который принадлежит по крайней мере еще 45-ти множествам — № 2, 3, 4, ... , 46, так как в противном случае общее число множеств не превосходило бы 44-454-1 = 1981, что не так. По нашему предположению, имеется множество, не содержащее элемента А. Оно пересекается по одному элементу с множествами № 1, 2, ..., 46 и поэтому содержит 46, а не 45 элементов. Противоречие. Ответ: 44-19854-1. 48.20. Проведем через три пары скрещивающихся ребер тетраэдра Т три пары параллельных плоскостей, получим парал- лелепипед П, диагоналями граней которого служат ребра Т. Объем П втрое больше объема Т, а расстояния между парал- лельными гранями П равны Ль Л2, Лз. Докажите сами, что объем П не меньше /11/12/13- (См. 3.03).
ПРИЛОЖЕНИЕ 1 XLIX олимпиада (1986 год) VI! класс 1. На листе прозрачной бумаги нарисован четырехугольник. Объяснить, как сложить этот лист (возможно, в несколько раз), чтобы определить, является ли исходный четырехугольник ром- бом. 2. Доказать, что ни для каких чисел х, у, г не могут одно- временно выполняться три неравенства Ixlcly — z|, lf/1 <|z—х|, |z| < |х — у\ 3. Три гнома живут в разных домах на плоскости и ходят со скоростями 1, 2 и 3 км/ч соответственно. Какое место для ежедневных встреч нужно им выбрать, чтобы сумма промежут- ков времени, необходимых на путь от своего дома до этого ме- ста (по прямой), была наименьшей? 4. Произведение некоторых 1986 натуральных чисел имеет ровно 1985 различных простых делителей. Доказать, что либо одно из этих чисел, либо произведение нескольких из них явля- ется квадратом натурального числа. 5. В кодовом замкё имеются ровно три кнопки с номерами I, 2, 3. Про код, открывающий замок, известно, что он трех- значен. Какое наименьшее число раз следует нажать на кнопки замка, чтобы он наверняка открылся? (Замок открывается, как только подряд и в правильном порядке нажаты все три цифры его кода.) VIII класс 6. См. задачу 1 с заменой ромба квадратом. 7. Найти все натуральные числа, непредставимые в виде разности квадратов каких-либо натуральных чисел. 8. Доказать, что если at = 1, ПРИ rt = 2, 3,10, то 0 <аю-л^< Ю-37°. 9. Квадратное поле разбито на 100 одинаковых квадратных участков, девять из которых поросли бурьяном. Известно, что бурьян за год распространяется на те и только те участки, у каждого из которых не менее двух соседних участков уже пора- жены бурьяном (участки называются соседними, если они имеют общую сторону). Доказать, что полностью все поле бурьяном никогда не зарастет. 283
10. Доказать, что система неравенств 1*1 > I# —Z-HI, \уI > lx—z + t |, И >1*—!/ + N, | /| >1* — y + z\ не имеет решений. IX класс 11. На листе бумаги отмечены точки Л, В, С, D. Распознаю- щее устройство может абсолютно точно выполнять два типа операций: а) измерять в сантиметрах расстояние между двумя заданными точками; б) сравнивать два заданных числа. Какое наименьшее число операций нужно выполнить этому устройству, чтобы можно было наверняка определить, является ли четырех- угольник ABCD прямоугольником? 12. Из точки М по плоскости с постоянной скоростью пол- зет муравей. Его путь представляет собой спираль, которая на- матывается на точку О и гомотетична некоторой своей части относительно этой точки. Сможет ли муравей пройти весь свой путь за конечное время? 13. Решить систему неравенств ( |х| < I//—z+/|, J \у\<1*—z+Л, I \z\ <|х — y+t\, ' |/| < |х — y + z\. (Ср. с задачей 10.) 14. Произведение некоторых 48 натуральных чисел имеет ров- но 10 различных простых делителей. Доказать, что из этих чисел можно выбрать такие четыре, произведение которых является квадратом натурального числа. (Ср. с задачей 4.) 15. На координатной плоскости нарисованы круги радиу- са с центрами в каждой точке, у которой обе координаты — целые числа. Доказать, что любая окружность радиуса 100 пере- сечет хотя бы один из нарисованных кругов. X класс 16. См. задачу 11 с заменой прямоугольника ABCD квад- ратом. 17. Биссектриса угла А треугольника АВС продолжена до пересечения в точке D с описанной вокруг него окружностью. Доказать, что АС). 18. Решить уравнение Xх =4(х>0). 284
19. Доказать, что ни для каких векторов а, Ь, с не могут одновременно выполняться три неравенства /3|а| < |6-с|, -^|5|<|с-а|, д/3|с| < |а-5|. (Ср. с задачей 2.) 20. Найти минимум по всем аир максимума функции у (х) = |cos % + a cos 2% + р cos Зх|. РЕШЕНИЯ УКАЗАНИЯ ОТВЕТЫ 49.01. Достаточно два раза перегнуть четырехугольник по его диагоналям. Если оба раза произошло совпадение наклады- ваемых треугольников, то четырехугольник — ромб. 49.02. Предположим, что указанные неравенства имеют ме- сто. Возведем почленно в квадрат каждое неравенство, пере- несем влево все правые части и разложим на множители полу- ченные разности квадратов. Получим: (X—y + z)(x+y — z)<0, (у — z + x)(y + z — х)<0, (2—x+y)(z+x — у)<0. Перемножив почленно все эти неравенства, получаем проти- воречие: ((* — У + г) (х+у — z) (у + z — х))2 < 0. 49.03. Этим местом встречи является дом первого гнома (который ходит со скоростью 1 км/ч). Для доказательства этого обозначим искомое место встречи гномов буквой Л, а дома зану- меруем цифрами 1, 2, 3 в соответствии с величинами скоростей гномов. Расстояния между домом 1 и домами 2 и 3 обозначим через а и Ь, а расстояния от точки А до домов 1, 2, 3 — через х, у и z соответственно. Тогда о х___i_jy_ __I 2 2^2’2’ 3 3 ’ 3 ’ откуда —Ч-—< —+ < х+^-4-— 2^3^2^2^3^ ~2“3 ’ причем равенство достигается при х = 0, т. е. когда точка А совпадает с домом 1. 49.04. Рассмотрим наборы исходных чисел, взятых во всех возможных количествах: по одному, по два, по три и т. д. Таких наборов, как нетрудно подсчитать, 21986 — 1. Числа в каж- дом таком наборе перемножим и полученное произведение пред- ставим в виде произведения наибольшего полного квадрата и не- скольких простых сомножителей (например, М = 2|6*3|5-513* I79 представляется в виде (28.37-56.174)2-3 - 5- 17, а М = 2|6-13|0 = = (28- 135)2). Сопоставим каждому набору исходных чисел тот на- бор простых чисел, который получается после выделения наиболь- 285
шего точного квадрата из их произведения (в рассмотренном примере числу N сопоставляется набор 3, 5, 17, а числу М — пус- той набор). Нетрудно подсчитать, что число различных наборов из 1985 простых делителей (по одному, по два, по три и т. д., включая и пустой набор) равно 21 , что меньше количества 21986—1 —наборов из исходных чисел. Поэтому каким-то двум наборам А и В из исходных чисел отвечает один и тот же набор Рь ..., pk простых делителей, т. е. Д=а2«р|*р2* ... -рл, B = b2-pi- ... *pk. Следовательно, произведение А» В есть точный квадрат. С другой стороны, А-В равно произведению чисел в наборе А и чисел в наборе В. Выбросив из наборов А и В их общую часть (произведение выбрасываемых чисел есть точный квадрат), получим, что произведение остальных чисел является точным квадратом, что и требовалось доказать. 49.05. Всего существует З3 = 27 различных трехзначных чи- сел, в записи которых участвуют цифры 1, 2, 3. Кроме первых двух цифр в последовательности нажатий кнопок, каждая из остальных цифр служит последней цифрой какого-то трехзначно- го числа. Следовательно, в искомой последовательности должно быть не менее 27+ 2 = 29 цифр. Двадцати девяти цифр уже доста- точно для открытая замка; например, годится последователь- ность 11123222133313121223113233211. 49.06. Если ABCD — ромб (см. решение задачи 49.1), то достаточно перегнуть бумагу так, чтобы вершина С совпала с вершиной В. Если при этом вершина D совпадет с Л, то ABCD — квадрат. 49.07. Решим уравнение х2 — у2 = п, где п — заданное нату- ральное число, х и у — неизвестные натуральные числа. Посколь- ку х2 —у2 = (х —у)(х + у), а числа х —у и х-|-у имеют одинако- вую четность, то и либо делится на 4, либо имеет вид 4й±1. Верно и обратное: все числа п вида 4Л и 4й±1 представимы в виде н = х2 —у2 (кроме чисел 1 и 4). Действительно, если (х —у) (х + у)=4Л, то достаточно взять х = й+1, у = й—1 (ис- ключение составляет к = 4, так как при этом у = 0 — не нату- ральное число); если же (х — у)(х + у) = 4£ ± 1 =2/-j- 1, то доста- точно взять х = /+ 1, у = 1 (исключение составляет п= 1, так как тогда у = 0). Следовательно, в требуемом виде непредставимы только числа л=1, и = 4 и все числа вида n = 4k-\-2. 49.08. Обозначим кп=——Докажем, что Xn + i = X2. Действительно, 1 rr- Л ____ ♦ I— + I । (6 Отсюда имеем: Х|0= Хд = Хв = ... =Х? , 286
т. е. Дю—/ <2-1 V’ Дю + <2 \ V2+1 ' ' Поскольку из неравенства, связывающего среднее арифметиче- ское и среднее геометрическое, следует, что (неравенство строгое, так как 09#=—-) » то Дю —V2 вю+у2 ою —V2 2 <2 Следовательно, 2 <2 (V2 + 0 Заметим, что (л/5+1)8 = (3 + 2 л/5)4=(17+12^2)2>(24 л/2)2 = = 1152>103, поэтому (V2+ 1)'024>(Ю3),28 = 10384>»2-^. Ю383 и г- 2 д/2 aio-V2 (^ + |)102< < 10 - 383. Таким образом, нами доказано даже более сильное, чем требу- ется в условии задачи, неравенство. Замечания. 1) Основная идея доказательства в этой за- даче та же, что и в задаче 16.37: после каждой итерации количе- ство знаков после запятой примерно удваивается, так что точ- ность вычислений экспоненциальная. 2) Однако из неравенства аю—-\/2<;1О_370 нельзя формаль- но заключить, что первые 370 знаков у чисел Ою и совпадают: для этого достаточно было бы удостовериться (лучше всего на ЭВМ), что в десятичной записи у2 цифра, стоящая после запя- той на 371-м месте, отлична от 9 (если она равна 9, то нельзя утверждать даже, что совпадает хотя бы одна цифра; вот контр- пример: 1— 0,99999999999= 10“|0). Как же в этом удостоверить- ся? Ведь, вычисляя по приведенному алгоритму, мы добиваемся только хорошей точности, но не точных цифр числа -\/2- Выход на самом деле очень прост: достаточно сделать еще одну, 11-ю, итерацию и убедиться, что 371-я цифра числа ап не равна 9. 49.09. Рассмотрим длину всей границы пораженной бурьяном области. Нетрудно проверить, что после возникновения новых «бурьянных» участков общая длина пораженной бурьяном обла- сти не увеличится. Поэтому если бы все поле когда-либо поросло бурьяном, то общая длина границы «бурьянной» области стала бы равной 40, хотя в начальный момент (а значит, и во все 287
последующие тоже) она не могла превосходить 4-9 = 36. Полу- чили противоречие. 49.10 и 13. Решение этих задач аналогично решению зада- чи 49.2. Проделаем следующие действия: 1) возведем все не- равенства в квадрат (знаки неравенства при этом не изменят- ся); 2) перенесем все правые части влево; 3) представим каж- дую получившуюся разность квадратов в виде произведения разности на сумму; 4) перемножим все полученные скобки в ле- вых частях неравенств. Все эти левые части отличны от нуля и имеют одинаковые знаки; значит, их произведение положитель- но. С другой стороны, оно равно — ((х —у+ z—f) (х + у —z-|-/)X Х( — x+y+z— /)(х —y + z+f))2, т- е- отрицательно. Противо- речие. 49.11 и 16. Чтобы определить, является ли ABCD прямо- угольником, достаточно проверить равенства Дв = С£), BC = AD и AC=BD— итого 9 операций (по 3 операции на каждое ра- венство: два измерения и одно сравнение). Прямоугольник ABCD будет квадратом, если АВ=ВС — для этого нужна еще одна, 10-я, операция сравнения длин отрезков АВ и ВС, Докажем, что меньшим числом операций в обоих случаях не обойтись, т. е. что все указанные операции обязательно нужно выполнить. Действительно, если мы не знаем, что какие-то две противо-- положные стороны четырехугольника равны, то нельзя даже утверждать, что ABCD — параллелограмм, ведь годится и равно- бедренная трапеция (ее диагонали равны). Поэтому проверка равенств AB = CD и BC=AD необходима. Если же мы не знаем, что AC=BD, то ABCD может быть произвольным параллело- граммом. Значит, необходима и проверка равенства диагоналей. Наконец, в случае квадрата нужна также проверка равенства двух соседних сторон. 49.12. Муравей проходит весь путь по спирали от М до О за конечное время. Действительно, если коэффициент гомотетии k<\, а первый виток муравей проходит за время /, то на остальные витки он потратит время kt, k2t, kst, ...; поэтому время на весь путь равно t (1 + k^k? + . 49.14. Решение аналогично решению задачи 49.04. Предста- вим произведение произвольной пары чисел (а, Ь) из данного набора в виде произведения квадрата натурального числа на произведение простых делителей в первых степенях (например, если a = 2I3-34- 193, 6 = 56-77- 19, то ab = K2-2-7, где /( = 26-32Х Х53-73- 192). Сопоставим паре (а, Ь) получившийся набор про- стых делителей. Всевозможных различных пар (а, Ь) в наборе из 48 чисел 6,48 = 48-—, а количество наборов из 10 простых делителей (по одному, по два, по три и т. д., включая пустой на- бор) 210. Так как 6?48>21(), то найдутся две различные пары (а, Ь) и (с, d) из набора, которым отвечает один и тот же набор 288
(pi, p2.. Pk) простых делителей (O^fe^IO). Следовательно, abed — точный квадрат. Если при этом пары (а, Ь) и (с, d) не имеют общего элемента, то числа a, b, с9 d искомые. Если же общий элемент есть, на- пример b=d, то тогда ас — точный квадрат. Выкинем на время пару (а, с) из рассмотрения. Тогда мы приходим к набору из 46 чи- сел, произведение которых имеет не более 10 различных простых делителей. Проведя те же рассуждения, что и выше, и учитывая, что верно также и неравенство Счб>210, приходим к выводу о сущест- вовании двух различных пар чисел (х, у) и (z, /) из набора, для которых xyzt — точный квадрат. Если общего элемента у этих пар нет, то х, у9 z9 t — искомые 4 числа; если же общий элемент есть, например х = /, то yz — точный квадрат. В этом случае искомой четверкой чисел является (а, с, у9 z). 49.15. На клетчатой плоскости со стороной клетки 1 выберем самую северную горизонтальную прямую y = k9 пересекающую фиксированную окружность радиуса 100 с центром О (прямая у = Л+1 окружности не пересекает). Если все узлы этой прямой лежат вне окружности, то легко подсчитать, что ближайший к окружности узел находится от нее на расстоянии, меньшем 1 » 1 —, поэтому она пересекает нарисованный круг радиуса — с центром в этом узле. Считаем поэтому в дальнейшем, что на прямой y — k некоторые узлы лежат внутри окружности. Выберем из них узел S, лежащий ближе всего к окружности. Через А обозначим ближайший к нему внешний узел на прямой y=k, т. е. АВ=\. Предположим, что окружность не пересекает нарисованных кругов радиуса с центрами А и В. Имеем тогда: ОЛ>Ю04“]у, 99<OB<Z 100 — , откуда ОЛ —Ов>-у-, ол2-ов2=(ол-ов) (од + ое)> 199 Если О' — проекция центра О на прямую y=k и О'В=х, то О'А = х-{-1, и (х-Ь I)2—х2 = ОЛ2— ОВ2>-^-, откуда О'В = — х>—- А тогда ОО'2 = ОВ2 - О'В2 <( 100 —i-)2 ~(^)2 < 992, откуда ОО'<99. Отсюда вытекает, что расстояние от центра О до прямой y = k+ 1, равное ОО'+ 1, меньше 99 + 1 = 100, т. е. наша окруж- ность радиуса 100 пересекает также и прямую */==£+1. Это противоречит сделанному вначале предположению и доказывает утверждение задачи. 289
49.17. Введем обозначения: АВ=а, AC=b, AD = l, Z.C=aa, = d — диаметр описанной вокруг треугольника АВС окружности. По теореме синусов длина хорды равна произведе- нию диаметра окружности на синус половины дуги, на кото- рую эта хорда опирается. Поскольку АВ = 2а, о АС=2р» = л —а + р, то a = d sin а, & = dsinp, l = d sin =d cos Цр. Из неравенства d cos > y-(d sin a -|- d sin P) и следует требуемое утверждение. 49*18. Очевидно, что х=^2— решение уравнения. Функций у=х“ монотонно возрастает на [1; 4-°°). поэтому на этом про- межутке других решений нет. На интервале (0; 1) решений тоже нет, так как на нем значения функции меньше 1. Итак, х=-^2 — единственное решение. 49.19. Возведем данные неравенства в квадрат и почленно сложим их все (учитывая, что для произвольного вектора х, \х\2=х-х=х2): 3 (а2 4- Ь2 4- с2) < 2 (а2 4- Ь2 4- с2)—2 (а • b 4- b • с + а • с), откуда (а4-64-с)2<0 — противоречие. 49.20. При всех аир справедливо неравенство шаху (х)> шах ({/(-£-) ; = max ( | ^4-f-| I 1-fi) i+i -fKf+ 4-тг) — ( 4--?-) I =-тг . поэтому min max у (x)^^-. Поло- жим теперь f(x)=cosx—|-cos3x и найдем точки экстремума этой функции из уравнения /' (х)=0: f' (х)= —sin x4--|-sin 3x=0, — 2 sin х 4-(3 sin х — 4sin3x)=0, откуда либо sinx = 0, x = kn, либо sin2x=-^-, х=±-^-4-/гл. Для функции y(x)=|f(x)| имеем: у (/гл)=-|*<"о^ > у( ±'Г_4"^Л) =-тг • Отсюда вытекает, О Лл \ О / что max у и поэтому min max — Окончательный ответ: min max у (%)=-— и достигается при а. р х »=о. Р=-т-
ПРИЛОЖЕНИЕ 2 Избранные задачи математических турниров В этом приложении приведены наиболее красивые задачи, предлагавшиеся в разные годы участникам математических кружков, победителям Московских олимпиад при отборе на Все- союзную олимпиаду, а также некоторые задачи из архива жюри Московских олимпиад, не попавшие ни на один из турниров и поэтому малоизвестные. После условия задачи стоит номер класса, школьникам которого она предназначалась. 1(7—9). Найти сумму цифр числа 123456789101112... ... 999998999999. Сколько в этом числе семерок? ________ 2(8—10). Найти наименьшее натуральное число ab ... с, не содержащее в своей десятичной записи нулей, которое в сумме со своим обращенным (т. е. с числом с... Ьа) дает число, цифры в котором получены перестановкой цифр исходного числа. 3(9—10). Существуют ли такие иррациональные числа аир, что аэ — число рациональное? 4(8—10). В вершинах куба поставлены числа 4-1 и — 1. В центре каждой грани поставлено число, равное произведению чисел в вершинах этой грани. Может ли сумма полученных 14 чи- сел оказаться равной нулю? а семерке? 5(8—10). В пространстве расположено конечное (но неизве- стное) количество звезд. Находясь в произвольной точке про- странства (пункте наблюдения), можно определить количество видимых из нее звезд и направления на них. Однако это наблю- дение может не дать достоверного числа всех звезд, так как звезды, вообще говоря, будут загораживать друг друга. Какое наименьшее число пунктов наблюдения следует выбрать, чтобы определить точное число звезд на небе? Решите ту же задачу и на плоскости. 6(7—9). На кольцевой автомобильной дороге стоят п одина- ковых автомашин. Общее количество бензина у всех этих автома- шин достаточно для того, чтобы одна из них смогла проехать по всей кольцевой дороге. При любом ли расположении автома- шин на дороге и распределении горючего по ним найдется такая автомашина, которая может, забирая по пути бензин у осталь- ных, проехать по всей кольцевой дороге? 7(8—10). Решить в целых неотрицательных числах уравнение х44-2х34-х2 — 11x4- 11 =у2. 8(8—10). Отметим на числовой прямой красным цветом все точки, отвечающие числам вида 81x4-Ю0у, где х и у — 291
целые числа, и синим карандашом остальные целые точки. Найти на прямой такую точку, что любые симметричные относи- тельно нее целые точки закрашены в разные цвета. 9(9—10). Доказать, что если целые числа x2t ...» хп, у\> ..., ут удовлетворяют неравенствам 1 < xi < х2 <... < хп < <... <; <Ут И Л'1 + ТО Х\Х2...Хп>У\У2 Ут- 10(10). Доказать, что любые два равновеликих многогран- ника можно разрезать на попарно равновеликие тетраэдры. 11(8—9). Даны квадрат ABCD и точка О внутри его. Дока- зать, что 135° < Z. О АВ + Z_ ОВС + Z. OCD + Z_ ODA < 225°. 12(10). В пространстве через точку О проведены п прямых так, что для любых двух из них найдется третья из того же на- бора, перпендикулярная первым двум прямым. Доказать, что: а) все прямые, кроме одной, лежат в одной плоскости, а эта последняя прямая перпендикулярна ей и б) п нечетно. 13(7—10). Квадратный пирог разделен двумя взаимно пер- пендикулярными прямыми на четыре части. Три из этих частей весят по 200 г. Сколько весит пирог? 14* (10). В п точках плоскости стоят прожекторы, освеща- ющие на плоскости углы он, аг, аЛ, причем он 4-а2 + ••• + = = 360°. Доказать, что прожекторы можно так поменять местами, не вращая, что они осветят всю плоскость. 15(8—9). Про выпуклый четырехугольник ABCD известно, что AC = BD, Z_ABD = 2 Z_ACD, AACD+AADB = 9Q°. Найти углы В н D этого четырехугольника. 16(9—10). Восемь точек лежат на поверхности куба с реб- ром 1. Расстояние между любыми двумя из них не меньше 1. Доказать, что эти точки — вершины куба. 17(9—10). а) Проекции тела на две плоскости в простран- стве — круги. Доказать, что эти круги одинаковы. б) Проекции выпуклого n-угольника на две непараллельные плоскости в пространстве — правильные n-угольники. Доказать, что эти проекции — равные п-угольники. 18(8—9). Конечно или бесконечно множество натуральных чисел п, таких, что сумма цифр числа 5" меньше 10,и()? 19(7—9). Доказать, что среди первых 10 миллионов цифр разложения \/2 ни одна цифра не встретится 5 000 001 раз подряд. 20(7—9). В Швамбрании 1000 аэродромов, все попарные рас- стояния между которыми различны. С каждого аэродрома под- нимаются по самолету и каждый самолет летит на ближайший аэродром. Какое наибольшее число самолетов может оказаться на произвольном аэродроме, если Швамбрания — а) плоскость? 6) сфера? 292
21(9—10). В кубическом сыре проделано несколько сфериче- ских дыр. Доказать, что можно разрезать сыр на такие выпук- лые многогранники, чтобы внутри каждого из них находилась ровно одна дыра. 22(10). Пусть sina=—. Доказать, что sin25a=~|j, где и э п — целое число, не кратное 5. 23(7—10). Три разноцветные лампочки — синяя, зеленая и красная — подсоединены как-то проводами к п переключателям. Каждый переключатель может находиться в одном из трех по- ложений. При любом положении переключателей горит ровно одна лампочка, но если переключить сразу все переключатели, загорится другая лампочка. Доказать, что цвет горящей лампоч- ки определяется одним фиксированным переключателем и не за- висит от состояния остальных. 24(8—10). По бесконечной шахматной доске с полями в ви- де квадратов со стороной 1 прыгает кузнечик, перемещаясь за каждый прыжок на а вправо и на р вверх. Доказать, что если числа а, р и у иррациональны, то кузнечик обязательно по- падет на черное поле. 25(9—10). Доказать, что tgTf+4sin^=ViL 26* (8—10). Найти все решения в натуральных числах урав- нения 520 (xyzt+xy+xz+zt+ 1)=577 (yzt+y + z). 27(7—8). Доказать, что если в каждый момент времени хотя бы один из десяти идущих равномерно будильников показывает правильное время, то среди них хотя бы один будильник идет правильно всегда. 28(9—10). Пространство разбито на одинаковые и одинако- во расположенные параллелепипеды. Доказать, что с каждым параллелепипедом имеют общую точку не менее 14 параллеле- пипедов. 29(8—10). Треугольная пластинка площади 1 двумя прямо- линейными разрезами, идущими от двух вершин к противопо- ложным сторонам, поделена на четыре части (три треугольника и четырехугольник), три из которых имеют равную площадь. Найти площадь каждой части. 30(8—9). Доказать, что если среднее арифметическое пер- вых 10,о,° цифр числа 2—-^2 заключено между 4-|- и 4-^-, то О о это же верно и для числа д/2—1. 31(8—10). Доказать, что в любой момент на поверхности Солнца (которая считается сферой) найдется точка, из которой видно не более трех планет (из девяти известных). 293
32(7—9). Имеется два сосуда. В первом находится 1 д воды а второй пустой. Из первого переливают половину имеющейся в нем воды во второй, затем из второго переливают треть имею- щейся в нем воды в первый, затем из первого переливают чет- верть имеющейся в нем воды во второй и т. д. Найти количество воды в первом сосуде после 12 345 переливаний. 33* (9—10). Доказать, что квадраты со сторонами М 3 ’ 1 2 ’ I 1 1 —, —, ..., —, ... можно без перекрытии разместить в квадрате со v 5 стороной —, но нельзя разместить ни в каком меньшем квадрате. 34(8—10). Даны деревянный шар, циркуль и лист бумаги. Начертить на листе бумаги циркулем окружность, радиус ко- торой равен радиусу шара. (На шаре циркулем можно чер- тить произвольные окружности.) 35* (9—10). Квадрат двумя способами разбит на 100 равно- великих частей. Доказать, что можно выбрать 100 точек так, чтобы в каждой части каждого разбиения оказалось ровно по одной точке. 36(8). Доказать, что число (,.+ ')(з.+4)(5.^)(7.+4)(9.+±)(„.+1) целое, и найти его. 37(8—10). Стол полностью покрыт 100 квадратными скатер- тями, в каждой из которых прожжена круглая дырка. Доказать, что уже какие-то три из этих скатертей полностью покрывают стол. 38(8—10). Дано 40 правильных игральных костей (сумма чисел на противоположных гранях кости равна 7). Они постав- лены друг на друга так, что получается параллелепипед. Верно ли, что их можно повернуть так вокруг вертикальной оси, что суммы чисел, стоящих на четырех боковых гранях параллеле- пипеда, будут равны между собой? 39(8—10). Даны две концентрические окружности и две па- раллельные хорды / и т, касающиеся внутренней окружности. На внешней окружности между / и т взята точка А и из нее про- ведены касательные к внутренней окружности. Точки пересече- ния касательных с хордами С и D, Доказать, что произведение AC-AD не зависит от положения точки А. 40(8—10). На бесконечной клетчатой бумаге со стороной клетки, равной 1, нарисована система параллелограммов с вер- шинами в узлах бумаги такая, что начало координат не явля- ется вершиной параллелограмма. Доказать, что существует такое целое Л, что если подвергнуть клетчатую бумагу растяжению 294
в k раз от начала координат, оставив параллелограммы на месте, то ни одна вершина параллелограмма не совпадет с узлами растянутой клетчатой сети. 41(8—9). Точки А, В и С двигаются равномерно по трем окружностям в одну и ту же сторону с одинаковой угловой ско- ростью. Как двигается центр тяжести треугольника АВС? 42(8—9). На доске выписаны числа 1, 2, 3, .... 1974. За один ход разрешается произвольные два числа из набора стереть, а вместо них написать в набор модуль их разности. После 1973 хо- дов останется одно число. Чему может быть равно это число? 43(8—9). Прямые а и b касаются окружности в точках А и В. На прямой а выбирается точка С, а на прямой Ь — точка D так, что отрезок АВ пересекается с отрезком CD в точке М. п СА DB Доказать, что 44(8—10). Число р простое. Дано р4-1 различных натураль- ных чисел. Доказать, что среди них найдется пара таких чисел х и у, что большее из этих чисел, деленное на их наибольший общий делитель, не меньше р4-1. 45(9—10). Числа аь аг.... а* таковы, что при всех х вы- полняется неравенство л S Ол cos — 1. п — I Доказать, что at 4-...-)-atC^- 46(9—10). Игра «Задумка» состоит в том, что нужно угадать за три вопроса задуманное пятизначное число, в записи которого имеются только нули и единицы. Угадывающий называет пяти- значное число, составленное из нулей и единиц, а загадавший говорит, в каком количестве разрядов названное число совпа- дает с задуманным. Сможет ли угадывающий определить заду- манное число? 47(9—10). На поверхности куба с ребром 1 расположена замкнутая ломаная линия. На каждой грани куба находится по крайней мере одно звено ломаной. Доказать, что длина ломаной не меньше чем 3-^2. 48(9—10). Доказать, что для любого натурального числа л^2 выполняются неравенства n(VMU-l)<i+4-+ ... +!-<!+„(!«). * п \ \п / 49(8—10). Какое наименьшее число полей можно вырезать из шахматной доски размером 9X9 так, чтобы по оставшимся полям шахматная ладья не смогла описать замкнутого пути, не проходя дважды по одному отрезку? 50(8—10). В круге радиуса 16 расположено 650 точек. Дока- зать, что найдется кольцо с внутренним радиусом 2 и внешним 295
радиусом 3, в котором лежит не менее 10 из данных точек 51(8—10). Существует ли такое натуральное число п, что любое рациональное число между 0 и 1 представляется в виде суммы п чисел, обратных к натуральным? 52(8—10). Правильный 2п-угольник вписан в правильный 2/г-угольник, т. е. каждая вершина 2п-угольника принадлежит контуру 2/г-угольника. Доказать, что 2k дрлтся на п. 53(9—10). Внутри куба расположен такой многогранник М, что его проекция на каждую грань куба заполняет эту грань. Доказать, что объем многогранника М не меньше одной трети объема куба. 54(9—10). Город, имеющий форму квадрата со стороной 10 км, разделен на п2 одинаковых квадратных кварталов. Квар- талы занумерованы числами от 1 до гг так, что два квартала, имеющие соседний номер, имеют общую сторону. Доказать, что велосипедист может найти нужный ему квартал, проехав не бо- лее 100 км. 55(8—10). В выпуклом пятиугольнике провели все диагонали, в результате чего он оказался разбитым на 10 треугольников и один пятиугольник. Из суммы площадей треугольников, приле- гающих к сторонам исходного пятиугольника, вычли площадь внутреннего пятиугольника; получилось число S. Потом анало- гичную разность вычислили для внутреннего пятиугольника; получилось число $. Доказать, что S>s. 56(8). Найти максимальное число вершин невынуклого неса- мопересекающегося n-угольника, из которых нельзя провести ни одной внутренней диагонали. 57(8). В целых точках числовой прямой расставлены какие-то натуральные числа. На первом шаге между каждыми двумя со- седними числами записывается их среднее арифметическое, а исходные числа стираются. На втором шаге с написанными числа- ми проделывается та же операция и т. д. Оказалось, что все числа, которые получаются на каждом шаге, натуральные. Мож- но ли утверждать, что на некотором шаге все числа были равны между собой? 58(9—10). Из 27 кубиков со стороной 1 составили куб раз- мером 3X3X3. Каждый кубик покрасили в белый или черный цвет. Через час пришел маляр и все кубики, имеющие четное число черных соседей, покрасил в белый цвет, а все осталь- ные — в черный. Эту процедуру маляр проделывал каждый час. Доказать, что через некоторое время все кубики станут белыми. 59(9—10). На плоскости расположено п материальных точек равной массы. Возьмем произвольную точку плоскости До и рас- смотрим те из наших точек, расстояние от которых до До мень- ше 1. Пусть Д| — центр масс этих точек. Теперь рассмотрим те из наших точек, расстояние от которых до Д| меньше 1; пусть 296
As — их центр масс. Аналогично поступим с Аг и найдем центр масс Аз точек, удаленных от А2 меньше чем на 1, и т. д. Возникает последовательность точек Ао, Аь А2, .... Доказать, что в этой по- следовательности все точки, начиная с некоторой, совпадают. Примечание. Если для некоторой точки Ak внутри круга радиуса 1 с центром Ak нет ни одной материальной точки, то полагаем, что A*=A*+i = .... 60(8). С упорядоченными наборами из п нулей и единиц разрешается проделывать две операции: изменять первую слева цифру, а также изменять цифру, следующую после первой (сле- ва) единицы. Доказать, что такими операциями из любого на- бора можно получить любой другой. 61(8). Четыре равные окружности Оь О2. Оз» лежат внут- ри треугольника, причем окружность О| касается двух сторон треугольника, окружность О2 — другой пары сторон этого тре- угольника, окружность Оз — третьей пары сторон, а окружность О4 касается первых трех окружностей. Доказать, что центр О4 ле- жит на одной прямой с центрами вписанной и описанной окруж- ностей треугольника. 62(8—10). На бесконечной клетчатой бумаге нарисованы две не пересекающиеся бесконечные в обе стороны ломаные, звенья которых идут по линиям бумаги и каждая из которых проходит через все узлы бумаги. Обязательно ли эти ломаные имеют общие звенья? 63(8—10). Обозначим через сумму п первых простых чи- сел. Доказать, что между числами и расположен полный квадрат. 64(8—10). На плоскости нарисованы квадрат и правильный треугольник. Доказать, что какое-то из расстояний от вершины квадрата до вершины треугольника иррационально. 65(9—10). Город, имеющий форму квадрата, разделен на п2 квадратных кварталов. По улицам между кварталами введено двустороннее движение, а вокруг города — одностороннее. Ве- лосипедист едет по городу, соблюдая правила уличного движе- ния, т. е. едет по правой стороне улицы и на перекрестках не поворачивает налево (на внешней односторонней улице он обязан ехать так, чтобы дома находились все время справа от него). При каких п можно утверждать, что велосипедист может объехать весь город, побывав на каждой стороне каждой улицы по одно- му разу (на внешней улице — на ее единственной стороне)? Постарайтесь найти возможно более широкий набор искомых значений п. 66(8). Восьмиугольник А1А2АзА4А5АбА7Ав вписан в окруж- ность. Оказалось, что А|А2||А5Аб» А2Аз||АбА7, А3А4||А7А8. Дока- зать, что А4А5 = А1А8. 67(8—9). Дан квадрат со стороной п. Его разбили на п2 квадратных клеток со стороной 1 каждая. Можно ли расставить в клетки п2 различных чисел так, чтобы в любом квадрате, сто- 297
роны которого идут по сторонам данного квадрата или по линиям разбиения, произведение чисел, стоящих на одной диагонали равнялось произведению чисел, стоящих на второй диагонали? 68(8—10). Могут ли три человека пройти из точек, находя- щихся на расстоянии 0,1 и 2 от начала дороги, до точек, находя- щихся на расстоянии 1000, 1001, 1002 от начала дороги, не обгоняя друг друга, так, чтобы последний человек всю дорогу видел первого, но ни в какой момент не видел второго человека? Дорога прямая, но не ровная. Ростом людей принебречь. 69(8). Обозначим через S (N) сумму цифр числа N. Доказать, что существует бесконечно много таких /V, что в десятичной записи N отсутствуют нули и: a) N делится на S (N); б) N де- лится на S 1. 70(8—10). Доказать, что из отрезков, имеющих длины 1, 2, 3, ...» 1980, можно составить выпуклый 1980-угольник, у которого все углы равны между собой. Верно ли аналогичное утвержде- ние для 1981-угольника? 71 (9—10). Функции fug определены на прямой, и при всех х и у выполняется равенство f(x-y)+fU+^)=2f(x)g(y). Доказать, что если функция f не равна тождественно нулю, то значения функции g (у) при всех у не меньше — 1. 72(9—10). Имеется полный граф: п точек, каждые две из которых соединены отрезком. Каждый отрезок покрашен либо красным, либо синим цветом, причем из любой точки в любую другую можно пройти, пользуясь как только синими путями, так и только красными. Доказать, что среди п точек найдутся такие четыре точки, что в полном графе, составленном из этих четырех точек и соединяющих их отрезков, будет выполнено то же свойство: из любой точки в любую другую можно пройти как только синими, так и только красными путями. 73(9—10). Многочлен Р(х) при всех неотрицательных х при- нимает неотрицательные значения. Обязательно ли найдутся такие два многочлена, Q (х) и /?(х), что Р (x) = (Q (x))2-f-(/? (х))2? 74(7—10). Решить в целых числах систему ( xz = 2yt, ( х2 —2r/2 = 6/2 —3z2. 75(10). Самолет совершил беспересадочный полет из города Г1 в город Г2. В течение всего полета за самолетом велось наблюдение со станций слежения А и В, расположенных на от- резке Г|Г2. а) Доказать, что найдется промежуток времени величиной в 1 с, в течение которого самолет сместился из некоторой точ- ки X траектории в точку Y на одинаковые углы относительно каждой из станций слежения (т. е. Z_XAY = 2.XBY). б) Доказать, что это утверждение перестает быть верным, 298
если хотя бы одна из станций слежения не лежит на отрезке Г|Г>, как бы близко точки А и В ни находились от точек Гн Г2 соответст- венно. Примечание. Самолет движется в пространстве; его ско- рость, вообще говоря, переменная; время всего полета превос- ходит 1 с; города Г1 и Г2, станции слежения Диви самолет — точки. 76(9—10). Бассейн имеет форму выпуклого четырехугольни- ка, в вершинах которого растут деревья. Каждое дерево отбрасы- вает тень, имеющую форму круга с центром в соответствующей вершине. Известно, что весь бассейн находится в тени. Дока- зать, что тень каких-то трех деревьев полностью покрывает треугольник, в вершинах которого эти деревья растут. 77(8—10). На прямой расположена колония из конечного числа бактерий. В моменты /=1, 2, 3, ... некоторые из бактерий могут погибать; новых бактерий не возникает ни в один момент. Погибают те и только те бактерии, от которых ни слева на рас- стоянии 1, ни справа на расстоянии нет бактерий. Существует ли такая колония бактерий, которая будет существовать вечно? 78(9—10). Существует ли конечное множество точек на пло- скости, для каждой из которых среди оставшихся найдется не менее 1000 точек, удаленных от нее на расстояние 1? 79* (10). Додекаэдр, вершины которого покрашены красной краской, начинают перекатывать по плоскости через его ребра; при этом вершины додекаэдра дают на плоскости красные точеч- ные отпечатки. Доказать, что для любого кружка произвольного радиуса г на плоскости додекаэдр можно перекатывать так, чтобы какой-то красный отпечаток одной из вершин попал в этот кружок. 80(9—10). Из трех стержней одинаковой длины изготовлена жесткая пространственная конструкция, в которой стержни не соприкасаются друг с другом, а только связаны нерастяжимыми нитями, прикрепленными к их концам. а) Какое наименьшее число нитей необходимо для этого? б) При каком соотношении между длинами стержней и дли- нами нитей можно изготовить такую конструкцию?
Литература 1. Б а л к М. Б. Геометрические приложения понятия о центре тяжести.— М.: Физматгиз, 1956. 2. Барр Ст. Россыпи головоломок.— М.: Мир, 1984. 3. Б и з а м Д., Г е р ц е г Я. Игра и логика.— М.: Мир, 1975. 4. Б и з а м Д., Г е р ц е г Я. Многоцветная логика.— М.: Мир 1978. 5. Г а р д н е р М. Математические головоломки и развлече- ния.— М.: Мир, 1971. 6. Г а р д н е р М. Математические досуги.— М.: Мир, 1972. 7. Гарднер М. Математические новеллы.— М.: Мир, 1974. 8. Г а р д н е р М. Есть идея! — М.: Мир, 1982. 9. Дьюдени Г. Э. Кентерберийские головоломки.— М.: Мир, 1979. 10. Дьюдени Г. Э. 520 головоломок.— М.: Мир, 1975. 11. ЗубелевичГ. И. Сборник задач Московских мате- матических олимпиад (V—VIII классы).— М.: Просвещение, 1971. 12. Избранные задачи (из журнала American Mathematical Monthly).—М.: Мир, 1977. 13. К о л м о г о р о в А. Н. О профессии математика.— М.: Изд-во МГУ, 1959. 14. Кэррол Л. История с узелками.— М.: Мир, 1973. 15. Кюршак Й, Нейкомм Д., Ха йоги Д., Ш у р а- н и Я. Венгерские математические олимпиады.— М.: Мир, 1976. 16. Линдгрен Г. Занимательные задачи на разрезание.— М.: Мир, 1977. 17. Лойд С. Математическая мозаика.— М.: Мир, 1980. 18. Математический цветник (Юбилейная книга к 65-летию М. Гарднера).— М.: Мир, 1983. 19. Морозова Е. А., Петраков И. С., Скворцов В. А. Международные математические олимпиады.— М.: Просвеще- ние, 1976. 20. Радемахер Г. Р.» Т е п л и ц О. Числа и фигуры.— М.: Физматгиз, 1962. 21. Сборник задач Московских математических олимпиад/ Сост. А. А. Леман.— М.: Просвещение, 1965. 22. С м а л л и а н Р. М. Как же называется эта книга? — М.: Мир, 1981. 23. С т р а ш е в и ч С., БровкинЕ. Польские математиче- ские олимпиады.— М.: Мир, 1978. 24. Тригг Ч. Задачи с изюминкой.— М.: Мир, 1975. 25. Ш к л я р с к и й Д. О., Чепцов Н. Н., Я гл ом И. М. Избранные задачи и теоремы элементарной математики, ч. I, арифметика и алгебра.—М.: Наука, 1965. 300
26. Ш к л я р с к и й Д. О., Ченцов Н. Н., Я гл ом И. М. Избранные задачи и теоремы элементарной математики, ч. П, геометрия (планиметрия).—М.: Гостехнздат, 1952. 27. Ш к л я р с к и й Д. О., Ч е н ц о в Н. Н., Я г л о м И. М. Избранные задачи и теоремы элементарной математики, ч. III, геометрия (стереометрия).— М.: Гостехнздат, 1954. 28. Ш т е й н г а у з Г. Задачи и размышления.— М.: Мир, 1974. 29. Эббот Э. Флатландия. Бюргер Д. Сфсрландия.— М.: Мир, 1976. 30. Я гл ом А. М., Я гл ом И. М. Неэлементарные задачи в элементарном изложении.— М.: Гостехнздат, 1954. 31. Я гл ом И. М., Болтянский В. Г. Выпуклые фигу- ры.— М.: Гостехнздат, 1951. 32. Я гл ом И. М. Геометрические преобразования, т. I.— М.: Гостехнздат, 1955; т. II.— М.: Гостехнздат, 1959. Статьи о Московских олимпиадах в журнале «Успехи математических наук» 33. Б о н ч к о в с к и й Р. Н. Вторая Московская математиче- ская олимпиада, 1936, т. II, с. 275—278. 34. Г а л ь п е р н С. А. IX Московская математическая олим- пиада школьников, 1946, т. I, вып. 3—4, с. 206—211. 35. Петровский И. Я., Ф е т и с о в А. И. X Московская математическая олимпиада школьников, 1947, т. II, вып. 5, с. 243—247. 36. К р е й н е с М. А., М о р о з о в а Е. А., Ченцов Н. Н. XIII Московская математическая олимпиада школьников, 1950, т. V, вып. 5, с. 204. 37. Д е л о н е Б. Н., В в е д е н с к а я Н. Д. XIV Московская школьная математическая олимпиада, 1951, т. VII, вып. 4, с. 180-184. 38. Рашевский П. К-, Введенская Н. Д., Коро- ле в Б. М. XV Московская школьная математическая олим- пиада, 1953, т. VIII. вып. 4. 39. М е н ь ш о в Д. Е., Шафаревич И. Р., Морозо- ва Е. А., Золотарев В. И. XVI Московская школьная ма- тематическая олимпиада, 1954, т. IX, вып. 3. с. 257—262. 40. Б а х в а л о в С. В., Ж и д к о в Н. П., Сафронов И. Д., ЛупановО. Б. XVII Московская школьная математическая олимпиада, 1955, т. X, вып. 1, с. 213—219. Статьи о Московских олимпиадах в журнале «М атематика в школе» 41. Фетисов V И. IX Математическая олимпиада школь- ников в Москве, 1947, № 1, с. 66—67. 301
42. Ф е т и с о в А. И. IX Математическая олимпиада уча- щихся московских школ, 1947, № 3, с. 54—60. 43. Задачи для математических кружков (задачи VI Москов- ской математической олимпиады школьников), 1948, № 6, с. 62. 44. Т а н а т а р. И. Я. ХП Математическая олимпиада уча- щихся средних школ г. Москвы (1-й тур), 1949, № 5, с. 42—47. 45. Та на тар И. Я. XII Математическая олимпиада уча- щихся средних школ г. Москвы (2-й тур), 1949, № 6, с. 36—42. 46. Та на тар И. Я. XIII Московская математическая олим- пиада, 1950, № 6, с. 54. 47. Решения задач, предлагавшихся на Всероссийской олим- пиаде по математике учащимся 7—10 классов, 1962, № 1, с. 86—90. 48. Васильев Н. Б. Задачи XXVIII Московской матема- тической олимпиады, 1965, № 3, с. 70— 75. 49. Гальперин Г. А. XXXV Московская математическая олимпиада, 1972, № 6, с. 58—64. 50. КирилловА. А.. Гальперин Г. А. XXXVI Мо- сковская математическая олимпиада, 1973, № 5, с. 79—82. 51. Г а л ь п е р и н Г. А. Задачи XXXVII Московской мате- матической олимпиады, 1974, № 6, с. 65—69. 52. КолмогоровА. Н., Гальперин Г. А. XXXVIII Московская математическая олимпиада, 1976, № 4, с. 68—72. 53. Тихомиров В. М., Гальперин Г. А. 42-й Мо- сковская математическая олимпиада, 1980, № 3, с. 63—66. Статьи о Московских олимпиадах в журнале «Математическое просвещение» 54. Гире а нов И. В. XIX школьная математическая олим- пиада в Москве, 1957, т. 1, с. 187—194. 55. О л е й н и к О. А., К и р и л л о в А. А. XX школьная математическая олимпиада в Москве, 1958, т. 3, с. 221—227. 56. Болтянский В. Г., Розендорн Э. Р. XXI школь- ная математическая олимпиада в Москве, 1961, т. 6, с. 301—309.
СОДЕРЖАНИЕ Предисловие редактора............................................. 3 Предисловие авторов............................................. 5 Из истории Московских олимпиад .... — Указания к работе с книгой ................................... 18 Часть I Задачи Московских математических олимпиад 1 олимпиада (1935 г.) 20 XXV олимпиада (1962 г.) 78 II олимпиада (1936 г.) 21 XXVI олимпиада (1963 г.) 81 III олимпиада (1937 г.) 22 XXVII олимпиада (1964 г.) 85 IV олимпиада (1938 г.) — XXVIII олимпиада (1965 г.) 89 V олимпиада (1939 г.) 23 XXIX олимпиада (1966 г.) 94 VI олимпиада (1940 г.) — XXX олимпиада (1967 г.) 95 VII олимпиада (1941 г.) 25 XXXI олимпиада (1968 г.) 99 VIII олимпиада (1945 г.) 27 XXXII олимпиада (1969 г.) 105 IX олимпиада (1946 г.) 28 XXXIII олимпиада (1970 г.) ПО X олимпиада (1947 г.) 31 XXXIV олимпиада (1971 г.) 116 XI олимпиада (1948 г.) 33 XXXV олимпиада (1972 г.) 120 хп олимпиада (1949 г.) 34 XXXVI олимпиада (1973 г.) 125 XIII олимпиада (1950 г.) 36 XXXV11 олимпиада (1974 г.) 128 XIV олимпиада (1951 г.) 38 XXXVIII олимпиада (1975 г.) 131 XV олимпиада (1952 г.) 40 XXXIX олимпиада (1976 г.) 133 XVI олимпиада (1953 г.) 43 XL олимпиада (1977 г.) 135 XVII олимпиада (1954 г.) 46 XL1 олимпиада (1978 г.) 138 XV1I1 олимпиада (1955 г.) 50 XL11 олимпиада (1979 г.) 139 XIX олимпиада (1956 г.) 54 XLI11 олимпиада (1980 г.) 141 XX олимпиада (1957 г.) 58 XLIV олимпиада (1981 г.) 143 XXI олимпиада (1958 г.) 61 XLV олимпиада (1982 г.) 144 ххп олимпиада (1959 г.) 66 XLV1 олимпиада (1983 г.) 146 XX111 олимпиада (1960 г.) 70 XLV11 олимпиада (1984 г.) 148 XXIV олимпиада (1961 г.) 73 XLV111 олимпиада (1985 г.) 150 Часть II Решения, указания, ответы............................153 Приложение 1 283 Приложение 2 291 Литература...........................................300
Григорий Александрович Гальперин Алексей Кириллович Толпыго МОСКОВСКИЕ МАТЕМАТИЧЕСКИЕ ОЛИМПИАДЫ Зав. редакцией Р. А. Хабиб Редактор Л. Н. Белоновская Младшие редакторы Л. Е. Козырева, Е. А. Сафронова Художник Е. С. Шабе ль ник Художественный редактор Е. Н. Карасик Технический редактор Г» В. Субочева Корректоры Н. Б. Гитлсвич, И. Н. Панкова ИБ № 9670 Сдано в набор 10.11.85. Подписано к печати 03.10.86. Формат 60 X 9О'/|6. Бум. кн.-журн. отеч. Гарнит. Литературная. Печать высокая. Усл. печ. л. 19 + 0,25 форз. Усл. кр.-отт. 20,31. Уч.-изд. л. 18,83 + 0,46 форз. Тираж 680 000 экз. Заказ 247. Цена 75 К0Пв Ордена Трудового Красного Знамени издательство «Просвещение» Государст- венного комитета РСФСР по делам издательств, полиграфии и книжной торговли. 129846, Москва, 3-й проезд Марьиной рощи, 41. Саратовский ордена Трудового Красного Знамени полиграфический комбинат Росглавполиграфпрома Государственного комитета РСФСР по делам изда- тельств, полиграфии и книжной торговли. 410004, Саратов, ул. Чернышевского, 59.